NCLEX Style Practice Questions - MedSurg DIABETES, Endocrine NCLEX Questions, Endocrine Nclex, Medical-Surgical Nursing Chapter 50 Endocrine Problems, NCLEX Med-Surg Endocrine Chapter Qs/Ans/Rationales, NCLEX Study - Med-Surg: Endocrine, MedSurg 2 Ex...
List the inflammatory mediators that are released from mast cells as they degranulate during an asthma attack
- leukotrienes : make airway hyperactive - histamine : vasodilation, construction of smooth muscle - cytokines : increases inflammation - prostaglandins : - nitric oxide : causes vasodilation
How should the nurse record smoking history on a pt who has smoked 2.5 packs a day for 10 years?
Pt has a 25 pack-year smoking history
What is are some signs of cor pulmonale?
- edema in the ankles - results from backup of fluid into the venous system as the right side of the heart fails - dyspnea with relatively normal lung sounds - distended neck veins - hepatomegaly - weight gain
The nurse is reinforcing discharge teaching with a client diagnosed with TB and has been on medication for 1.5 weeks. The nurse knows that the client has understood the information if which statement is made?
"I should not be contagious after 2-3 weeks of medication therapy."
What happens to the blood pH as a patient continues to have an asthma attack?
- airway becomes more obstructed and air gets trapped within the more distal airways - CO2 builds up as the patient tires - leads to respiratory acidosis
A nurse is instructing a client with type 2 diabetes mellitus regarding the pathophysiology of the client's condition. Which statement made by the client reflects to the nurse an understanding of the teaching?
"My cells are resistant to the effects of insulin." Resistance to insulin at cellular receptors in typical WRONG ANSWER: "My pancreas does not produce insulin." This is the mechanism associated with type 1 diabetes mellitus.
Why is sweat salty in patients with CF?
- failure of CFTR protein results in huge losses of sodium and chloride in sweat - CFTR protein fails to reabsorb chloride in the sweat duct --> 4x as much sodium chloride in sweat
What are the most common causes of COPD exacerbation?
- bacterial or viral respiratory infections
airborne route
* TEST* *Which identified the route of transmission of tuberculosis(TB)?* A. Hand to mouth B. the enteric route C. airborne route D. blood and body fluids
Diet recommendations for patients with COPD
- high protein and calorie to maintain weight and energy - fluid intake of at least 3 L per day
Peripheral cyanosis
- initial response to poor oxygenation and circulation in the body - causes vasoconstriction of peripheral blood vessels to ensure that the vital organs are receiving enough blood - earlobes, nail beds and finger tips are cool and pale or bluish
Progression of cystic fibrosis
- begins as bronchiolitis : tenacious mucus causes blockage in bronchioles and leads to chronic infections and inflammation - becomes bronchitis as the infections begin to involve more of the airways - pulmonary remodeling occurs from hypoxia and vasoconstriction of pulmonary arteries - pulmonary hypertension, formation of blebs and bullae --> hemoptysis - cor pulmonale
Effect of acetylcholine on smooth muscle of the airway
- causes contraction and increased mucus secretion - leads to bronchoconstriction
Describe what happens when cigarette smoke enters the lungs
- causes inflammation of airways -->release of immune cells and inflammatory mediators - causes remodeling of smaller airways - destroys capillary bed and causes deposit of collagen throughout airway
Effects of cigarette smoke on the airways
- irritates cells lining airway, causes hyperplasia of goblet cells --> increased mucus production - destroys cilia and reduces their action --> decreased airway clearance - causes abnormal dilation of alveoli and eventual destruction - causes airway remodeling
Asthma
- chronic inflammatory disorder - recurrent episodes of wheezing, breathlessness, chest tightness, and cough
What is a normal PT?
11.0-12.5 seconds
What is a normal aPTT?
30-40 seconds
Which nursing measure would be effective in preventing complications in a client with Addison's disease? 1. Restricting fluid intake 2. Offering foods high in potassium 3. Checking family support systems 4. Monitoring the blood glucose
4. Monitoring the blood glucose rationale The decrease in cortisol secretion that characterizes Addison's disease can result in hypoglycemia. Therefore monitoring the blood glucose would detect the presence of hypoglycemia so that it can be treated early to prevent complications. Fluid intake should be encouraged to compensate for dehydration. Potassium intake should be restricted because of hyperkalemia. Option 3 is not a priority for this client.
liver enzyme levels
A client with tuberculosis is being started on antituberculosis therapy with isoniazid. Before giving the client the first dose, the nurse ensures that which baseline study has been completed? A. electrolyte levels B. coagulation levels C. liver enzymes levels D. serum creatinine level
What parts are on a chest tube drainage system?
Client connector Drainage collection chambers Water seal chamber Air ventilation Suction control chamber Suction regulator
The nurse is assigned to care for a client after a left pneumonectomy. Which position is contraindicated for this patient?
Lateral position
Glomerulonephritis
A patient with a sore throat is to have a throat culture to establish whether the infection is being caused by Streptococcus pyogenes. If it is a streptococcal infection and the patient is not treated, what may the patient be at risk for? A. Cystitis B. Hepatitis C. Glaucoma D. Glomerulonephritis
A patient with acute dyspnea is scheduled for a spiral computed tomography (CT) scan. Which information obtained by the nurse is a priority to communicate to the health care provider before the CT? a. Allergy to shellfish b. Apical pulse of 104 c. Respiratory rate of 30 d. Oxygen saturation of 90%
ANS: A Because iodine-based contrast media is used during a spiral CT, the patient may need to have the CT scan without contrast or be premedicated before injection of the contrast media. The increased pulse, low oxygen saturation, and tachypnea all indicate a need for further assessment or intervention but do not indicate a need to modify the CT procedure.
B. pneumothorax
If a patient is exhibiting sudden chest pain or tightness, dyspnea, increase pulse and respirations, decreased BP, and absence of normal chest movements and breath sounds the patient is probably experencing an: A. myocardial infarction B. pneumothorax C. acute tonsilitis D. acute asthma attack
16. A 44-year-old patient is admitted with tetany. Which laboratory value should the nurse monitor? a. Total protein b. Blood glucose c. Ionized calcium d. Serum phosphate
ANS: C Tetany is associated with hypocalcemia. The other values would not be useful for this patient. DIF: Cognitive Level: Apply (application) REF: 1146 TOP: Nursing Process: Assessment MSC: NCLEX: Physiological Integrity
A patient with right lower-lobe pneumonia has been treated with IV antibiotics for 3 days. Which assessment data obtained by the nurse indicates that the treatment has been effective? a. Bronchial breath sounds are heard at the right base. b. The patient coughs up small amounts of green mucus. c. The patient's white blood cell (WBC) count is 9000/µL. d. Increased tactile fremitus is palpable over the right chest.
ANS: C The normal WBC count indicates that the antibiotics have been effective. All the other data suggest that a change in treatment is needed.
BiPAP is indicated for what patients?
ARDS client with COPD Heart failure Sleep apnea
2 Because thoracentesis involves the introduction of a catheter into the pleural space, there is a risk of pneumothorax. Thoracentesis does not carry a significant potential for causing bronchospasm, pulmonary edema, or respiratory acidosis. Text Reference - p. 493
After assisting the health care provider with a bedside thoracentesis, the nurse should continue to assess the patient for signs and symptoms of what? 1 Bronchospasm 2 Pneumothorax 3 Pulmonary edema 4 Respiratory acidosis
After a laryngectomy, which of the following assessments takes priority?
Airway patency
What actions should the nurse perform post thoracentesis?
Another chest x-ray
A pt is admitted to the hospital with SOB. The nurse notes increasing confusion and combativeness during the past hour. Which of the following actions is appropriate first?
Assess SpO2 and apply oxygen per protocol if indicated
When obtaining a health history from a patient with acute pancreatitis, the nurse asks the patient specifically about a history of a. cigarette smoking. b. alcohol use. c. diabetes mellitus. d. high-protein diet.
B Rationale: Alcohol use is one of the most common risk factors for pancreatitis in the United States. Cigarette smoking, diabetes, and high-protein diets are not risk factors.
Symptoms of Hypoglycemia.
Elevated blood pressure. Cold, clammy skin. Tremors.
A nurse is collecting data from a client who has Graves' disease. The nurse should expect the client to report what?
Difficulty sleeping. Graves' disease is a common form of hyperthyroidism, which can cause difficulty sleeping and anxiety.
Which of the following assessment findings does the nurse expect in the pt with emphysema?
Diminished breath sounds
A. emphysema
Diseases that are considered to be obstructive pulmonary disorders that cause chronic airflow limitatons(CAL) include: A.emphysema B. acute bronchitis C. pleurisy D. pulmonary tuberculosis
What are the three chambers of the chest drainage unit (CDU)?
Drainage collection chamber Water seal chamber Suction control chamber
When is bubbling abnormal in the CDU?
If there is continuous bubbling in the water seal chamber, then you have an air leak in the system. Never clap a chest tube without an order. - If could lead to a tension pnumothorax
D. Extreme temperature elevation and rapid pulse rate
In the postoperative period, the LPN/LVN should observe a patient who has had a thyroidectomy for which signs of thyroid crisis? A. Depression and fatigue B. Respiratory distress and hoarseness C. Twitching of muscles and severe convulsions D. Extreme temperature elevation and rapid pulse rate
What is the purpose of the chest drainage unit (CDU)?
It is to restore the normal vacuum pressure in the pleural space. The CDU does this by removing all air and fluid in a closed one-way system until the problem is corrected.
When are chest tubes indicated for a client?
Lung collapse
Flail chest occurs with what?
Multiple rib fractures
Thyroidectomy:
Semi Fowler and avoid hyperflexion and hyperextension of the neck
Which identifies the route of transmission of TB?
The airborne route
C. Good
The hemoglobin A1c test results of the patient are 8.2%. The nurse knows which word best describes the patient's glycemic control? A. Fair B. Poor C. Good D. Excellent
What is the purpose of the water seal chamber?
To promote one-way flow out of the pleural space which will air from moving back up the system and into the chest.
The nurse determines that the pt in acute adrenal insufficiency is responding favorably to trtmt when a. the patient appears alert and oriented b. the patient's urinary output has increased c. pulmonary edema is reduced as evidenced by clear lung sounds d. laboratory tests reveal serum elevations of K and glucose and a decrease in sodium
a. the patient appears alert and oriented (R- confusion, irritability, disorientation, or depressioni s often present in pt with Addison's dz, and a (+) response to therapy would be indicated by a return to alertness and orientation. Other indication of response to therapy would be a decreased urinary output, decreased serum potassium, and increased serum sodium and glucose. The pt with Addison's would be very dehydrated and volume-depleted and would not have pulmonary edema.)
The nurse is caring for the client diagnosed with ascites from hepatic cirrhosis. What information should the nurse report to the health-care provider? a. A decrease in the client's daily weight of one (1) pound. b. An increase in urine output after administration of a diuretic. c. An increase in abdominal girth of two (2) inches. d. A decrease in the serum direct bilirubin to 0.6 mg/dL.
c. An increase in abdominal girth of two (2) inches. Rationale: An increase in abdominal girth would indicate that the ascites is increasing, meaning that the client's condition is becoming more serious and should be reported to the health-care provider.
Which of the following diagnostic tests should be performed annually over age 50 to screen for colon cancer? a. Abdominal CT scan b. Abdominal x-ray c. Colonoscopy d. Fecal occult blood test
d. Fecal occult blood test Surface blood vessels of polyps and cancers are fragile and often bleed with the passage of stools. Abdominal x-ray and CT scan can help establish tumor size and metastasis. A colonoscopy can help locate a tumor as well as polyps, which can be removed before they become malignant.
Of which of the following symptoms might an older woman with diabetes mellitus complain? a. Anorexia b. Pain intolerance c. Weight loss d. Perineal itching
d. Perineal itching
A diagnosis of hyperglycemic hyperosmolar nonketotic coma (HHNC) is made for a patient with type 2 diabetes who is brought to the emergency department in an unresponsive state. The nurse will anticipate the need to a. administer glargine (Lantus) insulin. b. initiate oxygen by nasal cannula. c. insert a large-bore IV catheter. d. give 50% dextrose as a bolus.
C Rationale: HHNC is initially treated with large volumes of IV fluids to correct hypovolemia. Regular insulin is administered, not a long-acting insulin. There is no indication that the patient requires oxygen. Dextrose solutions will increase the patient's blood glucose and would be contraindicated.
The client tells the nurse that the client really misses having sugar with tea in the morning. What is an alternative that the nurse could advise them to help sweeten their drink. a. Oatrim c. sucralose b. Olestra d. tannin
C Aspartame is the generic name for a sweetener composed of two amino acids, phenylalanine and aspartic acid. Olestra and Oatrim are fat replacers and tannin is an acid found in some foods such as tea.
The nurse is having difficulty obtaining a capillary blood sample from a client's finger to measure blood glucose using a blood glucose monitor. Which procedure will increase the blood flow to the area to ensure an adequate specimen? a. Raise the hand on a pillow to increase venous flow. b. Pierce the skin with the lancet in the middle of the finger pad. c. Wrap the finger in a warm cloth for 30-60 seconds. d. Pierce the skin at a 45-degree angle.
c. Wrap the finger in a warm cloth for 30-60 seconds. The hand is lowered to increase venous flow. The finger is pierced lateral to the middle of the pad perpendicular to the skin surface.
When a client learned that the symptoms of diabetes were caused by high levels of blood glucose the client decided to stop eating carbohydrates. In this instance, the nurse would be concerned that the client would develop what complication? a. acidosis b. atherosclerosis c. glycosuria d. retinopathy
A When a client's carbohydrate consumption is inadequate ketones are produced from the breakdown of fat. These ketones lower the pH of the blood, potentially causing acidosis that can lead to a diabetic coma.
C. To evaluate the thyroid's ability to accumulate iodine
A patient is admitted to the hospital with a suspected diagnosis of hyperthyroidism. She asks the LPN/LVN about the thyroid scan for which she is scheduled that morning. The nurse should tell her that it is done for which purpose? A. To collect a blood sample for analysis B. To determine underactivity of the thyroid cells C. To evaluate the thyroid's ability to accumulate iodine D. To detect abnormal areas of growth or tissue in the thyroid
D. Insufficiencies of the mineralocorticoids and glucocorticoids
A patient is diagnosed as having Addison disease. This condition is the result of which disturbance? A. A secreting tumor of the adrenal cortex B. Overfunctioning of the pituitary and hypothalamus C. Excessive secretion of adrenocorticotropic hormone D. Insufficiencies of the mineralocorticoids and glucocorticoids
B. Asking the patient if she has recently eaten shrimp or clams D. Explaining to the patient that the dose of radioactivity she will be exposed to during testing is very small
A patient is scheduled for thyroid function tests in the morning. Patient preparation for this diagnostic procedure includes which interventions? (Select all that apply.) A. Ascertaining if the patient is claustrophobic Instructing the patient to discard her first morning urine B. Asking the patient if she has recently eaten shrimp or clams C. Keeping the patient on NPO status (nothing by mouth) after midnight D. Explaining to the patient that the dose of radioactivity she will be exposed to during testing is very small
C. "Fasting is required for 10 to 12 hours before the test."
A patient is scheduled to have a glucose tolerance test. What information should be included in the instructions provided to the patient? A. "Eat a normal-sized breakfast before the test." B. "Limit carbohydrate intake 5 to 7 days before the test." C. "Fasting is required for 10 to 12 hours before the test." D. "During the test, you may eat low-carbohydrate snacks."
C. Decreased ability to respond to stress
The nurse who works primarily with older adults knows that which of the following is a significant age-related change of the endocrine system? A. Change in glucose production B. Inconsistent hormone production C. Decreased ability to respond to stress D. Increased number of endocrine disorders
B. "I will draw up the regular insulin before the NPH."
A patient newly diagnosed with diabetes is learning to administer his injections of NPH and regular insulin. Which statement indicates that the patient understands the nurse's teaching regarding proper insulin administration? A. "I will draw up the NPH before the regular insulin." B. "I will draw up the regular insulin before the NPH." C. "I will give myself the NPH and the regular insulin in two different injections." D. "It doesn't matter which insulin I draw up first, as long as the amount is correct.
A. 2-hour test results of 175 mg/dL
A patient has completed glucose tolerance testing. The results are normal. Which result is reflective of this finding? A. 2-hour test results of 175 mg/dL B. 2-hour test results of less than 225 mg/dL C. Fasting blood glucose of 50 mg/dL D. Fasting blood glucose of 130 mg/dL
C. Vasopressin
A patient has diabetes insipidus (DI). The LPN/LVN should plan to provide information on which medication? A. Insulin B. Cortisone C. Vasopressin D. Fludrocortisone acetate
A patient with type 2 diabetes is scheduled for an outpatient coronary arteriogram. Which information obtained by the nurse when admitting the patient indicates a need for a change in the patient's regimen? a. The patient's most recent hemoglobin A1C was 6%. b. The patient takes metformin (Glucophage) every morning. c. The patient uses captopril (Capoten) for hypertension. d. The patient's admission blood glucose is 128 mg/dl.
B Rationale: To avoid lactic acidosis, metformin should not be used for 48 hours after IV contrast media are administered. The other patient data indicate that the patient is managing the diabetes appropriately.
C. Apply a moisturizing lotion to dry feet, but not in between the toes
The nurse is assisting with preparing a teaching plan for the client with diabetes mellitus regarding proper foot care. Which instruction should include in the plan of care? A. Soak the feet in hot water B. Avoid using soap to wash the feet C. Apply a moisturizing lotion to dry feet, but not in between the toes D. Always have a podiatrist cut your toesnails; never cut them yourself
C. Treat hypocalcemic tetany
The nurse is caring for a client after a thyroidectomy and notes that calcium gluconate is prescribed for the client. The nurse determines that this medication has been prescribed for which reason? A. Treat thyroid storm B. prevent cardiac irritability C. Treat hypocalcemic tetany D. Stimulate the release of parathyroid hormone
A. Cortisol can be classified as a glucocorticoid. B. This patient is at an increased risk for hyperglycemia. E. Cortisol works to counteract the body's response to inflammation
The nurse is caring for a patient with a deficiency of cortisol. The nurse correctly recognizes which information about this disorder? (Select all that apply.) A. Cortisol can be classified as a glucocorticoid. B. This patient is at an increased risk for hyperglycemia. C. Cortisol levels are controlled by the posterior pituitary gland. D. The patient has an increased chance of becoming overweight. E. Cortisol works to counteract the body's response to inflammation
D. Reassure the client that is is usually temporary condition
The nurse is collecting data regarding a client after a thyroidectomy and notes that the client has developed hoarseness and a weak voice, Which nursing action is appropriate? A. Check for signs of bleeding B. Administer calcium gluconate C. Notify the registered nurse immediately D. Reassure the client that is is usually temporary condition
C. Benign pituitary adenoma
The patient presents to the clinic with acromegaly, muscle weakness, and osteoporosis. Laboratory results show elevated growth hormone levels. These symptoms, along with computed tomography scan findings, most closely correlate with which disorder? A. Astrocytoma B. Craniopharyngioma C. Benign pituitary adenoma D. Malignant pituitary adenoma
A. Urine osmolality B. Serum osmolality
The patient presents to the emergency department with symptoms of syndrome of inappropriate antidiuretic hormone (SIADH). The nurse knows to educate the patient about which test(s) or procedure(s) that will be performed before a diagnosis can be made? (Select all that apply.) A. Urine osmolality B. Serum osmolality C. Glucose tolerance testing D. Magnetic resonance imaging (MRI) E. Radioactive iodine uptake test (RAIU)
D. Thirst, dry mucous membranes, and dry skin
When teaching a patient with type 1 diabetes about home care, the LPN/LVN would be sure to include which signs of diabetic ketoacidosis? A. Dark, scanty urine, and diarrhea B. Cool, clammy skin, and nervousness C. Hunger, headache, and tremulousness D. Thirst, dry mucous membranes, and dry skin
A client is brought to the emergency room in an unresponsive state, and a diagnosis of hyperglycemic hyperosmolar nonketotic syndrome is made. The nurse would immediately prepare to initiate which of the following anticipated physician's orders? a) endotracheal intubation b) 100 units of NPH insulin c) intravenous infusion of normal saline d) intravenous infusion of sodium bicarbonate
c) intravenous infusion of normal saline The primary goal of treatment in hyperglycemic hyperosmolar nonketotic syndrome (HHNS) is to rehydrate the client to restore fluid volume and to correct electrolyte deficiency. Intravenous fluid replacement is similar to that administered in diabetic ketoacidosis (DKA) and begins with IV infusion of normal saline. Regular insulin, not NPH insulin, would be administered. The use of sodium bicarbonate to correct acidosis is avoided because it can precipitate a further drop in serum potassium levels. Intubation and mechanical ventilation are not required to treat HHNS.
Venturi mask uses and limitations
- delivers precise, high flow rates of O2 - mask is uncomfortable - patient can talk but voice may be muffled
A nurse is reinforcing teaching with a client who has type 1 diabetes mellitus about exercise. An appropriate statement by the nurse is what?
"Always wear a medical alert identification tag when you exercise." Because exercise may actually potentiate the effects of insulin and cause the blood glucose levels to decrease, the client should always wear a medical alert identification tag in the event of a hypoglycemic response.
Which of the following responses is correct when a pt asks why the HCP did not order a new antiviral drug for flu symptoms that started three days ago?
"Antivirals work only if you start them within 48 hours after flu symptoms begin."
The nurse in the occupational health clinic prepares to administer the influenza vaccine by nasal spray to a 35-year-old female employee. Which question should the nurse ask before administration of this vaccine? "Are you allergic to chicken?" "Could you be pregnant now?" "Did you ever have influenza?" "Have you ever had hepatitis B?"
"Could you be pregnant now?" The live attenuated influenza vaccine (LAIV) is given by nasal spray and approved for healthy people age 2 years to 49 years. The LAIV is given only to nonpregnant, healthy people. The inactivated vaccine is given by injection and is approved for use in people 6 months or older. The inactivated vaccine can be used in pregnancy, in people with chronic conditions, or in people who are immunosuppressed. Influenza vaccination is contraindicated if the person has a history of Guillain-Barré syndrome or a hypersensitivity to eggs.
A client has a prescription to take guaifenesin every 4 hours, as needed. The nurse determines that the client understands the most effective use of this medication if the client makes which statement?
"I will take the tablet with a full glass of water."
The nurse teaches a 20-year-old female patient who is prescribed budesonide (Rhinocort) intranasal spray for seasonal allergic rhinitis. The nurse determines that medication teaching is successful if the patient makes which statement? "My liver function will be checked with blood tests every 2 to 3 months." "The medication will decrease the congestion within 3 to 5 minutes after use." "I may develop a serious infection because the medication reduces my immunity." "I will use the medication every day of the season whether I have symptoms or not."
"I will use the medication every day of the season whether I have symptoms or not." Budesonide should be started 2 weeks before pollen season starts and used on a regular basis, and not as needed. The spray acts to decrease inflammation and the effect is not immediate as with decongestant sprays. At recommended doses, budesonide has only local effects and will not result in immunosuppression or a systemic infection. Zafirlukast (Accolate) is a leukotriene receptor antagonist and may alter liver function tests (LFTs). LFTs must be monitored periodically in the patient taking zafirlukast.
The nurse teaches a 66-year-old man with hypertension and osteoarthritis about actions to prevent and control epistaxis. Which statement, if made by the patient, indicates further teaching is required? "I should avoid using ibuprofen (Motrin) for pain and discomfort." "It is important for me to take my blood pressure medication every day." "I will sit down and pinch the tip of my nose for at least 10 to 15 minutes." "If I get a nosebleed, I will lie down flat and raise my feet above my heart."
"If I get a nosebleed, I will lie down flat and raise my feet above my heart." A simple measure to control epistaxis (or a nosebleed) is for the patient to remain quiet in a sitting position. Another measure would be to apply direct pressure by pinching the entire soft lower portion of the nose for 10 to 15 minutes. Aspirin or nonsteroidal antiinflammatory drugs such as ibuprofen increase the bleeding time and should be avoided. Elevated blood pressure makes epistaxis more difficult to control. The patient should continue with antihypertensive medications as prescribed.
Which is the best explanation to a pt by a nurse for why a health care provider does not prescribe antibiotics for influenza?
"Influenza is caused by viruses."
A pt asks the nurse why he doesn't feel sick even though his TB test is positive. The nurse knows the pt has been diagnosed with LTBI. Which explanation is best to provide to the pt?
"You have TB infection, but not active disease. As long as your immune system stays strong, it can keep the infection from making you feel sick."
A nurse is reviewing discharge teaching with a client who has Cushing's syndrome. Which statement by the client indicates that the instructions related to dietary management were understood? 1. "I can eat foods that contain potassium." 2. "I will need to limit the amount of protein in my diet." 3. "I am fortunate that I can eat all the salty foods I enjoy." 4. "I am fortunate that I do not need to follow any special diet."
*1. "I can eat foods that contain potassium."* *rationale* A diet that is low in calories, carbohydrates, and sodium but ample in protein and potassium content is encouraged for a client with Cushing's syndrome. Such a diet promotes weight loss, the reduction of edema and hypertension, the control of hypokalemia, and the rebuilding of wasted tissue.
Which statement by the client would cause the nurse to suspect that the thyroid test results drawn on the client this morning may be inaccurate? 1. "I had a radionuclide test done 3 days ago." 2. "When I exercise I sweat more than normal." 3. "I drank some water before the blood was drawn." 4. "That hamburger I ate before the test sure tasted good."
*1. "I had a radionuclide test done 3 days ago."* *rationale* Option 1 indicates that a recent radionuclide scan had been performed. Recent radionuclide scans performed before the test can affect thyroid laboratory results. No food, fluid, or activity restrictions are required for this test, so options 2, 3, and 4 are incorrect.
A client has just been admitted with a diagnosis of myxedema coma. If all of the following interventions were prescribed, the nurse would place highest priority on completing which of the following first? 1. Administering oxygen 2. Administering thyroid hormone 3. Warming the client 4. Giving fluid replacement
*1. Administering oxygen* *rationale* As part of maintaining a patent airway, oxygen would be administered first. This would be quickly followed by fluid replacement, keeping the client warm, monitoring vital signs, and administering thyroid hormones.
A nurse is collecting data on a client with a diagnosis of hypothyroidism. Which of these behaviors, if present in the client's history, would the nurse determine as being likely related to the manifestations of this disorder? 1. Depression 2. Nervousness 3. Irritability 4. Anxiety
*1. Depression* *rationale* Hypothyroid clients experience a slow metabolic rate, and its manifestation includes apathy, fatigue, sleepiness, and depression. Options 2, 3, and 4 identify the clinical manifestations of hyperthyroidism.
A nurse is reviewing a plan of care for a client with Addison's disease. The nurse notes that the client is at risk for dehydration and suggests nursing interventions that will prevent this occurrence. Which nursing intervention is an appropriate component of the plan of care? *Select all that apply.* 1. Encouraging fluid intake of at least 3000 mL/day 2. Encouraging an intake of low-protein foods 3. Monitoring for changes in mental status 4. Monitoring intake and output 5. Maintaining a low-sodium diet
*1. Encouraging fluid intake of at least 3000 mL/day* *3. Monitoring for changes in mental status* *4. Monitoring intake and output* t *rationale* The client at risk for deficient fluid volume should be encouraged to eat regular meals and snacks and to increase the intake of sodium, protein, and complex carbohydrates. Oral replacement of sodium losses is necessary, and maintenance of adequate blood glucose levels is required.
A client with Addison's disease asks the nurse how a newly prescribed medication, fludrocortisone acetate (Florinef), will improve the condition. When formulating a response, the nurse should incorporate that a key action of this medication is to: 1. Help restore electrolyte balance. 2. Make the body produce more cortisol. 3. Replace insufficient circulating estrogens. 4. Alter the body's immune system functioning.
*1. Help restore electrolyte balance.* *rationale* Fludrocortisone acetate is a long-acting oral medication with mineralocorticoid and moderate glucocorticoid activity. It is prescribed for the long-term management of Addison's disease. Mineralocorticoids cause renal reabsorption of sodium and chloride ions and the excretion of potassium and hydrogen ions. These actions help restore electrolyte balance in the body. The other options are incorrect.
In planning nutrition for the client with hypoparathyroidism, which diet would be appropriate? 1. High in calcium and low phosphorous 2. Low in vitamins A, D, E, and K 3. High in sodium with no fluid restriction 4. Low in water and insoluble fiber
*1. High in calcium and low phosphorous* *rationale* Hypocalcemia is the end result of hypoparathyroidism resulting from either a lack of parathyroid hormone (PTH) secretion or ineffective PTH influence on tissue. Calcium is the major controlling factor of PTH secretion. Because of this, the diet needs to be high in calcium but low in phosphorus because these two electrolytes must exist in inverse proportions in the body. The other options are not dietary interventions with hypoparathyroidism.
A client with hypoparathyroidism has hypocalcemia. The nurse avoids giving the client the prescribed vitamin and calcium supplement with which of the following liquids? 1. Milk 2. Water 3. Iced tea 4. Fruit juice
*1. Milk* *rationale* Milk products are high in phosphates, which should be avoided by a client with hypoparathyroidism. Otherwise, calcium products are best absorbed with milk because the vitamin D in the milk promotes calcium absorption.
A nurse notes in the medical record that a client with Cushing's syndrome is experiencing fluid overload. Which interventions should be included in the plan of care? *Select all that apply.* 1. Monitoring daily weight 2. Monitoring intake and output 3. Maintaining a low-potassium diet 4. Monitoring extremities for edema 5. Maintaining a low-sodium diet
*1. Monitoring daily weight* *2. Monitoring intake and output* *4. Monitoring extremities for edema* *5. Maintaining a low-sodium diet* *rationale* The client with Cushing's syndrome experiencing fluid overload should be maintained on a high-potassium and low-sodium diet. Decreased sodium intake decreases renal retention of sodium and water. Monitoring weight, intake, output, and extremities for edema are all appropriate interventions for such a nursing diagnosis.
A nurse is caring for a client with a diagnosis of hypoparathyroidism. The nurse reviews the laboratory results drawn on the client and notes that the calcium level is extremely low. The nurse would expect to note which of the following on data collection of the client? 1. Positive Trousseau's sign 2. Negative Chvostek's sign 3. Unresponsive pupils 4. Hyperactive bowel sounds
*1. Positive Trousseau's sign* *rationale* Hypoparathyroidism is related to a lack of parathyroid hormone secretion or to a decreased effectiveness of parathyroid hormone on target tissues. The end result of this disorder is hypocalcemia. When serum calcium levels are critically low, the client may exhibit positive Chvostek's and Trousseau's signs, which indicate potential tetany. Options 2, 3, and 4 are not related to the presence of hypocalcemia.
A health care provider has prescribed propylthiouracil (PTU) for a client with hyperthyroidism, and the nurse assists in developing a plan of care for the client. A priority nursing measure to be included in the plan regarding this medication is to monitor the client for: 1. Signs and symptoms of hypothyroidism 2. Signs and symptoms of hyperglycemia 3. Relief of pain 4. Signs of renal toxicity
*1. Signs and symptoms of hypothyroidism* *rationale* Excessive dosing with propylthiouracil may convert the client from a hyperthyroid state to a hypothyroid state. If this occurs, the dosage should be reduced. Temporary administration of thyroid hormone may be required. Propylthiouracil is not used for pain and does not cause hyperglycemia or renal toxicity.
A nurse is preparing to discharge a client who has had a parathyroidectomy. When teaching the client about the prescribed oral calcium supplement, what information should the nurse include? 1. Take the calcium 30 to 60 minutes following a meal. 2. Avoid sunlight because it can cause skin color change. 3. Store the calcium in the refrigerator to maintain potency. 4. Check the pulse daily and hold the dosage if it is below 60 beats per minute.
*1. Take the calcium 30 to 60 minutes following a meal.* *rationale* Oral calcium supplements can be taken 30 to 60 minutes after meals to enhance their absorption and decrease gastrointestinal irritation. All the other options are unrelated to oral calcium therapy.
An older client with a history of hyperparathyroidism and severe osteoporosis is hospitalized. The nurse caring for the client plans first to address which problem? 1. The possibility of injury 2. Constipation 3. Urinary retention 4. Need for teaching about the disorder
*1. The possibility of injury* *rationale* The client with severe osteoporosis as a result of hyperparathyroidism is at risk for injury as a result of pathological fractures that can occur from bone demineralization. The client may also have a risk for constipation from the disease process but this is a lesser priority than client safety. The client may or may not have urinary elimination problems, depending on other factors in the client's history. There is no information in the question to support whether the client needs teaching.
A nurse is monitoring a client following a thyroidectomy for signs of hypocalcemia. Which of the following signs, if noted in the client, likely indicates the presence of hypocalcemia? 1. Tingling around the mouth 2. Negative Chvostek's sign 3. Flaccid paralysis 4. Bradycardia
*1. Tingling around the mouth* *rationale* Following a thyroidectomy, the nurse assesses the client for signs of hypocalcemia and tetany. Early signs include tingling around the mouth and fingertips, muscle twitching or spasms, palpitations or dysrhythmias, and positive Chvostek's and Trousseau's signs. Options 2, 3, and 4 are not signs of hypocalcemia.
During routine postoperative assessment of a client who has undergone hypophysectomy, the client complains of thirst and frequent urination. Knowing the expected complications of this surgery, the nurse would next check the: 1. Urine specific gravity 2. Serum glucose 3. Respiratory rate 4. Blood pressure
*1. Urine specific gravity* *rationale* Following hypophysectomy, diabetes insipidus can occur temporarily because of antidiuretic hormone (ADH) deficiency. This deficiency is related to surgical manipulation. The nurse should assess specific gravity and notify the registered nurse if the results are less than 1.005. Although options 2, 3, and 4 may be components of the assessment, the nurse would next check urine specific gravity.
A nurse has reinforced dietary instructions to a client with a diagnosis of hypoparathyroidism. The nurse instructs the client to include which of the following items in the diet? 1. Vegetables 2. Meat 3. Fish 4. Cereals
*1. Vegetables* *rationale* The client with hypoparathyroidism is instructed to follow a calcium-rich diet and to restrict the amount of phosphorus in the diet. The client should limit meat, poultry, fish, eggs, cheese, and cereals. Vegetables are allowed in the diet.
A nurse is caring for a client with pheochromocytoma. The client is scheduled for an adrenalectomy. During the preoperative period, the priority nursing action would be to monitor the: 1. Vital signs 2. Intake and output 3. Blood urea nitrogen (BUN) level 4. Urine for glucose and acetone
*1. Vital signs* *rationale* Hypertension is the hallmark of pheochromocytoma. Severe hypertension can precipitate a brain attack (stroke) or sudden blindness. Although all of the options are accurate nursing interventions for the client with pheochromocytoma, the priority nursing action is to monitor the vital signs, particularly the blood pressure.
A nurse is collecting data on a client with hyperparathyroidism. Which of the following questions would elicit the accurate information about this condition from the client? 1. "Do you have tremors in your hands?" 2. "Are you experiencing pain in your joints?" 3. "Have you had problems with diarrhea lately?" 4. "Do you notice swelling in your legs at night?"
*2. "Are you experiencing pain in your joints?"* *rationale* Hyperparathyroidism causes an oversecretion of parathyroid hormone (PTH), which causes excessive osteoblast growth and activity within the bones. When bone reabsorption is increased, calcium is released from the bones into the blood, causing hypercalcemia. The bones suffer demineralization as a result of calcium loss, leading to bone and joint pain, and pathological fractures.
A nursing instructor asks a student to describe the pathophysiology that occurs in Cushing's disease. Which statement by the student indicates an accurate understanding of this disorder? 1. "Cushing's disease is characterized by an oversecretion of insulin." 2. "Cushing's disease is characterized by an oversecretion of glucocorticoid hormones." 3. "Cushing's disease is characterized by an undersecretion of corticotropic hormones." 4. "Cushing's disease is characterized by an undersecretion of glucocorticoid hormones."
*2. "Cushing's disease is characterized by an oversecretion of glucocorticoid hormones."* *rationale* Cushing's syndrome is characterized by an oversecretion of glucocorticoid hormones. Addison's disease is characterized by the failure of the adrenal cortex to produce and secrete adrenocortical hormones. Options 1 and 4 are inaccurate regarding Cushing's syndrome.
A nurse is reinforcing home care instructions to a client with a diagnosis of Cushing's syndrome. Which statement reflects a need for further client education? 1. "Taking my medications exactly as prescribed is essential." 2. "I need to read the labels on any over-the-counter medications I purchase." 3. "My family needs to be familiar with the signs and symptoms of hypoadrenalism." 4. "I could experience the signs and symptoms of hyperadrenalism because of Cushing's."
*2. "I need to read the labels on any over-the-counter medications I purchase."* *rationale* The client with Cushing's syndrome should be instructed to take the medications exactly as prescribed. The nurse should emphasize the importance of continuing medications, consulting with the health care provider before purchasing any over-the-counter medications, and maintaining regular follow-up care. The nurse should also instruct the client in the signs and symptoms of both hypoadrenalism and hyperadrenalism.
A licensed practical nurse (LPN) is assisting a high school nurse in conducting a session with female adolescents regarding the menstrual cycle. The LPN tells the adolescents that the normal duration of the menstrual cycle is about: 1. 14 days 2. 28 days 3. 30 days 4. 45 days
*2. 28 days* *rationale* The normal duration of the menstrual cycle is about 28 days, although it may range from 20 to 45 days. The first day of the menstrual period is counted as day 1 of the woman's cycle. Options 1, 3, and 4 are incorrect.
Which of the following clients is at risk for developing thyrotoxicosis? 1. A client with hypothyroidism 2. A client with Graves' disease who is having surgery 3. A client with diabetes mellitus scheduled for debridement of a foot ulcer 4. A client with diabetes insipidus scheduled for an invasive diagnostic test
*2. A client with Graves' disease who is having surgery* *rationale* Thyrotoxicosis is usually seen in clients with Graves' disease with the symptoms precipitated by a major stressor. This complication typically occurs during periods of severe physiological or psychological stress such as trauma, sepsis, the birth process, or major surgery. It also must be recognized as a potential complication following a thyroidectomy.
A nurse is caring for a client with Addison's disease. The nurse checks the vital signs and determines that the client has orthostatic hypotension. The nurse determines that this finding relates to which of the following? 1. A decrease in cortisol release 2. A decreased secretion of aldosterone 3. An increase in epinephrine secretion 4. Increased levels of androgens
*2. A decreased secretion of aldosterone* *rationale* A decreased secretion of aldosterone results in a limited reabsorption of sodium and water; therefore the client experiences fluid volume deficit. A decrease in cortisol, an increase in epinephrine, and an increase in androgen secretion do not result in orthostatic hypotension.
While collecting data on a client being prepared for an adrenalectomy, the nurse obtains a temperature reading of 100.8° F. The nurse analyzes this temperature reading as: 1. Within normal limits 2. A finding that needs to be reported immediately 3. An expected finding caused by the operative stress response 4. Slightly abnormal but an insignificant finding
*2. A finding that needs to be reported immediately* *rationale* An adrenalectomy is performed because of excess adrenal gland function. Excess cortisol production impairs the immune response, which puts the client at risk for infection. Because of this, the client needs to be protected from infection, and minor variations in normal vital sign values must be reported so that infections are detected early, before they become overwhelming. In addition, the surgeon may elect to postpone surgery in the event of a fever because it can be indicative of infection. Options 1, 3, and 4 are not correct interpretations.
A nurse is reviewing the postoperative prescriptions for a client who had a transsphenoidal hypophysectomy. Which health care provider's prescription, if noted on the record, indicates the need for clarification? 1. Instruct the client about the need for a Medic-Alert bracelet. 2. Apply a loose dressing if any clear drainage is noted. 3. Monitor vital signs and neurological status. 4. Instruct the client to avoid blowing the nose.
*2. Apply a loose dressing if any clear drainage is noted.* *rationale* The nurse should observe for clear nasal drainage, constant swallowing, and a severe, persistent, generalized, or frontal headache. These signs and symptoms indicate cerebrospinal fluid leak into the sinuses. If clear drainage is noted following this procedure, the health care provider needs to be notified immediately. Options 1, 3, and 4 indicate appropriate postoperative interventions.
A client is admitted with a diagnosis of pheochromocytoma. The nurse would monitor which of the following to detect the most common sign of pheochromocytoma? 1. Skin temperature 2. Blood pressure 3. Urine ketones 4. Weight
*2. Blood pressure* *rationale* Hypertension is the major symptom associated with pheochromocytoma and is monitored by taking the client's blood pressure. Glycosuria, weight loss, and diaphoresis are other clinical manifestations of pheochromocytoma; however, hypertension is the most common sign.
A client with a pituitary tumor will undergo transsphenoidal hypophysectomy. The nurse includes which priority item in the preoperative teaching plan for the client? 1. Brushing the teeth vigorously and frequently is important to minimize bacteria in the mouth. 2. Blowing the nose following surgery is prohibited. 3. A small area will be shaved at the base of the neck. 4. It will be necessary to cough and deep breathe following the surgery.
*2. Blowing the nose following surgery is prohibited.* *rationale* The approach used for this surgery is the oronasal route, specifically where the upper lip meets the gum. The surgeon then uses a route through the sphenoid sinus to get to the pituitary gland. The client is not allowed to blow the nose, sneeze, or cough vigorously because these activities could raise intracranial pressure. The client also is not allowed to brush the teeth, to avoid disrupting the surgical site. Alternate methods for performing mouth care are used.
A nurse is caring for a client diagnosed with hyperparathyroidism who is prescribed furosemide (Lasix). The nurse reinforces dietary instructions to the client. Which of the following is an appropriate instruction? 1. Increase dietary intake of calcium. 2. Drink at least 2 to 3 L of fluid daily. 3. Eat sparely when experiencing nausea. 4. Decrease dietary intake of potassium.
*2. Drink at least 2 to 3 L of fluid daily.* *rationale* The aim of treatment in the client with hyperparathyroidism is to increase the renal excretion of calcium and decrease gastrointestinal absorption and bone resorption. This is aided by the sufficient intake of fluids. Dietary restriction of calcium may be used as a component of therapy. The parathyroid is responsible for calcium production, and the term, "hyperparathyroidism" can be indicative of an increase in calcium. The client should eat foods high in potassium, especially if the client is taking furosemide. Limiting nutrients is not advisable.
When caring for a client diagnosed with pheochromocytoma, what information should the nurse know when assisting with planning care? 1. Profound hypotension may occur. 2. Excessive catecholamines are released. 3. The condition is not curable and is treated symptomatically. 4. Hypoglycemia is the primary presenting symptom.
*2. Excessive catecholamines are released.* *rationale* Pheochromocytoma is a catecholamine-producing tumor of the adrenal gland and causes secretion of excessive amounts of epinephrine and norepinephrine. Hypertension is the principal manifestation, and the client has episodes of a high blood pressure accompanied by pounding headaches. The excessive release of catecholamine also results in excessive conversion of glycogen into glucose in the liver. Consequently, hyperglycemia and glucosuria occur during attacks. Pheochromocytoma is curable. The primary treatment is surgical removal of one or both of the adrenal glands, depending on whether the tumor is unilateral or bilateral.
A client scheduled for a thyroidectomy says to the nurse, "I am so scared to get cut in my neck." Based on the client's statement, the nurse determines that the client is experiencing which problem? 1. Inadequate knowledge about the surgical procedure 2. Fear about impending surgery 3. Embarrassment about the changes in personal appearance 4. Lack of support related to the surgical procedure
*2. Fear about impending surgery* *rationale* The client is having a difficult time coping with the scheduled surgery. The client is able to express fears but is scared. No data in the question support options 1, 3, and 4.
A nurse is monitoring a client with Graves' disease for signs of thyrotoxic crisis (thyroid storm). Which of the following signs and symptoms, if noted in the client, will alert the nurse to the presence of this crisis? *Select all that apply.* 1. Bradycardia 2. Fever 3. Sweating 4. Agitation 5. Pallor
*2. Fever* *3. Sweating* *4. Agitation* *rationale* Thyrotoxic crisis (thyroid storm) is an acute, potentially life-threatening state of extreme thyroid activity that represents a breakdown in the body's tolerance to a chronic excess of thyroid hormones. The clinical manifestations include fever greater than 100° F, severe tachycardia, flushing and sweating, and marked agitation and restlessness. Delirium and coma can occur.
A nurse is caring for a client with pheochromocytoma. The client asks for a snack and something warm to drink. The appropriate choice for this client to meet nutritional needs would be which of the following? 1. Crackers with cheese and tea 2. Graham crackers and warm milk 3. Toast with peanut butter and cocoa 4. Vanilla wafers and coffee with cream and sugar
*2. Graham crackers and warm milk* *rationale* The client with pheochromocytoma needs to be provided with a diet that is high in vitamins, minerals, and calories. Of particular importance is that food or beverages that contain caffeine (e.g., chocolate, coffee, tea, and cola) are prohibited.
A nurse is caring for a client after thyroidectomy and monitoring for signs of thyroid storm. The nurse understands that which of the following is a manifestation associated with this disorder? 1. Bradycardia 2. Hypotension 3. Constipation 4. Hypothermia
*2. Hypotension* *rationale* Clinical manifestations associated with thyroid storm include a fever as high as 106° F (41.1° C), severe tachycardia, profuse diarrhea, extreme vasodilation, hypotension, atrial fibrillation, hyperreflexia, abdominal pain, diarrhea, and dehydration. With this disorder, the client's condition can rapidly progress to coma and cardiovascular collapse.
A nurse is caring for a client following an adrenalectomy and is monitoring for signs of adrenal insufficiency. Which of the following, if noted in the client, indicates signs and symptoms related to adrenal insufficiency? *Select all that apply.* 1. Double vision 2. Hypotension 3. Mental status changes 4. Weakness 5. Fever
*2. Hypotension* *3. Mental status changes* *4. Weakness* *5. Fever* *rationale* The nurse should be alert to signs and symptoms of adrenal insufficiency in a client following adrenalectomy. These signs and symptoms include weakness, hypotension, fever, and mental status changes. Double vision is generally not associated with this condition.
Triggers of asthma
- allergens - exposure to cold air - vigourous exercise - air pollutants like cigarette smoke, vehicle exhaust - frequent respiratory infections - history of other respiratory illnesses like allergic rhinitis
A client with Cushing's disease is being admitted to the hospital after a stab wound to the abdomen. The nurse plans care and places highest priority on which potential problem? 1. Nervousness 2. Infection 3. Concern about appearance 4. Inability to care for self
*2. Infection* *rationale* The client with a stab wound has a break in the body's first line of defense against infection. The client with Cushing's disease is at great risk for infection because of excess cortisol secretion and subsequent impaired antibody function and decreased proliferation of lymphocytes. The client may also have a potential for the problems listed in the other options but these are not the highest priority at this time.
A nurse is collecting data on a client admitted to the hospital with a diagnosis of myxedema. Which data collection technique will provide data necessary to support the admitting diagnosis? 1. Auscultation of lung sounds 2. Inspection of facial features 3. Percussion of the thyroid gland 4. Palpation of the adrenal glands
*2. Inspection of facial features* *rationale* Inspection of facial features will reveal the characteristic coarse features, presence of edema around the eyes and face, and a blank expression that are characteristic of myxedema. The techniques in the remaining options will not reveal any data that would support the diagnosis of myxedema.
A nurse is providing discharge instructions to a client who had a unilateral adrenalectomy. Which of the following will be a component of the instructions? 1. The reason for maintaining a diabetic diet 2. Instructions about early signs of a wound infection 3. Teaching regarding proper application of an ostomy pouch 4. The need for lifelong replacement of all adrenal hormones
*2. Instructions about early signs of a wound infection* *rationale* A client who is undergoing a unilateral adrenalectomy will be placed on corticosteroids temporarily to avoid a cortisol deficiency. These medications will be gradually weaned in the postoperative period until they are discontinued. Because of the anti-inflammatory properties of corticosteroids, clients who undergo an adrenalectomy are at increased risk for developing wound infections. Because of this increased risk for infection, it is important for the client to know measures to prevent infection, early signs of infection, and what to do if an infection is present. Options 1, 3, and 4 are incorrect instructions.
A nurse is caring for a postoperative parathyroidectomy client. Which of the following would require the nurse's immediate attention? 1. Incisional pain 2. Laryngeal stridor 3. Difficulty voiding 4. Abdominal cramps
*2. Laryngeal stridor* *rationale* During the postoperative period, the nurse carefully observes the client for signs of hemorrhage, which cause swelling and the compression of adjacent tissue. Laryngeal stridor is a harsh, high-pitched sound heard on inspiration and expiration that is caused by the compression of the trachea and that leads to respiratory distress. It is an acute emergency situation that requires immediate attention to avoid the complete obstruction of the airway.
A nurse is caring for a client with a diagnosis of myasthenia gravis. The health care provider plans to perform an Enlon test on the client to determine the presence of cholinergic crisis. In addition to planning care for the client during this testing, which of the following will the nurse ensure is at the bedside? 1. Cardiac monitor 2. Oxygen equipment 3. Vial of protamine sulfate and a syringe 4. Potassium injection and a liter of normal saline solution
*2. Oxygen equipment* *rationale* An Enlon test is performed to distinguish between myasthenic and cholinergic crisis. Following administration of Enlon, if symptoms intensify, the crisis is cholinergic. Because the symptoms of cholinergic crisis will worsen with the administration of Enlon, atropine sulfate and oxygen should be immediately available whenever Enlon is used.
A nurse is collecting data from a client who is being admitted to the hospital for a diagnostic workup for primary hyperparathyroidism. The nurse understands that which client complaint would be characteristic of this disorder? 1. Diarrhea 2. Polyuria 3. Polyphagia 4. Weight gain
*2. Polyuria* *rationale* Hypercalcemia is the hallmark of hyperparathyroidism. Elevated serum calcium levels produce osmotic diuresis (polyuria). This diuresis leads to dehydration and the client would lose weight. Options 1, 3, and 4 are gastrointestinal (GI) symptoms but are not associated with the common GI symptoms typical of hyperparathyroidism (nausea, vomiting, anorexia, constipation).
The anticipated intended effect of fludrocortisone acetate (Florinef) for the treatment of Addison's disease is to: 1. Stimulate the immune response. 2. Promote electrolyte balance. 3. Stimulate thyroid production. 4. Stimulate thyrotropin production.
*2. Promote electrolyte balance.* *rationale* Florinef is a long-acting oral medication with mineralocorticoid and moderate glucocorticoid activity used for long-term management of Addison's disease. Mineralocorticoids act on the renal distal tubules to enhance the reabsorption of sodium and chloride ions and the excretion of potassium and hydrogen ions. In small doses, fludrocortisone acetate causes sodium retention and increased urinary potassium excretion. The client rapidly can develop hypotension and fluid and electrolyte imbalance if the medication is discontinued abruptly. Options 1, 3, and 4 are not associated with the effects of this medication.
What would the nurse anticipate being included in the plan of care for a client who has been diagnosed with Graves' disease? 1. Provide a high-fiber diet. 2. Provide a restful environment. 3. Provide three small meals per day. 4. Provide the client with extra blankets.
*2. Provide a restful environment.* *rationale* Because of the hypermetabolic state, the client with Graves' disease needs to be provided with an environment that is restful both physically and mentally. Six full meals a day that are well balanced and high in calories are required, because of the accelerated metabolic rate. Foods that increase peristalsis (e.g., high-fiber foods) need to be avoided. These clients suffer from heat intolerance and require a cool environment.
A nurse is caring for a client with hypothyroidism who is overweight. Which food items would the nurse suggest to include in the plan? 1. Peanut butter, avocado, and red meat 2. Skim milk, apples, whole-grain bread, and cereal 3. Organ meat, carrots, and skim milk 4. Seafood, spinach, and cream cheese
*2. Skim milk, apples, whole-grain bread, and cereal* *rationale* Clients with hypothyroidism may have a problem with being over-weight because of their decreased metabolic need. They should consume foods from all food groups, which will provide them with the necessary nutrients; however, the foods should be low in calories. Option 2 is the only option that identifies food items that are low in calories.
Airway remodeling
- results from chronic inflammation of the airways --> structural changes in the bronchial wall - fibrosis, hypertrophy and loss of elasticity can occur - leads to continued inflammation and excessive mucus production
When caring for a client who is having clear drainage from his nares after transsphenoidal hypophysectomy, which action by the nurse is appropriate? 1. Lower the head of the bed. 2. Test the drainage for glucose. 3. Obtain a culture of the drainage. 4. Continue to observe the drainage.
*2. Test the drainage for glucose.* *rationale* After hypophysectomy, the client should be monitored for rhinorrhea, which could indicate a cerebrospinal fluid (CSF) leak. If this occurs, the drainage should be collected and tested for glucose, indicating the presence of CSF. The head of the bed should not be lowered to prevent increased intracranial pressure. Clear nasal drainage would not indicate the need for a culture. Continuing to observe the drainage without taking action could result in a serious complication.
A client who returned to the nursing unit 8 hours ago after hypophysectomy has clear drainage saturating the nasal dressing. The nurse should take which action first? 1. Continue to observe for further drainage. 2. Test the drainage for glucose. 3. Put the head of the bed flat. 4. Test the drainage for occult blood.
*2. Test the drainage for glucose.* *rationale* Following hypophysectomy the client should be monitored for rhinorrhea (clear nasal drainage), which could indicate a cerebrospinal fluid (CSF) leak. If this occurs, the drainage should be collected and tested for the presence of CSF by testing it for glucose. CSF tests positive for glucose, whereas true nasal secretions would not. It is not necessary to test drainage that is clear for occult blood. The head of the bed should not be lowered, to prevent a rise in intracranial pressure. Continuing to observe the drainage without taking action could put the client at risk for developing a serious complication.
A nurse has reinforced instructions to the client with hyperparathyroidism regarding home care measures related to exercise. Which statement by the client indicates a need for further instruction? *Select all that apply.* 1. "I enjoy exercising but I need to be careful." 2. "I need to pace my activities throughout the day." 3. "I need to limit playing football to only the weekends." 4. "I should gauge my activity level by my energy level." 5. "I should exercise in the evening to encourage a good sleep pattern."
*3. "I need to limit playing football to only the weekends."* *5. "I should exercise in the evening to encourage a good sleep pattern."* *rationale* The client should be instructed to avoid high-impact activity or contact sports such as football. Exercising late in the evening may interfere with restful sleep. The client with hyperparathyroidism should pace activities throughout the day and plan for periods of uninterrupted rest. The client should plan for at least 30 minutes of walking each day to support calcium movement into the bones. The client should be instructed to use energy level as a guide to activity.
A client with Cushing's syndrome verbalizes concern to the nurse regarding the appearance of the buffalo hump that has developed. Which response by the nurse is appropriate? 1. "Don't be concerned, this problem can be covered with clothing." 2. "This is permanent, but looks are deceiving and not that important." 3. "Usually, these physical changes slowly improve following treatment." 4. "Try not to worry about it. There are other things to be concerned about."
*3. "Usually, these physical changes slowly improve following treatment."* *rationale* The client with Cushing's syndrome should be reassured that most physical changes resolve with treatment. Options 1, 2, and 4 are not therapeutic responses.
A nurse is caring for a client with pheochromocytoma. Which data would indicate a potential complication associated with this disorder? 1. A urinary output of 50 mL/hr 2. A coagulation time of 5 minutes 3. Congestion heard on auscultation of the lungs 4. A blood urea nitrogen (BUN) level of 20 mg/dL
*3. Congestion heard on auscultation of the lungs* *rationale* The complications associated with pheochromocytoma include hypertensive retinopathy and nephropathy, myocarditis, congestive heart failure (CHF), increased platelet aggregation, and stroke. Death can occur from shock, stroke, renal failure, dysrhythmias, or dissecting aortic aneurysm. Congestion heard on auscultation of the lungs is indicative of CHF. A urinary output of 50 mL/hr is an appropriate output; the nurse would become concerned if the output were less than 30 mL/hr. A coagulation time of 5 minutes is normal. A BUN level of 20 mg/dL is a normal finding.
A maternity nursing instructor asks a nursing student to identify the hormones that are produced by the ovaries. Which of the following, if identified by the student, indicates an understanding of the hormones produced by this endocrine gland? 1. Oxytocin 2. Luteinizing hormone (LH) 3. Estrogen and progesterone 4. Follicle-stimulating hormone (FSH)
*3. Estrogen and progesterone* *rationale* The ovaries are the endocrine glands that produce estrogen and progesterone. Oxytocin is produced by the posterior pituitary gland and stimulates the uterus to produce contractions. LH and FSH are produced by the anterior pituitary gland.
A nurse is preparing to provide instructions to a client with Addison's disease regarding diet therapy. The nurse understands that which of the following diets would likely be prescribed for this client? 1. Low-protein diet 2. Low-sodium diet 3. High-sodium diet 4. Low-carbohydrate diet
*3. High-sodium diet* *rationale* A high-sodium, high-complex carbohydrate, and high-protein diet will be prescribed for the client with Addison's disease. To prevent excess fluid and sodium loss, the client is instructed to maintain an adequate salt intake of up to 8 g of sodium daily and to increase salt intake during hot weather, before strenuous exercise, and in response to fever, vomiting, or diarrhea.
A client is diagnosed with hyperparathyroidism. The nurse teaching the client about dietary alterations to manage the disorder tells the client to limit which of the following foods in the diet? 1. Bananas 2. Oatmeal 3. Ice cream 4. Chicken breast
*3. Ice cream* *rationale* The client with hyperparathyroidism is likely to have elevated calcium levels. This client should reduce intake of dairy products such as milk, cheese, ice cream, or yogurt. Apples, bananas, chicken, oatmeal, and pasta are low-calcium foods.
Guillain-Barre Syndrome - what is it, and how can this cause pulmonary obstruction?
- autoimmune disorder where the immune system attacks the myelin sheaths of peripheral nerves - causes loss of nervous function to the arms and legs - eventually works its way to the upper body and lungs, cutting off the innervation that produces respirations - respirations cease --> alveoli collapse and air exchange stops
Cystic Fibrosis
- autosomal recessive disease that alters the transport of sodium and chloride in and out of epithelial cells
A1-Antitrypsin deficiency
- autosomal recessive disorder that affects the lungs and liver - deficiency of A-1 antitrypsin means that lung tissue will be destroyed during inflammatory processes - results in destruction of alveolar walls
A nurse would expect to note which interventions in the plan of care for a client with hypothyroidism? *Select all that apply.* 1. Provide a cool environment for the client. 2. Instruct the client to consume a high-fat diet. 3. Instruct the client about thyroid replacement therapy. 4. Encourage the client to consume fluids and high-fiber foods in the diet. 5. Instruct the client to contact the health care provider if episodes of chest pain occur. 6. Inform the client that iodine preparations will be prescribed to treat the disorder.
*3. Instruct the client about thyroid replacement therapy.* *4. Encourage the client to consume fluids and high-fiber foods in the diet.* *5. Instruct the client to contact the health care provider if episodes of chest pain occur.* *rationale* The clinical manifestations of hypothyroidism are the result of decreased metabolism from low levels of thyroid hormone. Interventions are aimed at replacement of the hormones and providing measures to support the signs and symptoms related to a decreased metabolism. The nurse encourages the client to consume a well-balanced diet that is low in fat for weight reduction and high in fluids and high-fiber foods to prevent constipation. The client often has cold intolerance and requires a warm environment. The client would notify the health care provider if chest pain occurs since it could be an indication of overreplacement of thyroid hormone. Iodine preparations are used to treat hyperthyroidism. These medications decrease blood flow through the thyroid gland and reduce the production and release of thyroid hormone.
A client with newly diagnosed Cushing's syndrome expresses concern about personal appearance, specifically about the "buffalo hump" that has developed at the base of the neck. When counseling the client about this manifestation, the nurse should incorporate the knowledge that: 1. This is a permanent feature. 2. It can be minimized by wearing tight clothing. 3. It may slowly improve with treatment of the disorder. 4. It will quickly disappear once medication therapy is started.
*3. It may slowly improve with treatment of the disorder.* *rationale* The client with Cushing's syndrome should be reassured that most physical changes resolve over time with treatment. The other options are incorrect.
A client has been diagnosed with hypoparathyroidism. The nurse teaches the client to include foods in the diet that are: 1. High in phosphorus and low in calcium 2. Low in phosphorus and low in calcium 3. Low in phosphorus and high in calcium 4. High in phosphorus and high in calcium
*3. Low in phosphorus and high in calcium* *rationale* Hypoparathyroidism results in hypocalcemia. A therapeutic diet for this disorder is one that is high in calcium but low in phosphorus because these two electrolytes have inverse proportions in the body. All of the other options are unrelated to this disorder and are incorrect.
A nurse reviews a plan of care for a postoperative client following a thyroidectomy and notes that the client is at risk for breathing difficulty. Which of the following nursing interventions will the nurse suggest to include in the plan of care? 1. Maintain a supine position. 2. Encourage coughing and deep breathing exercises. 3. Monitor neck circumference frequently. 4. Maintain a pressure dressing on the operative site.
*3. Monitor neck circumference frequently.* *rationale* Following a thyroidectomy, the client should be placed in an upright position to facilitate air exchange. The nurse should assist the client with deep breathing exercises, but coughing is minimized to prevent tissue damage and stress to the incision. A pressure dressing is not placed on the operative site because it could affect breathing. The nurse should monitor the dressing closely and should loosen the dressing if necessary. Neck circumference is monitored at least every 4 hours to assess for postoperative edema.
Which nursing action would be appropriate to implement when a client has a diagnosis of pheochromocytoma? 1. Weigh the client. 2. Test the client's urine for glucose. 3. Monitor the client's blood pressure. 4. Palpate the client's skin to determine warmth.
*3. Monitor the client's blood pressure.* *rationale* Hypertension is the major symptom that is associated with pheochromocytoma. The blood pressure status is monitored by taking the client's blood pressure. Glycosuria, weight loss, and diaphoresis are also clinical manifestations of pheochromocytoma, but hypertension is the major symptom.
A nurse assists in developing a plan of care for a client with hyperparathyroidism receiving calcitonin-human (Cibacalcin). Which outcome has the highest priority regarding this medication? 1. Relief of pain 2. Absence of side effects 3. Reaching normal serum calcium levels 4. Verbalization of appropriate medication knowledge
*3. Reaching normal serum calcium levels* *rationale* Hypercalcemia can occur in clients with hyperparathyroidism, and calcitonin is used to lower plasma calcium level. The highest priority outcome in this client situation would be a reduction in serum calcium level. Option 1 is unrelated to this medication. Although options 2 and 4 are expected outcomes, they are not the highest priority for administering this medication.
A preoperative client is scheduled for adrenalectomy to remove a pheochromocytoma. The nurse would most closely monitor which of the following items in the preoperative period? 1. Intake and output 2. Blood urea nitrogen (BUN) 3. Vital signs 4. Urine glucose and ketones
*3. Vital signs* 4. Urine glucose and ketones *rationale* Hypertension is the hallmark of pheochromocytoma. Severe hypertension can precipitate a stroke or sudden blindness. Although all the items are appropriate nursing assessments for the client with pheochromocytoma, the priority is to monitor the vital signs, especially the blood pressure.
A health care provider prescribes a 24-hour urine collection for vanillylmandelic acid (VMA). The nurse instructs the client in the procedure for the collection of the urine. Which statement by the client would indicate a need for further instruction? 1. "I will start the collection in 2 days. I cannot eat or drink any tea, chocolate, vanilla, or fruit until the test is completed." 2. "When I start the collection, I will urinate and discard that specimen." 3. "I will pour the urine into the collection bottle each time I urinate and refrigerate the urine." 4. "I can take any medications if I need to before the collection."
*4. "I can take any medications if I need to before the collection."* *rationale* Because a 24-hour urine collection is a timed quantitative determination, it is essential that the client start the test with an empty bladder. Therefore the client is instructed to void and discard the first urine and note the time and start the test. The 24-hour urine specimen collection bottle must be kept on ice or refrigerated. In a VMA collection, the client is instructed to avoid tea, chocolate, vanilla, and all fruits for 2 days before urine collection begins. Also clients are reminded not to take certain medications for 2 to 3 days before the test.
What is the primary cause of COPD?
- cigarette smoke
A client with myxedema has changes in intellectual function such as impaired memory, decreased attention span, and lethargy. The client's husband is upset and shares his concerns with the nurse. Which statement by the nurse is helpful to the client's husband? 1. "Would you like me to ask the health care provider for a prescription for a stimulant?" 2. "Give it time. I've seen dozens of clients with this problem that fully recover." 3. "I don't blame you for being frustrated, because the symptoms will only get worse." 4. "It's obvious that you are concerned about your wife's condition, but the symptoms may improve with continued therapy."
*4. "It's obvious that you are concerned about your wife's condition, but the symptoms may improve with continued therapy."* *rationale* Using therapeutic communication techniques, the nurse acknowledges the husband's concerns and conveys that the client's symptoms are common with myxedema. With thyroid hormone therapy, these symptoms should decrease, and cognitive function often returns to normal. Option 1 is not helpful, and it blocks further communication. Option 3 is pessimistic and untrue. Option 2 is not appropriate and offers false reassurance.
A client with pheochromocytoma is scheduled for surgery and says to the nurse, "I'm not sure that surgery is the best thing to do." What response by the nurse is appropriate? 1. "I think you are making the right decision to have the surgery." 2. "You are very ill. Your health care provider has made the correct decision." 3. "There is no reason to worry. Your health care provider is a wonderful surgeon." 4. "You have concerns about the surgical treatment for your condition."
*4. "You have concerns about the surgical treatment for your condition."* *rationale* Paraphrasing is restating the client's message in the nurse's own words. Option 4 addresses the therapeutic communication technique of paraphrasing. The client is reaching out for understanding. In option 3, the nurse is offering a false reassurance, and this type of response will block communication. Option 2 also represents a communication block because it reflects a lack of the client's right to an opinion. In option 1, the nurse is expressing approval, which can be harmful to a nurse-client relationship.
The nurse caring for a client who has had a subtotal thyroidectomy reviews the plan of care and determines which problem is the priority for this client in the immediate postoperative period? 1. Dehydration 2. Infection 3. Urinary retention 4. Bleeding
*4. Bleeding* *rationale* Hemorrhage is one of the most severe complications that can occur following thyroidectomy. The nurse must frequently check the neck dressing for bleeding and monitor vital signs to detect early signs of hemorrhage, which could lead to shock. T3 and T4 do not regulate fluid volumes in the body. Infection is a concern for any postoperative client but is not the priority in the immediate postoperative period. Urinary retention can occur in postoperative clients as a result of medication and anesthesia but is not the priority from the options provided.
A nurse working on an endocrine nursing unit understands that which correct concept is used in planning care? 1. Clients with Cushing's syndrome are likely to experience episodic hypotension. 2. Clients with hyperthyroidism must be monitored for weight gain. 3. Clients who have diabetes insipidus should be assessed for fluid excess. 4. Clients who have hyperparathyroidism should be protected against falls.
*4. Clients who have hyperparathyroidism should be protected against falls.* *rationale* Hyperparathyroidism is a disease that involves excess secretion of parathyroid hormone (PTH). Elevation of PTH causes excess calcium to be removed from the bones. There is a decline in bone mass, which may cause a fracture if a fall occurs. Cushing's syndrome is likely to cause hypertension. Clients with hypothyroidism must be monitored for weight gain and clients with hyperthyroidism must be monitored for weight loss. Clients who have diabetes insipidus should be assessed for fluid deficit.
A nursing student notes in the medical record that a client with Cushing's syndrome is experiencing body image disturbances. The need for additional education regarding this problem is identified when the nursing student suggests which nursing intervention? 1. Encouraging the client's expression of feelings 2. Evaluating the client's understanding of the disease process 3. Encouraging family members to share their feelings about the disease process 4. Evaluating the client's understanding that the body changes need to be dealt with
*4. Evaluating the client's understanding that the body changes need to be dealt with* *rationale* Evaluating the client's understanding that the body changes that occur in this disorder need to be dealt with is an inappropriate nursing intervention. This option does not address the client's feelings. Options 1, 2, and 3 are appropriate because they address the client and family feelings regarding the disorder.
Which clinical manifestation should the nurse expect to note when assessing a client with Addison's disease? 1. Edema 2. Obesity 3. Hirsutism 4. Hypotension
*4. Hypotension* *rationale* Common manifestations of Addison's disease include postural hypotension from fluid loss, syncope, muscle weakness, anorexia, nausea, vomiting, abdominal cramps, weight loss, depression, and irritability. The manifestations in options 1, 2, and 3 are not associated with Addison's disease.
A nurse is caring for a client following a thyroidectomy. The client tells the nurse that she is concerned because of voice hoarseness. The client asks the nurse whether the hoarseness will subside. The nurse appropriately tells the client that the hoarseness: 1. Indicates nerve damage 2. Is harmless but permanent 3. Will worsen before it subsides 4. Is normal and will gradually subside
*4. Is normal and will gradually subside* *rationale* Hoarseness that develops in the postoperative period is usually the result of laryngeal pressure or edema and will resolve within a few days. The client should be reassured that the effects are transitory. Options 1, 2, and 3 are incorrect.
Which nursing measure would be effective in preventing complications in a client with Addison's disease? 1. Restricting fluid intake 2. Offering foods high in potassium 3. Checking family support systems 4. Monitoring the blood glucose
*4. Monitoring the blood glucose* *rationale* The decrease in cortisol secretion that characterizes Addison's disease can result in hypoglycemia. Therefore monitoring the blood glucose would detect the presence of hypoglycemia so that it can be treated early to prevent complications. Fluid intake should be encouraged to compensate for dehydration. Potassium intake should be restricted because of hyperkalemia. Option 3 is not a priority for this client.
Tripod position
- compensatory position that people with COPD assume to breathe more effectively - sit upright with arms supported in a fixed structure like an overbed table
List some drug classes used for long-term control of asthma
- corticosteroids: fluticasone, prednisone - leukotriene modifiers: montelukast - anti IgE: omalizumab
A client with Graves' disease has exophthalmos and is experiencing photophobia. Which intervention would best assist the client with this problem? 1. Administering methimazole (Tapazole) every 8 hours 2. Lubricating the eyes with tap water every 2 to 4 hours 3. Instructing the client to avoid straining or heavy lifting 4. Obtaining dark glasses for the client
*4. Obtaining dark glasses for the client* *rationale* Because photophobia (light intolerance) accompanies this disorder, dark glasses are helpful in alleviating the symptom. Medical therapy for Graves' disease does not help alleviate the clinical manifestation of exophthalmos. Other interventions may be used to relieve the drying that occurs from not being able to completely close the eyes; however, the question is asking what the nurse can do for photophobia. Tap water, which is hypotonic, could actually cause more swelling to the eye because it could pull fluid into the interstitial space. In addition, the client is at risk for developing an eye infection because the solution is not sterile. There is no need to prevent straining with exophthalmos.
A nurse is caring for a client experiencing thyroid storm. Which of the following would be a priority concern for this client? 1. Inability to cope with the treatment plan 2. Lack of sexual drive 3. Self-consciousness about body appearance 4. Potential for cardiac disturbances
*4. Potential for cardiac disturbances* *rationale* Clients in thyroid storm are experiencing a life-threatening event, which is associated with uncontrolled hyperthyroidism. It is characterized by high fever, severe tachycardia, delirium, dehydration, and extreme irritability. The signs and symptoms of the disorder develop quickly, and therefore emergency measures must be taken to prevent death. These measures include maintaining hemodynamic status and patency of airway as well as providing adequate ventilation. Options 1, 2, and 3 are not a priority in the care of the client in thyroid storm.
A nurse is collecting data regarding a client after a thyroidectomy and notes that the client has developed hoarseness and a weak voice. Which nursing action is appropriate? 1. Check for signs of bleeding. 2. Administer calcium gluconate. 3. Notify the registered nurse immediately. 4. Reassure the client that this is usually a temporary condition.
*4. Reassure the client that this is usually a temporary condition.* *rationale* Weakness and hoarseness of the voice can occur as a result of trauma of the laryngeal nerve. If this develops, the client should be reassured that the problem will subside in a few days. Unnecessary talking should be discouraged. It is not necessary to notify the registered nurse immediately. These signs do not indicate bleeding or the need to administer calcium gluconate.
A nurse is caring for a postoperative adrenalectomy client. Which of the following does the nurse specifically monitor for in this client? 1. Peripheral edema 2. Bilateral exophthalmos 3. Signs and symptoms of hypocalcemia 4. Signs and symptoms of hypovolemia
*4. Signs and symptoms of hypovolemia* *rationale* Following adrenalectomy, the client is at risk for hypovolemia. Aldosterone, secreted by the adrenal cortex, plays a major role in fluid volume balance by retaining sodium and water. A deficiency of adrenocortical hormones does not cause the clinical manifestations noted in options 1, 2, and 3.
bronchoscopy
*TEST* *A nurse is caring for several patients who are scheduled for diagnostic testing for respiratory disorders. The patient who needs postprocedural care that includes frequent vital signs is the patient who had: * A. capnography B. D-Dimer test C. a ventilation and perfusion scan D. bronchoscopy
Albuterol
*TEST* *A patient with asthma is suddenly experiencing difficulty breathing, tachypnea, and wheezing. Which medication listed on the medication administration record, administered through an inhaler, should the nurse administer to this patient?* A.Albuterol B. Cromolyn C. Salmeterol D. Formoterol
Negative
*TEST* *The nurse is reading the results of a tuberculin skin test on a client with no documented health problems. The site has no induration and a 1-mm area of ecchymosis. Which interpretation should the nurse make of these results? * A. Positive B. Negative C. Uncertain D. Borderline
Do not suction the patient for more than 10 to 15 seconds
*TEST* *The nurse is suctioning a patient who is unable to expectorate respiratory secretions from his tracheotomy. How can the nurse avoid the serious consequences of removing oxygen when suctioning this patient?* A. Keep suction pressure between 110 and 120 mm Hg. B. Avoid giving the patient oxygen just before suctioning. C. Apply suction as the catheter is advanced into the trachea. D. Do not suction the patient for more than 10 to 15 seconds.
Have the patient apply direct pressure by pinching his nose for 10 to 15 minutes
*TEST* A patient's nose begins to bleed. Which action should the LPN/LVN take?* A. Tell the patient to keep swallowing. B. Apply warm compresses to the nose and face. C. Encourage the patient to blow his nose at frequent intervals. D. Have the patient apply direct pressure by pinching his nose for 10 to 15 minutes
Bronchodilators used for COPD
- B2 adrenergic agonist like albuterol - anticholinergic agents - methylxanthines
Absorption atelectasis
- aalveoli are usually full of nitrogen after oxygen has been absorbed into the capillaries - keeps alveoli open and prevent them from collapsing - high flow O2 pushes all the nitrogen out and fills alveoli with only O2 - in the event of airway obstruction, all of the O2 will be absorbed and there will be nothing left to keep the alveoli open --> collapse of airway
Emphysema
- abnormal, permanent enlargement of the alveoli - involves destruction of alveolar walls without obvious fibrosis
Blebs
- air spaces adjacent to pleurae - blister-like pockets that form on the surface of the lung - makes the outside of the lung look lumpy
Give an example of a short acting bronchodilator(SABA)
- albuterol
Describe the pathophysiology of an asthma attack caused by allergies
- allergen lands on the smooth muscle of the bronchi or bonchioles - cell responds to antigen by signalling immune cells (IgE and mast cells) - nearby mast cells degranulate and release inflammatory mediators into the nearby tissues - causes vasodilation and increased capillary permeability to respond to perceived threat = swelling and edema of airways - more immune cells respond and infiltrate into smooth muscle --> bronchospams, edema, mucus secretion, thickening of airway - bronchi become hyperresponsive - airway obstructs
Role of leukotriene modifiers in asthma management
- drugs either block leukotriene receptors or inhibit leukotriene synthesis - stops leukotrienes from causing inflammation, bronchoconstriction and airway edema - not used for acute asthma attacks
Examples of leukotriene modifiers
- drugs ending in -lukast - montelukast, zafirlukast - zileuton
List a medication that can be given in an emergency where a person has gone into anaphylactic shock and cannot breathe
- epinephrine IM
Anastamoses
- formation of connections between blood vessels - occurs in bronchiectasis and other lung illnesses where perfusion to the airways is impaired - results in hemoptysis as these new connections are fragile
What causes bronchiectasis?
- in children, typically cystic fibrosis - in adults, bacterial infections of the lungs that aren't treated or receive delayed treatment
Bullae
- large air pockets within the lung tissue - makes the grape shape of alveoli look like balloons
How do ABGs affect pulmonary circulation?
- low PaO2 = causes constriction of blood vessels, remodeling of vascular tissue, increased red blood cell synthesis --> thicker, more viscous blood - low pH and high CO2 = cause even more constriction -
What organs does cystic fibrosis affect?
- lungs - GI tract (pancreas and biliary tract) - reproductive system
Effect of CF in the pancreas
- mucus plugs the exocrine glands - causes atrophy and cyst formation within the pancreas - exocrine function is lost --> lack of lipase, amylase, trypsin etc - fat malabsorption and inability to digest food --> weight loss and malnutrition
How does the CFTR protein affect people with CF?
- mutated gene causes the protein to be created in a way that blocks the ion channel - epithelial cells create secretions that are low in sodium chloride (meaning that they are also low in water content) - results in failure to reabsorb chloride and retention of sodium - mucus becomes thick and sticky and plugs up the glands in affected organs - glands atrophy and result in organ failure
Inflammatory mediators in COPD
- neutrophils - macrophages - lymphocytes - all of these invite more inflammatory mediators to the lungs, causing structural changes
Can gas exchange occur in blebs and bullae?
- no - don't contain the capillary bed that normally surrounds alveoli
Oxygen toxicity
- occurs after prolonged exposure to high concentrations of O2 - oxygen radicals and damage to alveolar capillary membranes can cause a severe inflammatory response - results in severe pulmonary edema, shunting of blood, and hypoxemia - results in ARDS
Example of an anti-IgE drug
- omalizumab
Simple face mask uses and limitations
- only used for short periods (causes tight seal around nose and mouth) - can tolerate 6-12 L O2
ABGs late in an asthma attack
- pH = decreased (more acidic) - PaO2 = significantly decreased - PaCO2 = increased significantly
ABGs early in an asthma attack
- pH = increased (more basic) - PaO2 = decreased - PaCO2 = decreased
ABGs in late COPD
- pH = low normal - PaO2 = low normal - PaCO2 = high normal/ high - HCO3- = high normal (to compensate for respiratory acidosis)
During asthma, which part of the autonomic nervous system is responsible for airway obstruction?
- parasympathetic NS - overactive PSNS causes increased release of acetylcholine
At the beginning of an asthma attack, is the patient in acidosis or alkalosis?
- patient experiences respiratory alkalosis - during asthma attack, the patient hyperventilates and blows off CO2
Why does the chest become barrel shaped in a person with COPD?
- peripheral airways become obstructed and air gets progressively trapped during expiration - as residual air volume increases, some alveoli begin to detach from the bronchioles and become dead space - loss of elastic recoil - chest hyperexpands to try and push out the residual air
Bronchiectasis
- permanent dilation of the bronchi caused by chronic infection and inflammation - destroys elastic and muscular structures that support the bronchial wall
Signs and symptoms of bronchiectasis
- persistent cough with consistent production of tenacious, purulent sputum - hemoptysis - pleuritic chest pain - dyspnea - wheezing - clubbing of fingernails - weight loss - anemia - variety of adventitious breath sounds on auscultation
Examples of corticosteroids used for asthma
- prednisone - methylprednisolone - fluticasone - beclomethasone - budesonide
Chronic bronchitis
- presence of a chronic productive cough for 3 months in a year for 2 consecutive years
Role of anti IgE drugs in asthma
- prevent IgE from binding to mast cells, stopping the release of chemical mediators - reserved for very severe asthma that hasn't responded to inhaled corticosteroids
What is COPD? List the two classifications
- progressive inflammation and damage to the airways 1. Chronic bronchitis 2. Emphysema
Pursed-lip breathing is done to:
- prolong expiration and promote CO2 clearance
Central cyanosis - what causes it, give signs and symptoms
- prolonged hypoxia - indicates serious heart and lung issues, as they are unable to supply oxygenated blood to even vital organs - will see bluish mottling of the lips, oral mucosa and tongue - chest may appear pale and feel cool
Role of CFTR protein
- protein found in the epithelial cells of the airways, GI tract, ducts in the liver, pancreas and sweat glands - regulates sodium and chloride channels
Role of corticosteroids in asthma management
- reduce chronic inflammation - reduce hyperresponsiveness of bronchi - block the late phase response of asthma (secondary attack that occurs 4-6 hours after exposure to trigger) -
Cor pulmonale
- right sides heart failure secondary to pulmonary hypertension that occurs in COPD - increased pressure within pulmonary circulation --> backup and increased pressure within right ventricle - right ventricle hypertrophies to compensate for increased resistance - eventually decompensates and begins to fail
Describe the uses and limitations of nasal cannulas for oxygen delivery
- safe and simple, allows more freedom of movement - patient can eat, talk and cough while wearing device - not effective for >6L O2
Role of A1-Antitrypsin
- serum protein produced by the liver, found in the lungs - protects normal lung tissue from being broken down by proteases during inflammation caused by cigarette smoking and infections
Rescue inhalers use what class of medication?
- short acting beta 2 agonists (SABA) - often referred to as bronchodilators - stimulate the B adrenergic receptor in bronchioles, causing bronchodilation
What happens to the pulmonary vascular system in COPD?
- small pulmonary arteries constrict from hypoxia - structure begins to change -->thickening of vascular smooth muscle - loss of alveolar walls and surrounding capillaries --> increased pressure in pulmonary circulation - all of this results in pulmonary hypertension
What is the most important treatment for people with COPD?
- smoking cessation
Describe the effect of CF in the lungs
- starts as a disease of the smaller airways --> larger airways --> destruction of the lungs - thick, tenacious mucus + decreased cilia motility = bronchioles become obstructed with mucus - causes air trapping and hyperinflation of lungs
Diagnosis of CF
- sweat chloride test - any value greater than 60 mmol/L are considered positive
Partial and Non Rebreather Mask uses and limitations
- useful for short term therapy (24hrs) - has a reservoir bag that allows patient to rebreathe 1/3 of exhaled air which is still rich in oxygen and easier to I hale than flowing O2 - O2 rate must be enough to keep bag from collapsing during inspiration
How long does a patient need to be on corticosteroids before they begin to take effect?
- usually 1 to 2 weeks before full therapeutic effect is achieved
Describe what happens during bronchiectasis
- walls of the bronchial system get damaged through an infection - mucociliary mechanism is damaged --> bacteria and mucus can accumulate in the injured pockets - decreased ability to clear mucus as it builds up and thickens - results in decreased expiratory airflow and lung obstruction
Signs and symptoms of an asthma attack
- wheezing - cough - dyspnea - chest tightness - thick mucus secretions - anxiety - patient sitting upright or slightly forward to compensate
A client has just been admitted with a diagnosis of myxedema coma. If all of the following interventions were prescribed, the nurse would place highest priority on completing which of the following first? 1. Administering oxygen 2. Administering thyroid hormone 3. Warming the client 4. Giving fluid replacement
1. Administering oxygen As part of maintaining a patent airway, oxygen would be administered first. This would be quickly followed by fluid replacement, keeping the client warm, monitoring vital signs, and administering thyroid hormones.
A nurse is collecting data on a client with a diagnosis of hypothyroidism. Which of these behaviors, if present in the client's history, would the nurse determine as being likely related to the manifestations of this disorder? 1. Depression 2. Nervousness 3. Irritability 4. Anxiety
1. Depression Hypothyroid clients experience a slow metabolic rate, and its manifestation includes apathy, fatigue, sleepiness, and depression. Options 2, 3, and 4 identify the clinical manifestations of hyperthyroidism.
A client with Addison's disease asks the nurse how a newly prescribed medication, fludrocortisone acetate (Florinef), will improve the condition. When formulating a response, the nurse should incorporate that a key action of this medication is to: 1. Help restore electrolyte balance. 2. Make the body produce more cortisol. 3. Replace insufficient circulating estrogens. 4. Alter the body's immune system functioning.
1. Help restore electrolyte balance. Fludrocortisone acetate is a long-acting oral medication with mineralocorticoid and moderate glucocorticoid activity. It is prescribed for the long-term management of Addison's disease. Mineralocorticoids cause renal reabsorption of sodium and chloride ions and the excretion of potassium and hydrogen ions. These actions help restore electrolyte balance in the body. The other options are incorrect.
In planning nutrition for the client with hypoparathyroidism, which diet would be appropriate? 1. High in calcium and low phosphorous 2. Low in vitamins A, D, E, and K 3. High in sodium with no fluid restriction 4. Low in water and insoluble fiber
1. High in calcium and low phosphorous Hypocalcemia is the end result of hypoparathyroidism resulting from either a lack of parathyroid hormone (PTH) secretion or ineffective PTH influence on tissue. Calcium is the major controlling factor of PTH secretion. Because of this, the diet needs to be high in calcium but low in phosphorus because these two electrolytes must exist in inverse proportions in the body. The other options are not dietary
A client with hypoparathyroidism has hypocalcemia. The nurse avoids giving the client the prescribed vitamin and calcium supplement with which of the following liquids? 1. Milk 2. Water 3. Iced tea 4. Fruit juice
1. Milk rationale Milk products are high in phosphates, which should be avoided by a client with hypoparathyroidism. Otherwise, calcium products are best absorbed with milk because the vitamin D in the milk promotes calcium absorption.
A nurse is monitoring the results of periodic serum laboratory studies drawn on a client with diabetic ketoacidosis (DKA) receiving an insulin infusion. The nurse determines that which of the following values needs to be reported? 1. Potassium 3.1 mEq/L 2. Calcium 9.2 mg/dL 3. Sodium 137 mEq/L 4. Serum osmolality 288 mOsm/kg H2O
1. Potassium 3.1 mEq/L The client with DKA initially becomes hyperkalemic as potassium leaves the cells in response to lowered pH. Once fluid replacement and insulin therapy are started, the potassium level drops quickly. This occurs because potassium is carried into the cells along with glucose and insulin and because potassium is excreted in the urine once rehydration has occurred. Thus the nurse carefully monitors the results of serum potassium levels and reports hypokalemia (option 1) promptly. The other laboratory values are within the normal ranges.
A nurse is preparing to discharge a client who has had a parathyroidectomy. When teaching the client about the prescribed oral calcium supplement, what information should the nurse include? 1. Take the calcium 30 to 60 minutes following a meal. 2. Avoid sunlight because it can cause skin color change. 3. Store the calcium in the refrigerator to maintain potency. 4. Check the pulse daily and hold the dosage if it is below 60 beats per
1. Take the calcium 30 to 60 minutes following a meal. Oral calcium supplements can be taken 30 to 60 minutes after meals to enhance their absorption and decrease gastrointestinal irritation. All the other options are unrelated to oral calcium therapy.
An older client with a history of hyperparathyroidism and severe osteoporosis is hospitalized. The nurse caring for the client plans first to address which problem? 1. The possibility of injury 2. Constipation 3. Urinary retention 4. Need for teaching about the disorder
1. The possibility of injury The client with severe osteoporosis as a result of hyperparathyroidism is at risk for injury as a result of pathological fractures that can occur from bone demineralization. The client may also have a risk for constipation from the disease process but this is a lesser priority than client safety. The client may or may not have urinary elimination problems, depending on other factors in the client's history. There is no information in the question to support whether the client needs teaching.
During routine postoperative assessment of a client who has undergone hypophysectomy, the client complains of thirst and frequent urination. Knowing the expected complications of this surgery, the nurse would next check the: 1. Urine specific gravity 2. Serum glucose 3. Respiratory rate 4. Blood pressure
1. Urine specific gravity Following hypophysectomy, diabetes insipidus can occur temporarily because of antidiuretic hormone (ADH) deficiency. This deficiency is related to surgical manipulation. The nurse should assess specific gravity and notify the registered nurse if the results are less than 1.005. Although options 2, 3, and 4 may be components of the assessment, the nurse would next check urine specific gravity.
A client with diabetes mellitus is scheduled to have a fasting blood glucose level determined in the morning. The nurse tells the client not to eat or drink after midnight. When the client asks for further information, the nurse clarifies by stating that which of the following would be acceptable to take before the test? 1. Water 2. Coffee without any milk 3. Tea without any sugar 4. Clear liquids such as apple juice
1. Water When a client is scheduled for a fasting blood glucose level, the client should not eat or drink anything except water after midnight. This is needed to ensure accurate test results, which form the basis for adjustments or continuance of treatment. Options 2, 3, and 4 are inaccurate, and the client should not consume these items before the test.
The nurse is caring for a client with emphysema receiving oxygen. The nurse should check the oxygen flow rate to ensure the client does not exceed how many L/min of oxygen?
2
A nurse has just supervised a newly diagnosed diabetes mellitus client self-inject NPH insulin at 7:30 ᴀᴍ. The nurse reviews the time frames for peak insulin action with the client, telling the client to be especially watchful for a hypoglycemic reaction between: 1. 7:30 ᴀᴍ and 9:30 ᴀᴍ 2. 1:30 ᴘᴍ and 7:30 ᴘᴍ 3. 8:30 ᴘᴍ and 12:00 ᴀᴍ 4. 2:30 ᴀᴍ and 4:30 ᴀᴍ
2. 1:30 ᴘᴍ and 7:30 ᴘᴍ NPH is an intermediate-acting insulin. It begins to work in 1 to 2 hours (onset), peaks in 6 to 12 hours, and lasts for 18 to 24 hours (duration). Hypoglycemic reactions most likely occur during peak time, which in this case is option 2.
A nurse is caring for a client with Addison's disease. The nurse checks the vital signs and determines that the client has orthostatic hypotension. The nurse determines that this finding relates to which of the following? 1. A decrease in cortisol release 2. A decreased secretion of aldosterone 3. An increase in epinephrine secretion 4. Increased levels of androgens
2. A decreased secretion of aldosterone A decreased secretion of aldosterone results in a limited reabsorption of sodium and water; therefore the client experiences fluid volume deficit. A decrease in cortisol, an increase in epinephrine, and an increase in androgen secretion do not result in orthostatic hypotension.
.While collecting data on a client being prepared for an adrenalectomy, the nurse obtains a temperature reading of 100.8° F. The nurse analyzes this temperature reading as: 1. Within normal limits 2. A finding that needs to be reported immediately 3. An expected finding caused by the operative stress response 4. Slightly abnormal but an insignificant finding
2. A finding that needs to be reported immediately An adrenalectomy is performed because of excess adrenal gland function. Excess cortisol production impairs the immune response, which puts the client at risk for infection. Because of this, the client needs to be protected from infection, and minor variations in normal vital sign values must be reported so that infections are detected early, before they become overwhelming. In addition, the surgeon may elect to postpone surgery in the event of a fever because it can be indicative of infection. Options 1, 3, and 4 are not correct interpretations.
A client with a pituitary tumor will undergo transsphenoidal hypophysectomy. The nurse includes which priority item in the preoperative teaching plan for the client? 1. Brushing the teeth vigorously and frequently is important to minimize bacteria in the mouth. 2. Blowing the nose following surgery is prohibited. 3. A small area will be shaved at the base of the neck. 4. It will be necessary to cough and deep breathe following the surgery.
2. Blowing the nose following surgery is prohibited. The approach used for this surgery is the oronasal route, specifically where the upper lip meets the gum. The surgeon then uses a route through the sphenoid sinus to get to the pituitary gland. The client is not allowed to blow the nose, sneeze, or cough vigorously because these activities could raise intracranial pressure. The client also is not allowed to brush the teeth, to avoid disrupting the surgical site. Alternate methods for performing mouth care are used.
A client with diabetes mellitus demonstrates acute anxiety when admitted to the hospital for the treatment of hyperglycemia. The appropriate intervention to decrease the client's anxiety would be to: 1. Administer a sedative. 2. Convey empathy, trust, and respect toward the client. 3. Ignore the signs and symptoms of anxiety so that they will soon disappear. 4. Make sure the client knows all the correct medical terms so that he or she can understand what is happening.
2. Convey empathy, trust, and respect toward the client. The appropriate intervention is to address the client's feelings related to the anxiety and to convey empathy, trust, and respect toward the client. Administering a sedative is not the most appropriate intervention. The nurse should not ignore the client's anxious feelings. A client will not relate to medical terms, particularly when anxiety exists.
A client scheduled for a thyroidectomy says to the nurse, "I am so scared to get cut in my neck." Based on the client's statement, the nurse determines that the client is experiencing which problem? 1. Inadequate knowledge about the surgical procedure 2. Fear about impending surgery 3. Embarrassment about the changes in personal appearance 4. Lack of support related to the surgical procedure
2. Fear about impending surgery The client is having a difficult time coping with the scheduled surgery. The client is able to express fears but is scared. No data in the question support options 1, 3, and 4.
A client newly diagnosed with diabetes mellitus takes NPH insulin every day at 7:00 ᴀᴍ. The nurse has taught the client how to recognize the signs of hypoglycemia. The nurse determines that the client understands the information presented if the client watches for which of the following signs in the late afternoon? 1. Nausea and vomiting, and abdominal pain 2. Hunger; shakiness; and cool, clammy skin 3. Drowsiness; red, dry skin; and fruity breath odor 4. Increased urination; thirst; and rapid, deep breathing
2. Hunger; shakiness; and cool, clammy skin The client taking NPH insulin obtains peak medication effects approximately 6 to 12 hours after administration. At the time that the medication peaks, the client is at risk of hypoglycemia if food intake is insufficient. The nurse should teach the client to watch for signs and symptoms of hypoglycemia including anxiety, confusion, difficulty concentrating, blurred vision, cold sweating, headache, increased pulse, shakiness, and hunger. The other options list various signs and symptoms of hyperglycemia.
A nurse is collecting data from a client who is being admitted to the hospital for a diagnostic workup for primary hyperparathyroidism. The nurse understands that which client complaint would be characteristic of this disorder? 1. Diarrhea 2. Polyuria 3. Polyphagia 4. Weight gain
2. Polyuria Hypercalcemia is the hallmark of hyperparathyroidism. Elevated serum calcium levels produce osmotic diuresis (polyuria). This diuresis leads to dehydration and the client would lose weight. Options 1, 3, and 4 are gastrointestinal (GI) symptoms but are not associated with the common GI symptoms typical of hyperparathyroidism (nausea, vomiting, anorexia, constipation).
The anticipated intended effect of fludrocortisone acetate (Florinef) for the treatment of Addison's disease is to: 1. Stimulate the immune response. 2. Promote electrolyte balance. 3. Stimulate thyroid production. 4. Stimulate thyrotropin production.
2. Promote electrolyte balance Florinef is a long-acting oral medication with mineralocorticoid and moderate glucocorticoid activity used for long-term management of Addison's disease. Mineralocorticoids act on the renal distal tubules to enhance the reabsorption of sodium and chloride ions and the excretion of potassium and hydrogen ions. In small doses, fludrocortisone acetate causes sodium retention and increased urinary potassium excretion. The client rapidly can develop hypotension and fluid and electrolyte imbalance if the medication is discontinued abruptly. Options 1, 3, and 4 are not associated with the effects of this medication.
36. RN observes a nursing assistant (NA) caring for a patient after a hypophysectomy. Which action by the NA requires that the RN intervene? a. The NA lowers the HOB to the flat position. b. The NA cautions the patient to avoid coughing. c. The NA cleans the patient's mouth with a swab. d. NA collects a urine specimen for specific gravity
A R: HOB should be elevated about 30 degrees to decrease pressure on the sella turcica and avoid headaches. The other actions by the NA are appropriate after this surgery. (Cognitive Level: Application Text : p. 1293 NProcess: Implementation NCLEX: Safe and Effective Care Enviro)
A nurse is caring for a client with hypothyroidism who is overweight. Which food items would the nurse suggest to include in the plan? 1. Peanut butter, avocado, and red meat 2. Skim milk, apples, whole-grain bread, and cereal 3. Organ meat, carrots, and skim milk 4. Seafood, spinach, and cream cheese
2. Skim milk, apples, whole-grain bread, and cereal Clients with hypothyroidism may have a problem with being over-weight because of their decreased metabolic need. They should consume foods from all food groups, which will provide them with the necessary nutrients; however, the foods should be low in calories. Option 2 is the only option that identifies food items that are low in calories.
A client who returned to the nursing unit 8 hours ago after hypophysectomy has clear drainage saturating the nasal dressing. The nurse should take which action first? 1. Continue to observe for further drainage. 2. Test the drainage for glucose. 3. Put the head of the bed flat. 4. Test the drainage for occult blood.
2. Test the drainage for glucose. Following hypophysectomy the client should be monitored for rhinorrhea (clear nasal drainage), which could indicate a cerebrospinal fluid (CSF) leak. If this occurs, the drainage should be collected and tested for the presence of CSF by testing it for glucose. CSF tests positive for glucose, whereas true nasal secretions would not. It is not necessary to test drainage that is clear for occult blood. The head of the bed should not be lowered, to prevent a rise in intracranial pressure. Continuing to observe the drainage without taking action could put the client at risk for developing a serious complication.
Glucagon hydrochloride injection would most likely be prescribed for which disorder? 1. Thyroid crisis 2. Type 1 diabetes mellitus 3. Hypoadrenalism 4. Excess growth hormone secretion
2. Type 1 diabetes mellitus Glucagon hydrochloride is a medication that can be administered subcutaneously or intramuscularly. It is prescribed to stimulate the liver to release glucose when a client is experiencing hypoglycemia and unable to take oral glucose replacement. It is important to teach a person other than the client how to administer the medication because the client's symptoms may prevent self-injection. Therefore options 1, 3, and 4 are incorrect.
What is a therapeutic INR?
2.0-3.0
A client with Cushing's syndrome verbalizes concern to the nurse regarding the appearance of the buffalo hump that has developed. Which response by the nurse is appropriate? 1. "Don't be concerned, this problem can be covered with clothing." 2. "This is permanent, but looks are deceiving and not that important." 3. "Usually, these physical changes slowly improve following treatment." 4. "Try not to worry about it. There are other things to be concerned about."
3. "Usually, these physical changes slowly improve following treatment." The client with Cushing's syndrome should be reassured that most physical changes resolve with treatment. Options 1, 2, and 4 are not therapeutic responses.
The wife of a client with diabetes mellitus who takes insulin calls the nurse in a health care provider's office about her husband. She states that her husband is sleepy and that his skin is warm and flushed. She adds that his breathing is faster than normal and his pulse rate seems fast. Which of the following should the nurse tell the woman to do first? 1. Call an ambulance. 2. Take his temperature. 3. Check his blood glucose level. 4. Drive him to the health care provider's office.
3. Check his blood glucose The client's signs and symptoms are consistent with hyperglycemia. The wife should first obtain a blood glucose reading, which the nurse would then report to the health care provider. Option 1 or 4 may be done at a later time if required. Option 2 is unrelated to the client's immediate problem.
A client has a blood glucose level drawn for suspected hyperglycemia. After interviewing the client, the nurse determines that the client ate lunch approximately 2 hours before the blood specimen was drawn. The laboratory reports that the blood glucose to be 180 mg/dL, and the nurse analyzes this result to be: 1. Normal 2. Lower than the normal value 3. Elevated from the normal value 4. A dangerously high value requiring immediate health care provider notification
3. Elevated from the normal value Normal fasting blood glucose values range from 70 to 120 mg/dL. A 2-hour postprandial blood glucose level should be less than 140 mg/dL. In this situation, the blood glucose value was 180 mg/dL 2 hours after the client ate, which is an elevated value as compared to normal. Although the result may be reported to the health care provider, it is not a dangerously high one.
A client is brought to the emergency department with suspected diabetic ketoacidosis (DKA). Which of the following findings would the nurse note as being consistent with this diagnosis? 1. High serum glucose level and an increase in pH 2. Low serum potassium and high serum bicarbonate level 3. High serum glucose level and low serum bicarbonate level 4. Decreased urine output and Kussmaul's respirations
3. High serum glucose level and low serum bicarbonate level In DKA the blood glucose level is higher than 250 mg/dL, and ketones are present in the blood and urine. The arterial pH is low, less than 7.35. The plasma bicarbonate is also low. The client would exhibit polyuria and Kussmaul's respirations. The potassium level usually is elevated as a result of dehydration.
A client is diagnosed with hyperparathyroidism. The nurse teaching the client about dietary alterations to manage the disorder tells the client to limit which of the following foods in the diet? 1. Bananas 2. Oatmeal 3. Ice cream 4. Chicken breast
3. Ice cream The client with hyperparathyroidism is likely to have elevated calcium levels. This client should reduce intake of dairy products such as milk, cheese, ice cream, or yogurt. Apples, bananas, chicken, oatmeal, and pasta are low-calcium foods.
A client with newly diagnosed Cushing's syndrome expresses concern about personal appearance, specifically about the "buffalo hump" that has developed at the base of the neck. When counseling the client about this manifestation, the nurse should incorporate the knowledge that: 1. This is a permanent feature. 2. It can be minimized by wearing tight clothing. 3. It may slowly improve with treatment of the disorder. 4. It will quickly disappear once medication therapy is started.
3. It may slowly improve with treatment of the disorder. The client with Cushing's syndrome should be reassured that most physical changes resolve over time with treatment. The other options are incorrect.
A male client recently diagnosed with diabetes mellitus requiring insulin tells the clinic nurse that he is traveling by air throughout the next week. The client asks the nurse for any suggestions about managing the disorder while traveling. The nurse tells the client to: 1. Obtain referrals to health care providers in the destination cities. 2. Check the blood glucose every 2 hours during the flight. 3. Keep snacks in carry-on luggage to prevent hypoglycemia during the flight. 4. Pad the insulin and syringes against breakage and place in a suitcase to be stowed.
3. Keep snacks in carry-on luggage to prevent hypoglycemia during the flight. A frequent concern of diabetics during air travel is the availability of food at times that correspond with the timing and peak action of the client's insulin. For this reason, the nurse may suggest that the client have carbohydrate snacks on hand. Insulin equipment and supplies should always be placed in carry-on luggage (not stowed). This provides ready access to treat hyperglycemia, if needed, and prevents loss of equipment if luggage is lost. Options 1 and 2 are unnecessary.
A client has been diagnosed with hypoparathyroidism. The nurse teaches the client to include foods in the diet that are: 1. High in phosphorus and low in calcium 2. Low in phosphorus and low in calcium 3. Low in phosphorus and high in calcium 4. High in phosphorus and high in calcium
3. Low in phosphorus and high in calcium Hypoparathyroidism results in hypocalcemia. A therapeutic diet for this disorder is one that is high in calcium but low in phosphorus because these two electrolytes have inverse proportions in the body. All of the other options are unrelated to this disorder and are incorrect.
A nurse is discussing foot care with a diabetic client and spouse. The nurse includes which of the following during this informational session? 1. There is decreased risk of infection when feet are soaked in hot water. 2. Lanolin should be applied to dry feet, especially the heels and between the toes. 3. The toenails should be cut straight across. 4. Strong soap should be used to decrease skin bacteria.
3. The toenails should be cut straight across. The client should be instructed to cut the toenails straight across. The client should not soak the feet in hot water, to prevent burns. The client should be instructed to wash the feet daily using a mild soap. Moisturizing lotion can be applied to the feet but should not be placed between the toes.
O2 needs to be humidified when delivering more than :
35 to 50% O2
A client with myxedema has changes in intellectual function such as impaired memory, decreased attention span, and lethargy. The client's husband is upset and shares his concerns with the nurse. Which statement by the nurse is helpful to the client's husband? 1. "Would you like me to ask the health care provider for a prescription for a stimulant?" 2. "Give it time. I've seen dozens of clients with this problem that fully recover." 3. "I don't blame you for being frustrated, because the symptoms will only get worse." 4. "It's obvious that you are concerned about your wife's condition, but the symptoms may improve with continued therapy."
4. "It's obvious that you are concerned about your wife's condition, but the symptoms may improve with continued therapy." rationale Using therapeutic communication techniques, the nurse acknowledges the husband's concerns and conveys that the client's symptoms are common with myxedema. With thyroid hormone therapy, these symptoms should decrease, and cognitive function often returns to normal. Option 1 is not helpful, and it blocks further communication. Option 3 is pessimistic and untrue. Option 2 is not appropriate and offers false reassurance.
A nurse is collecting data from a client with type 2 diabetes mellitus. Which statement by the client indicates an understanding of the medication regimen? 1. "I should take my metformin (Glucophage) only if my blood glucose is elevated." 2. "By taking these medications, I am able to eat more." 3. "When I become ill, I need to increase the number of pills I take." 4. "The medication that I am taking helps release the insulin I already make."
4. "The medication that I am taking helps release the insulin I already make." rationale Clients with type 2 diabetes mellitus have decreased or impaired insulin secretion. Oral hypoglycemic agents are given to these clients to facilitate glucose use and need to be taken on a regular schedule as prescribed. To maintain normal blood glucose levels throughout the day, oral hypoglycemic agents such as metformin are not taken on an as-needed basis depending on the blood glucose levels. Insulin injections may be given during times of stress-induced hyperglycemia. Oral insulin is not available or effective because of the breakdown of the insulin by digestion.
3. Following a transsphenoidal resection of a pituitary tumor, an important N assessment is a. monitoring hourly urine output. b. checking the dressings for serous drainage. c. palpating for dependent pitting edema. d. obtaining continuous pulse oximetry.
A R: After pituitary surgery, the pt is at risk for diabetes insipidus caused by cerebral edema and monitoring of urine output and urine specific gravity is essential. There will be no dressing when transsphenoidal approach is used. The pt is at risk for dehydration, not volume overload. The pt is not at high risk for problems with oxygenation, and continuous pulse oximetry is not needed.
A hospitalized client is newly diagnosed with diabetes mellitus. The client must take both NPH and Regular insulin for glucose control. The nurse develops a teaching plan to help the client meet which outcome as a first step in managing the disease? 1. Avoid all strenuous exercise. 2. Maintain health at an optimum level. 3. Lose 40 pounds to achieve ideal body weight. 4. Adjust insulin according to capillary blood glucose levels.
4. Adjust insulin according to capillary blood glucose levels. rationale There are many learning goals for the client who is newly diagnosed with diabetes mellitus. The client must learn dietary control, medication management, and proper exercise in order to control the disease. As a first step, the client learns to adjust medication (insulin) according to blood glucose results as prescribed by the health care provider. The client should then focus on long-term dietary control and weight loss, which will often lead to a decreased need for insulin. At the same time that diet is being controlled, the client should begin a regular exercise program to aid in weight loss.
A nurse participating in a free health screening at the local mall obtains a random blood glucose level of 200 mg/dL on an otherwise healthy client. The nurse tells the client to do which of the following as a next step? 1. Seek treatment for diabetes mellitus. 2. Ask the pharmacist about starting insulin therapy. 3. Begin blood glucose monitoring three times a day. 4. Call the health care provider to have the value rechecked as soon as possible.
4. Call the health care provider to have the value rechecked as soon as possible. rationale Adult diabetes mellitus can be diagnosed either by symptoms (polydipsia, polyuria, polyphagia) or by laboratory values. Diabetes is also diagnosed by an abnormal glucose tolerance test, when random plasma glucose levels are greater than 200 mg/dL, or fasting plasma glucose levels are greater than 140 mg/dL on two separate occasions. Further confirmation of this result is needed to ensure appropriate diagnosis and therapy.
A nurse working on an endocrine nursing unit understands that which correct concept is used in planning care? 1. Clients with Cushing's syndrome are likely to experience episodic hypotension. 2. Clients with hyperthyroidism must be monitored for weight gain. 3. Clients who have diabetes insipidus should be assessed for fluid excess. 4. Clients who have hyperparathyroidism should be protected against falls.
4. Clients who have hyperparathyroidism should be protected against falls. rationale Hyperparathyroidism is a disease that involves excess secretion of parathyroid hormone (PTH). Elevation of PTH causes excess calcium to be removed from the bones. There is a decline in bone mass, which may cause a fracture if a fall occurs. Cushing's syndrome is likely to cause hypertension. Clients with hypothyroidism must be monitored for weight gain and clients with hyperthyroidism must be monitored for weight loss. Clients who have diabetes insipidus should be assessed for fluid deficit.
A client newly diagnosed with diabetes mellitus is having difficulty learning the technique of blood glucose measurement. The nurse should teach the client to do which of the following to perform the procedure properly? 1. Wash the hands first using cold water. 2. Puncture the center of the finger pad. 3. Puncture the finger as deeply as possible. 4. Let the arm hang dependently and milk the digit.
4. Let the arm hang dependently and milk the digit. rationale Before doing a fingerstick for blood glucose measurement, the client should first wash the hands. Warm water should be used to stimulate the circulation to the area. The finger is punctured near the side, not the center, because there are fewer nerve endings along the side of the finger. The puncture is only deep enough to obtain an adequately sized drop of blood; excessively deep punctures may lead to pain and bruising. The arm should be allowed to hang dependently, and the finger may be milked to promote obtaining a good-size blood drop.
A nurse is planning to instruct a client with diabetes mellitus who has hypertension about "sick day management." Which of the following does the nurse avoid putting on a list of easily consumed carbohydrate-containing beverages for use when the client cannot tolerate food orally? 1. Cola 2. Ginger ale 3. Apple juice 4. Mineral water
4. Mineral water Diabetic clients should take in approximately 15 g of carbohydrate every 1 to 2 hours when unable to tolerate food because of illness. Each of the beverages listed in options 1, 2, and 3 provides approximately 13 to 15 g of carbohydrate in a half-cup serving. Mineral water is incorrect for two reasons. First, it contains sodium and should not be used by the client with hypertension. Second, it is not a source of carbohydrates.
A nurse is caring for a client with type 1 diabetes mellitus who is hyperglycemic. Which problem would the nurse consider first, when planning care for this client? 1. The need for knowledge about the diagnosis 2. Insomnia 3. Lack of appetite 4. Signs of dehydration
4. Signs of dehydration rationale Hyperglycemia can develop into ketoacidosis in the client with type 1 diabetes mellitus. Polyuria develops as the body attempts to get rid of the excess glucose, and the client will lose large amounts of fluid. Because glucose is hyperosmotic, fluid is pulled from the tissue. Nausea and vomiting can occur as a result of hyperglycemia and can lead to a loss of sodium and water. Water also is lost from the lungs in an attempt to get rid of excess carbon dioxide. The severe dehydration that occurs can lead to hypovolemic shock. Of the problems listed, dehydration is considered first.
A nurse reviews the nursing care plan of an older client with diabetic neuropathy of the lower extremities as a result of type 2 diabetes mellitus. The nurse plans care, knowing that which problem has the highest priority for this client? 1. Pain as a result of intermittent claudication 2. Lack of self-confidence as a result of impaired ability to walk 3. Lack of self-esteem as a result of perceived loss of abilities 4. The possibility of injury as a result of decreased sensation in the legs and feet
4. The possibility of injury as a result of decreased sensation in the legs and feet rationale The client with diabetic neuropathy of the lower extremities has diminished ability to feel sensations in the legs and feet. This client is at risk for tissue injury and for falls as a result of this nervous system impairment. Thus the highest priority problem is option 4, which can be determined using Maslow's Hierarchy of Needs theory. Options 2 and 3 represent problems that are more psychosocial in nature, and as such are secondary needs using Maslow's theory. Option 1 is incorrect because intermittent claudication is not directly associated with diabetic neuropathy.
increase calories, proteins, vitamins and minerals
A patient with emphysema may lose weight despite having an adequate caloric intake. What advice should the nurse give the patient regarding ways to maintain an optimal weight? A. Increase activity level to stimulate appetite. B. Increase calories, protein, vitamins, and minerals. C. Continue the same caloric intake but increase fat intake. D. increase amounts of complex carbohydrates and decrease fats.
A 63-year-old patient is newly diagnosed with type 2 diabetes. When developing an education plan, the nurse's first action should be to a. assess the patient's perception of what it means to have type 2 diabetes. b. demonstrate how to check glucose using capillary blood glucose monitoring. c. ask the patient's family to participate in the diabetes education program. d. discuss the need for the patient to actively participate in diabetes management.
A Rationale: Before planning education, the nurse should assess the patient's interest in and ability to self-manage the diabetes. After assessing the patient, the other nursing actions may be appropriate, but planning needs to be individualized to each patient.
A patient with type 2 diabetes that is controlled with diet and metformin (Glucophage) also has severe rheumatoid arthritis (RA). During an acute exacerbation of the patient's arthritis, the health care provider prescribes prednisone (Deltasone) to control inflammation. The nurse will anticipate that the patient may a. require administration of insulin while taking prednisone. b. develop acute hypoglycemia during the RA exacerbation. c. have rashes caused by metformin-prednisone interactions. d. need a diet higher in calories while receiving prednisone.
A Rationale: Glucose levels increase when patients are taking CORTICOsteroids, and insulin may be required to control blood glucose. Hypoglycemia is not a complication of RA exacerbation or prednisone use. Rashes are not an adverse effect caused by taking metformin and prednisone simultaneously. The patient is likely to have an increased appetite when taking prednisone, but it will be important to avoid weight gain for the patient with RA.
Intramuscular glucagon is administered to an unresponsive patient for treatment of hypoglycemia. Which action should the nurse take after the patient regains consciousness? a. Give the patient a snack of cheese and crackers. b. Have the patient drink a glass of orange juice or nonfat milk. c. Administer a continuous infusion of 5% dextrose for 24 hours. d. Assess the patient for symptoms of hyperglycemia.
A Rationale: Rebound hypoglycemia can occur after glucagon administration, but having a meal containing complex carbohydrates plus protein and fat will help prevent hypoglycemia. Orange juice and nonfat milk will elevate blood sugar rapidly, but the cheese and crackers will stabilize blood sugar. Administration of glucose intravenously might be used in patients who were unable to take in nutrition orally. The patient should be assessed for symptoms of hypoglycemia after glucagon administration.
Cardiac monitoring is initiated for a patient in diabetic ketoacidosis (DKA). The nurse recognizes that this measure is important to identify a. electrocardiographic (ECG) changes and dysrhythmias related to hypokalemia. b. fluid overload resulting from aggressive fluid replacement. c. the presence of hypovolemic shock related to osmotic diuresis. d. cardiovascular collapse resulting from the effects of hyperglycemia.
A Rationale: The hypokalemia associated with metabolic acidosis can lead to potentially fatal dysrhythmias such as ventricular tachycardia and ventricular fibrillation, which would be detected with ECG monitoring. Fluid overload, hypovolemia, and cardiovascular collapse are possible complications of DKA, but cardiac monitoring would not detect theses.
A patient with type 1 diabetes has been using self-monitoring of blood glucose (SMBG) as part of diabetes management. During evaluation of the patient's technique of SMBG, the nurse identifies a need for additional teaching when the patient a. chooses a puncture site in the center of the finger pad. b. washes the puncture site using soap and water. c. says the result of 130 mg indicates good blood sugar control. d. hangs the arm down for a minute before puncturing the site.
A Rationale: The patient is taught to choose a puncture site at the side of the finger pad. The other patient actions indicate that teaching has been effective.
A patient with cancer of the liver has severe ascites, and the health care provider plans a paracentesis to relieve the fluid pressure on the diaphragm. To prepare the patient for the procedure, the nurse a. asks the patient to empty the bladder. b. positions the patient on the right side. c. obtains informed consent for the procedure. d. assists the patient to lie flat in bed.
A Rationale: The patient should empty the bladder to decrease the risk of bladder perforation during the procedure. The patient would be positioned in Fowler's position and would not be able to lie flat without compromising breathing. The health care provider is responsible for obtaining informed consent.
While hospitalized and recovering from an episode of diabetic ketoacidosis, the patient calls the nurse and reports feeling anxious, nervous, and sweaty. Based on the patient's report, the nurse should a. obtain a glucose reading using a finger stick. b. administer 1 mg glucagon subcutaneously. c. have the patient eat a candy bar. d. have the patient drink 4 ounces of orange juice.
A Rationale: The patient's clinical manifestations are consistent with hypoglycemia and the initial action should be to check the patient's glucose with a finger stick or order a stat blood glucose. If the glucose is low, the patient should ingest a rapid-acting carbohydrate, such as orange juice. Glucagon might be given if the patient's symptoms become worse or if the patient is unconscious. Candy bars contain fat, which would slow down the absorption of sugar and delay the response to treatment.
When assessing the patient experiencing the onset of symptoms of type 1 diabetes, which question should the nurse ask? a. "Have you lost any weight lately?" b. "Do you crave fluids containing sugar?" c. "How long have you felt anorexic?" d. "Is your urine unusually dark-colored?"
A Rationale: Weight loss occurs because the body is no longer able to absorb glucose and starts to break down protein and fat for energy. The patient is thirsty but does not necessarily crave sugar- containing fluids. Increased appetite is a classic symptom of type 1 diabetes. With the classic symptom of polyuria, urine will be very dilute.
A client has recently been diagnosed with Type I diabetes and asks the nurse for help formulating a nutrition plan. Which of the following recommendations would the nurse make to help the client increase calorie consumption to offset absorption problems? a. Eating small meals with two or three snacks may be more helpful in maintaining blood glucose levels than three large meals. b. Eat small meals with two or three snacks throughout the day to keep blood glucose levels steady c. Increase consumption of simple carbohydrates d. Skip meals to help lose weight
A Eating small meals with two or three snacks may be more helpful in maintaining blood glucose levels than three large meals.
How does dry suction work?
If a dry suction system is used, water is not used to regulate the pressure and therefore has no bubbling. A dial is used to set the desired negative pressure. Once again, increasing the vacuum wall suction will not increase the pressure.
9. A patient with symptoms of DI is admitted to the hospital for evaluation and treatment of the condition. An appropriate nursing diagnosis for the patient is a. insomnia related to waking at night to void. b. risk for impaired skin integrity related to generalized edema. c. excess fluid volume related to intake greater than output. d. activity intolerance related to muscle cramps and weakness.
A R: Nocturia occurs as a result of the polyuria caused by diabetes insipidus. Edema will not be expected because dehydration is a concern with polyuria. The pt drinks large amnts of fluid to compensate for losses experienced from diuresis. The pt's fluid and electrolyte status remain normal as long as the patient's oral intake can keep up w fluid losses, muscle cramps and weakness arent concerns.
24. After neck surgery, a patient develops hypoparathyroidism. The nurse should plan to teach the patient about a. calcium supplementation to normalize serum calcium levels. b. including whole grains in the diet to prevent constipation. c. use of bisphosphonates to reduce bone demineralization. d. having a high fluid intake to decrease risk for nephrolithiasis.
A R: Oral calcium supplements are used to maintain the serum calcium in normal range and prevent the complications of hypocalcemia. Whole-grain foods decrease calcium absorption and will not be recommended. Bisphosphonates will lower serum calcium level further by preventing calcium from being reabsorbed from bone. Kidney stones are not a complication of hypoparathyroidism and low calcium levels.
18. A patient with hyperthyroidism is treated with radioactive iodine (RAI) at a clinic. Before the patient is discharged, the nurse instructs the pt a. to monitor for symptoms of hypothyroidism, such as easy bruising and cold intolerance. b. to discontinue the antithyroid medications taken before the radioactive therapy. c. that symptoms of hyperthyroidism should be relieved in about a week. d. about radioactive precautions to take with urine, stool, and other body secretions.
A R: There is a high incidence of post-radiation hypothyroidism after RAI, and the pt should be monitored for symptoms of hypothyroidism. RAI has a delayed response, with maximum effect not seen for 2-3 months, and pt will continue to take antithyroid medications during this time. The therapeutic dose of radioactive iodine is low enough that no radiation safety precautions are needed.
8. A patient is hospitalized with possible SIADH. The patient is confused and reports a headache, muscle cramps, and twitching. The nurse would expect the initial laboratory results to include a a. serum sodium of 125 mEq/L (125 mmol/L). b. hematocrit of 52%. c. blood urea nitrogen (BUN) of 22 mg/dl (11.5 mmol/L). d. serum chloride of 110 mEq/L (110 mmol/L).
A R: When water is retained, the serum sodium level will drop below normal, causing the CMs reported by the patient. The hematocrit will decrease because of the dilution caused by water retention. The BUN is not helpful in diagnosis of SIADH and this BUN value is increased. The serum chloride level will usually decrease along with the sodium level. This chloride value is elevated.
8. To evaluate both oxygenation and ventilation in a patient with acute respiratory failure, the nurse uses the findings revealed with a. arterial blood gas (ABG) analysis. b. hemodynamic monitoring. c. chest x-rays. d. pulse oximetry.
A Rationale: ABG analysis is useful because it provides information about both oxygenation and ventilation and assists with determining possible etiologies and appropriate treatment. The other tests may also provide useful information about patient status but will not indicate whether the patient has hypoxemia, hypercapnia, or both. Cognitive Level: Comprehension Text Reference: p. 1805 Nursing Process: Assessment NCLEX: Physiological Integrity
19. Which information obtained by the nurse when assessing a patient with acute respiratory distress syndrome (ARDS) who is being treated with mechanical ventilation and high levels of positive end-expiratory pressure (PEEP) indicates a complication of ventilator therapy is occurring? a. The patient has subcutaneous emphysema. b. The patient has a sinus bradycardia, rate 52. c. The patient's PaO2 is 50 mm Hg and the SaO2 is 88%. d. The patient has bronchial breath sounds in both the lung fields.
A Rationale: Complications of positive-pressure ventilation (PPV) and PEEP include subcutaneous emphysema. Bradycardia, hypoxemia, and bronchial breath sounds are all concerns, but they are not caused by PPV and PEEP. Cognitive Level: Application Text Reference: p. 1816 Nursing Process: Assessment NCLEX: Physiological Integrity
A patient in the outpatient clinic has positive serologic testing for anti-HCV. Which action by the nurse is appropriate? a. Schedule the patient for HCV genotype testing. b. Teach the patient that the HCV will resolve in 2 to 4 months. c. Administer immune globulin and the HCV vaccine. d. Instruct the patient on self-administration of -interferon.
A Rationale: Genotyping of HCV has an important role in managing treatment and is done before drug therapy with -interferon or other medications is started. HCV has a high percentage of conversion to the chronic state so the nurse should not teach the patient that the HCV will resolve in 2 to 4 months. Immune globulin or vaccine is not available for HCV.
2. The nurse will monitor for clinical manifestations of hypercapnia when a patient in the emergency department has a. chest trauma and multiple rib fractures. b. carbon monoxide poisoning after a house fire. c. left-sided ventricular failure and acute pulmonary edema. d. tachypnea and acute respiratory distress syndrome (ARDS).
A Rationale: Hypercapnia is caused by poor ventilatory effort, which occurs in chest trauma when rib fractures (or flail chest) decrease lung ventilation. Carbon monoxide poisoning, acute pulmonary edema, and ARDS are more commonly associated with hypoxemia. Cognitive Level: Application Text Reference: p. 1800 Nursing Process: Assessment NCLEX: Physiological Integrity
14. When admitting a patient in possible respiratory failure with a high PaCO2, which assessment information will be of most concern to the nurse? a. The patient is somnolent. b. The patient's SpO2 is 90%. c. The patient complains of weakness. d. The patient's blood pressure is 162/94.
A Rationale: Increasing somnolence will decrease the patient's respiratory rate and further increase the PaCO2 and respiratory failure. Rapid action is needed to prevent respiratory arrest. An SpO2 of 90%, weakness, and elevated blood pressure all require ongoing monitoring but are not indicators of possible impending respiratory arrest. Cognitive Level: Application Text Reference: p. 1804 Nursing Process: Assessment NCLEX: Physiological Integrity
How does the suction control chamber work?
If the client needs suction to remove air and fluid, this chamber controls the amount of pressure applied. Sterile water is placed in this chamber up to the 20 cm line. This is the usual prescribed amount. Turn on the wall vacuum suction until you have slow gentle continuous bubbling.
After the home health nurse has taught a patient and family about how to use glargine and regular insulin safely, which action by the patient indicates that the teaching has been successful? a. The patient disposes of the open insulin vials after 4 weeks. b. The patient draws up the regular insulin in the syringe and then draws up the glargine. c. The patient stores extra vials of both types of insulin in the freezer until needed. d. The patient's family prefills the syringes weekly and stores them in the refrigerator.
A Rationale: Insulin can be stored at room temperature for 4 weeks. Glargine should not be mixed with other insulins or prefilled and stored. Freezing alters the insulin molecule and should not be done.
The nurse identifies the collaborative problem of potential complication: electrolyte imbalance for a patient with severe acute pancreatitis. Assessment findings that alert the nurse to electrolyte imbalances associated with acute pancreatitis include a. muscle twitching and finger numbness. b. paralytic ileus and abdominal distention. c. hypotension. d. hyperglycemia.
A Rationale: Muscle twitching and finger numbness indicate hypocalcemia, a potential complication of acute pancreatitis. The other data indicate other complications of acute pancreatitis but are not indicators of electrolyte imbalance.
A patient using a split mixed-dose insulin regimen asks the nurse about the use of intensive insulin therapy to achieve tighter glucose control. The nurse should teach the patient that a. intensive insulin therapy requires three or more injections a day in addition to an injection of a basal long-acting insulin. b. intensive insulin therapy is indicated only for newly diagnosed type 1 diabetics who have never experienced ketoacidosis. c. studies have shown that intensive insulin therapy is most effective in preventing the macrovascular complications characteristic of type 2 diabetes. d. an insulin pump provides the best glucose control and requires about the same amount of attention as intensive insulin therapy.
A Rationale: Patients using intensive insulin therapy must check their glucose level four to six times daily and administer insulin accordingly. A previous episode of ketoacidosis is not a contraindication for intensive insulin therapy. Intensive insulin therapy is not confined to type 2 diabetics and would prevent microvascular changes as well as macrovascular changes. Intensive insulin therapy and an insulin pump are comparable in glucose control.
7. When teaching a patient with chronic SIADH about long-term management of the disorder, the nurse determines that additional instruction is needed when the patient says, a. "I need to shop for foods that are low in sodium and avoid adding salt to foods." b. "I should weigh myself daily and report any sudden weight loss or gain." c. "I need to limit my fluid intake to no more than 1 quart of liquids a day." d. "I will eat foods high in potassium because the diuretics cause potassium loss."
A Rationale: Pts with SIADH are at risk for hyponatremia, and a sodium supplement may be prescribed. The other pt statements are correct and indicate successful teaching has occurred.
22. A patient with acute respiratory distress syndrome (ARDS) has progressed to the fibrotic phase. The patient's family members are anxious about the patient's condition and are continuously present at the hospital. In addressing the family's concerns, it is important for the nurse to a. support the family and help them understand the realistic expectation that the patient's chance for survival is poor. b. inform the family that home health nurses will be able to help them maintain the mechanical ventilation at home after patient discharge. c. refer the family to social support services and case management to plan for transfer of the patient to a long-term care facility. d. provide hope and encouragement to the family because the patient's disease process has started to resolve.
A Rationale: The chance for survival is poor when the patient progresses to the fibrotic stage because permanent damage to the alveoli has occurred. Because of continued severe hypoxemia, the patient is not a candidate for home health or long-term care. The fibrotic stage indicates a poor patient prognosis, not the resolution of the ARDS process. Cognitive Level: Application Text Reference: p. 1814 Nursing Process: Implementation NCLEX: Psychosocial Integrity
A homeless patient with severe anorexia, fatigue, jaundice, and hepatomegaly is diagnosed with viral hepatitis and has just been admitted to the hospital. In planning care for the patient, the nurse assigns the highest priority to the patient outcome of a. maintaining adequate nutrition. b. establishing a stable home environment. c. increasing activity level. d. identifying the source of exposure to hepatitis.
A Rationale: The highest priority outcome is to maintain nutrition because adequate nutrition is needed for hepatocyte regeneration. Finding a home for the patient and identifying the source of the infection would be appropriate activities, but they do not have as high a priority as having adequate nutrition. Although the patient's activity level will be gradually increased, rest is indicated during the acute phase of hepatitis.
5. A patient is brought to the emergency department unconscious following a barbiturate overdose. Which potential complication will the nurse include when developing the plan of care? a. Hypercapnic respiratory failure related to decreased ventilatory effort b. Hypoxemic respiratory failure related to diffusion limitations c. Hypoxemic respiratory failure related to shunting of blood d. Hypercapnic respiratory failure related to increased airway resistance
A Rationale: The patient with an opioid overdose develops hypercapnic respiratory failure as a result of the decrease in respiratory rate and depth. Diffusion limitations, blood shunting, and increased airway resistance are not the primary pathophysiology causing the respiratory failure. Cognitive Level: Application Text Reference: p. 1800 Nursing Process: Diagnosis NCLEX: Physiological Integrity
Amitriptyline (Elavil) is prescribed for a diabetic patient with peripheral neuropathy who has burning foot pain occurring mostly at night. Which information should the nurse include when teaching the patient about the new medication? a. Amitriptyline will help prevent the transmission of pain impulses to the brain. b. Amitriptyline will improve sleep and make you less aware of nighttime pain. c. Amitriptyline will decrease the depression caused by the pain. d. Amitriptyline will correct some of the blood vessel changes that cause pain.
A Rationale: Tricyclic antidepressants decrease the transmission of pain impulses to the spinal cord and brain. Tricyclics also improve sleep quality and are used for depression, but that is not the major purpose for their use in diabetic neuropathy. The blood vessel changes that contribute to neuropathy are not affected by tricyclics.
What are signs and symptoms of flail chest?
Pain Paradoxical chest wall movement (see-saw chest); chest sucks inwardly on inspiration and puffs out on expiration. To assess chest symmetry, always stand at foot of bed to observe how the chest is rising and falling Dyspnea, cyanosis Increased pulse
D (Feedback: Immediately after the fracture, the nurse applies ice and encourages the patient to keep the head elevated. The nurse instructs the patient to apply ice packs to the nose to decrease swelling. Dependent positioning would exacerbate bleeding and the nose is not irrigated. Occlusive dressings are not used.)
A 42-year-old patient is admitted to the ED after an assault. The patient received blunt trauma to the face and has a suspected nasal fracture. Which of the following interventions should the nurse perform? A) Administer nasal spray and apply an occlusive dressing to the patient's face. B) Position the patient's head in a dependent position. C) Irrigate the patient's nose with warm tap water. D) Apply ice and keep the patient's head elevated.
C (Feedback: Obstructive sleep apnea occurs in men, especially those who are older and overweight. Symptoms include excessive daytime sleepiness, insomnia, and snoring. Daytime sleepiness and difficulty going to sleep at night are not indications of tonsillitis or adenoiditis. This patient's symptoms are not suggestive of laryngeal cancer.)
A 45-year-old obese man arrives in a clinic with complaints of daytime sleepiness, difficulty going to sleep at night, and snoring. The nurse should recognize the manifestations of what health problem? A) Adenoiditis B) Chronic tonsillitis C) Obstructive sleep apnea D) Laryngeal cancer
A 60 year old patient has an abrupt onset of anorexia, nausea and vomiting, hepatomegaly, and abnormal liver function studies. Serologic testing is negative for viral causes of hepatitis. During assessment of the patient, it is most important for the nurse to question the patient regarding A. any prior exposure to people with jaundice B. the use of all prescription and OTC (over the counter) medications C. treatment of chronic diseases with corticosteriods D. exposure to children recently immunized for hepatitis B
A and D assess for exposure to hepatitis. Hepatitis was ruled out this is inappropriate. C is incorrect because corticosteroids do not commonly cause liver disease B is correct because overdose of medications can cause liver disease.
The family will receive prophylactic therapy and the client will not be contagious after 2 to 3 weeks of medication therapy
A client being discharged from the hospital to home with a diagnosis of TB is worried about the possibility of infecting family members and others. Which information should reassure the client that contaminating family members and others is not likely? A. The family does not need therapy, and client will not be contagious after 1 month of medication therapy B. The family does not need therapy, and the client will not be contagious after 6 consecutive weeks of medication therapy C. The family will receive prophylactic therapy, and the client will not be contagious after 1 continuous week of medication therapy D. The family will receive prophylactic therapy, and the client will not be contagious after 2 to 3 weeks of medication therapy
B. Maintain a patent airway
A client is admitted to the ER and a diagnosis of myxedema coma is made. Which action should the nurse prepare to carry out initially? A. Warm the client B. Maintain a patent airway C. Monitor Intravenous fluids D. Administer thyroid hormone
D." I should not exercise in the late afternoon"
A client with type 1 diabetes mellitus calls the nurse to report recurrent episodes of hypoglycemia. Which statement by the client indicated a correct understanding of Humulin N insulin and exercise? A. " I should not exercise after lunch" B. " I should not exercise after breakfast." C." I should not exercise in the late evening" D." I should not exercise in the late afternoon"
The nurse is caring for several clients with respiratory disorders. Which client is at least risk for developing TB infection?
A man who is an inspector for the US Postal Service
B (Feedback: Antimicrobial agents (antibiotics) should not be used because they do not affect the virus or reduce the incidence of bacterial complications. In addition, their inappropriate use has been implicated in development of organisms resistant to therapy. It would be inappropriate to tell the patient that the physician will not respond to her request.)
A mother calls the clinic asking for a prescription for Amoxicillin for her 2-year-old son who has what the nurse suspects to be viral rhinitis. What should the nurse explain to this mother? A) "I will relay your request promptly to the doctor, but I suspect that she won't get back to you if it's a cold." B) "I'll certainly inform the doctor, but if it is a cold, antibiotics won't be used because they do not affect the virus." C) "I'll phone in the prescription for you since it can be prescribed by the pharmacist." D) "Amoxicillin is not likely the best antibiotic, but I'll call in the right prescription for you."
1, 2, 4 The symptoms of inadequate oxygenation in the patient include cyanosis, diaphoresis, and tachypnea. Cyanosis indicates inadequate perfusion due to compromised oxygenation. Diaphoresis and tachypnea occur due to sympathetic stimulation to compensate for inadequate oxygenation. Anemia occurs gradually and does not suddenly cause inadequate oxygenation. Hypertension does not indicate inadequate oxygenation in the patient. Text Reference - p. 479
A nurse is assessing a patient with dengue. The patient suddenly develops acute respiratory distress syndrome (ARDS). Which patient findings indicate inadequate oxygenation? Select all that apply. 1 Cyanosis 2 Diaphoresis 3 Anemia 4 Tachypnea 5 Hypertension
2 The patient has an increased respiratory rate with increased tidal volume; pH is 6, which indicates respiratory acidosis. In metabolic acidosis, the symptoms include rapid, regular, and deep inspirations. In respiratory and metabolic alkalosis, pH exceeds 7.5. Text Reference - p. 480
A nurse is caring for a patient experiencing respiratory distress. The patient has a respiratory rate of 24 breaths/minute. Arterial blood gas (ABG) analysis reveals a pH of 6, and the spirometric measurement of tidal volume is 600 mL. What do these values and symptoms indicate? 1 Metabolic acidosis 2 Respiratory acidosis 3 Respiratory alkalosis 4 Metabolic alkalosis
wheezes
A nurse is caring for a patient who has asthma. Which lung sound would the nurse expect to hear when auscultating this patients lung fields? A. fine crackles B. stridor C. pleural friction rub D. wheezes
1, 2, 4 The different triggers for asthmatic attack include perfumes, strong odors, and cold air. They cause reflex bronchoconstriction, and the patient should avoid them to prevent asthma attacks. Cold water and bacteria do not trigger asthma. Text Reference - p. 481
A nurse is caring for a patient with asthma. What instructions should the nurse provide to the patient to help prevent triggers of bronchoconstriction? Select all that apply. 1 Avoid wearing perfumes. 2 Avoid exposure to strong odors. 3 Do not drink cold water. 4 Avoid exposure to cold air. 5 Avoid people with certain infections
1, 2 Blood samples for ABG analysis are collected from the radial and femoral arteries, because they carry oxygenated blood. The pulmonary artery carries deoxygenated blood and is used for the measurement of mixed venous blood gas. The superior vena cava is used to measure the central venous pressure (CVP). The femoral vein is not used for ABG analysis, because it carries deoxygenated blood. Text Reference - p. 478
A nurse is caring for a patient with bronchitis. The health care provider has advised arterial blood gas (ABG) analysis. The blood sample can be obtained from which blood vessels? Select all that apply. 1 Radial artery 2 Femoral artery 3 Pulmonary artery 4 Superior vena cava 5 Femoral vein
1, 3, 5 Pneumonia will present with egophony, bronchophony, and whispering pectoriloquy. Egophony is a test to assess breath sounds. It is positive when the patient is asked to pronounce "E" but instead says "A." In bronchophony, the patient is asked to repeat "ninety-nine" several times in a row. If the words are easily understood and are clear and loud, it indicates an abnormal finding. In pectoriloquy, the patient is asked to whisper "one-two-three." If the whisper is heard clearly and distinctly, it indicates an abnormal finding. Wheezes are heard in asthma when there is bronchoconstriction. Stridor is heard in laryngeal diseases due to the obstruction of the larynx or trachea. Text Reference - p. 488
A nurse is caring for a patient with pneumonia. The nurse is most likely to auscultate what breath sounds when assessing the patient's lungs? Select all that apply. 1 Egophony 2 Wheezes 3 Bronchophony 4 Stridor 5 Whispering pectoriloquy
sitting on the side of the bed, leaning on an overbed table
A nurse is reinforcing instructions to a hospitalized client with a diagnosis of emphysema about positions that will enhance the effectiveness of breathing during dyspneic episodes, Which position should the nurse instruct the client to assume? A. side-laying in bed B. sitting in a recliner chair C.sitting up in bed at a 90 degree angle D.sitting on the side of the bed, leaning on a overbed table
A (Feedback: Chronic pharyngitis is common in adults who live and work in dusty surroundings, use the voice to excess, suffer from chronic chough, and habitually use alcohol and tobacco. Caffeine and spicy foods have not been linked to chronic pharyngitis. GERD is not a noted risk factor.)
A nurse practitioner has provided care for three different patients with chronic pharyngitis over the past several months. Which patients are at greatest risk for developing chronic pharyngitis? A) Patients who are habitual users of alcohol and tobacco B) Patients who are habitual users of caffeine and other stimulants C) Patients who eat a diet high in spicy foods D) Patients who have gastrointestinal reflux disease (GERD)
1 Nasal hairs help in filtration of air before the air enters the airways. Particles more than 5 µm in size do not get inside the airway due to their large size and are less dangerous to the airways. Text Reference - p. 480
A nurse recalls that some airborne particles are small enough to move past the nasal hairs, reach the alveoli, and become dangerous to the patient. The nurse is aware that a particle is too large to reach the alveoli if the particle is what size? 1 Greater than 5 µm in size 2 Greater than 7 µm in size 3 Greater than 9 µm in size 4 Greater than 11 µm in size
biots respiration
A nurse should observe for and report which abnormal breathing pattern that is most likely to occur in patients with increased intracranial pressure? A. cheyne-stokes respiration B. kussmauls respiration C. biot's respiration D. apnustic respiration
C (Feedback: Nursing care for patients with viral pharyngitis focuses on symptomatic management. Antibiotics are not prescribed for viral etiologies. Surgery is not indicated in the treatment of viral pharyngitis. Chronic hoarseness is not a common sequela of viral pharyngitis, so teaching ways to prevent it would be of no use in this instance.)
A nursing student is discussing a patient with viral pharyngitis with the preceptor at the walk-in clinic. What should the preceptor tell the student about nursing care for patients with viral pharyngitis? A) Teaching focuses on safe and effective use of antibiotics. B) The patient should be preliminarily screened for surgery. C) Symptom management is the main focus of medical and nursing care. D) The focus of care is resting the voice to prevent chronic hoarseness.
D (Feedback: If pressure to the midline septum does not stop the bleeding for epistaxis, additional treatment of silver nitrate application, Gelfoam, electrocautery, or vasoconstrictors may be used. Suction may be used to visualize the nasal septum, but it does not alleviate the bleeding. Irrigation with a hypertonic solution is not used to treat epistaxis.)
A patient comes to the ED and is admitted with epistaxis. Pressure has been applied to the patient's midline septum for 10 minutes, but the bleeding continues. The nurse should anticipate using what treatment to control the bleeding? A) Irrigation with a hypertonic solution B) Nasopharyngeal suction C) Normal saline application D) Silver nitrate application
1 Prolactin is secreted by the pituitary gland. Thyroxine hormone is secreted by the thyroid gland. The kidney secretes erythropoietin. Parathormone is secreted by parathyroid gland. Text Reference - p. 1192
A patient has a dysfunction of the pituitary gland. Which hormone secretion does the nurse recognize may be altered in this patient? 1 Prolactin 2 Thyroxine 3 Erythropoietin 4 Parathormone
4 ]Coughing up blood is called hemoptysis. Vomiting blood is known as hematemesis. Both conditions are difficult to differentiate. A pH test of the mucus may show an acidic reaction if the blood is from the stomach. A detailed history may not help the nurse to differentiate between hemoptysis and hematemesis, because the symptoms are similar. Hematologic tests may indicate the levels of hemoglobin but do not help in differentiating between both conditions. SpO2 helps in measuring the oxygen saturation levels but does not help in differentiating between hemoptysis and hematemesis. Text Reference - p. 483
A patient has a history of coughing up blood. How can the nurse differentiate between hemoptysis and hematemesis? 1 By asking the history in detail 2 By performing hematologic tests 3 By measuring SpO2 4 By performing a mucus pH test
2, 3, 4, 1 Diabetes insipidus is caused by abnormalities in antidiuretic hormone levels. In patients with diabetes insipidus, the levels of antidiuretic hormone is reduced; this leads to decreased reabsorption of water, increasing the urine output, which reduces the intravascular fluid volume and elevates the osmolality in the blood. Text Reference - p. 1194
A patient has developed diabetes insipidus. Arrange the events in the order they occur in this patient. 1. Increase in serum osmolality 2. Decrease in antidiuretic hormone 3. Decrease in water reabsorption 4. Decrease in intravascular fluid volume
A (Feedback: Patients with nasotracheal and nasogastric tubes in place are at risk for development of sinus infections. Thus, accurate assessment of patients with these tubes is critical. Use of a nasogastric tube is not associated with the development of the other listed pathologies.)
A patient has had a nasogastric tube in place for 6 days due to the development of paralytic ileus after surgery. In light of the prolonged presence of the nasogastric tube, the nurse should prioritize assessments related to what complication? A) Sinus infections B) Esophageal strictures C) Pharyngitis D) Laryngitis
B (Feedback: The incidence of distant metastasis with squamous cell carcinoma of the head and neck (including larynx cancer) is relatively low. The patient's prognosis is determined by the oncologist, but the patient has asked a general question and it would be inappropriate to refuse a response. The nurse must not downplay the patient's concerns.)
A patient has just been diagnosed with squamous cell carcinoma of the neck. While the nurse is doing health education, the patient asks, "Does this kind of cancer tend to spread to other parts of the body?" What is the nurse's best response? A) "In many cases, this type of cancer spreads to other parts of the body." B) "This cancer usually does not spread to distant sites in the body." C) "You will have to speak to your oncologist about that." D) "Squamous cell carcinoma is nothing to be concerned about, so try to focus on your health."
A. Semi-Fowler
A patient has just returned from the postanesthesia care unit after a hypophysectomy. The nurse knows to keep the patient in which position? A. Semi-Fowler B. Left lateral Sims C. Supine with no pillows D. Reverse Trendelenburg
2 Elevating the head of the bed to a 30-degree angle alleviates pressure on the sella turcica and thereby relieves the headache. Placing the bed parallel to the floor does not alleviate pressure on the sella turcica. Glucose levels are used to check cerebrospinal fluid leakage. Vigorous coughing and sneezing should be avoided to prevent cerebrospinal fluid leakage. Text Reference - p. 1191
A patient has undergone surgery for acromegaly. After surgery, the patient is experiencing severe headache. What action should be taken to provide relief from the headache? 1 The bed should be placed parallel to the floor. 2 The head of the bed should be elevated to 30-degree angle. 3 The glucose level of the patient should be maintained. 4 The patient should be told to avoid coughing and sneezing
C (Feedback: A liquid or soft diet is provided during the acute stage of the disease, depending on the patient's appetite and the degree of discomfort that occurs with swallowing. The patient is encouraged to drink as much fluid as possible (at least 2 to 3 L/day). There is no need for increased potassium or protein intake.)
A patient is being treated for bacterial pharyngitis. Which of the following should the nurse recommend when promoting the patient's nutrition during treatment? A) A 1.5 L/day fluid restriction B) A high-potassium, low-sodium diet C) A liquid or soft diet D) A high-protein diet
2 Asthma involves bronchospasms, which can be triggered by many factors including pollens inhaled during outdoor activities such as gardening. Wheezes are continuous, high-pitched squeaking or musical sounds caused by the rapid vibration and narrowing of bronchial walls. If the patient has wheezing sounds during auscultation, it indicates the patient may have asthma. Rhonchi sounds are continuous rumbling, snoring, or rattling sounds caused by obstruction of large airways with secretions. This would be seen in instances of cystic fibrosis. Fine crackles are series of short-duration, discontinuous, high-pitched sounds heard just before the end of inspiration, as seen in cases of pulmonary fibrosis and interstitial edema. Coarse crackles are long-duration, discontinuous, and low-pitched and they are usually caused by air passing through an airway intermittently occluded by mucus, unstable bronchial walls, or folds of mucosa. Coarse crackles can be heard in conditions such as heart failure and pulmonary edema. Text Reference - p. 489
A patient is brought to the emergency department with chest tightness and acute dyspnea after an afternoon of gardening. As the nurse auscultates the patient's lungs, which finding would indicate a need for asthma testing? 1 Rhonchi 2 Wheezes 3 Fine crackles 4 Coarse crackles
1 A mass in the neck may cause tracheal diversion to the opposite side of the mass. Therefore, if the patient has a neck mass in the right side, the nurse is likely to find tracheal deviation to the left side, away from the mass, during palpation. The tracheal deviation would be to the right side, or toward the mass, in the case of lobar atelectasis. The nurse is not likely to see any effect on the patient's chest expansion or diaphragm movement because a neck mass would be at a higher anatomical level. Chest expansion would be asymmetrical in instances of atelectasis or a collapsed lung. The diaphragm would be dysfunctional in the instance of phrenic nerve injury. Test-Taking Tip: Look for contradictory answer choices. Since both answers cannot be correct, it is likely that one is definitely incorrect and it can be immediately eliminated. Text Reference - p. 486
A patient is diagnosed with a mass on the right side of the neck. What physical change does the nurse find during a physical examination of this patient? 1 The trachea is deviated to the left. 2 The trachea is deviated to the right. 3 The chest expansion is asymmetrical. 4 The diaphragm is unable to contract smoothly.
1 Brain tumor is one possible cause for central diabetes insipidus, which occurs due to the interference with antidiuretic hormone synthesis, transport, or release. In cases of renal damage and drug therapy with lithium, there would be an inadequate renal response to antidiuretic hormone despite the presence of adequate antidiuretic hormone, which leads to nephrogenic diabetes insipidus. A structural lesion in the thirst center may cause primary diabetes insipidus, which can be a result of excessive water intake. Text Reference - p. 1194
A patient is diagnosed with central diabetes insipidus and states not knowing how the illness was acquired. What does the nurse recognize as a possible cause of this disorder? 1 The presence of a brain tumor 2 Renal damage from long-standing hypertension 3 Drug therapy with lithium for bipolar disorder 4 Structural lesion in the thirst center
4 The syndrome of inappropriate antidiuretic hormone (SIADH) is characterized by inappropriate secretion of ADH, which disrupts the fluid and electrolyte balance. Increased intravascular volume is one of the characteristic features of SIADH. Decreased ADH, excessive urine output, and increased serum osmolality are the features of diabetes insipidus. Text Reference - p. 1193
A patient is diagnosed with syndrome of inappropriate antidiuretic hormone (SIADH) after a head injury. What condition does the nurse suspect that correlates with this disorder? 1 Decreased antidiuretic hormone (ADH) 2 Excessive urine output 3 Increased serum osmolality 4 Increased intravascular volume
1 In addition to the patient history and physical examination, the oral glucose tolerance test is a specific test for acromegaly. As growth hormone secretion is normally inhibited by glucose, measurement of glucose nonsuppresibility is required. Growth hormone concentration normally falls during the oral glucose tolerance test, but in patients with acromegaly, the growth hormone levels do not fall. Hepatomegaly is enlargement of the liver. Spleenomegaly is enlargement of the spleen. Dactylomegaly is enlargement of the toes and fingers. Text Reference - p. 1190
A patient is instructed to ingest 75 g of glucose orally as a part of an oral glucose tolerance test. In addition, growth hormone measurements are taken consecutively at 30, 60, 90, and 120 minutes. Glucose levels and growth hormone levels are found to be constant during the test. What does the nurse infer from these findings? 1 Acromegaly 2 Hepatomegaly 3 Splenomegaly 4 Dactylomegaly
1, 3, 5 The nurse should ensure the consent form has been signed and administer sedatives if prescribed. For a bronchoscopy, the patient is required to be on NPO for 6 to 12 hours before the procedure. Renal function is not related to bronchoscopy, so assessment of BUN and serum creatinine levels is not needed. Removing metals from the body may not be necessary, because the procedure does not involve x-rays. Text Reference - p. 492
A patient is scheduled for a biopsy through bronchoscopy. What nursing interventions are appropriate for this patient? Select all that apply. 1 Ensure that an informed consent has been signed. 2 Assess blood urea nitrogen (BUN) and serum creatinine levels. 3 Administer sedative, if prescribed. 4 Instruct patient to remove all metal from the body. 5 Instruct patient to be on NPO status for 6 to 12 hours before the test.
4 CT scans may be often performed with contrast medium. These contrast media are excreted through urine; therefore, it is important for the patient to have optimal renal function to prevent accumulation of the contrast media in the body. The normal range of serum creatinine level is 0.6 to 1.3 mg/dL; therefore, a serum creatinine level of 3.0 mg/dL is very high and indicates renal dysfunction. As a result, the diagnostic test should not be performed on the patient. Hematocrit of 50%, PaCO2 of 40 mm Hg, and hemoglobin of 14.0 g/dL are within normal ranges. Text Reference - p. 491
A patient is scheduled for a computed tomography (CT) scan with contrast medium. After reviewing the patient's laboratory reports, the nurse contacts the primary health care provider and the CT scan is cancelled. Which laboratory parameter would have made the diagnostic test unsafe for the patient? 1 Hematocrit 50% 2 PaCO2 40mm Hg 3 Hemoglobin 14.0 g/dL 4 Serum creatinine 3.0 mg/dL
blood- tinged sputum
A patient is to have a bronchoscopy. The LPN/LVN should expect which finding in the postprocedure period? A.Difficulty breathing B. Blood-tinged sputum C. Elevated temperature D. Elevated blood pressure
increase fluid intake to 2500 ml a day
A patient presents at the emergency room complaining of severe throat pain "that's so bad I can hardly swallow. It feels like there's a huge lump in my throat." The patient is diagnosed with severe pharyngitis. What would the nurse include in patient teaching regarding this condition? A. Increase fluid intake to 2500 mL a day. B. Refrain from taking hot baths or showers. C. Limit the amount of fruit juice in the patient's diet. D. Decrease the humidity in the patient's environment
3 Dyspnea, cyanosis, fine crackles, and dullness on percussion all support the diagnosis of pulmonary edema. Wheezing and hyperresonance on percussion support the diagnosis of asthma. Tachypnea, diminished or absent breath sounds, and dullness on percussion support the diagnosis of pleural effusion. Tachypnea, crackles, and resonance on percussion support the diagnosis of pulmonary fibrosis. Test-Taking Tip: Read the question carefully before looking at the answers. Emphasize on the symptoms and the abnormal condition of the patient to answer correctly. Text Reference - p. 490
A patient presents in the emergency department with dyspnea. The nurse notes a bluish discoloration of the patient's lips, fine crackles on auscultation, and dullness upon percussion of the lung fields. Which diagnosis does the nurse anticipate? 1 Asthma 2 Pleural effusion 3 Pulmonary edema 4 Pulmonary fibrosis
C. "Hypoglycemia has been successfully treated by diet modifications
A patient recently diagnosed as having hypoglycemia says, "Hypoglycemia! I can't live with that. My neighbor, Joseph, had that and he acted crazy!" Which response by the nurse is most appropriate? A. "You seem to be overreacting to the problem." B. "You're right; it would be difficult to live with hypoglycemia." C. "Hypoglycemia has been successfully treated by diet modifications." D. "Taking care of yourself years ago would have prevented the problem"
2 Prolactinomas are among the most common type of pituitary adenomas; compression of the chiasm nerve is a complication of this condition and can cause visual problems with signs of increased intracranial pressure, including headache, nausea, and vomiting. Patients with acromegaly, diabetes insipidus, and panhypopituitarism are not associated with the complication of a compressed optic chiasm. Test-Taking Tip: Be certain to answer every question. You must arrive at one correct or one "best" answer. If you must, "guess" between two alternatives or eliminate the two or three answers you know are wrong first. Text Reference - p. 1192
A patient reports to the nurse having visual problems, and is found to have a compressed optic chiasm. Which condition could be the reason for this complication? 1 Acromegaly 2 Prolactinomas 3 Diabetes insipidus 4 Panhypopituitarism
B (Feedback: Colds are highly contagious because virus is shed for about 2 days before the symptoms appear and during the first part of the symptomatic phase. Antibiotic resistance is not relevant to viral illnesses and OTC medications do not have a "rebound" effect. Genetic factors do not exist.)
A patient states that her family has had several colds during this winter and spring despite their commitment to handwashing. The high communicability of the common cold is attributable to what factor? A) Cold viruses are increasingly resistant to common antibiotics. B) The virus is shed for 2 days prior to the emergence of symptoms. C) A genetic predisposition to viral rhinitis has recently been identified. D) Overuse of OTC cold remedies creates a "rebound" susceptibility to future colds.
A client has an endocrine system dysfunction of the pancreas. The nurse anticipates that the client will exhibit impaired secretion of which of the following substances? 1. Amylase 2. Lipase 3. Trypsin 4. Insulin
Ans. 4 rationale The pancreas produces both endocrine and exocrine secretions as part of its normal function. The organ secretes insulin as a key endocrine hormone to regulate the blood glucose level. Other pancreatic endocrine hormones are glucagon and somatostatin. The exocrine pancreas produces digestive enzymes such as amylase, lipase, and trypsin.
3 Following a lung biopsy through TTNA, the patient should be sent for chest x-ray to rule out a pneumothorax, which is a common complication of the procedure. Only after the chest x-ray is done, can the patient can be told to rest or do deep breathing exercises. Oxygen saturation levels are generally monitored throughout the procedure. Test-Taking Tip: A psychologic technique used to boost your test-taking confidence is to look into a mirror whenever you pass one and say out loud, "I know the material, and I'll do well on the test." Try it; many students have found that it works because it reduces "test anxiety." Text Reference - p. 493
A patient suspected of having lung cancer has undergone lung biopsy through transthoracic needle aspiration (TTNA). What is the priority nursing action for this patient immediately following the procedure? 1 Allow the patient to take a rest. 2 Instruct the patient to do deep breathing. 3 Send the patient for a chest x-ray as prescribed. 4 Measure oxygen saturation levels.
The patient will need his aminophylline dosage adjusted
A patient taking aminophylline tells the nurse that he is going to begin a smoking cessation program when he is discharged from the hospital. Why should the nurse tell this patient to notify his physician if his smoking pattern changes? A. The patient will need his aminophylline dosage adjusted. B. The patient will not derive as much benefit from inhaler use. C. The patient will require an increase in antitussive medication. D. The patient will no longer require annual influenza immunization.
D (Feedback: For a patient diagnosed with acute sinusitis, the nurse should instruct the patient that hot packs, increasing fluid intake, and elevating the head of the bed can promote drainage. Applying a mustard poultice will not promote sinus drainage. Postural drainage is used to remove bronchial secretions.)
A patient visiting the clinic is diagnosed with acute sinusitis. To promote sinus drainage, the nurse should instruct the patient to perform which of the following? A) Apply a cold pack to the affected area. B) Apply a mustard poultice to the forehead. C) Perform postural drainage. D) Increase fluid intake.
Pneumonia
A patient who experienced high fever and chills, a productive cough, chest pain, general malaise, and aching muscles during the past week is admitted to the hospital. The nurse realizes these symptoms correspond most closely with which disease? A. Pneumonia B. Type A influenza C. Pleurisy with effusion D. Streptococcus empyema
A. Keeping environmental stimuli to a minimum
A patient who has hyperthyroidism exhibits symptoms of anxiety, nervousness, and agitation. Which intervention should be included in the patient's care? A. Keeping environmental stimuli to a minimum B. Encouraging questions about options for treatment C. Stressing the importance of complying with the treatment regimen D. Maintaining the temperature of the room slightly above normal
B. Regular
A patient who is undergoing surgery will have an intravenous solution to which insulin will be added. Which type of insulin must be used? A. Lente B. Regular C. Ultralente D. Neutral protamine Hagedorn (NPH)
1 Tracheal deviation is a medical emergency when it is caused by a tension pneumothorax. Tactile fremitus increases with pneumonia or pulmonary edema and decreases in pleural effusion or lung hyperinflation. Diminished chest movement occurs with barrel chest, restrictive disease, and neuromuscular disease. Text Reference - p. 489
A patient who was injured playing football presents to the emergency department (ED). During the assessment, the nurse is palpating the patient's chest. Which finding is a medical emergency? 1 Trachea moved to the left 2 Increased tactile fremitus 3 Decreased tactile fremitus 4 Diminished chest movement
Hypoxic drive is necessary for breathing.
A patient with COPD asks the nurse to turn his oxygen up from 3 L/min via nasal cannula to 5 L/min. The nurse explains to the patient that she cannot turn his oxygen up this high. What is the reason the oxygen cannot be increased to 5 L/min? A. Hypoxic drive is necessary for breathing. B. Hypercapnic drive is necessary for breathing. C. Higher concentrations may result in a severe headache. D. Higher levels will be required later for arterial blood gases (ABGs).
3 Somatotropin is also called growth hormone (GH). Excessive secretion of GH results in overgrowth of soft tissues and bones resulting in acromegaly. Cortisol produces a number of physiologic effects, such as increasing blood glucose levels, potentiating the action of catecholamines on blood vessels, and inhibiting the inflammatory response. Thyroxine acts as a precursor to triiodothyronine, which regulates metabolic rate of all cells and processes of cell growth and tissue differentiation. Excessive secretion of cortisol, thyroxine, and triiodothyronine do not result in acromegaly. Text Reference - p. 1190
A patient with a pituitary tumor has developed excessive height, and increased hat size and shoe size. Which hormone does the nurse determine is secreting excessively? 1 Cortisol 2 Thyroxine 3 Somatotropin 4 Triidothyronine
4 In contrast to inspiration, expiration is passive. Elastic recoil is the tendency for the lungs to relax after being stretched or expanded. The elasticity of lung tissue is due to the elastin fibers found in the alveolar walls and surrounding the bronchioles and capillaries. The elastic recoil of the chest wall and lungs allows the chest to passively decrease in volume. Intrathoracic pressure rises, causing air to move out of the lungs. Lungs do not expel air by use of abdominal muscles. An increase in carbon dioxide and decrease in oxygen in the blood do not promote expiration. Text Reference - p. 478
A patient with a respiratory condition asks "I know how air gets into my lungs. But how does it get out of them?" The nurse bases the answer on the knowledge that air moves out of the lungs because of 1 Contraction of the accessory abdominal muscles 2 Increased carbon dioxide and decreased oxygen in the blood 3 Stimulation of the respiratory muscles by the chemoreceptors 4 Increase in intrathoracic pressure as elastic recoil of the lungs occurs
C. "Inspect each foot daily for cuts, cracks, blisters, or abrasions."
A patient with diabetes asks her nurse about foot care when she is discharged home. What is the nurse's best response? A. "Cut your toenails in a V shape to prevent ingrown toenails." B. "Soak your feet in hot water each night before going to bed." C. "Inspect each foot daily for cuts, cracks, blisters, or abrasions." D. "There are no special instructions for your feet when you have diabetes"
A. Diabetic neuropathy
A patient with diabetes is admitted to the emergency department with complaints of lack of feeling, yet debilitating pain in his legs and feet, constipation, and sexual impotence. These symptoms most closely correlate with which disorder? A. Diabetic neuropathy B. Diabetic retinopathy C. Diabetic ketoacidosis D. Diabetic nephropathy
4 The most common cause for the development of pleural effusion in the patient suffering from malignancy is lymphatic drainage blocked by malignant cells. Bacterial infection is unlikely in the absence of other signs. An allergic reaction may not lead to pleural effusion. Malignancy is not the cause of raised BP. Text Reference - p. 478
A patient with lung cancer reports chest pain. The nurse assesses the patient and finds the pain to be related to pleural effusion. What could be the reason for the development of pleural effusion in this patient? 1 Bacterial infection due to compromised immunity 2 Allergic reaction to chemotherapy 3 Increased blood pressure (BP) due to malignancy 4 Lymphatic drainage blocked by malignant cells
1, 2, 3, 5 Monitoring the papillary response helps rule out any visual changes after transsphenoidal hypophysectomy. Observing the patient for signs of bleeding is of the utmost importance as hemorrhage can cause complications. The strength of the extremities is monitored to rule out postoperative neurologic complications, such as ataxia. Elevating the head of the patient's bed to a 30-degree angle alleviates pressure on the sella turcica and decreases headaches, which are a frequent postoperative problem. Avoiding tooth brushing for at least 10 days helps protect the suture line. Text Reference - p. 1191
A patient with pituitary adenoma underwent transsphenoidal hypophysectomy. What nursing actions are most effective for prevention of complications? Select all that apply. 1 Monitoring the pupillary response 2 Elevating the head of the patient's bed 3 Observing the patient for any signs of bleeding 4 Advising the patient to brush his or her teeth twice daily 5 Monitoring extremity strength to detect neurologic complications
1 Priorities for assessment are the patient's airway and breathing, both of which may be compromised after bronchoscopy by laryngeal edema. These assessment parameters supersede the importance of LOC, pain, heart rate, and blood pressure, although the nurse should be assessing these also. Text Reference - p. 492
A patient with recurrent shortness of breath has just had a bronchoscopy. What is a priority nursing action immediately following the procedure? 1 Monitor the patient for laryngeal edema. 2 Assess the patient's level of consciousness (LOC). 3 Monitor and manage the patient's level of pain. 4 Assess the patient's heart rate and blood pressure
C. She can play tennis, but she will need to eat more before she plays
A patient with type 1 diabetes mellitus (DM) plays tennis and asks if she will be able to continue with that sport. The nurse should base his response on which information? A. It would be better to take up walking or some quiet sport. B. She can play tennis, but she will need an extra dose of insulin. C. She can play tennis, but she will need to eat more before she plays. D. She cannot play tennis because heavy exercise is not permitted with this type of diabetes.
D (Feedback: In patients receiving transesophageal puncture, a valve is placed in the tracheal stoma to divert air into the esophagus and out the mouth. Once the puncture is surgically created and has healed, a voice prosthesis (Blom-Singer®) is fitted over the puncture site. A nasogastric tube and belching are not required. An artificial pharynx is not used.)
A patient's total laryngectomy has created a need for alaryngeal speech which will be achieved through the use of tracheoesophageal puncture. What action should the nurse describe to the patient when teaching him about this process? A) Training on how to perform controlled belching B) Use of an electronically enhanced artificial pharynx C) Insertion of a specialized nasogastric tube D) Fitting for a voice prosthesis
1, 2, 5 The most common conditions presenting with pleural rub are pleurisy, pneumonia, and pulmonary infarct. Pleural rub is caused by the rubbing together of the two layers of the lungs. Asthma and bronchitis present with wheezes and do not manifest as pleural rub. Text Reference - p. 490
A senior nurse is teaching nursing students to auscultate for adventitious sounds. One of the students auscultates a pleural friction rub. What are the common conditions in which a pleural friction rub is present? Select all that apply. 1 Pleurisy 2 Pneumonia 3 Asthma 4 Bronchitis 5 Pulmonary infarct
When auscultating a patient's chest while the patient takes a deep breath, the nurse hears loud, high-pitched, "blowing" sounds at both lung bases. The nurse will document these as A) Abnormal sounds. B) Normal sounds. C) Vesicular sounds. D) Adventitious sounds.
A) Abnormal sounds. Feedback: The description indicates that the nurse hears bronchial breath sounds that are abnormal when heard at the lung base. Adventitious sounds are extra breath sounds such as crackles, wheezes, rhonchi, and friction rubs. Vesicular sounds are low-pitched, soft sounds heard over all lung areas except the major bronchi.
A patient with bacterial pneumonia has rhonchi and thick sputum. Which action will the nurse use to promote airway clearance? A) Assist the patient to splint the chest when coughing. B) Educate the patient about the need for fluid restrictions. C) Instruct the patient on the pursed lip breathing technique. D) Encourage the patient to wear the nasal oxygen cannula.
A) Assist the patient to splint the chest when coughing. Feedback: Coughing is less painful and more likely to be effective when the patient splints the chest during coughing. Fluids should be encouraged to help liquefy secretions. Nasal oxygen will improve gas exchange, but will not improve airway clearance. Pursed lip breathing is used to improve gas exchange in patients with COPD, but will not improve airway clearance.
When assessing a patient's sleep-rest pattern related to respiratory health, the nurse would ask if the patient: (Select all that apply.) A. Has trouble falling asleep B. Awakens abruptly during the night C. Sleeps more than 8 hours per night D. Has to sleep with the head elevated
A,B,D The patient with sleep apnea may have insomnia and/or abrupt awakenings. Patients with cardiovascular disease (e.g., heart failure that may affect respiratory health) may need to sleep with the head elevated on several pillows (orthopnea). Sleeping more than 8 hours per night is not indicative of impaired respiratory health.
A patient with bacterial pneumonia has rhonchi and thick sputum. What is the nurse's most appropriate action to promote airway clearance? a. Assist the patient to splint the chest when coughing. b. Teach the patient about the need for fluid restrictions. c. Encourage the patient to wear the nasal oxygen cannula. d. Instruct the patient on the pursed lip breathing technique.
ANS: A Coughing is less painful and more likely to be effective when the patient splints the chest during coughing. Fluids should be encouraged to help liquefy secretions. Nasal oxygen will improve gas exchange, but will not improve airway clearance. Pursed lip breathing is used to improve gas exchange in patients with COPD, but will not improve airway clearance.
A patient is having inspiratory stridor (crowing respiration) and the nurse suspects he is experiencing a laryngospasm. Which of the following would be most appropriate to implement for a patient experiencing a laryngospasm? A. Administer 100% oxygen. B. Position the patient in high Fowler's position. C. Insert a 16-gauge (large-bore) IV needle. D. Activate the emergency response team (code blue team) to the patient's room.
A. Administer 100% oxygen.A nurse should immediately administer 100% oxygen to the patient until the airway is fully reestablished, the larynx relaxes, and the spasms stop. Activating the emergency response team is not an immediate nursing action at this time because the nurse can administer the oxygen without the assistance of others. Positioning the patient in high Fowler's will not address the patient's need for immediate reoxygenation because of the patient's compromised respiratory state. Insertion of an IV device is not the first priority response but should be implemented after the nurse has assessed that the airway is stable.
The nurse is caring for a patient admitted to the hospital with pneumonia. Upon assessment, the nurse notes a temperature of 101.4° F, a productive cough with yellow sputum and a respiratory rate of 20. Which of the following nursing diagnosis is most appropriate based upon this assessment? A. Hyperthermia related to infectious illness B. Ineffective thermoregulation related to chilling C. Ineffective breathing pattern related to pneumonia D. Ineffective airway clearance related to thick secretions
A. Hyperthermia related to infectious illness Because the patient has spiked a temperature and has a diagnosis of pneumonia, the logical nursing diagnosis is hyperthermia related to infectious illness. There is no evidence of a chill, and her breathing pattern is within normal limits at 20 breaths per minute. There is no evidence of ineffective airway clearance from the information given because the patient is expectorating sputum.
If a nurse is caring for an 80-year-old patient with a temperature of 100.4° F, crackles at the right lung base, pain with deep inspiration, and dyspnea, which of the following orders is the nurse's priority? A. Sputum specimen for culture and sensitivity B. Codeine 15 mg orally every 6 hours as needed C. Incentive spirometer every 2 hours while awake D. Amoxicillin (Amoxil) 500 mg orally 4 times a day
A. Sputum specimen for culture and sensitivity The patient presents with signs of a respiratory infection. To initiate the most effective therapy, the health care prescriber must know the pathogen causing the infection. Therefore, the sputum specimen is the nurse's priority. If the antibiotic is administered before the specimen is obtained, the results of the culture might not be as accurate and could impair the effectiveness of therapy. After the specimen is obtained, the nurse can administer codeine for coughing and begin the incentive spirometry to mobilize secretions and improve the patient's ability to expectorate the secretions.
The nurse is caring for a postoperative patient with sudden onset of respiratory distress. The physician orders a STAT ventilation-perfusion scan. Which of the following explanations should the nurse provide to the patient about the procedure? A. This test involves injection of a radioisotope to outline the blood vessels in the lungs, followed by inhalation of a radioisotope gas. B. This test will use special technology to examine cross sections of the chest with use of a contrast dye. C. This test will use magnetic fields to produce images of the lungs and chest. D. This test involves injecting contrast dye into a blood vessel to outline the blood vessels of the lungs.
A. This test involves injection of a radioisotope to outline the blood vessels in the lungs, followed by inhalation of a radioisotope gas.A ventilation-perfusion scan has two parts. In the perfusion portion, a radioisotope is injected into the blood and the pulmonary vasculature is outlined. In the ventilation part, the patient inhales a radioactive gas that outlines the alveoli.
A patient with acute exacerbation of COPD needs to receive precise amounts of oxygen. Which of the following types of equipment should the nurse prepare to use? A. Venturi mask B. Partial non-rebreather mask C. Oxygen tent D. Nasal cannula
A. Venturi mask The Venturi mask delivers precise concentrations of oxygen and should be selected whenever this is a priority concern. The other methods are less precise in terms of amount of oxygen delivered.
What are signs and symptom of fractures of the ribs and sternum?
Pain & tenderness Crepitus (bones grating together) Shallow respirations Respiraotory acidosis
A patient is admitted to the hospital with fever, chills, a productive cough with rusty sputum, and pleuritic chest pain. Pneumococcal pneumonia is suspected. An appropriate nursing diagnosis for the patient based on the patient's manifestations is A. hyperthermia related to acute infectious process. B. chronic pain related to ineffective pain management. C. risk for injury related to disorientation and confusion. D. ineffective airway clearance related to retained secretions.
A. hyperthermia related to acute infectious process. The patient with pneumococcal pneumonia is acutely ill with fever and the systemic manifestations of fever, such as chills, thirst, headache, and malaise. Interventions that monitor temperature and aid in lowering body temperature are appropriate. Ineffective airway clearance would be manifested by adventitious breath sounds and difficulty producing secretions. Disorientation and confusion are not noted in this patient and are not typical unless the patient is very hypoxemic. Pleuritic pain is an acute pain that is due to inflammation of the pleura.
To ensure the correct amount of oxygen delivery for a patient receiving 35% oxygen via a Venturi mask, it is most important that the nurse A. keep the air-entrainment ports clean and unobstructed. B. apply an adaptor to increase humidification of the oxygen. C. drain moisture condensation from the oxygen tubing every hour. D. keep the flow rate high enough to keep the bag from collapsing during inspiration.
A. keep the air-entrainment ports clean and unobstructed. Oxygen is delivered to a small jet in the center of a wide-based cone. Air is entrained (pulled through) openings in the cone as oxygen flows through the small jet. The degree of restriction or narrowness of the jet determines the amount of entrainment and the dilution of pure oxygen with room air and thus the concentration of oxygen. Although applying an adaptor can increase the humidification with the Venturi mask, it is not the best answer, because an open port is essential to proper functioning. Draining moisture condensation from the oxygen tubing is performed as often as needed, not on an hourly schedule. A plastic face mask with a reservoir bag needs to have sufficient flow rate to keep the bag inflated.
Select all that apply. Which of the following is included in a comprehensive respiratory assessment? A. Pulse oximetry B. Chest auscultation C. Apical radial pulse D. Nail-bed assessment E. Evaluation of respiratory effort F. Rate and character of respirations
ABDEF The total assessment of the respiratory system includes pulse oximetry; auscultation; skin and nail-bed assessment for the detection of cyanosis; and rate, character, and degree of effort of respirations. The apical radial pulse is a cardiac assessment.
Select all that apply. During initial assessment, a nurse should record which of the following manifestations of respiratory distress? A. Tachypnea B. Nasal flaring C. Thready pulse D. Panting or grunting E. Use of intercostal muscles F. An inspiratory-to-expiratory ratio of 1:2
AD Manifestations of respiratory distress include tachypnea, grunting and panting on respiration, central cyanosis, use of accessory muscles, and flaring nares.
The nurse notes new onset confusion in an older patient who is normally alert and oriented. In which order should the nurse take the following actions? (Put a comma and a space between each answer choice [A, B, C, D].) a. Obtain the oxygen saturation. b. Check the patient's pulse rate. c. Document the change in status. d. Notify the health care provider
ANS: A, B, D, C Assessment for physiologic causes of new onset confusion such as pneumonia, infection, or perfusion problems should be the first action by the nurse. Airway and oxygenation should be assessed first, then circulation. After assessing the patient, the nurse should notify the health care provider. Finally, documentation of the assessments and care should be done.
21. The nurse is caring for a 63-year-old with a possible pituitary tumor who is scheduled for a computed tomography (CT) scan with contrast. Which information about the patient is most important to discuss with the health care provider before the test? a. History of renal insufficiency b. Complains of chronic headache c. Recent bilateral visual field loss d. Blood glucose level of 134 mg/dL
ANS: A Because contrast media may cause acute kidney injury in patients with poor renal function, the health care provider will need to prescribe therapies such as IV fluids to prevent this complication. The other findings are consistent with the patient's diagnosis of a pituitary tumor. DIF: Cognitive Level: Apply (application) REF: 1148 OBJ: Special Questions: Prioritization TOP: Nursing Process: Assessment MSC: NCLEX: Physiological Integrity
The clinic nurse teaches a patient with a 42 pack-year history of cigarette smoking about lung disease. Which information will be most important for the nurse to include? a. Options for smoking cessation b. Reasons for annual sputum cytology testing c. Erlotinib (Tarceva) therapy to prevent tumor risk d. Computed tomography (CT) screening for lung cancer
ANS: A Because smoking is the major cause of lung cancer, the most important role for the nurse is teaching patients about the benefits of and means of smoking cessation. CT scanning is currently being investigated as a screening test for high-risk patients. However, if there is a positive finding, the person already has lung cancer. Erlotinib may be used in patients who have lung cancer, but it is not used to reduce the risk of developing cancer.
19. After being hit by a baseball, a patient arrives in the emergency department with a possible nasal fracture. Which finding by the nurse is most important to report to the health care provider? a. Clear nasal drainage b. Complaint of nasal pain c. Bilateral nose swelling and bruising d. Inability to breathe through the nose
ANS: A Clear nasal drainage may indicate a meningeal tear with leakage of cerebrospinal fluid. This would place the patient at risk for complications such as meningitis. The other findings are typical with a nasal fracture and do not indicate any complications.
The nurse administers prescribed therapies for a patient with cor pulmonale and right-sided heart failure. Which assessment would best evaluate the effectiveness of the therapies? a. Observe for distended neck veins. b. Auscultate for crackles in the lungs. c. Palpate for heaves or thrills over the heart. d. Review hemoglobin and hematocrit values.
ANS: A Cor pulmonale is right ventricular failure caused by pulmonary hypertension, so clinical manifestations of right ventricular failure such as peripheral edema, jugular venous distention, and right upper-quadrant abdominal tenderness would be expected. Crackles in the lungs are likely to be heard with left-sided heart failure. Findings in cor pulmonale include evidence of right ventricular hypertrophy on electrocardiogram ECG and an increase in intensity of the second heart sound. Heaves or thrills are not common with cor pulmonale. Chronic hypoxemia leads to polycythemia and increased total blood volume and viscosity of the blood. The hemoglobin and hematocrit values are more likely to be elevated with cor pulmonale than decreased.
On auscultation of a patient's lungs, the nurse hears low-pitched, bubbling sounds during inhalation in the lower third of both lungs. How should the nurse document this finding? a. Inspiratory crackles at the bases b. Expiratory wheezes in both lungs c. Abnormal lung sounds in the apices of both lungs d. Pleural friction rub in the right and left lower lobes
ANS: A Crackles are low-pitched, bubbling sounds usually heard on inspiration. Wheezes are high-pitched sounds. They can be heard during the expiratory or inspiratory phase of the respiratory cycle. The lower third of both lungs are the bases, not apices. Pleural friction rubs are grating sounds that are usually heard during both inspiration and expiration
1. A 22-year-old patient is being seen in the clinic with increased secretion of the anterior pituitary hormones. The nurse would expect the laboratory results to show a. increased urinary cortisol. b. decreased serum thyroxine. c. elevated serum aldosterone levels. d. low urinary catecholamines excretion.
ANS: A Increased secretion of adrenocorticotropic hormone (ACTH) by the anterior pituitary gland will lead to an increase in serum and urinary cortisol levels. An increase, rather than a decrease, in thyroxine level would be expected with increased secretion of thyroid stimulating hormone (TSH) by the anterior pituitary. Aldosterone and catecholamine levels are not controlled by the anterior pituitary. DIF: Cognitive Level: Understand (comprehension) REF: 1149 TOP: Nursing Process: Assessment MSC: NCLEX: Physiological Integrity
The nurse assesses the chest of a patient with pneumococcal pneumonia. Which finding would the nurse expect? a. Increased tactile fremitus b. Dry, nonproductive cough c. Hyperresonance to percussion d. A grating sound on auscultation
ANS: A Increased tactile fremitus over the area of pulmonary consolidation is expected with bacterial pneumonias. Dullness to percussion would be expected. Pneumococcal pneumonia typically presents with a loose, productive cough. Adventitious breath sounds such as crackles and wheezes are typical. A grating sound is more representative of a pleural friction rub rather than pneumonia.
6. The nurse is caring for a mechanically ventilated patient with a cuffed tracheostomy tube. Which action by the nurse would best determine if the cuff has been properly inflated? a. Use a manometer to ensure cuff pressure is at an appropriate level. b. Check the amount of cuff pressure ordered by the health care provider. c. Suction the patient first with a fenestrated inner cannula to clear secretions. d. Insert the decannulation plug before the nonfenestrated inner cannula is removed.
ANS: A Measurement of cuff pressure using a manometer to ensure that cuff pressure is 20 mm Hg or lower will avoid compression of the tracheal wall and capillaries. Never insert the decannulation plug in a tracheostomy tube until the cuff is deflated and the nonfenestrated inner cannula is removed. Otherwise, the patient's airway is occluded. A health care provider's order is not required to determine safe cuff pressure. A nonfenestrated inner cannula must be used to suction a patient to prevent tracheal damage occurring from the suction catheter passing through the fenestrated openings.
An older patient is receiving standard multidrug therapy for tuberculosis (TB). The nurse should notify the health care provider if the patient exhibits which finding? a. Yellow-tinged skin b. Orange-colored sputum c. Thickening of the fingernails d. Difficulty hearing high-pitched voices
ANS: A Noninfectious hepatitis is a toxic effect of isoniazid (INH), rifampin, and pyrazinamide, and patients who develop hepatotoxicity will need to use other medications. Changes in hearing and nail thickening are not expected with the four medications used for initial TB drug therapy. Presbycusis is an expected finding in the older adult patient. Orange discoloration of body fluids is an expected side effect of rifampin and not an indication to call the health care provider.
18. The nurse is caring for a hospitalized older patient who has nasal packing in place to treat a nosebleed. Which assessment finding will require the most immediate action by the nurse? a. The oxygen saturation is 89%. b. The nose appears red and swollen. c. The patient's temperature is 100.1° F (37.8° C). d. The patient complains of level 8 (0 to 10 scale) pain.
ANS: A Older patients with nasal packing are at risk of aspiration or airway obstruction. An O2 saturation of 89% should alert the nurse to further assess for these complications. The other assessment data also indicate a need for nursing action but not as immediately as the low O2 saturation.
The nurse cares for a patient who has just had a thoracentesis. Which assessment information obtained by the nurse is a priority to communicate to the health care provider? a. Oxygen saturation is 88%. b. Blood pressure is 145/90 mm Hg. c. Respiratory rate is 22 breaths/minute when lying flat. d. Pain level is 5 (on 0 to 10 scale) with a deep breath.
ANS: A Oxygen saturation would be expected to improve after a thoracentesis. A saturation of 88% indicates that a complication such as pneumothorax may be occurring. The other assessment data also indicate a need for ongoing assessment or intervention, but the low oxygen saturation is the priority.
A patient experiences a chest wall contusion as a result of being struck in the chest with a baseball bat. The emergency department nurse would be most concerned if which finding is observed during the initial assessment? a. Paradoxic chest movement b. Complaint of chest wall pain c. Heart rate of 110 beats/minute d. Large bruised area on the chest
ANS: A Paradoxic chest movement indicates that the patient may have flail chest, which can severely compromise gas exchange and can rapidly lead to hypoxemia. Chest wall pain, a slightly elevated pulse rate, and chest bruising all require further assessment or intervention, but the priority concern is poor gas exchange.
8. A nurse obtains a health history from a patient who has a 35 pack-year smoking history. The patient complains of hoarseness and tightness in the throat and difficulty swallowing. Which question is most important for the nurse to ask? a. "How much alcohol do you drink in an average week?" b. "Do you have a family history of head or neck cancer?" c. "Have you had frequent streptococcal throat infections?" d. "Do you use antihistamines for upper airway congestion?"
ANS: A Prolonged alcohol use and smoking are associated with the development of laryngeal cancer, which the patient's symptoms and history suggest. Family history is not a risk factor for head or neck cancer. Frequent antihistamine use would be asked about if the nurse suspected allergic rhinitis, but the patient's symptoms are not suggestive of this diagnosis. Streptococcal throat infections also may cause these clinical manifestations, but patients with this type of infection will also have pain and a fever.
A client has been taking isoniazid for 2 months. The client c/o numbness, paresthesias, and tingling in the extremeties. The nurse interprets that the client is experiencing which problem?
Peripheral neuritis
A patient is admitted to the emergency department complaining of sudden onset shortness of breath and is diagnosed with a possible pulmonary embolus. How should the nurse prepare the patient for diagnostic testing to confirm the diagnosis? a. Start an IV so contrast media may be given. b. Ensure that the patient has been NPO for at least 6 hours. c. Inform radiology that radioactive glucose preparation is needed. d. Instruct the patient to undress to the waist and remove any metal objects.
ANS: A Spiral computed tomography (CT) scans are the most commonly used test to diagnose pulmonary emboli, and contrast media may be given IV. A chest x-ray may be ordered but will not be diagnostic for a pulmonary embolus. Preparation for a chest x-ray includes undressing and removing any metal. Bronchoscopy is used to detect changes in the bronchial tree, not to assess for vascular changes, and the patient should be NPO 6 to 12 hours before the procedure. Positron emission tomography (PET) scans are most useful in determining the presence of malignancy, and a radioactive glucose preparation is used.
12. Which action should the nurse take first when a patient develops a nosebleed? a. Pinch the lower portion of the nose for 10 minutes. b. Pack the affected nare tightly with an epistaxis balloon. c. Obtain silver nitrate that will be needed for cauterization. d. Apply ice compresses over the patient's nose and cheeks.
ANS: A The first nursing action for epistaxis is to apply direct pressure by pinching the nostrils. Application of cold packs may decrease blood flow to the area, but will not be sufficient to stop bleeding. Cauterization and nasal packing are medical interventions that may be needed if pressure to the nares does not stop the bleeding, but these are not the first actions to take for a nosebleed.
The nurse assesses a patient with chronic obstructive pulmonary disease (COPD) who has been admitted with increasing dyspnea over the last 3 days. Which finding is most important for the nurse to report to the health care provider? a. Respirations are 36 breaths/minute. b. Anterior-posterior chest ratio is 1:1. c. Lung expansion is decreased bilaterally. d. Hyperresonance to percussion is present.
ANS: A The increase in respiratory rate indicates respiratory distress and a need for rapid interventions such as administration of oxygen or medications. The other findings are common chronic changes occurring in patients with COPD.
15. Which patient in the ear, nose, and throat (ENT) clinic should the nurse assess first? a. A 23-year-old who is complaining of a sore throat and has a muffled voice b. A 34-year-old who has a "scratchy throat" and a positive rapid strep antigen test c. A 55-year-old who is receiving radiation for throat cancer and has severe fatigue d. A 72-year-old with a history of a total laryngectomy whose stoma is red and inflamed
ANS: A The patient's clinical manifestation of a muffled voice suggests a possible peritonsillar abscess that could lead to an airway obstruction requiring rapid assessment and potential treatment. The other patients do not have diagnoses or symptoms that indicate any life-threatening problems.
A patient who was admitted the previous day with pneumonia complains of a sharp pain of 7 (based on 0 to 10 scale) "whenever I take a deep breath." Which action will the nurse take next? a. Auscultate breath sounds. b. Administer the PRN morphine. c. Have the patient cough forcefully. d. Notify the patient's health care provider.
ANS: A The patient's statement indicates that pleurisy or a pleural effusion may have developed and the nurse will need to listen for a pleural friction rub and/or decreased breath sounds. Assessment should occur before administration of pain medications. The patient is unlikely to be able to cough forcefully until pain medication has been administered. The nurse will want to obtain more assessment data before calling the health care provider
11. The nurse completes discharge instructions for a patient with a total laryngectomy. Which statement by the patient indicates that additional instruction is needed? a. "I must keep the stoma covered with an occlusive dressing at all times." b. "I can participate in most of my prior fitness activities except swimming." c. "I should wear a Medic-Alert bracelet that identifies me as a neck breather." d. "I need to be sure that I have smoke and carbon monoxide detectors installed."
ANS: A The stoma may be covered with clothing or a loose dressing, but this is not essential. An occlusive dressing will completely block the patient's airway. The other patient comments are all accurate and indicate that the teaching has been effective.
Following assessment of a patient with pneumonia, the nurse identifies a nursing diagnosis of ineffective airway clearance. Which assessment data best supports this diagnosis? a. Weak, nonproductive cough effort b. Large amounts of greenish sputum c. Respiratory rate of 28 breaths/minute d. Resting pulse oximetry (SpO2) of 85%
ANS: A The weak, nonproductive cough indicates that the patient is unable to clear the airway effectively. The other data would be used to support diagnoses such as impaired gas exchange and ineffective breathing pattern.
The nurse is caring for a patient who has a right-sided chest tube after a right lower lobectomy. Which nursing action can the nurse delegate to the unlicensed assistive personnel (UAP)? a. Document the amount of drainage every eight hours. b. Obtain samples of drainage for culture from the system. c. Assess patient pain level associated with the chest tube. d. Check the water-seal chamber for the correct fluid level.
ANS: A UAP education includes documentation of intake and output. The other actions are within the scope of practice and education of licensed nursing personnel.
7. Which statement by the patient indicates that the teaching has been effective for a patient scheduled for radiation therapy of the larynx? a. "I will need to buy a water bottle to carry with me." b. "I should not use any lotions on my neck and throat." c. "Until the radiation is complete, I may have diarrhea." d. "Alcohol-based mouthwashes will help clean oral ulcers."
ANS: A Xerostomia can be partially alleviated by drinking fluids at frequent intervals. Radiation will damage tissues at the site being radiated but should not affect the abdominal organs, so loose stools are not a usual complication of head and neck radiation therapy. Frequent oral rinsing with non-alcohol-based rinses is recommended. Prescribed lotions and sunscreen may be used on radiated skin, although they should not be used just before the radiation therapy.
A patient who has just been admitted with community-acquired pneumococcal pneumonia has a temperature of 101.6° F with a frequent cough and is complaining of severe pleuritic chest pain. Which prescribed medication should the nurse give first? a. Codeine b. Guaifenesin (Robitussin) c. Acetaminophen (Tylenol) d. Piperacillin/tazobactam (Zosyn)
ANS: D Early initiation of antibiotic therapy has been demonstrated to reduce mortality. The other medications are also appropriate and should be given as soon as possible, but the priority is to start antibiotic therapy.
The nurse is educating a pregnant client who has gestational diabetes. Which of the following statements should the nurse make to the client? Select all that apply. a. Cakes, candies, cookies, and regular soft drinks should be avoided. b. Gestational diabetes increases the risk that the mother will develop diabetes later in life. c. Gestational diabetes usually resolves after the baby is born. d. Insulin injections may be necessary. e. The baby will likely be born with diabetes f. The mother should strive to gain no more weight during the pregnancy.
ANS: A, B, C, D Gestational diabetes can occur between the 16th and 28th week of pregnancy. If not responsive to diet and exercise, insulin injections may be necessary. Concentrated sugars should be avoided. Weight gain should continue, but not in excessive amounts. Usually, gestational diabetes disappears after the infant is born. However, diabetes can develop 5 to 10 years after the pregnancy.
Which factors will the nurse consider when calculating the CURB-65 score for a patient with pneumonia (select all that apply)? a. Age b. Blood pressure c. Respiratory rate d. Oxygen saturation e. Presence of confusion f. Blood urea nitrogen (BUN) level
ANS: A, B, C, E, F Data collected for the CURB-65 are mental status (confusion), BUN (elevated), blood pressure (decreased), respiratory rate (increased), and age (65 and older). The other information is also essential to assess, but are not used for CURB-65 scoring.
2. The nurse is reviewing the medical records for five patients who are scheduled for their yearly physical examinations in September. Which patients should receive the inactivated influenza vaccination (select all that apply)? a. A 76-year-old nursing home resident b. A 36-year-old female patient who is pregnant c. A 42-year-old patient who has a 15 pack-year smoking history d. A 30-year-old patient who takes corticosteroids for rheumatoid arthritis e. A 24-year-old patient who has allergies to penicillin and cephalosporins
ANS: A, B, D Current guidelines suggest that healthy individuals between 6 months and age 49 receive intranasal immunization with live, attenuated influenza vaccine. Individuals who are pregnant, residents of nursing homes, or are immunocompromised or who have chronic medical conditions should receive inactivated vaccine by injection. The corticosteroid use by the 30-year-old increases the risk for infection.
1. The clinic nurse is teaching a patient with acute sinusitis. Which interventions should the nurse plan to include in the teaching session (select all that apply)? a. Decongestants can be used to relieve swelling. b. Blowing the nose should be avoided to decrease the nosebleed risk. c. Taking a hot shower will increase sinus drainage and decrease pain. d. Saline nasal spray can be made at home and used to wash out secretions. e. You will be more comfortable if you keep your head in an upright position.
ANS: A, C, D, E The steam and heat from a shower will help thin secretions and improve drainage. Decongestants can be used to relieve swelling. Patients can use either over-the-counter (OTC) sterile saline solutions or home-prepared saline solutions to thin and remove secretions. Maintaining an upright posture decreases sinus pressure and the resulting pain. Blowing the nose after a hot shower or using the saline spray is recommended to expel secretions.
MULTIPLE RESPONSE 1. Which statements will the nurse include when teaching a patient who is scheduled for oral glucose tolerance testing in the outpatient clinic (select all that apply)? a. "You will need to avoid smoking before the test." b. "Exercise should be avoided until the testing is complete." c. "Several blood samples will be obtained during the testing." d. "You should follow a low-calorie diet the day before the test." e. "The test requires that you fast for at least 8 hours before testing."
ANS: A, C, E Smoking may affect the results of oral glucose tolerance tests. Blood samples are obtained at baseline and at 30, 60, and 120 minutes. Accuracy requires that the patient be fasting before the test. The patient should consume at least 1500 calories/day for 3 days before the test. The patient should be ambulatory and active for accurate test results. DIF: Cognitive Level: Apply (application) REF: 1150 TOP: Nursing Process: Planning MSC: NCLEX: Physiological Integrity
19. The nurse is caring for a 45-year-old male patient during a water deprivation test. Which finding is most important for the nurse to communicate to the health care provider? a. The patient complains of intense thirst. b. The patient has a 5-lb (2.3 kg) weight loss. c. The patient's urine osmolality does not increase. d. The patient feels dizzy when sitting on the edge of the bed.
ANS: B A drop in the weight of more than 2 kg indicates severe dehydration, and the test should be discontinued. The other assessment data are not unusual with this test. DIF: Cognitive Level: Apply (application) REF: 1148 OBJ: Special Questions: Prioritization TOP: Nursing Process: Assessment MSC: NCLEX: Physiological Integrity
When caring for a patient who is hospitalized with active tuberculosis (TB), the nurse observes a student nurse who is assigned to take care of a patient. Which action, if performed by the student nurse, would require an intervention by the nurse? a. The patient is offered a tissue from the box at the bedside. b. A surgical face mask is applied before visiting the patient. c. A snack is brought to the patient from the unit refrigerator. d. Hand washing is performed before entering the patient's room.
ANS: B A high-efficiency particulate-absorbing (HEPA) mask, rather than a standard surgical mask, should be used when entering the patient's room because the HEPA mask can filter out 100% of small airborne particles. Hand washing before entering the patient's room is appropriate. Because anorexia and weight loss are frequent problems in patients with TB, bringing food to the patient is appropriate. The student nurse should perform hand washing after handling a tissue that the patient has used, but no precautions are necessary when giving the patient an unused tissue.
A patient with idiopathic pulmonary arterial hypertension (IPAH) is receiving nifedipine (Procardia). Which assessment would best indicate to the nurse that the patient's condition is improving? a. Blood pressure (BP) is less than 140/90 mm Hg. b. Patient reports decreased exertional dyspnea. c. Heart rate is between 60 and 100 beats/minute. d. Patient's chest x-ray indicates clear lung fields.
ANS: B Because a major symptom of IPAH is exertional dyspnea, an improvement in this symptom would indicate that the medication was effective. Nifedipine will affect BP and heart rate, but these parameters would not be used to monitor the effectiveness of therapy for a patient with IPAH. The chest x-ray will show clear lung fields even if the therapy is not effective.
Which of the following communication methods is inappropriate for the pt with a total laryngectomy?
Placing a finger over the stoma
How does continuous positive airway pressure (CPAP) work?
Pressure is delivered continuously during spontaneous breathing, for both inspiration and expiration.
After the nurse has received change-of-shift report, which patient should the nurse assess first? a. A patient with pneumonia who has crackles in the right lung base b. A patient with possible lung cancer who has just returned after bronchoscopy c. A patient with hemoptysis and a 16-mm induration with tuberculin skin testing d. A patient with chronic obstructive pulmonary disease (COPD) and pulmonary function testing (PFT) that indicates low forced vital capacity
ANS: B Because the cough and gag are decreased after bronchoscopy, this patient should be assessed for airway patency. The other patients do not have clinical manifestations or procedures that require immediate assessment by the nurse.
20. A 35-year-old female patient with a possible pituitary adenoma is scheduled for a computed tomography (CT) scan with contrast media. Which patient information is most important for the nurse to communicate to the health care provider before the test? a. Bilateral poor peripheral vision b. Allergies to iodine and shellfish c. Recent weight loss of 20 pounds d. Complaint of ongoing headaches
ANS: B Because the usual contrast media is iodine-based, the health care provider will need to know about the allergy before the CT scan. The other findings are common with any mass in the brain such as a pituitary adenoma. DIF: Cognitive Level: Apply (application) REF: 1148 OBJ: Special Questions: Prioritization TOP: Nursing Process: Assessment MSC: NCLEX: Physiological Integrity
17. Which information about a 30-year-old patient who is scheduled for an oral glucose tolerance test should be reported to the health care provider before starting the test? a. The patient reports having occasional orthostatic dizziness. b. The patient takes oral corticosteroids for rheumatoid arthritis. c. The patient has had a 10-pound weight gain in the last month. d. The patient drank several glasses of water an hour previously.
ANS: B Corticosteroids can affect blood glucose results. The other information will be provided to the health care provider but will not affect the test results. DIF: Cognitive Level: Apply (application) REF: 1142 OBJ: Special Questions: Prioritization TOP: Nursing Process: Assessment MSC: NCLEX: Physiological Integrity
3. A 30-year-old patient seen in the emergency department for severe headache and acute confusion is found to have a serum sodium level of 118 mEq/L. The nurse will anticipate the need for which diagnostic test? a. Urinary 17-ketosteroids b. Antidiuretic hormone level c. Growth hormone stimulation test d. Adrenocorticotropic hormone level
ANS: B Elevated levels of antidiuretic hormone will cause water retention and decrease serum sodium levels. The other tests would not be helpful in determining possible causes of the patient's hyponatremia. DIF: Cognitive Level: Apply (application) REF: 1136 TOP: Nursing Process: Planning MSC: NCLEX: Physiological Integrity
An experienced nurse instructs a new nurse about how to care for a patient with dyspnea caused by a pulmonary fungal infection. Which action by the new nurse indicates a need for further teaching? a. Listening to the patient's lung sounds several times during the shift b. Placing the patient on droplet precautions and in a private hospital room c. Increasing the oxygen flow rate to keep the oxygen saturation above 90% d. Monitoring patient serology results to identify the specific infecting organism
ANS: B Fungal infections are not transmitted from person to person. Therefore no isolation procedures are necessary. The other actions by the new nurse are appropriate
A patient has just been admitted with probable bacterial pneumonia and sepsis. Which order should the nurse implement first? a. Chest x-ray via stretcher b. Blood cultures from two sites c. Ciprofloxacin (Cipro) 400 mg IV d. Acetaminophen (Tylenol) rectal suppository
ANS: B Initiating antibiotic therapy rapidly is essential, but it is important that the cultures be obtained before antibiotic administration. The chest x-ray and acetaminophen administration can be done last.
A diabetic patient's arterial blood gas (ABG) results are pH 7.28; PaCO2 34 mm Hg; PaO2 85 mm Hg; HCO3- 18 mEq/L. The nurse would expect which finding? a. Intercostal retractions b. Kussmaul respirations c. Low oxygen saturation (SpO2) d. Decreased venous O2 pressure
ANS: B Kussmaul (deep and rapid) respirations are a compensatory mechanism for metabolic acidosis. The low pH and low bicarbonate result indicate metabolic acidosis. Intercostal retractions, a low oxygen saturation rate, and a decrease in venous O2 pressure would not be caused by acidosis.
Which action is appropriate for the nurse to delegate to unlicensed assistive personnel (UAP)? a. Listen to a patient's lung sounds for wheezes or rhonchi. b. Label specimens obtained during percutaneous lung biopsy. c. Instruct a patient about how to use home spirometry testing. d. Measure induration at the site of a patient's intradermal skin test.
ANS: B Labeling of specimens is within the scope of practice of UAP. The other actions require nursing judgment and should be done by licensed nursing personnel.
18. Which action by a new registered nurse (RN) caring for a patient with a goiter and possible hyperthyroidism indicates that the charge nurse needs to do more teaching? a. The RN checks the blood pressure on both arms. b. The RN palpates the neck thoroughly to check thyroid size. c. The RN lowers the thermostat to decrease the temperature in the room. d. The RN orders nonmedicated eye drops to lubricate the patient's bulging eyes.
ANS: B Palpation can cause the release of thyroid hormones in a patient with an enlarged thyroid and should be avoided. The other actions by the new RN are appropriate when caring for a patient with an enlarged thyroid. DIF: Cognitive Level: Apply (application) REF: 1144 OBJ: Special Questions: Delegation TOP: Nursing Process: Evaluation MSC: NCLEX: Safe and Effective Care Environment
The nurse receives change-of-shift report on the following four patients. Which patient should the nurse assess first? a. A 23-year-old patient with cystic fibrosis who has pulmonary function testing scheduled b. A 46-year-old patient on bed rest who is complaining of sudden onset of shortness of breath c. A 77-year-old patient with tuberculosis (TB) who has four antitubercular medications due in 15 minutes d. A 35-year-old patient who was admitted the previous day with pneumonia and has a temperature of 100.2° F (37.8° C)
ANS: B Patients on bed rest who are immobile are at high risk for deep vein thrombosis (DVT). Sudden onset of shortness of breath in a patient with a DVT suggests a pulmonary embolism and requires immediate assessment and action such as oxygen administration. The other patients should also be assessed as soon as possible, but there is no indication that they may need immediate action to prevent clinical deterioration.
The nurse knows that the pt understands teaching related to prevention of influenza transmission when the pt demonstrates which behaviors?
Washing hands frequently; Covering the nose and mouth when sneezing or coughing; Avoid sharing eating utensils with others
The nurse provides discharge instructions to a patient who was hospitalized for pneumonia. Which statement, if made by the patient, indicates a good understanding of the instructions? a. "I will call the doctor if I still feel tired after a week." b. "I will continue to do the deep breathing and coughing exercises at home." c. "I will schedule two appointments for the pneumonia and influenza vaccines." d. "I'll cancel my chest x-ray appointment if I'm feeling better in a couple weeks."
ANS: B Patients should continue to cough and deep breathe after discharge. Fatigue is expected for several weeks. The Pneumovax and influenza vaccines can be given at the same time in different arms. Explain that a follow-up chest x-ray needs to be done in 6 to 8 weeks to evaluate resolution of pneumonia
A patient with a chronic cough has a bronchoscopy. After the procedure, which intervention by the nurse is most appropriate? a. Elevate the head of the bed to 80 to 90 degrees. b. Keep the patient NPO until the gag reflex returns. c. Place on bed rest for at least 4 hours after bronchoscopy. d. Notify the health care provider about blood-tinged mucus.
ANS: B Risk for aspiration and maintaining an open airway is the priority. Because a local anesthetic is used to suppress the gag/cough reflexes during bronchoscopy, the nurse should monitor for the return of these reflexes before allowing the patient to take oral fluids or food. Blood-tinged mucus is not uncommon after bronchoscopy. The patient does not need to be on bed rest, and the head of the bed does not need to be in the high-Fowler's position.
17. Which nursing action could the registered nurse (RN) working in a skilled care hospital unit delegate to an experienced licensed practical/vocational nurse (LPN/LVN) caring for a patient with a permanent tracheostomy? a. Assess the patient's risk for aspiration. b. Suction the tracheostomy when needed. c. Teach the patient about self-care of the tracheostomy. d. Determine the need for replacement of the tracheostomy tube.
ANS: B Suctioning of a stable patient can be delegated to LPNs/LVNs. Patient assessment and patient teaching should be done by the RN.
The nurse teaches a patient about the transmission of pulmonary tuberculosis (TB). Which statement, if made by the patient, indicates that teaching was effective? a. "I will avoid being outdoors whenever possible." b. "My husband will be sleeping in the guest bedroom." c. "I will take the bus instead of driving to visit my friends." d. "I will keep the windows closed at home to contain the germs."
ANS: B Teach the patient how to minimize exposure to close contacts and household members. Homes should be well ventilated, especially the areas where the infected person spends a lot of time. While still infectious, the patient should sleep alone, spend as much time as possible outdoors, and minimize time in congregate settings or on public transportation.
16. The nurse obtains the following assessment data on an older patient who has influenza. Which information will be most important for the nurse to communicate to the health care provider? a. Fever of 100.4° F (38° C) b. Diffuse crackles in the lungs c. Sore throat and frequent cough d. Myalgia and persistent headache
ANS: B The crackles indicate that the patient may be developing pneumonia, a common complication of influenza, which would require aggressive treatment. Myalgia, headache, mild temperature elevation, and sore throat with cough are typical manifestations of influenza and are treated with supportive care measures such as over-the-counter (OTC) pain relievers and increased fluid intake.
A patient is admitted to the emergency department with an open stab wound to the left chest. What is the first action that the nurse should take? a. Position the patient so that the left chest is dependent. b. Tape a nonporous dressing on three sides over the chest wound. c. Cover the sucking chest wound firmly with an occlusive dressing. d. Keep the head of the patient's bed at no more than 30 degrees elevation.
ANS: B The dressing taped on three sides will allow air to escape when intrapleural pressure increases during expiration, but it will prevent air from moving into the pleural space during inspiration. Placing the patient on the left side or covering the chest wound with an occlusive dressing will allow trapped air in the pleural space and cause tension pneumothorax. The head of the bed should be elevated to 30 to 45 degrees to facilitate breathing
14. Following a laryngectomy a patient coughs violently during suctioning and dislodges the tracheostomy tube. Which action should the nurse take first? a. Cover stoma with sterile gauze and ventilate through stoma. b. Attempt to reinsert the tracheostomy tube with the obturator in place. c. Assess the patient's oxygen saturation and notify the health care provider. d. Ventilate the patient with a manual bag and face mask until the health care provider arrives.
ANS: B The first action should be to attempt to reinsert the tracheostomy tube to maintain the patient's airway. Assessing the patient's oxygenation is an important action, but it is not the most appropriate first action in this situation. Covering the stoma with a dressing and manually ventilating the patient may be an appropriate action if the nurse is unable to reinsert the tracheostomy tube. Ventilating with a facemask is not appropriate for a patient with a total laryngectomy because there is a complete separation between the upper airway and the trachea.
After 2 months of tuberculosis (TB) treatment with isoniazid (INH), rifampin (Rifadin), pyrazinamide (PZA), and ethambutol, a patient continues to have positive sputum smears for acid-fast bacilli (AFB). Which action should the nurse take next? a. Teach about treatment for drug-resistant TB treatment. b. Ask the patient whether medications have been taken as directed. c. Schedule the patient for directly observed therapy three times weekly. d. Discuss with the health care provider the need for the patient to use an injectable antibiotic.
ANS: B The first action should be to determine whether the patient has been compliant with drug therapy because negative sputum smears would be expected if the TB bacillus is susceptible to the medications and if the medications have been taken correctly. Assessment is the first step in the nursing process. Depending on whether the patient has been compliant or not, different medications or directly observed therapy may be indicated. The other options are interventions based on assumptions until an assessment has been completed.
The nurse is preparing a list of home care instructions for the client who has been hospitalized and treated for TB. Which instructions should the nurse reinforce? (SATA)
Activities should be resumed gradually; A sputum culture is needed every 2-4 weeks once medication therapy is initiated; Respiratory isolation is not necessary because family members have already been exposed; Cover the mouth and nose when coughing or sneezing and confine used tissues to plastic bags
A pt with recurrent pneumothorax is scheduled to have pleurodesis done 1 hour. Which nursing intervention should take priority at this time?
Administer a prn analgesic as ordered
The nurse monitors a patient after chest tube placement for a hemopneumothorax. The nurse is most concerned if which assessment finding is observed? a. A large air leak in the water-seal chamber b. 400 mL of blood in the collection chamber c. Complaint of pain with each deep inspiration d. Subcutaneous emphysema at the insertion site
ANS: B The large amount of blood may indicate that the patient is in danger of developing hypovolemic shock. An air leak would be expected immediately after chest tube placement for a pneumothorax. Initially, brisk bubbling of air occurs in this chamber when a pneumothorax is evacuated. The pain should be treated but is not as urgent a concern as the possibility of continued hemorrhage. Subcutaneous emphysema should be monitored but is not unusual in a patient with pneumothorax. A small amount of subcutaneous air is harmless and will be reabsorbed.
A patient with newly diagnosed lung cancer tells the nurse, "I don't think I'm going to live to see my next birthday." Which response by the nurse is best? a. "Would you like to talk to the hospital chaplain about your feelings?" b. "Can you tell me what it is that makes you think you will die so soon?" c. "Are you afraid that the treatment for your cancer will not be effective?" d. "Do you think that taking an antidepressant medication would be helpful?"
ANS: B The nurse's initial response should be to collect more assessment data about the patient's statement. The answer beginning "Can you tell me what it is" is the most open-ended question and will offer the best opportunity for obtaining more data. The answer beginning, "Are you afraid" implies that the patient thinks that the cancer will be immediately fatal, although the patient's statement may not be related to the cancer diagnosis. The remaining two answers offer interventions that may be helpful to the patient, but more assessment is needed to determine whether these interventions are appropriate.
22. When assessing a patient with a sore throat, the nurse notes anterior cervical lymph node swelling, a temperature of 101.6° F (38.7° C), and yellow patches on the tonsils. Which action will the nurse anticipate taking? a. Teach the patient about the use of expectorants. b. Use a swab to obtain a sample for a rapid strep antigen test. c. Discuss the need to rinse the mouth out after using any inhalers. d. Teach the patient to avoid use of nonsteroidal antiinflammatory drugs (NSAIDs).
ANS: B The patient's clinical manifestations are consistent with streptococcal pharyngitis and the nurse will anticipate the need for a rapid strep antigen test and/or cultures. Because patients with streptococcal pharyngitis usually do not have a cough, use of expectorants will not be anticipated. Rinsing the mouth out after inhaler use may prevent fungal oral infections, but the patient's assessment data are not consistent with a fungal infection. NSAIDs are frequently prescribed for pain and fever relief with pharyngitis.
The nurse develops a plan of care to prevent aspiration in a high-risk patient. Which nursing action will be most effective? a. Turn and reposition immobile patients at least every 2 hours. b. Place patients with altered consciousness in side-lying positions. c. Monitor for respiratory symptoms in patients who are immunosuppressed. d. Insert nasogastric tube for feedings for patients with swallowing problems.
ANS: B The risk for aspiration is decreased when patients with a decreased level of consciousness are placed in a side-lying or upright position. Frequent turning prevents pooling of secretions in immobilized patients but will not decrease the risk for aspiration in patients at risk. Monitoring of parameters such as breath sounds and oxygen saturation will help detect pneumonia in immunocompromised patients, but it will not decrease the risk for aspiration. Conditions that increase the risk of aspiration include decreased level of consciousness (e.g., seizure, anesthesia, head injury, stroke, alcohol intake), difficulty swallowing, and nasogastric intubation with or without tube feeding. With loss of consciousness, the gag and cough reflexes are depressed, and aspiration is more likely to occur. Other high-risk groups are those who are seriously ill, have poor dentition, or are receiving acid-reducing medications.
14. A nurse will teach a patient who is scheduled to complete a 24-hour urine collection for 17-ketosteroids to a. insert and maintain a retention catheter. b. keep the specimen refrigerated or on ice. c. drink at least 3 L of fluid during the 24 hours. d. void and save that specimen to start the collection.
ANS: B The specimen must be kept on ice or refrigerated until the collection is finished. Voided or catheterized specimens are acceptable for the test. The initial voided specimen is discarded. There is no fluid intake requirement for the 24-hour collection. DIF: Cognitive Level: Apply (application) REF: 1150 TOP: Nursing Process: Implementation MSC: NCLEX: Physiological Integrity
7. During the physical examination of a 36-year-old female, the nurse finds that the patient's thyroid gland cannot be palpated. The most appropriate action by the nurse is to a. palpate the patient's neck more deeply. b. document that the thyroid was nonpalpable. c. notify the health care provider immediately. d. teach the patient about thyroid hormone testing.
ANS: B The thyroid is frequently nonpalpable. The nurse should simply document the finding. There is no need to notify the health care provider immediately about a normal finding. There is no indication for thyroid-stimulating hormone (TSH) testing unless there is evidence of thyroid dysfunction. Deep palpation of the neck is not appropriate. DIF: Cognitive Level: Apply (application) REF: 1145 TOP: Nursing Process: Assessment MSC: NCLEX: Physiological Integrity
A patient with acute shortness of breath is admitted to the hospital. Which action should the nurse take during the initial assessment of the patient? a. Ask the patient to lie down to complete a full physical assessment. b. Briefly ask specific questions about this episode of respiratory distress. c. Complete the admission database to check for allergies before treatment. d. Delay the physical assessment to first complete pulmonary function tests.
ANS: B When a patient has severe respiratory distress, only information pertinent to the current episode is obtained, and a more thorough assessment is deferred until later. Obtaining a comprehensive health history or full physical examination is unnecessary until the acute distress has resolved. Brief questioning and a focused physical assessment should be done rapidly to help determine the cause of the distress and suggest treatment. Checking for allergies is important, but it is not appropriate to complete the entire admission database at this time. The initial respiratory assessment must be completed before any diagnostic tests or interventions can be ordered.
When caring for a patient who is 3 hours postoperative laryngectomy, what is the nurse's highest priority assessment? Patient comfort Airway patency Incisional drainage Blood pressure and heart rate
Airway patency Remember the ABCs with prioritization. Airway patency is always the highest priority and is essential for a patient undergoing surgery surrounding the upper respiratory system. Comfort, drainage, and vital signs follow the ABCs in priority.
11. A 40-year-old male patient has been newly diagnosed with type 2 diabetes mellitus. Which information about the patient will be most useful to the nurse who is helping the patient develop strategies for successful adaptation to this disease? a. Ideal weight b. Value system c. Activity level d. Visual changes
ANS: B When dealing with a patient with a chronic condition such as diabetes, identification of the patient's values and beliefs can assist the health care team in choosing strategies for successful lifestyle change. The other information also will be useful, but is not as important in developing an individualized plan for the necessary lifestyle changes. DIF: Cognitive Level: Apply (application) REF: 1143-1144 TOP: Nursing Process: Assessment MSC: NCLEX: Psychosocial Integrity
A patient with a pleural effusion is scheduled for a thoracentesis. Which action should the nurse take to prepare the patient for the procedure? a. Start a peripheral IV line to administer the necessary sedative drugs. b. Position the patient sitting upright on the edge of the bed and leaning forward. c. Obtain a large collection device to hold 2 to 3 liters of pleural fluid at one time. d. Remove the water pitcher and remind the patient not to eat or drink anything for 6 hours.
ANS: B When the patient is sitting up, fluid accumulates in the pleural space at the lung bases and can more easily be located and removed. The patient does not usually require sedation for the procedure, and there are no restrictions on oral intake because the patient is not sedated or unconscious. Usually only 1000 to 1200 mL of pleural fluid is removed at one time. Rapid removal of a large volume can result in hypotension, hypoxemia, or pulmonary edema.
A patient is scheduled for a computed tomography (CT) of the chest with contrast media. Which assessment findings should the nurse immediately report to the health care provider (select all that apply)? a. Patient is claustrophobic. b. Patient is allergic to shellfish. c. Patient recently used a bronchodilator inhaler. d. Patient is not able to remove a wedding band. e. Blood urea nitrogen (BUN) and serum creatinine levels are elevated.
ANS: B, E Because the contrast media is iodine-based and may cause dehydration and decreased renal blood flow, asking about iodine allergies (such as allergy to shellfish) and monitoring renal function before the CT scan are necessary. The other actions are not contraindications for CT of the chest, although they may be for other diagnostic tests, such as magnetic resonance imaging (MRI) or pulmonary function testing (PFT).
8. Which laboratory value should the nurse review to determine whether a patient's hypothyroidism is caused by a problem with the anterior pituitary gland or with the thyroid gland? a. Thyroxine (T4) level b. Triiodothyronine (T3) level c. Thyroid-stimulating hormone (TSH) level d. Thyrotropin-releasing hormone (TRH) level
ANS: C A low TSH level indicates that the patient's hypothyroidism is caused by decreased anterior pituitary secretion of TSH. Low T3 and T4 levels are not diagnostic of the primary cause of the hypothyroidism. TRH levels indicate the function of the hypothalamus. DIF: Cognitive Level: Apply (application) REF: 1151 TOP: Nursing Process: Assessment MSC: NCLEX: Physiological Integrity
The nurse notes that a patient has incisional pain, a poor cough effort, and scattered rhonchi after a thoracotomy. Which action should the nurse take first? a. Assist the patient to sit upright in a chair. b. Splint the patient's chest during coughing. c. Medicate the patient with prescribed morphine. d. Observe the patient use the incentive spirometer.
ANS: C A major reason for atelectasis and poor airway clearance in patients after chest surgery is incisional pain (which increases with deep breathing and coughing). The first action by the nurse should be to medicate the patient to minimize incisional pain. The other actions are all appropriate ways to improve airway clearance but should be done after the morphine is given.
A patient in metabolic alkalosis is admitted to the emergency department, and pulse oximetry (SpO2) indicates that the O2 saturation is 94%. Which action should the nurse take next? a. Administer bicarbonate. b. Complete a head-to-toe assessment. c. Place the patient on high-flow oxygen. d. Obtain repeat arterial blood gases (ABGs).
ANS: C Although the O2 saturation is adequate, the left shift in the oxyhemoglobin dissociation curve will decrease the amount of oxygen delivered to tissues, so high oxygen concentrations should be given. Bicarbonate would worsen the patient's condition. A head-to-toe assessment and repeat ABGs may be implemented. However, the priority intervention is to give high-flow oxygen.
10. A 60-year-old patient is taking spironolactone (Aldactone), a drug that blocks the action of aldosterone on the kidney, for hypertension. The nurse will monitor for a. increased serum sodium. b. decreased urinary output. c. elevated serum potassium. d. evidence of fluid overload.
ANS: C Because aldosterone increases the excretion of potassium, a medication that blocks aldosterone will tend to cause hyperkalemia. Aldosterone also promotes the reabsorption of sodium and water in the renal tubules, so spironolactone will tend to cause increased urine output, a decreased or normal serum sodium level, and signs of dehydration. DIF: Cognitive Level: Apply (application) REF: 1140 TOP: Nursing Process: Evaluation MSC: NCLEX: Physiological Integrity
5. A patient with a tracheostomy has a new order for a fenestrated tracheostomy tube. Which action should the nurse include in the plan of care in collaboration with the speech therapist? a. Leave the tracheostomy inner cannula inserted at all times. b. Place the decannulation cap in the tube before cuff deflation. c. Assess the ability to swallow before using the fenestrated tube. d. Inflate the tracheostomy cuff during use of the fenestrated tube.
ANS: C Because the cuff is deflated when using a fenestrated tube, the patient's risk for aspiration should be assessed before changing to a fenestrated tracheostomy tube. The decannulation cap is never inserted before cuff deflation because to do so would obstruct the patient's airway. The cuff is deflated and the inner cannula removed to allow air to flow across the patient's vocal cords when using a fenestrated tube.
4. Which question will provide the most useful information to a nurse who is interviewing a patient about a possible thyroid disorder? a. "What methods do you use to help cope with stress?" b. "Have you experienced any blurring or double vision?" c. "Have you had a recent unplanned weight gain or loss?" d. "Do you have to get up at night to empty your bladder?"
ANS: C Because thyroid function affects metabolic rate, changes in weight may indicate hyperfunction or hypofunction of the thyroid gland. Nocturia, visual difficulty, and changes in stress level are associated with other endocrine disorders. DIF: Cognitive Level: Apply (application) REF: 1143 TOP: Nursing Process: Assessment MSC: NCLEX: Physiological Integrity
A patient who has a right-sided chest tube following a thoracotomy has continuous bubbling in the suction-control chamber of the collection device. Which action by the nurse is most appropriate? a. Document the presence of a large air leak. b. Notify the surgeon of a possible pneumothorax. c. Take no further action with the collection device. d. Adjust the dial on the wall regulator to decrease suction.
ANS: C Continuous bubbling is expected in the suction-control chamber and indicates that the suction-control chamber is connected to suction. An air leak would be detected in the water-seal chamber. There is no evidence of pneumothorax. Increasing or decreasing the vacuum source will not adjust the suction pressure. The amount of suction applied is regulated by the amount of water in this chamber and not by the amount of suction applied to the system.
5. A 29-year-old patient in the outpatient clinic will be scheduled for blood cortisol testing. Which instruction will the nurse provide? a. "Avoid adding any salt to your foods for 24 hours before the test." b. "You will need to lie down for 30 minutes before the blood is drawn." c. "Come to the laboratory to have the blood drawn early in the morning." d. "Do not have anything to eat or drink before the blood test is obtained."
ANS: C Cortisol levels are usually drawn in the morning, when levels are highest. The other instructions would be given to patients who were having other endocrine testing. DIF: Cognitive Level: Apply (application) REF: 1149 TOP: Nursing Process: Implementation MSC: NCLEX: Physiological Integrity
A patient has acute bronchitis with a nonproductive cough and wheezes. Which topic should the nurse plan to include in the teaching plan? a. Purpose of antibiotic therapy b. Ways to limit oral fluid intake c. Appropriate use of cough suppressants d. Safety concerns with home oxygen therapy
ANS: C Cough suppressants are frequently prescribed for acute bronchitis. Because most acute bronchitis is viral in origin, antibiotics are not prescribed unless there are systemic symptoms. Fluid intake is encouraged. Home oxygen is not prescribed for acute bronchitis, although it may be used for chronic bronchitis.
A patient is diagnosed with both human immunodeficiency virus (HIV) and active tuberculosis (TB) disease. Which information obtained by the nurse is most important to communicate to the health care provider? a. The Mantoux test had an induration of 7 mm. b. The chest-x-ray showed infiltrates in the lower lobes. c. The patient is being treated with antiretrovirals for HIV infection. d. The patient has a cough that is productive of blood-tinged mucus.
ANS: C Drug interactions can occur between the antiretrovirals used to treat HIV infection and the medications used to treat TB. The other data are expected in a patient with HIV and TB.
The nurse completes a shift assessment on a patient admitted in the early phase of heart failure. When auscultating the patient's lungs, which finding would the nurse most likely hear? a. Continuous rumbling, snoring, or rattling sounds mainly on expiration b. Continuous high-pitched musical sounds on inspiration and expiration c. Discontinuous, high-pitched sounds of short duration heard on inspiration d. A series of long-duration, discontinuous, low-pitched sounds during inspiration
ANS: C Fine crackles are likely to be heard in the early phase of heart failure. Fine crackles are discontinuous, high-pitched sounds of short duration heard on inspiration. Rhonchi are continuous rumbling, snoring, or rattling sounds mainly on expiration. Course crackles are a series of long-duration, discontinuous, low-pitched sounds during inspiration. Wheezes are continuous high-pitched musical sounds on inspiration and expiration.
The nurse teaches a patient about pulmonary function testing (PFT). Which statement, if made by the patient, indicates teaching was effective? a. "I will use my inhaler right before the test." b. "I won't eat or drink anything 8 hours before the test." c. "I should inhale deeply and blow out as hard as I can during the test." d. "My blood pressure and pulse will be checked every 15 minutes after the test."
ANS: C For PFT, the patient should inhale deeply and exhale as long, hard, and fast as possible. The other actions are not needed with PFT. The administration of inhaled bronchodilators should be avoided 6 hours before the procedure.
The nurse provides preoperative instruction for a patient scheduled for a left pneumonectomy for cancer of the lung. Which information should the nurse include about the patient's postoperative care? a. Positioning on the right side b. Bed rest for the first 24 hours c. Frequent use of an incentive spirometer d. Chest tube placement with continuous drainage
ANS: C Frequent deep breathing and coughing are needed after chest surgery to prevent atelectasis. To promote gas exchange, patients after pneumonectomy are positioned on the surgical side. Early mobilization decreases the risk for postoperative complications such as pneumonia and deep vein thrombosis. In a pneumonectomy, chest tubes may or may not be placed in the space from which the lung was removed. If a chest tube is used, it is clamped and only released by the surgeon to adjust the volume of serosanguineous fluid that will fill the space vacated by the lung. If the cavity overfills, it could compress the remaining lung and compromise the cardiovascular and pulmonary function. Daily chest x-rays can be used to assess the volume and space.
9. The nurse reviews a patient's glycosylated hemoglobin (Hb A1C) results to evaluate a. fasting preprandial glucose levels. b. glucose levels 2 hours after a meal. c. glucose control over the past 90 days. d. hypoglycemic episodes in the past 3 months.
ANS: C Glycosylated hemoglobin testing measures glucose control over the last 3 months. Glucose testing before/after a meal or random testing may reveal impaired glucose tolerance and indicate prediabetes, but it is not done on patients who already have a diagnosis of diabetes. There is no test to evaluate for hypoglycemic episodes in the past. DIF: Cognitive Level: Understand (comprehension) REF: 1150 TOP: Nursing Process: Evaluation MSC: NCLEX: Physiological Integrity
10. A patient who had a total laryngectomy has a nursing diagnosis of hopelessness related to loss of control of personal care. Which information obtained by the nurse is the best indicator that this identified problem is resolving? a. The patient lets the spouse provide tracheostomy care. b. The patient allows the nurse to suction the tracheostomy. c. The patient asks how to clean the tracheostomy stoma and tube. d. The patient uses a communication board to request "No Visitors."
ANS: C Independently caring for the laryngectomy tube indicates that the patient has regained control of personal care and hopelessness is at least partially resolved. Letting the nurse and spouse provide care and requesting no visitors may indicate that the patient is still experiencing hopelessness.
The nurse observes a student who is listening to a patient's lungs who is having no problems with breathing. Which action by the student indicates a need to review respiratory assessment skills? a. The student starts at the apices of the lungs and moves to the bases. b. The student compares breath sounds from side to side avoiding bony areas. c. The student places the stethoscope over the posterior chest and listens during inspiration. d. The student instructs the patient to breathe slowly and a little more deeply than normal through the mouth.
ANS: C Listening only during inspiration indicates the student needs a review of respiratory assessment skills. At each placement of the stethoscope, listen to at least one cycle of inspiration and expiration. During chest auscultation, instruct the patient to breathe slowly and a little deeper than normal through the mouth. Auscultation should proceed from the lung apices to the bases, comparing opposite areas of the chest, unless the patient is in respiratory distress or will tire easily. If so, start at the bases (see Fig. 26-7). Place the stethoscope over lung tissue, not over bony prominences.
A patient who is taking rifampin (Rifadin) for tuberculosis calls the clinic and reports having orange discolored urine and tears. Which is the best response by the nurse? a. Ask if the patient is experiencing shortness of breath, hives, or itching. b. Ask the patient about any visual abnormalities such as red-green color discrimination. c. Explain that orange discolored urine and tears are normal while taking this medication. d. Advise the patient to stop the drug and report the symptoms to the health care provider.
ANS: C Orange-colored body secretions are a side effect of rifampin. The patient does not have to stop taking the medication. The findings are not indicative of an allergic reaction. Alterations in red-green color discrimination commonly occurs when taking ethambutol (Myambutol), which is a different TB medication.
15. Which additional information will the nurse need to consider when reviewing the laboratory results for a patient's total calcium level? a. The blood glucose is elevated. b. The phosphate level is normal. c. The serum albumin level is low. d. The magnesium level is normal.
ANS: C Part of the total calcium is bound to albumin so hypoalbuminemia can lead to misinterpretation of calcium levels. The other laboratory values will not affect total calcium interpretation. DIF: Cognitive Level: Apply (application) REF: 1149 TOP: Nursing Process: Assessment MSC: NCLEX: Physiological Integrity
13. The nurse will teach a patient to plan to minimize physical and emotional stress while the patient is undergoing a. a water deprivation test. b. testing for serum T3 and T4 levels. c. a 24-hour urine test for free cortisol. d. a radioactive iodine (I-131) uptake test.
ANS: C Physical and emotional stress can affect the results of the free cortisol test. The other tests are not impacted by stress. DIF: Cognitive Level: Apply (application) REF: 1150 TOP: Nursing Process: Planning MSC: NCLEX: Physiological Integrity
An occupational health nurse works at a manufacturing plant where there is potential exposure to inhaled dust. Which action, if recommended by the nurse, will be most helpful in reducing the incidence of lung disease? a. Treat workers with pulmonary fibrosis. b. Teach about symptoms of lung disease. c. Require the use of protective equipment. d. Monitor workers for coughing and wheezing.
ANS: C Prevention of lung disease requires the use of appropriate protective equipment such as masks. The other actions will help in recognition or early treatment of lung disease but will not be effective in prevention of lung damage. Repeated exposure eventually results in diffuse pulmonary fibrosis. Fibrosis is the result of tissue repair after inflammation.
The health care provider writes an order for bacteriologic testing for a patient who has a positive tuberculosis skin test. Which action should the nurse take? a. Teach about the reason for the blood tests. b. Schedule an appointment for a chest x-ray. c. Teach about the need to get sputum specimens for 2 to 3 consecutive days. d. Instruct the patient to expectorate three specimens as soon as possible.
ANS: C Sputum specimens are obtained on 2 to 3 consecutive days for bacteriologic testing for M. tuberculosis. The patient should not provide all the specimens at once. Blood cultures are not used for tuberculosis testing. A chest x-ray is not bacteriologic testing. Although the findings on chest x-ray examination are important, it is not possible to make a diagnosis of TB solely based on chest x-ray findings because other diseases can mimic the appearance of TB.
While caring for a patient with respiratory disease, the nurse observes that the patient's SpO2 drops from 93% to 88% while the patient is ambulating in the hallway. What is the priority action of the nurse? a. Notify the health care provider. b. Document the response to exercise. c. Administer the PRN supplemental O2. d. Encourage the patient to pace activity.
ANS: C The drop in SpO2 to 85% indicates that the patient is hypoxemic and needs supplemental oxygen when exercising. The other actions are also important, but the first action should be to correct the hypoxemia.
The nurse is caring for a patient with idiopathic pulmonary arterial hypertension (IPAH) who is receiving epoprostenol (Flolan). Which assessment information requires the most immediate action by the nurse? a. The oxygen saturation is 94%. b. The blood pressure is 98/56 mm Hg. c. The patient's central IV line is disconnected. d. The international normalized ratio (INR) is prolonged.
ANS: C The half-life of this drug is 6 minutes, so the nurse will need to restart the infusion as soon as possible to prevent rapid clinical deterioration. The other data also indicate a need for ongoing monitoring or intervention, but the priority action is to reconnect the infusion.
Which action by the nurse will be most effective in decreasing the spread of pertussis in a community setting? a. Providing supportive care to patients diagnosed with pertussis b. Teaching family members about the need for careful hand washing c. Teaching patients about the need for adult pertussis immunizations d. Encouraging patients to complete the prescribed course of antibiotics
ANS: C The increased rate of pertussis in adults is thought to be due to decreasing immunity after childhood immunization. Immunization is the most effective method of protecting communities from infectious diseases. Hand washing should be taught, but pertussis is spread by droplets and contact with secretions. Supportive care does not shorten the course of the disease or the risk for transmission. Taking antibiotics as prescribed does assist with decreased transmission, but patients are likely to have already transmitted the disease by the time the diagnosis is made.
inserting a tube for breathing
The nurse is caring for a patient going to surgery for a tracheostomy. What is the purpose of a tracheostomy? A. Inserting a tube for feeding B. Inserting a tube for breathing C. Inserting a tube for bile drainage D. Inserting a tube for gastric drainage
Employee health test results reveal a tuberculosis (TB) skin test of 16-mm induration and a negative chest x-ray for a staff nurse working on the pulmonary unit. The nurse has no symptoms of TB. Which information should the occupational health nurse plan to teach the staff nurse? a. Standard four-drug therapy for TB b. Need for annual repeat TB skin testing c. Use and side effects of isoniazid (INH) d. Bacille Calmette-Guérin (BCG) vaccine
ANS: C The nurse is considered to have a latent TB infection and should be treated with INH daily for 6 to 9 months. The four-drug therapy would be appropriate if the nurse had active TB. TB skin testing is not done for individuals who have already had a positive skin test. BCG vaccine is not used in the United States for TB and would not be helpful for this individual, who already has a TB infection.
3. The nurse discusses management of upper respiratory infections (URI) with a patient who has acute sinusitis. Which statement by the patient indicates that additional teaching is needed? a. "I can take acetaminophen (Tylenol) to treat my discomfort." b. "I will drink lots of juices and other fluids to stay well hydrated." c. "I can use my nasal decongestant spray until the congestion is all gone." d. "I will watch for changes in nasal secretions or the sputum that I cough up."
ANS: C The nurse should clarify that nasal decongestant sprays should be used for no more than 3 days to prevent rebound vasodilation and congestion. The other responses indicate that the teaching has been effective.
4. A nurse who is caring for patient with a tracheostomy tube in place has just auscultated rhonchi bilaterally. If the patient is unsuccessful in coughing up secretions, what action should the nurse take? a. Encourage increased incentive spirometer use. b. Encourage the patient to increase oral fluid intake. c. Put on sterile gloves and use a sterile catheter to suction. d. Preoxygenate the patient for 3 minutes before suctioning.
ANS: C This patient needs suctioning now to secure a patent airway. Sterile gloves and a sterile catheter are used when suctioning a tracheostomy. Preoxygenation for 3 minutes is not necessary. Incentive spirometer (IS) use opens alveoli and can induce coughing, which can mobilize secretions. However, the patient with a tracheostomy may not be able to use an incentive spirometer. Increasing oral fluid intake would not moisten and help mobilize secretions in a timely manner.
The nurse analyzes the results of a patient's arterial blood gases (ABGs). Which finding would require immediate action? a. The bicarbonate level (HCO3-) is 31 mEq/L. b. The arterial oxygen saturation (SaO2) is 92%. c. The partial pressure of CO2 in arterial blood (PaCO2) is 31 mm Hg. d. The partial pressure of oxygen in arterial blood (PaO2) is 59 mm Hg.
ANS: D All the values are abnormal, but the low PaO2 indicates that the patient is at the point on the oxyhemoglobin dissociation curve where a small change in the PaO2 will cause a large drop in the O2 saturation and a decrease in tissue oxygenation. The nurse should intervene immediately to improve the patient's oxygenation.
A patient with pneumonia has a fever of 101.4° F (38.6° C), a nonproductive cough, and an oxygen saturation of 88%. The patient complains of weakness, fatigue, and needs assistance to get out of bed. Which nursing diagnosis should the nurse assign as the highest priority? a. Hyperthermia related to infectious illness b. Impaired transfer ability related to weakness c. Ineffective airway clearance related to thick secretions d. Impaired gas exchange related to respiratory congestion
ANS: D All these nursing diagnoses are appropriate for the patient, but the patient's oxygen saturation indicates that all body tissues are at risk for hypoxia unless the gas exchange is improved.
20. A patient arrives in the ear, nose, and throat clinic complaining of a piece of tissue being "stuck up my nose" and with foul-smelling nasal drainage from the right nare. Which action should the nurse take first? a. Notify the clinic health care provider. b. Obtain aerobic culture specimens of the drainage. c. Ask the patient about how the cotton got into the nose. d. Have the patient occlude the left nare and blow the nose.
ANS: D Because the highest priority action is to remove the foreign object from the nare, the nurse's first action should be to assist the patient to remove the object. The other actions are also appropriate but should be done after attempting to clear the nose.
When assessing the respiratory system of an older patient, which finding indicates that the nurse should take immediate action? a. Weak cough effort b. Barrel-shaped chest c. Dry mucous membranes d. Bilateral crackles at lung bases
ANS: D Crackles in the lower half of the lungs indicate that the patient may have an acute problem such as heart failure. The nurse should immediately accomplish further assessments, such as oxygen saturation, and notify the health care provider. A barrel-shaped chest, hyperresonance to percussion, and a weak cough effort are associated with aging. Further evaluation may be needed, but immediate action is not indicated. An older patient has a less forceful cough and fewer and less functional cilia. Mucous membranes tend to be drier.
2. Which statement by a 50-year-old female patient indicates to the nurse that further assessment of thyroid function may be necessary? a. "I notice my breasts are tender lately." b. "I am so thirsty that I drink all day long." c. "I get up several times at night to urinate." d. "I feel a lump in my throat when I swallow."
ANS: D Difficulty in swallowing can occur with a goiter. Nocturia is associated with diseases such as diabetes mellitus, diabetes insipidus, or chronic kidney disease. Breast tenderness would occur with excessive gonadal hormone levels. Thirst is a sign of disease such as diabetes. DIF: Cognitive Level: Apply (application) REF: 1144 TOP: Nursing Process: Assessment MSC: NCLEX: Physiological Integrity
An alcoholic and homeless patient is diagnosed with active tuberculosis (TB). Which intervention by the nurse will be most effective in ensuring adherence with the treatment regimen? a. Arrange for a friend to administer the medication on schedule. b. Give the patient written instructions about how to take the medications. c. Teach the patient about the high risk for infecting others unless treatment is followed. d. Arrange for a daily noon meal at a community center where the drug will be administered.
ANS: D Directly observed therapy is the most effective means for ensuring compliance with the treatment regimen, and arranging a daily meal will help ensure that the patient is available to receive the medication. The other nursing interventions may be appropriate for some patients but are not likely to be as helpful for this patient.
Blood sugar is well controlled when Hemoglobin A1C is: a. Below 7% b. Between 12%-15% c. Less than 180 mg/dL d. Between 90 and 130 mg/dL
a. Below 7% A1c measures the percentage of hemoglobin that is glycated and determines average blood glucose during the 2 to 3 months prior to testing. Used as a diagnostic tool, A1C levels of 6.5% or higher on two tests indicate diabetes. A1C of 6% to 6.5% is considered prediabetes.
12. An 18-year-old male patient with a small stature is scheduled for a growth hormone stimulation test. In preparation for the test, the nurse will obtain a. ice in a basin. b. glargine insulin. c. a cardiac monitor. d. 50% dextrose solution.
ANS: D Hypoglycemia is induced during the growth hormone stimulation test, and the nurse should be ready to administer 50% dextrose immediately. Regular insulin is used to induce hypoglycemia (glargine is never given IV). The patient does not require cardiac monitoring during the test. Although blood samples for some tests must be kept on ice, this is not true for the growth hormone stimulation test. DIF: Cognitive Level: Apply (application) REF: 1147 TOP: Nursing Process: Planning MSC: NCLEX: Physiological Integrity
A patient who has a history of chronic obstructive pulmonary disease (COPD) was hospitalized for increasing shortness of breath and chronic hypoxemia (SaO2 levels of 89% to 90%). In planning for discharge, which action by the nurse will be most effective in improving compliance with discharge teaching? a. Start giving the patient discharge teaching on the day of admission. b. Have the patient repeat the instructions immediately after teaching. c. Accomplish the patient teaching just before the scheduled discharge. d. Arrange for the patient's caregiver to be present during the teaching.
ANS: D Hypoxemia interferes with the patient's ability to learn and retain information, so having the patient's caregiver present will increase the likelihood that discharge instructions will be followed. Having the patient repeat the instructions will indicate that the information is understood at the time, but it does not guarantee retention of the information. Because the patient is likely to be distracted just before discharge, giving discharge instructions just before discharge is not ideal. The patient is likely to be anxious and even more hypoxemic than usual on the day of admission, so teaching about discharge should be postponed.
Which intervention will the nurse include in the plan of care for a patient who is diagnosed with a lung abscess? a. Teach the patient to avoid the use of over-the-counter expectorants. b. Assist the patient with chest physiotherapy and postural drainage. c. Notify the health care provider immediately about any bloody or foul-smelling sputum. d. Teach about the need for prolonged antibiotic therapy after discharge from the hospital.
ANS: D Long-term antibiotic therapy is needed for effective eradication of the infecting organisms in lung abscess. Chest physiotherapy and postural drainage are not recommended for lung abscess because they may lead to spread of the infection. Foul smelling and bloody sputum are common clinical manifestations in lung abscess. Expectorants may be used because the patient is encouraged to cough
The nurse completes discharge teaching for a patient who has had a lung transplant. The nurse evaluates that the teaching has been effective if the patient makes which statement? a. "I will make an appointment to see the doctor every year." b. "I will stop taking the prednisone if I experience a dry cough." c. "I will not worry if I feel a little short of breath with exercise." d. "I will call the health care provider right away if I develop a fever."
ANS: D Low-grade fever may indicate infection or acute rejection so the patient should notify the health care provider immediately if the temperature is elevated. Patients require frequent follow-up visits with the transplant team. Annual health care provider visits would not be sufficient. Home oxygen use is not an expectation after lung transplant. Shortness of breath should be reported. Low-grade fever, fatigue, dyspnea, dry cough, and oxygen desaturation are signs of rejection. Immunosuppressive therapy, including prednisone, needs to be continued to prevent rejection.
1. The nurse teaches a patient about discharge instructions after a rhinoplasty. Which statement, if made by the patient, indicates that the teaching was successful? a. "I can take 800 mg ibuprofen for pain control." b. "I will safely remove and reapply nasal packing daily." c. "My nose will look normal after 24 hours when the swelling goes away." d. "I will keep my head elevated for 48 hours to minimize swelling and pain."
ANS: D Maintaining the head in an elevated position will decrease the amount of nasal swelling. NSAIDs, such as ibuprofen, increase the risk for postoperative bleeding and should not be used postoperatively. The patient would not be taught to remove or reapply nasal packing, which is usually removed by the surgeon on the day after surgery. Although return to a preinjury appearance is the goal of the surgery, it is not always possible to achieve this result, especially in the first few weeks after surgery.
A lobectomy is scheduled for a patient with stage I non-small cell lung cancer. The patient tells the nurse, "I would rather have chemotherapy than surgery." Which response by the nurse is most appropriate? a. "Are you afraid that the surgery will be very painful?" b. "Did you have bad experiences with previous surgeries?" c. "Surgery is the treatment of choice for stage I lung cancer." d. "Tell me what you know about the various treatments available."
ANS: D More assessment of the patient's concerns about surgery is indicated. An open-ended response will elicit the most information from the patient. The answer beginning, "Surgery is the treatment of choice" is accurate, but it discourages the patient from sharing concerns about surgery. The remaining two answers indicate that the nurse has jumped to conclusions about the patient's reasons for not wanting surgery. Chemotherapy is the primary treatment for small cell lung cancer. In non-small cell lung cancer, chemotherapy may be used in the treatment of nonresectable tumors or as adjuvant therapy to surgery
A patient is admitted with active tuberculosis (TB). The nurse should question a health care provider's order to discontinue airborne precautions unless which assessment finding is documented? a. Chest x-ray shows no upper lobe infiltrates. b. TB medications have been taken for 6 months. c. Mantoux testing shows an induration of 10 mm. d. Three sputum smears for acid-fast bacilli are negative.
ANS: D Negative sputum smears indicate that Mycobacterium tuberculosis is not present in the sputum, and the patient cannot transmit the bacteria by the airborne route. Chest x-rays are not used to determine whether treatment has been successful. Taking medications for 6 months is necessary, but the multidrug-resistant forms of the disease might not be eradicated after 6 months of therapy. Repeat Mantoux testing would not be done because the result will not change even with effective treatment.
A comatose client with an admitting diagnosis of diabetic ketoacidosis (DKA) has a blood glucose of 368 mg/dL, arterial pH of 7.2, arterial bicarbonate of 14 mEq/L, and a positive for serum ketones. The diagnosis is supported by which noted data? 1. Hypertension 2. Fruity breath odor 3. Slow regular breathing 4. Moist mucous membranes
Ans. 2 rationale Diabetic ketoacidotic coma is usually identified with a fruity breath odor, dry cracked mucous membranes, hypotension, and rapid deep breathing.
21. The nurse is caring for a patient who has acute pharyngitis caused by Candida albicans. Which action is appropriate for the nurse to include in the plan of care? a. Avoid giving patient warm liquids to drink. b. Assess patient for allergies to penicillin antibiotics. c. Teach the patient about the need to sleep in a warm, dry environment. d. Teach patient to "swish and swallow" prescribed oral nystatin (Mycostatin).
ANS: D Oral or pharyngeal fungal infections are treated with nystatin solution. The goal of the "swish and swallow" technique is to expose all of the oral mucosa to the antifungal agent. Warm liquids may be soothing to a sore throat. The patient should be taught to use a cool mist humidifier. There is no need to assess for penicillin/cephalosporin allergies because Candida albicans infection is treated with antifungals.
6. A 61-year-old female patient admitted with pneumonia has a total serum calcium level of 13.3 mg/dL (3.3 mmol/L). The nurse will anticipate the need to teach the patient about testing for _____ levels. a. calcitonin b. catecholamine c. thyroid hormone d. parathyroid hormone
ANS: D Parathyroid hormone is the major controller of blood calcium levels. Although calcitonin secretion is a countermechanism to parathyroid hormone, it does not play a major role in calcium balance. Catecholamine and thyroid hormone levels do not affect serum calcium level. DIF: Cognitive Level: Apply (application) REF: 1136 | 1149 TOP: Nursing Process: Planning MSC: NCLEX: Physiological Integrity
The nurse is performing tuberculosis (TB) skin tests in a clinic that has many patients who have immigrated to the United States. Which question is most important for the nurse to ask before the skin test? a. "Is there any family history of TB?" b. "How long have you lived in the United States?" c. "Do you take any over-the-counter (OTC) medications?" d. "Have you received the bacille Calmette-Guérin (BCG) vaccine for TB?"
ANS: D Patients who have received the BCG vaccine will have a positive Mantoux test. Another method for screening (such as a chest x-ray) will need to be used in determining whether the patient has a TB infection. The other information also may be valuable but is not as pertinent to the decision about doing TB skin testing.
The nurse supervises unlicensed assistive personnel (UAP) who are providing care for a patient with right lower lobe pneumonia. The nurse should intervene if which action by UAP is observed? a. UAP splint the patient's chest during coughing. b. UAP assist the patient to ambulate to the bathroom. c. UAP help the patient to a bedside chair for meals. d. UAP lower the head of the patient's bed to 15 degrees.
ANS: D Positioning the patient with the head of the bed lowered will decrease ventilation. The other actions are appropriate for a patient with pneumonia.
The nurse provides discharge teaching for a patient who has two fractured ribs from an automobile accident. Which statement, if made by the patient, would indicate that teaching has been effective? a. "I am going to buy a rib binder to wear during the day." b. "I can take shallow breaths to prevent my chest from hurting." c. "I should plan on taking the pain pills only at bedtime so I can sleep." d. "I will use the incentive spirometer every hour or two during the day."
ANS: D Prevention of the complications of atelectasis and pneumonia is a priority after rib fracture. This can be ensured by deep breathing and coughing. Use of a rib binder, shallow breathing, and taking pain medications only at night are likely to result in atelectasis
The nurse admits a patient who has a diagnosis of an acute asthma attack. Which statement indicates that the patient may need teaching regarding medication use? a. "I have not had any acute asthma attacks during the last year." b. "I became short of breath an hour before coming to the hospital." c. "I've been taking Tylenol 650 mg every 6 hours for chest-wall pain." d. "I've been using my albuterol inhaler more frequently over the last 4 days."
ANS: D The increased need for a rapid-acting bronchodilator should alert the patient that an acute attack may be imminent and that a change in therapy may be needed. The patient should be taught to contact a health care provider if this occurs. The other data do not indicate any need for additional teaching.
13. A nurse is caring for a patient who has had a total laryngectomy and radical neck dissection. During the first 24 hours after surgery what is the priority nursing action? a. Monitor for bleeding. b. Maintain adequate IV fluid intake. c. Suction tracheostomy every eight hours. d. Keep the patient in semi-Fowler's position.
ANS: D The most important goals after a laryngectomy and radical neck dissection are to maintain the airway and ensure adequate oxygenation. Keeping the patient in a semi-Fowler's position will decrease edema and limit tension on the suture lines to help ensure an open airway. Maintenance of IV fluids and monitoring for bleeding are important, but maintaining an open airway is the priority. Tracheostomy care and suctioning should be provided as needed. During the immediate postoperative period, the patient with a laryngectomy requires frequent suctioning of the tracheostomy tube.
2. The nurse plans to teach a patient how to manage allergic rhinitis. Which information should the nurse include in the teaching plan? a. Hand washing is the primary way to prevent spreading the condition to others. b. Use of oral antihistamines for 2 weeks before the allergy season may prevent reactions. c. Corticosteroid nasal sprays will reduce inflammation, but systemic effects limit their use. d. Identification and avoidance of environmental triggers are the best way to avoid symptoms.
ANS: D The most important intervention is to assist the patient in identifying and avoiding potential allergens. Intranasal corticosteroids (not oral antihistamines) should be started several weeks before the allergy season. Corticosteroid nasal sprays have minimal systemic absorption. Acute viral rhinitis (the common cold) can be prevented by washing hands.
A patient with a possible pulmonary embolism complains of chest pain and difficulty breathing. The nurse finds a heart rate of 142 beats/minute, blood pressure of 100/60 mmHg, and respirations of 42 breaths/minute. Which action should the nurse take first? a. Administer anticoagulant drug therapy. b. Notify the patient's health care provider. c. Prepare patient for a spiral computed tomography (CT). d. Elevate the head of the bed to a semi-Fowler's position.
ANS: D The patient has symptoms consistent with a pulmonary embolism (PE). Elevating the head of the bed will improve ventilation and gas exchange. The other actions can be accomplished after the head is elevated (and oxygen is started). A spiral CT may be ordered by the health care provider to identify PE. Anticoagulants may be ordered after confirmation of the diagnosis of PE.
An hour after a thoracotomy, a patient complains of incisional pain at a level 7 (based on 0 to 10 scale) and has decreased left-sided breath sounds. The pleural drainage system has 100 mL of bloody drainage and a large air leak. Which action is best for the nurse to take next? a. Milk the chest tube gently to remove any clots. b. Clamp the chest tube momentarily to check for the origin of the air leak. c. Assist the patient to deep breathe, cough, and use the incentive spirometer. d. Set up the patient controlled analgesia (PCA) and administer the loading dose of morphine.
ANS: D The patient is unlikely to take deep breaths or cough until the pain level is lower. A chest tube output of 100 mL is not unusual in the first hour after thoracotomy and would not require milking of the chest tube. An air leak is expected in the initial postoperative period after thoracotomy.
When assessing a patient who has just arrived after an automobile accident, the emergency department nurse notes tachycardia and absent breath sounds over the right lung. For which intervention will the nurse prepare the patient? a. Emergency pericardiocentesis b. Stabilization of the chest wall with tape c. Administration of an inhaled bronchodilator d. Insertion of a chest tube with a chest drainage system
ANS: D The patient's history and absent breath sounds suggest a right-sided pneumothorax or hemothorax, which will require treatment with a chest tube and drainage. The other therapies would be appropriate for an acute asthma attack, flail chest, or cardiac tamponade, but the patient's clinical manifestations are not consistent with these problems
After change-of-shift report, which patient should the nurse assess first? a. 72-year-old with cor pulmonale who has 4+ bilateral edema in his legs and feet b. 28-year-old with a history of a lung transplant and a temperature of 101° F (38.3° C) c. 40-year-old with a pleural effusion who is complaining of severe stabbing chest pain d. 64-year-old with lung cancer and tracheal deviation after subclavian catheter insertion
ANS: D The patient's history and symptoms suggest possible tension pneumothorax, a medical emergency. The other patients also require assessment as soon as possible, but tension pneumothorax will require immediate treatment to avoid death from inadequate cardiac output or hypoxemia.
The nurse prepares a patient with a left-sided pleural effusion for a thoracentesis. How should the nurse position the patient? a. Supine with the head of the bed elevated 30 degrees b. In a high-Fowler's position with the left arm extended c. On the right side with the left arm extended above the head d. Sitting upright with the arms supported on an over bed table
ANS: D The upright position with the arms supported increases lung expansion, allows fluid to collect at the lung bases, and expands the intercostal space so that access to the pleural space is easier. The other positions would increase the work of breathing for the patient and make it more difficult for the health care provider performing the thoracentesis
The laboratory has just called with the arterial blood gas (ABG) results on four patients. Which result is most important for the nurse to report immediately to the health care provider? a. pH 7.34, PaO2 82 mm Hg, PaCO2 40 mm Hg, and O2 sat 97% b. pH 7.35, PaO2 85 mm Hg, PaCO2 45 mm Hg, and O2 sat 95% c. pH 7.46, PaO2 90 mm Hg, PaCO2 32 mm Hg, and O2 sat 98% d. pH 7.31, PaO2 91 mm Hg, PaCO2 50 mm Hg, and O2 sat 96%
ANS: D These ABGs indicate uncompensated respiratory acidosis and should be reported to the health care provider. The other values are normal or close to normal.
The nurse palpates the posterior chest while the patient says "99" and notes absent fremitus. Which action should the nurse take next? a. Palpate the anterior chest and observe for barrel chest. b. Encourage the patient to turn, cough, and deep breathe. c. Review the chest x-ray report for evidence of pneumonia. d. Auscultate anterior and posterior breath sounds bilaterally.
ANS: D To assess for tactile fremitus, the nurse should use the palms of the hands to assess for vibration when the patient repeats a word or phrase such as "99." After noting absent fremitus, the nurse should then auscultate the lungs to assess for the presence or absence of breath sounds. Absent fremitus may be noted with pneumothorax or atelectasis. The vibration is increased in conditions such as pneumonia, lung tumors, thick bronchial secretions, and pleural effusion. Turning, coughing, and deep breathing is an appropriate intervention for atelectasis, but the nurse needs to first assess breath sounds. Fremitus is decreased if the hand is farther from the lung or the lung is hyperinflated (barrel chest).The anterior of the chest is more difficult to palpate for fremitus because of the presence of large muscles and breast tissue.
9. A patient scheduled for a total laryngectomy and radical neck dissection for cancer of the larynx asks the nurse, "Will I be able to talk normally after surgery?" What is the best response by the nurse? a. "You will breathe through a permanent opening in your neck, but you will not be able to communicate orally." b. "You won't be able to talk right after surgery, but you will be able to speak again after the tracheostomy tube is removed." c. "You won't be able to speak as you used to, but there are artificial voice devices that will give you the ability to speak normally." d. "You will have a permanent opening into your neck, and you will need to have rehabilitation for some type of voice restoration."
ANS: D Voice rehabilitation is planned after a total laryngectomy, and a variety of assistive devices are available to restore communication. Although the ability to communicate orally is changed, it would not be appropriate to tell a patient that this ability would be lost. Artificial voice devices do not permit normal-sounding speech. In a total laryngectomy, the vocal cords are removed, so normal speech is impossible.
A nurse is reinforcing teaching with a client who has Addison's disease about appropriate snack foods. The nurse evaluates the teaching to be effective when the client identifies what as an appropriate snack choice?
American cheese with saltine crackers. Addison's disease is a hormone deficiency caused by damage to the outer later of the adrenal gland (adrenal cortex). Addison's disease occurs when the adrenal glands do not produce enough of the hormone cortisol and, in some cases, the hormone aldosterone. The client with Addison's disease requires a diet high in salt, carbohydrates, and protein and low in potassium. American cheese provides protein and salt. Saltine crackers provide carbohydrates and salt.
A nurse is assisting with preparing a teaching plan for the client with diabetes mellitus regarding proper foot care. Which instruction should be included in the plan of care? 1. Soak the feet in hot water. 2. Avoid using soap to wash the feet. 3. Apply a moisturizing lotion to dry feet, but not between the toes. 4. Always have a podiatrist cut your toenails; never cut them yourself.
Ans 3 The client should use a moisturizing lotion on his or her feet, but should avoid applying the lotion between the toes. The client should also be instructed to not soak the feet and to avoid hot water to prevent burns. The client may cut the toenails straight across and even with the toe itself, but he or she should consult a podiatrist if the toenails are thick or hard to cut or if his or her vision is poor. The client should be instructed to wash the feet daily with a mild soap.
Which client complaint would alert the nurse to a possible hypoglycemic reaction? 1. Tremors 2. Anorexia 3. Hot, dry skin 4. Muscle cramps
Ans. 1 rationale Decreased blood glucose levels produce automatic nervous system symptoms, which are classically manifested as nervousness, irritability, and tremors. Option 3 is more likely to occur with hyperglycemia. Options 2 and 4 are unrelated to the signs of hypoglycemia.
A client is admitted to the hospital with a diagnosis of diabetic ketoacidosis (DKA). The initial serum glucose level was 950 mg/dL. Intravenous (IV) insulin was started along with rehydration with IV normal saline. The serum glucose level is now 240 mg/dL. The nurse who is assisting in caring for the client obtains which of the following items, anticipating a health care provider's prescription? 1. IV infusion containing 5% dextrose 2. NPH insulin and a syringe for subcutaneous injection 3. An ampule of 50% dextrose 4. Phenytoin (Dilantin) for prevention of seizures
Ans. 1 rationale During management of DKA, when the blood glucose level falls to 300 mg/dL, the infusion rate is reduced and 5% dextrose is added to maintain a blood glucose level of about 250 mg/dL, or until the client recovers from ketosis. NPH insulin is not used to treat DKA; 50% dextrose is used to treat hypoglycemia. Phenytoin is not a normal treatment measure in DKA.
A health care provider has prescribed propylthiouracil (PTU) for a client with hyperthyroidism, and the nurse assists in developing a plan of care for the client. A priority nursing measure to be included in the plan regarding this medication is to monitor the client for: 1. Signs and symptoms of hypothyroidism 2. Signs and symptoms of hyperglycemia 3. Relief of pain 4. Signs of renal toxicity
Ans. 1 rationale Excessive dosing with propylthiouracil may convert the client from a hyperthyroid state to a hypothyroid state. If this occurs, the dosage should be reduced. Temporary administration of thyroid hormone may be required. Propylthiouracil is not used for pain and does not cause hyperglycemia or renal toxicity.
A nurse is monitoring a client following a thyroidectomy for signs of hypocalcemia. Which of the following signs, if noted in the client, likely indicates the presence of hypocalcemia? 1. Tingling around the mouth 2. Negative Chvostek's sign 3. Flaccid paralysis 4. Bradycardia
Ans. 1 rationale Following a thyroidectomy, the nurse assesses the client for signs of hypocalcemia and tetany. Early signs include tingling around the mouth and fingertips, muscle twitching or spasms, palpitations or dysrhythmias, and positive Chvostek's and Trousseau's signs. Options 2, 3, and 4 are not signs of hypocalcemia.
A nurse is caring for a client with a diagnosis of hypoparathyroidism. The nurse reviews the laboratory results drawn on the client and notes that the calcium level is extremely low. The nurse would expect to note which of the following on data collection of the client? 1. Positive Trousseau's sign 2. Negative Chvostek's sign 3. Unresponsive pupils 4. Hyperactive bowel sounds
Ans. 1 rationale Hypoparathyroidism is related to a lack of parathyroid hormone secretion or to a decreased effectiveness of parathyroid hormone on target tissues. The end result of this disorder is hypocalcemia. When serum calcium levels are critically low, the client may exhibit positive Chvostek's and Trousseau's signs, which indicate potential tetany. Options 2, 3, and 4 are not related to the presence of hypocalcemia.
A client with diabetes mellitus is being discharged following treatment for hyperglycemic hyperosmolar nonketotic syndrome (HHNS) precipitated by acute illness. The client states to the nurse, "I will call the doctor next time I can't eat for more than a day or so." The nurse plans care, understanding that which of the following accurately reflects this client's level of knowledge? 1. The client needs immediate education before discharge. 2. The client's statement is accurate, but knowledge should be evaluated further. 3. The client's statement is inaccurate, and the client should be scheduled for outpatient diabetic counseling. 4. The client requires follow-up teaching regarding the administration of insulin.
Ans. 1 rationale If the client becomes ill and cannot retain fluids or food for a period of 4 hours, the health care provider should be notified. The client's statement in this question indicates a need for immediate education to prevent HHNS, a life-threatening emergency situation.
Which statement by the client would cause the nurse to suspect that the thyroid test results drawn on the client this morning may be inaccurate? 1. "I had a radionuclide test done 3 days ago." 2. "When I exercise I sweat more than normal." 3. "I drank some water before the blood was drawn." 4. "That hamburger I ate before the test sure tasted good."
Ans. 1 rationale Option 1 indicates that a recent radionuclide scan had been performed. Recent radionuclide scans performed before the test can affect thyroid laboratory results. No food, fluid, or activity restrictions are required for this test, so options 2, 3, and 4 are incorrect.
A nurse is monitoring a client who has been newly diagnosed with diabetes mellitus for signs of complications. Which of the following, if exhibited by the client, would indicate hyperglycemia and thus warrant health care provider notification? 1. Polyuria 2. Bradycardia 3. Diaphoresis 4. Hypertension
Ans. 1 rationale The classic symptoms of hyperglycemia include polydipsia, polyuria, and polyphagia. Options 2, 3, and 4 are not signs of hyperglycemia.
A nurse has reinforced home care measures to a client diagnosed with diabetes mellitus regarding exercise and insulin administration. Which statement by the client indicates a need for further instruction? 1. "I should perform my exercise at peak insulin time." 2. "I should always carry a quick-acting carbohydrate when I exercise." 3. "I should always wear a Medic-Alert bracelet especially when I exercise." 4. "I should avoid exercising at times when a hypoglycemic reaction is likely to occur."
Ans. 1 rationale The client should be instructed to avoid exercise at peak insulin time because this is when a hypoglycemic reaction is likely to occur. If exercise is performed at this time, the client should be instructed to eat an hour before the exercise and drink a carbohydrate liquid. Options 2, 3, and 4 are correct statements regarding exercise, insulin, and diabetic control.
A nurse has reinforced dietary instructions to a client with a diagnosis of hypoparathyroidism. The nurse instructs the client to include which of the following items in the diet? 1. Vegetables 2. Meat 3. Fish 4. Cereals
Ans. 1 rationale The client with hypoparathyroidism is instructed to follow a calcium-rich diet and to restrict the amount of phosphorus in the diet. The client should limit meat, poultry, fish, eggs, cheese, and cereals. Vegetables are allowed in the diet.
A nurse notes in the medical record that a client with Cushing's syndrome is experiencing fluid overload. Which interventions should be included in the plan of care? Select all that apply. 1. Monitoring daily weight 2. Monitoring intake and output 3. Maintaining a low-potassium diet 4. Monitoring extremities for edema 5. Maintaining a low-sodium diet
Ans. 1,2,4,5 rationale The client with Cushing's syndrome experiencing fluid overload should be maintained on a high-potassium and low-sodium diet. Decreased sodium intake decreases renal retention of sodium and water. Monitoring weight, intake, output, and extremities for edema are all appropriate interventions for such a nursing diagnosis.
A nurse is reviewing a plan of care for a client with Addison's disease. The nurse notes that the client is at risk for dehydration and suggests nursing interventions that will prevent this occurrence. Which nursing intervention is an appropriate component of the plan of care? Select all that apply. 1. Encouraging fluid intake of at least 3000 mL/day 2. Encouraging an intake of low-protein foods 3. Monitoring for changes in mental status 4. Monitoring intake and output 5. Maintaining a low-sodium diet
Ans. 1,3,4 rationale The client at risk for deficient fluid volume should be encouraged to eat regular meals and snacks and to increase the intake of sodium, protein, and complex carbohydrates. Oral replacement of sodium losses is necessary, and maintenance of adequate blood glucose levels is required.
A nurse is providing discharge instructions to a client who had a unilateral adrenalectomy. Which of the following will be a component of the instructions? 1. The reason for maintaining a diabetic diet 2. Instructions about early signs of a wound infection 3. Teaching regarding proper application of an ostomy pouch 4. The need for lifelong replacement of all adrenal hormones
Ans. 2 rationale A client who is undergoing a unilateral adrenalectomy will be placed on corticosteroids temporarily to avoid a cortisol deficiency. These medications will be gradually weaned in the postoperative period until they are discontinued. Because of the anti-inflammatory properties of corticosteroids, clients who undergo an adrenalectomy are at increased risk for developing wound infections. Because of this increased risk for infection, it is important for the client to know measures to prevent infection, early signs of infection, and what to do if an infection is present. Options 1, 3, and 4 are incorrect instructions.
When caring for a client who is having clear drainage from his nares after transsphenoidal hypophysectomy, which action by the nurse is appropriate? 1. Lower the head of the bed. 2. Test the drainage for glucose. 3. Obtain a culture of the drainage. 4. Continue to observe the drainage.
Ans. 2 rationale After hypophysectomy, the client should be monitored for rhinorrhea, which could indicate a cerebrospinal fluid (CSF) leak. If this occurs, the drainage should be collected and tested for glucose, indicating the presence of CSF. The head of the bed should not be lowered to prevent increased intracranial pressure. Clear nasal drainage would not indicate the need for a culture. Continuing to observe the drainage without taking action could result in a serious complication.
A nurse is caring for a client with a diagnosis of myasthenia gravis. The health care provider plans to perform an Enlon test on the client to determine the presence of cholinergic crisis. In addition to planning care for the client during this testing, which of the following will the nurse ensure is at the bedside? 1. Cardiac monitor 2. Oxygen equipment 3. Vial of protamine sulfate and a syringe 4. Potassium injection and a liter of normal saline solution
Ans. 2 rationale An Enlon test is performed to distinguish between myasthenic and cholinergic crisis. Following administration of Enlon, if symptoms intensify, the crisis is cholinergic. Because the symptoms of cholinergic crisis will worsen with the administration of Enlon, atropine sulfate and oxygen should be immediately available whenever Enlon is used.
What would the nurse anticipate being included in the plan of care for a client who has been diagnosed with Graves' disease? 1. Provide a high-fiber diet. 2. Provide a restful environment. 3. Provide three small meals per day. 4. Provide the client with extra blankets.
Ans. 2 rationale Because of the hypermetabolic state, the client with Graves' disease needs to be provided with an environment that is restful both physically and mentally. Six full meals a day that are well balanced and high in calories are required, because of the accelerated metabolic rate. Foods that increase peristalsis (e.g., high-fiber foods) need to be avoided. These clients suffer from heat intolerance and require a cool environment.
A nurse is caring for a client after thyroidectomy and monitoring for signs of thyroid storm. The nurse understands that which of the following is a manifestation associated with this disorder? 1. Bradycardia 2. Hypotension 3. Constipation 4. Hypothermia
Ans. 2 rationale Clinical manifestations associated with thyroid storm include a fever as high as 106° F (41.1° C), severe tachycardia, profuse diarrhea, extreme vasodilation, hypotension, atrial fibrillation, hyperreflexia, abdominal pain, diarrhea, and dehydration. With this disorder, the client's condition can rapidly progress to coma and cardiovascular collapse.
A nurse is collecting data on a client with hyperparathyroidism. Which of the following questions would elicit the accurate information about this condition from the client? 1. "Do you have tremors in your hands?" 2. "Are you experiencing pain in your joints?" 3. "Have you had problems with diarrhea lately?" 4. "Do you notice swelling in your legs at night?"
Ans. 2 rationale Hyperparathyroidism causes an oversecretion of parathyroid hormone (PTH), which causes excessive osteoblast growth and activity within the bones. When bone reabsorption is increased, calcium is released from the bones into the blood, causing hypercalcemia. The bones suffer demineralization as a result of calcium loss, leading to bone and joint pain, and pathological fractures.
A client with type 1 diabetes mellitus takes NPH insulin every morning and checks the blood glucose level four times per day. The client tells the nurse that yesterday the late afternoon blood glucose was 60 mg/dL and that she "felt funny." Which statement by the client would indicate an understanding of this occurrence? 1. "My blood glucose levels are running low because I'm tired." 2. "I forgot to take my usual afternoon snack yesterday." 3. "I took less insulin this morning so I won't feel funny today." 4. "I don't know why I have to check my blood glucose four times a day. That seems too much."
Ans. 2 rationale Hypoglycemia is a blood glucose level of 60 mg/dL or less. The causes are multiple, but, in this case, omitting the afternoon snack is the cause. Fatigue and self-adjustment of dose are incorrect options. Recommended blood glucose testing for the client with type 1 diabetes mellitus is at least four times a day.
A nurse is collecting data on a client admitted to the hospital with a diagnosis of myxedema. Which data collection technique will provide data necessary to support the admitting diagnosis? 1. Auscultation of lung sounds 2. Inspection of facial features 3. Percussion of the thyroid gland 4. Palpation of the adrenal glands
Ans. 2 rationale Inspection of facial features will reveal the characteristic coarse features, presence of edema around the eyes and face, and a blank expression that are characteristic of myxedema. The techniques in the remaining options will not reveal any data that would support the diagnosis of myxedema.
After several diagnostic tests, a client is diagnosed with diabetes insipidus. The nurse understands that which symptom is indicative of this disorder? 1. Diarrhea 2. Polydipsia 3. Weight gain 4. Blurred vision
Ans. 2 rationale Polydipsia and polyuria are classic symptoms of diabetes insipidus. The urine is pale in color, and its specific gravity is low. Anorexia and weight loss occur. Diarrhea, weight loss, and blurred vision are not manifestations of the disorder.
A nurse reinforces teaching with a client with diabetes mellitus regarding differentiating between hypoglycemia and ketoacidosis. The client demonstrates an understanding of the teaching by stating that glucose will be taken if which symptom develops? 1. Polyuria 2. Shakiness 3. Blurred vision 4. Fruity breath odor
Ans. 2 rationale Shakiness is a sign of hypoglycemia, and it would indicate the need for food or glucose. Fruity breath odor, blurred vision, and polyuria are signs of hyperglycemia.
A nurse is caring for a client diagnosed with hyperparathyroidism who is prescribed furosemide (Lasix). The nurse reinforces dietary instructions to the client. Which of the following is an appropriate instruction? 1. Increase dietary intake of calcium. 2. Drink at least 2 to 3 L of fluid daily. 3. Eat sparely when experiencing nausea. 4. Decrease dietary intake of potassium.
Ans. 2 rationale The aim of treatment in the client with hyperparathyroidism is to increase the renal excretion of calcium and decrease gastrointestinal absorption and bone resorption. This is aided by the sufficient intake of fluids. Dietary restriction of calcium may be used as a component of therapy. The parathyroid is responsible for calcium production, and the term, "hyperparathyroidism" can be indicative of an increase in calcium. The client should eat foods high in potassium, especially if the client is taking furosemide. Limiting nutrients is not advisable.
A nurse is reinforcing home care instructions to a client with a diagnosis of Cushing's syndrome. Which statement reflects a need for further client education? 1. "Taking my medications exactly as prescribed is essential." 2. "I need to read the labels on any over-the-counter medications I purchase." 3. "My family needs to be familiar with the signs and symptoms of hypoadrenalism." 4. "I could experience the signs and symptoms of hyperadrenalism because of Cushing's."
Ans. 2 rationale The client with Cushing's syndrome should be instructed to take the medications exactly as prescribed. The nurse should emphasize the importance of continuing medications, consulting with the health care provider before purchasing any over-the-counter medications, and maintaining regular follow-up care. The nurse should also instruct the client in the signs and symptoms of both hypoadrenalism and hyperadrenalism.
A client with Cushing's disease is being admitted to the hospital after a stab wound to the abdomen. The nurse plans care and places highest priority on which potential problem? 1. Nervousness 2. Infection 3. Concern about appearance 4. Inability to care for self
Ans. 2 rationale The client with a stab wound has a break in the body's first line of defense against infection. The client with Cushing's disease is at great risk for infection because of excess cortisol secretion and subsequent impaired antibody function and decreased proliferation of lymphocytes. The client may also have a potential for the problems listed in the other options but these are not the highest priority at this time.
A client is in metabolic acidosis caused by diabetic ketoacidosis (DKA). The nurse prepares for the administration of which of the following medications as a primary treatment for this problem? 1. Potassium 2. Regular insulin 3. Sodium bicarbonate 4. Calcium gluconate
Ans. 2 rationale The primary treatment for any acid-base imbalance is treatment of the underlying disorder that caused the problem. In this case, the underlying cause of the metabolic acidosis is anaerobic metabolism as a result of the lack of ability to use circulating glucose. Administration of regular insulin corrects this problem.
A nurse is collecting data regarding a client after a thyroidectomy and notes that the client has developed hoarseness and a weak voice. Which nursing action is appropriate? 1. Check for signs of bleeding. 2. Administer calcium gluconate. 3. Notify the registered nurse immediately. 4. Reassure the client that this is usually a temporary
Ans. 4 Weakness and hoarseness of the voice can occur as a result of trauma of the laryngeal nerve. If this develops, the client should be reassured that the problem will subside in a few days. Unnecessary talking should be discouraged. It is not necessary to notify the registered nurse immediately. These signs do not indicate bleeding or the need to administer calcium gluconate.
A nurse has reinforced instructions to the client with hyperparathyroidism regarding home care measures related to exercise. Which statement by the client indicates a need for further instruction? Select all that apply. 1. "I enjoy exercising but I need to be careful." 2. "I need to pace my activities throughout the day." 3. "I need to limit playing football to only the weekends." 4. "I should gauge my activity level by my energy level." 5. "I should exercise in the evening to encourage a good sleep pattern."
Ans. 2, 5 rationale The client should be instructed to avoid high-impact activity or contact sports such as football. Exercising late in the evening may interfere with restful sleep. The client with hyperparathyroidism should pace activities throughout the day and plan for periods of uninterrupted rest. The client should plan for at least 30 minutes of walking each day to support calcium movement into the bones. The client should be instructed to use energy level as a guide to activity.
A nurse is monitoring a client with Graves' disease for signs of thyrotoxic crisis (thyroid storm). Which of the following signs and symptoms, if noted in the client, will alert the nurse to the presence of this crisis? Select all that apply. 1. Bradycardia 2. Fever 3. Sweating 4. Agitation 5. Pallor
Ans. 2,3,4 rationale Thyrotoxic crisis (thyroid storm) is an acute, potentially life-threatening state of extreme thyroid activity that represents a breakdown in the body's tolerance to a chronic excess of thyroid hormones. The clinical manifestations include fever greater than 100° F, severe tachycardia, flushing and sweating, and marked agitation and restlessness. Delirium and coma can occur.
A nurse is caring for a client following an adrenalectomy and is monitoring for signs of adrenal insufficiency. Which of the following, if noted in the client, indicates signs and symptoms related to adrenal insufficiency? Select all that apply. 1. Double vision 2. Hypotension 3. Mental status changes 4. Weakness 5. Fever
Ans. 2,3,4,5 rationale The nurse should be alert to signs and symptoms of adrenal insufficiency in a client following adrenalectomy. These signs and symptoms include weakness, hypotension, fever, and mental status changes. Double vision is generally not associated with this condition.
A nurse is preparing to provide instructions to a client with Addison's disease regarding diet therapy. The nurse understands that which of the following diets would likely be prescribed for this client? 1. Low-protein diet 2. Low-sodium diet 3. High-sodium diet 4. Low-carbohydrate diet
Ans. 3 rationale A high-sodium, high-complex carbohydrate, and high-protein diet will be prescribed for the client with Addison's disease. To prevent excess fluid and sodium loss, the client is instructed to maintain an adequate salt intake of up to 8 g of sodium daily and to increase salt intake during hot weather, before strenuous exercise, and in response to fever, vomiting, or diarrhea.
A nurse caring for a client scheduled for a transsphenoidal hypophysectomy to remove a tumor in the pituitary gland assists to develop a plan of care for the client. The nurse suggests including which specific information in the preoperative teaching plan? 1. Hair will need to be shaved. 2. Deep breathing and coughing will be needed after surgery. 3. Toothbrushing will not be permitted for at least 2 weeks following surgery. 4. Spinal anesthesia is used.
Ans. 3 rationale Based on the location of the surgical procedure, spinal anesthesia would not be used. In addition, the hair would not be shaved. Although coughing and deep breathing are important, specific to this procedure is avoiding toothbrushing to prevent disruption of the surgical site. Also, coughing may disrupt the surgical site.
A nurse is caring for a client newly diagnosed with diabetes mellitus. The client asks the nurse whether eating at a restaurant will affect the diabetic control and whether this is allowed. Which nursing response is appropriate? 1. "You really should not eat in restaurants." 2. "If you plan to eat in a restaurant, you need to avoid carbohydrates." 3. "You should order a half-portion meal and have fresh fruit for dessert." 4. "You should increase your daily dose of insulin by half on the day you plan to eat out."
Ans. 3 rationale Clients with diabetes mellitus are instructed to make adjustments in their total daily intake to plan for meals at restaurants or parties. Some useful strategies include ordering half portions, salads with dressing on the side, fresh fruit for dessert, and baked or steamed entrées. Clients are not instructed to avoid any food group or to increase their prescribed insulin dosage.
A nurse provides dietary instructions to a client with diabetes mellitus regarding the prescribed diabetic diet. Which statement, if made by the client, indicates the need for further teaching? 1. "I'll eat a balanced meal plan." 2. "I need to drink diet soft drinks." 3. "I need to buy special dietetic foods." 4. "I will snack on fruit instead of cake."
Ans. 3 rationale It is important to emphasize to the client and family that they are not eating a diabetic diet but rather following a balanced meal plan. Adherence to nutrition principles is an important component of diabetic management, and an individualized meal plan should be developed for the client. It is not necessary for the client to purchase special dietetic foods.
A client with diabetes mellitus calls the clinic nurse to report that the blood glucose level is 150 mg/dL. After obtaining further data from the client, the nurse determines that the client ate lunch approximately 2 hours ago. The nurse would interpret these results to be: 1. Normal 2. Lower than the normal value 3. Slightly higher than the normal value 4. A value that indicates immediate health care provider notification
Ans. 3 rationale Normal fasting blood glucose values range from 70 to 120 mg/dL. The 2-hour postprandial blood glucose level should be less than 140 mg/dL. In this situation, the blood glucose value was 150 mg/dL 2 hours after the client ate, which is slightly elevated above normal. This value does not require health care provider notification.
A nurse is assisting in preparing a plan of care for the client with diabetes mellitus and plans to reinforce the client's understanding regarding the symptoms of hypoglycemia. Which symptoms will the nurse review? 1. Slow pulse; lethargy; and warm, dry skin 2. Elevated pulse; lethargy; and warm, dry skin 3. Elevated pulse; shakiness; and cool, clammy skin 4. Slow pulse, confusion, and increased urine output
Ans. 3 rationale Symptoms of mild hypoglycemia include tachycardia; shakiness; and cool, clammy skin. Options 1, 2, and 4 are not symptoms of hypoglycemia.
A nurse is providing instructions to a client newly diagnosed with diabetes mellitus. The nurse gives the client a list of the signs of hyperglycemia. Which of the following specific signs of this complication should be included on the list? 1. Decreased urine output 2. Profuse sweating 3. Increased thirst 4. Shakiness
Ans. 3 rationale The classic signs of hyperglycemia include polydipsia, polyuria, and polyphagia. Profuse sweating and shakiness would be noted in a hypoglycemic condition.
A client with Graves' disease has exophthalmos and is experiencing photophobia. Which intervention would best assist the client with this problem? 1. Administering methimazole (Tapazole) every 8 hours 2. Lubricating the eyes with tap water every 2 to 4 hours 3. Instructing the client to avoid straining or heavy lifting 4. Obtaining dark glasses for the client
Ans. 4 rationale Because photophobia (light intolerance) accompanies this disorder, dark glasses are helpful in alleviating the symptom. Medical therapy for Graves' disease does not help alleviate the clinical manifestation of exophthalmos. Other interventions may be used to relieve the drying that occurs from not being able to completely close the eyes; however, the question is asking what the nurse can do for photophobia. Tap water, which is hypotonic, could actually cause more swelling to the eye because it could pull fluid into the interstitial space. In addition, the client is at risk for developing an eye infection because the solution is not sterile. There is no need to prevent straining with exophthalmos.
An adult client just admitted to the hospital with heart failure also has a history of diabetes mellitus. The nurse calls the health care provider to verify a prescription for which medication that the client was taking before admission? 1. NPH insulin 2. Regular insulin 3. Acarbose (Precose) 4. Chlorpropamide
Ans. 4 rationale Chlorpropamide is an oral hypoglycemic agent that exerts an antidiuretic effect and should be administered cautiously or avoided in the client with cardiac impairment or fluid retention. It is a first-generation sulfonylurea. Insulin does not cause or aggravate fluid retention. Acarbose is a miscellaneous oral hypoglycemic agent.
A nurse is caring for a client experiencing thyroid storm. Which of the following would be a priority concern for this client? 1. Inability to cope with the treatment plan 2. Lack of sexual drive 3. Self-consciousness about body appearance 4. Potential for cardiac disturbances
Ans. 4 rationale Clients in thyroid storm are experiencing a life-threatening event, which is associated with uncontrolled hyperthyroidism. It is characterized by high fever, severe tachycardia, delirium, dehydration, and extreme irritability. The signs and symptoms of the disorder develop quickly, and therefore emergency measures must be taken to prevent death. These measures include maintaining hemodynamic status and patency of airway as well as providing adequate ventilation. Options 1, 2, and 3 are not a priority in the care of the client in thyroid storm.
A nursing student notes in the medical record that a client with Cushing's syndrome is experiencing body image disturbances. The need for additional education regarding this problem is identified when the nursing student suggests which nursing intervention? 1. Encouraging the client's expression of feelings 2. Evaluating the client's understanding of the disease process 3. Encouraging family members to share their feelings about the disease process 4. Evaluating the client's understanding that the body changes need to be dealt with
Ans. 4 rationale Evaluating the client's understanding that the body changes that occur in this disorder need to be dealt with is an inappropriate nursing intervention. This option does not address the client's feelings. Options 1, 2, and 3 are appropriate because they address the client and family feelings regarding the disorder.
A nurse is caring for a postoperative adrenalectomy client. Which of the following does the nurse specifically monitor for in this client? 1. Peripheral edema 2. Bilateral exophthalmos 3. Signs and symptoms of hypocalcemia 4. Signs and symptoms of hypovolemia
Ans. 4 rationale Following adrenalectomy, the client is at risk for hypovolemia. Aldosterone, secreted by the adrenal cortex, plays a major role in fluid volume balance by retaining sodium and water. A deficiency of adrenocortical hormones does not cause the clinical manifestations noted in options 1, 2, and 3.
A nurse is monitoring a client receiving glipizide (Glucotrol). Which outcome indicates an ineffective response from the medication? 1. A decrease in polyuria 2. A decrease in polyphagia 3. A fasting plasma glucose of 100 mg/dL 4. A glycosylated hemoglobin level of 12%
Ans. 4 rationale Glipizide (Glucotrol) is an oral hypoglycemic agent administered to decrease the serum glucose level and the signs and symptoms of hyperglycemia. Therefore, a decrease in both polyuria and polyphagia would indicate a therapeutic response. Laboratory values are also used to monitor a client's response to treatment. A fasting blood glucose level of 100 mg/dL is within normal limits. However, glycosylated hemoglobin of 12% indicates poor glycemic control.
The nurse caring for a client who has had a subtotal thyroidectomy reviews the plan of care and determines which problem is the priority for this client in the immediate postoperative period? 1. Dehydration 2. Infection 3. Urinary retention 4. Bleeding
Ans. 4 rationale Hemorrhage is one of the most severe complications that can occur following thyroidectomy. The nurse must frequently check the neck dressing for bleeding and monitor vital signs to detect early signs of hemorrhage, which could lead to shock. T3 and T4 do not regulate fluid volumes in the body. Infection is a concern for any postoperative client but is not the priority in the immediate postoperative period. Urinary retention can occur in postoperative clients as a result of medication and anesthesia but is not the priority from the options provided.
A nurse is caring for a client following a thyroidectomy. The client tells the nurse that she is concerned because of voice hoarseness. The client asks the nurse whether the hoarseness will subside. The nurse appropriately tells the client that the hoarseness: 1. Indicates nerve damage 2. Is harmless but permanent 3. Will worsen before it subsides 4. Is normal and will gradually subside
Ans. 4 rationale Hoarseness that develops in the postoperative period is usually the result of laryngeal pressure or edema and will resolve within a few days. The client should be reassured that the effects are transitory. Options 1, 2, and 3 are incorrect.
A client who is managing diabetes mellitus with insulin injections asks the nurse for information about any necessary changes in her diet to avoid hyperinsulinism. Which of the following diets would be appropriate for the client? 1. Low-fiber, high-fat diet 2. Limit carbohydrate intake to three meals per day 3. Large amounts of carbohydrates between low protein meals 4. Small frequent meals with protein, fat, and carbohydrates at each meal
Ans. 4 rationale The definition of hyperinsulinism is an excessive insulin secretion in response to carbohydrate-rich foods leading to hypoglycemia. It is often treated with a diet that provides for limited stimulation of the pancreas. Carbohydrates can produce a rapid rise in blood glucose levels. However, carbohydrates are necessary in the diet. Proteins do not stimulate insulin secretion. Fats are needed in the diet to provide calories. The best diet for hyperinsulinism will contain proteins and fats whenever carbohydrates are consumed and delivered in frequent but portion-controlled meals. Diets high in soluble fiber may be beneficial.
A client is brought to the emergency department in an unresponsive state, and a diagnosis of hyperglycemic hyperosmolar nonketotic syndrome (HHNS) is made. The nurse who is assisting to care for the client obtains which of the following immediately in preparation for the treatment of this syndrome? 1. NPH insulin 2. A nasal cannula 3. Intravenous (IV) infusion of sodium bicarbonate 4. IV infusion of normal saline
Ans. 4 rationale The primary goal of treatment is to rehydrate the client to restore fluid volume and to correct electrolyte deficiency. IV fluid replacement is similar to that administered in diabetic ketoacidosis (DKA) and begins with IV infusion of normal saline. Regular insulin, not NPH insulin would be administered. The use of sodium bicarbonate to correct acidosis is avoided because it can precipitate a further drop in serum potassium levels. A nasal cannula for oxygen administration is not necessarily required to treat HHNS.
A nurse is reviewing discharge teaching with a client who has Cushing's syndrome. Which statement by the client indicates that the instructions related to dietary management were understood? 1. "I can eat foods that contain potassium." 2. "I will need to limit the amount of protein in my diet." 3. "I am fortunate that I can eat all the salty foods I enjoy." 4. "I am fortunate that I do not need to follow any special diet."
Ans.1 rationale A diet that is low in calories, carbohydrates, and sodium but ample in protein and potassium content is encouraged for a client with Cushing's syndrome. Such a diet promotes weight loss, the reduction of edema and hypertension, the control of hypokalemia, and the rebuilding of wasted tissue.
A client with a diagnosis of diabetic ketoacidosis (DKA) is being treated in the emergency department. Which of the following findings would the nurse expect to note as confirming this diagnosis? 1. Elevated blood glucose and low plasma bicarbonate 2. Decreased urine output 3. Increased respirations and an increase in pH 4. Coma
Ans.1 rationale In DKA, the arterial pH is less than 7.35, plasma bicarbonate is less than 15 mEq/L, the blood glucose level is higher than 250 mg/dL, and ketones are present in the blood and urine. The client would be experiencing polyuria and Kussmaul's respirations. Coma may occur if DKA is not treated, but coma would not confirm the diagnosis.
A nursing instructor asks a student to describe the pathophysiology that occurs in Cushing's disease. Which statement by the student indicates an accurate understanding of this disorder? 1. "Cushing's disease is characterized by an oversecretion of insulin." 2. "Cushing's disease is characterized by an oversecretion of glucocorticoid hormones." 3. "Cushing's disease is characterized by an undersecretion of corticotropic hormones." 4. "Cushing's disease is characterized by an undersecretion of glucocorticoid hormones."
Ans.2 rationale Cushing's syndrome is characterized by an oversecretion of glucocorticoid hormones. Addison's disease is characterized by the failure of the adrenal cortex to produce and secrete adrenocortical hormones. Options 1 and 4 are inaccurate regarding Cushing's syndrome.
Which of the following clients is at risk for developing thyrotoxicosis? 1. A client with hypothyroidism 2. A client with Graves' disease who is having surgery 3. A client with diabetes mellitus scheduled for debridement of a foot ulcer 4. A client with diabetes insipidus scheduled for an invasive diagnostic test
Ans.2 rationale Thyrotoxicosis is usually seen in clients with Graves' disease with the symptoms precipitated by a major stressor. This complication typically occurs during periods of severe physiological or psychological stress such as trauma, sepsis, the birth process, or major surgery. It also must be recognized as a potential complication following a thyroidectomy.
A nurse is reinforcing instructions with a client with diabetes mellitus who is recovering from diabetic ketoacidosis (DKA) regarding measures to prevent a recurrence. Which instruction is important for the nurse to emphasize? 1. Eat six small meals daily. 2. Test the urine ketone levels. 3. Monitor blood glucose levels frequently. 4. Receive appropriate follow-up health care.
Ans.3 rationale Client education after DKA should emphasize the need for home glucose monitoring four to five times per day. It is also important to instruct the client to notify the health care provider when illness occurs. The presence of urinary ketones indicates that DKA has already occurred. The client should eat well-balanced meals with snacks, as prescribed.
Following hypophysectomy, a client complains of being very thirsty and having to urinate frequently. The initial nursing action is to: 1. Document the complaints. 2. Increase fluid intake. 3. Check the urine specific gravity. 4. Check for urinary glucose.
Ans.3 rationale Following hypophysectomy, diabetes insipidus can occur temporarily because of antidiuretic hormone deficiency. This deficiency is related to surgical manipulation. The nurse should check the urine for specific gravity and report the results if they are less than 1.005. Urinary glucose and diabetes mellitus is not a concern here. In this situation, increasing fluid intake would require a health care provider's prescription. The client's complaint would be documented but not as an initial action.
A nurse assists in developing a plan of care for a client with hyperparathyroidism receiving calcitonin-human (Cibacalcin). Which outcome has the highest priority regarding this medication? 1. Relief of pain 2. Absence of side effects 3. Reaching normal serum calcium levels 4. Verbalization of appropriate medication knowledge
Ans.3 rationale Hypercalcemia can occur in clients with hyperparathyroidism, and calcitonin is used to lower plasma calcium level. The highest priority outcome in this client situation would be a reduction in serum calcium level. Option 1 is unrelated to this medication. Although options 2 and 4 are expected outcomes, they are not the highest priority for administering this medication.
A newly diagnosed type 1 diabetic patient likes to run 3 miles several mornings a week. Which teaching will the nurse implement about exercise for this patient? a. "You should not take the morning NPH insulin before you run." b. "Plan to eat breakfast about an hour before your run." c. "Afternoon running is less likely to cause hypoglycemia." d. "You may want to run a little farther if your glucose is very high."
B Rationale: Blood sugar increases after meals, so this will be the best time to exercise. NPH insulin will not peak until mid-afternoon and is safe to take before a morning run. Running can be done in either the morning or afternoon. If the glucose is very elevated, the patient should postpone the run.
A nurse would expect to note which interventions in the plan of care for a client with hypothyroidism? Select all that apply. 1. Provide a cool environment for the client. 2. Instruct the client to consume a high-fat diet. 3. Instruct the client about thyroid replacement therapy. 4. Encourage the client to consume fluids and high-fiber foods in the diet. 5. Instruct the client to contact the health care provider if episodes of chest pain occur. 6. Inform the client that iodine preparations will be prescribed to treat the disorder.
Ans.3,4,5 rationale The clinical manifestations of hypothyroidism are the result of decreased metabolism from low levels of thyroid hormone. Interventions are aimed at replacement of the hormones and providing measures to support the signs and symptoms related to a decreased metabolism. The nurse encourages the client to consume a well-balanced diet that is low in fat for weight reduction and high in fluids and high-fiber foods to prevent constipation. The client often has cold intolerance and requires a warm environment. The client would notify the health care provider if chest pain occurs since it could be an indication of overreplacement of thyroid hormone. Iodine preparations are used to treat hyperthyroidism. These medications decrease blood flow through the thyroid gland and reduce the production and release of thyroid hormone.
A nurse enters the room of a client with type 1 diabetes mellitus and finds the client difficult to arouse. The client's skin is warm and flushed, and the pulse and respiratory rate are elevated from the client's baseline. The nurse would immediately: 1. Prepare for the administration of an insulin drip. 2. Give the client a glass of orange juice. 3. Prepare for the administration of a bolus dose of 50% dextrose. 4. Check the client's capillary blood glucose.
Ans.4 rationale The nurse must first obtain a blood glucose reading to determine the client's problem. Options 2 and 3 would be implemented as needed in the treatment of hypoglycemia. Insulin therapy is guided by blood glucose measurement.
A patient is admitted to the hospital with a diagnosis of primary hyperparathyroidism. A nurse checking the patient's lab results would expect which of the following changes in laboratory findings? A. Elevated serum calcium. B. Low serum parathyroid hormone (PTH). C. Elevated serum vitamin D. D. Low urine calcium.
Answer: A The parathyroid glands regulate the calcium level in the blood. In hyperparathyroidism, the serum calcium level will be elevated. Parathyroid hormone levels may be high or normal but not low. The body will lower the level of vitamin D in an attempt to lower calcium. Urine calcium may be elevated, with calcium spilling over from elevated serum levels. This may cause renal stones.
When obtaining a health history from a patient with acute pancreatitis, the nurse asks the patient specifically about a history of A. smoking B. alcohol use C. diabetes mellitus D. high-fat dietary intake
Answer: B pancreatitis is associated with alcoholism
A patient is being discharged from the emergency department after being treated for epistaxis. In teaching the family first aid measures in the event the epistaxis would recur, what measures should the nurse suggest (select all that apply)? Tilt patient's head backwards. Apply ice compresses to the nose. Tilt head forward while lying down. Pinch the entire soft lower portion of the nose. Partially insert a small gauze pad into the bleeding nostril.
Apply ice compresses to the nose. Pinch the entire soft lower portion of the nose. First aid measures to control epistaxis include placing the patient in a sitting position, leaning forward. Pinching the soft lower portion of the nose or inserting a small gauze pad into the bleeding nostril should stop the bleeding within 15 minutes. Tilting the head back or forward does not stop the bleeding, but rather allows the blood to enter the nasopharynx, which could result in aspiration or nausea/vomiting from swallowing blood. Lying down also will not decrease the bleeding.
D (Feedback: The nurse informs the patient about the need to take the full course of any prescribed antibiotic. Antibiotics should be taken for the entire 10-day period to eliminate the microorganisms. A patient should never be instructed to keep leftover antibiotics for use at a later time. Even if the fever or other symptoms are gone, the medications should be continued. Antibiotics do not need to be disposed of in a biohazard receptacle, though they should be discarded appropriately.)
As a clinic nurse, you are caring for a patient who has been prescribed an antibiotic for tonsillitis and has been instructed to take the antibiotic for 10 days. When you do a follow-up call with this patient, you are informed that the patient is feeling better and is stopping the medication after taking it for 4 days. What information should you provide to this patient? A) Keep the remaining tablets for an infection at a later time. B) Discontinue the medications if the fever is gone. C) Dispose of the remaining medication in a biohazard receptacle. D) Finish all the antibiotics to eliminate the organism completely.
What interventions should the nurse implement for a closed drainage system?
Assess dressing - It must be kept tight and intact Listen to lung sounds bilaterally Monitor pulse oximetry and report anything <90 Record drainage every hour for 24 hours and then every 8 hours Notify primary healthcare provider of 100 mL of drainage or greater in one hour, and if there is a change in color to bright red. Deep breathe, cough, and use incentive spirometer What for fever, increased WBCs, and drainage because they could develop infection at the insertion site Watch daily for chest x-rays for re-expansion
1. A pt with suspected acromegaly is seen at the clinic. To assist in making the diagnosis, which question should the nurse ask? a. "Have you had a recent head injury?" b. "Do you have to wear larger shoes now?" c. "Is there any family history of acromegaly?" d. "Are you experiencing tremors or anxiety?"
B Rationale: Acromegaly causes an enlargement of the hands and feet. Head injury and family history are not risk factors for acromegaly. Tremors and anxiety are not clinical manifestations of acromegaly. (Cognitive Level: Application Text Reference: p. 1291 NProcess: Assessment NCLEX: Physiological Integrity)
A patient with type 1 diabetes who uses glargine (Lantus) and lispro (Humalog) insulin develops a sore throat, cough, and fever. When the patient calls the clinic to report the symptoms and a blood glucose level of 210 mg/dl, the nurse advises the patient to a. use only the lispro insulin until the symptoms of infection are resolved. b. monitor blood glucose every 4 hours and notify the clinic if it continues to rise. c. decrease intake of carbohydrates until glycosylated hemoglobin is less than 7%. d. limit intake to non-calorie-containing liquids until the glucose is within the usual range.
B Rationale: Infection and other stressors increase blood glucose levels and the patient will need to test blood glucose frequently, treat elevations appropriately with insulin, and call the health care provider if glucose levels continue to be elevated. Discontinuing the glargine will contribute to hyperglycemia and may lead to DKA. Decreasing carbohydrate or caloric intake is not appropriate as the patient will need more calories when ill. Glycosylated hemoglobins are not used to test for short-term alterations in blood glucose.
A patient with cirrhosis has a massive hemorrhage from esophageal varices. In planning care for the patient, the nurse gives the highest priority to the goal of a. controlling bleeding. b. maintenance of the airway. c. maintenance of fluid volume. d. relieving the patient's anxiety.
B Rationale: Maintaining gas exchange has the highest priority because oxygenation is essential for life. The airway is compromised by the bleeding in the esophagus and aspiration easily occurs. The other goals would also be important for this patient, but they are not as high a priority as airway maintenance.
A patient with cirrhosis has 4+ pitting edema of the feet and legs and massive ascites. The data indicate that it is most important for the nurse to monitor the patient's a. temperature. b. albumin level. c. hemoglobin. d. activity level.
B Rationale: The low oncotic pressure caused by hypoalbuminemia is a major pathophysiologic factor in the development of ascites and edema. The other parameters should also be monitored, but they are not contributing factors to the patient's current symptoms.
A patient with type 2 diabetes has sensory neuropathy of the feet and legs and peripheral vascular disease evidenced by decreased peripheral pulses and dependent rubor. The nurse teaches the patient that a. the feet should be soaked in warm water on a daily basis. b. flat-soled leather shoes are the best choice to protect the feet from injury. c. heating pads should always be set at a very low temperature. d. over-the-counter (OTC) callus remover may be used to remove callus and prevent pressure.
B Rationale: The patient is taught to avoid high heels and that leather shoes are preferred. The feet should be washed, but not soaked, in warm water daily. Heating pad use should be avoided. Commercial callus and corn removers should be avoided; the patient should see a specialist to treat these problems.
A patient is admitted with an abrupt onset of jaundice, nausea and vomiting, hepatomegaly, and abnormal liver function studies. Serologic testing is negative for viral causes of hepatitis. Which question by the nurse is most appropriate? a. "Have you been around anyone with jaundice?" b. "Do you use any prescription or over-the-counter (OTC) drugs?" c. "Are you taking corticosteroids for any reason?" d. "Is there any history of IV drug use?"
B Rationale: The patient's symptoms, lack of antibodies for hepatitis, and the ABRUPT onset of symptoms suggest toxic hepatitis, which can be caused by commonly used OTC drugs such as acetaminophen (Tylenol). Exposure to a jaundiced individual and a history of IV drug use are risk factors for VIRAL hepatitis. Corticosteroid use does not cause the symptoms listed.
The nurse identifies a nursing diagnosis of risk for impaired skin integrity for a patient with cirrhosis who has ascites and 4+ pitting edema of the feet and legs. An appropriate nursing intervention for this problem is to a. restrict dietary protein intake. b. arrange for a pressure-relieving mattress. c. perform passive range of motion QID. d. turn the patient every 4 hours.
B Rationale: The pressure-relieving mattress will decrease the risk for skin breakdown for this patient. Dietary protein intake may be increased in patients with ascites to improve oncotic pressure. Turning the patient every 4 hours will not be adequate to maintain skin integrity. Passive range of motion will not take pressure off areas like the sacrum that are vulnerable to breakdown.
33. A pt is taking high doses of prednisone to control the symptoms of an acute exacerbation of systemic lupus erythematosus. When teaching the pt about use of prednisone, which information is most important for the nurse to include? a. Call the doctor if you experience any mood alterations with the prednisone. b. Do not stop taking the prednisone suddenly; it should be decreased gradually. c. Weigh yourself daily to monitor for weight gain caused by water or increased fat. d. Check your temperature daily because prednisone can hide signs of infection.
B R: Acute adrenal insufficiency may occur if exogenous glucocorticoids are suddenly stopped. Mood alterations and weight gain are possible adverse effects of glucocorticoid use, but these are not life-threatening effects. Glucocorticoids do mask the signs of infection, but temperature elevation tends to be suppressed, so other signs of infection should be monitored. (Cognitive Level: Application Text Reference: p. 1314 NProcess: Implementation NCLEX: Physiological Integrity)
17. The first nursing action indicated when a patient returns to the surgical nursing unit following a thyroidectomy is to a. check the dressing for bleeding. b. assess respiratory rate and effort. c. support the patient's head with pillows. d. take the blood pressure and pulse.
B R: Airway obstruction is a possible complication after thyroidectomy because of swelling or bleeding at the site or tetany, and priority nursing action is to assess airway. The other actions are also part of the standard nursing care post-thyroidectomy but are not as high in priority.
30. A pt is hospitalized with acute adrenal insufficiency. The nurse determines that the pt is responding favorably to treatment upon finding a. decreasing serum sodium. b. decreasing serum potassium. c. decreasing blood glucose. d. increasing urinary output.
B R: CMs of Addison's disease include hyperkalemia and a decrease in potassium level indicates improvement. Decreasing serum sodium and decreasing blood glucose indicate that treatment has not been effective. Changes in urinary output are not an effective way of monitoring treatment for Addison's disease.
23. Following a thyroidectomy, a patient develops carpal spasm while the nurse is taking a blood pressure on the left arm. Which action by the nurse is appropriate? a. Administer the ordered muscle relaxant. b. Have the patient rebreathe using a paper bag. c. Start oxygen at 2 to 3 L/min per cannula. d. Give the ordered oral calcium supplement.
B R: Carpal spasm after a thyroidectomy suggests that pt has hypocalcaemia caused by damage to the parathyroid glands. The symptoms of hypocalcemia will be temporarily reduced by having the patient breath into a paper bag, which will raise the PaCO2 and create a more acidic pH. The muscle relaxant will not impact on ionized calcium level. There is no indication that the patient is experiencing laryngeal stridor or needs oxygen. IV calcium supplements will be given to normalize calcium level quickly.
28. When providing postoperative care for a patient who has had bilateral adrenalectomy, which assessment information obtained by the nurse is most important to communicate to HCP? a. The blood glucose is 156 mg/dl. b. The patient's blood pressure is 102/50. c. The patient has 5/10 incisional pain. d. The lungs have bibasilar crackles.
B R: During immediate postoperative period, marked fluctuation in cortisol levels may occur and the nurse must be alert for signs of acute adrenal insufficiency such as hypotension. nurse should also address elevated glucose, incisional pain, and crackles with appropriate collaborative or nursing actions, but prevention and treatment of acute adrenal insufficiency is the priority after adrenalectomy.
39. When developing a plan of care for a pt with SIADH, which interventions will the nurse include? a. Encourage fluids to 2000 ml/day. b. Offer patient hard candies to suck on. c. Monitor for increased peripheral edema. d. Keep head of bed elevated to 30 degrees.
B R: Sucking on hard candies decreases thirst for patient on a fluid restriction. Pts with SIADH are on fluid restrictions of 800-1000 ml/day. Peripheral edema isnt seen w SIADH. HOB is elevated no more than 10 degrees to increase left atrial filling pressure and decrease ADH release. (Cognitive Level: Application Text Reference: p. 1296 NProcess: Planning NCLEX: Physiological Integrity)
31. A pt is admitted to the hospital in addisonian crisis 1 month after a diagnosis of Addison's disease. The nurse identifies the nursing diagnosis of ineffective therapeutic regimen management related to lack of knowledge of management of condition when the patient says, a. "I double my dose of hydrocortisone on the days that I go for a run." b. "I had the stomach flu earlier this week and couldn't take the hydrocortisone." c. "I frequently eat at restaurants, and so my food has a lot of added salt." d. "I do yoga exercises almost every day to help me reduce stress and relax."
B R: The need for hydrocortisone replacement is increased with stressors such as illness, and the patient needs to be taught to call the health care provider because medication and IV fluids and electrolytes may need to be given. The other patient statements indicate appropriate management of the Addison's disease.
16. The nurse identifies a nursing dx of risk for injury: corneal ulceration related to inability to close the eyelids secondary to exophthalmos for a patient with Graves' disease. An appropriate nursing intervention for this problem is to a. teach the patient to blink every few seconds to lubricate the cornea. b. elevate the head of the patient's bed to reduce periorbital fluid. c. apply eye patches to protect the cornea from irritation. d. place cold packs on the eyes to relieve pain and swelling.
B R: The patient should sit upright as much as possible to promote fluid drainage from the periorbital area. With exophthalmos, the pt is unable to close eyes completely. Lubrication of the eyes, rather than eye patches, will protect the eyes from developing corneal scarring. The swelling of the eye is not caused by excessive blood flow to the eye, so cold packs will not be helpful.
15. A few hours after returning to the surgical nursing unit, a patient who has undergone a subtotal thyroidectomy develops laryngeal stridor and a cramp in the right hand. The nurse anticipates that intervention will include a. administration of IV morphine. b. administration of IV calcium gluconate. c. endotracheal intubation with mechanical ventilation. d. immediate tracheostomy and manual ventilation.
B R: The pt's CMs are consistent with tetany caused by hypocalcemia resulting from damage to the parathyroid glands during surgery. Tracheostomy may be needed if the calcium does not resolve the stridor. There is no indication that morphine is needed. Endotracheal intubation may be done, but only if calcium is not effective in correcting stridor
2. During preoperative teaching for a patient scheduled for transsphenoidal hypophysectomy for treatment of a pituitary adenoma, the nurse instructs the patient about the need to a. remain on bed rest for the first 48 hours after the surgery. b. avoid brushing the teeth for at least 10 days after the surgery. c. cough and deep-breathe every 2 hrs postoperatively. d. be positioned flat with sandbags at the head postoperatively.
B R: To avoid disruption of the suture line, the patient should avoid brushing the teeth for 10 days after surgery. It is not necessary to remain on bed rest after this surgery. Coughing is discouraged because it may cause leakage of cerebrospinal fluid (CSF) from the suture line. The head of the bed should be elevated 30 degrees to reduce pressure on the sella turcica and decrease the risk for headaches. (Cognitive Level: Application Text Reference: p. 1293 NProcess: Implementation NCLEX: Physiological Integrity)
20. While hospitalized for a fractured femur, a 68-year-old pt is diagnosed with hypothyroidism. Which of these medications on the original admission orders will the nurse need to consult with the HCP about before it is administered? a. Docusate (Colace) b. Diazepam (Valium) c. Ibuprofen (Motrin) d. Cefoxitin (Mefoxin)
B R: Worsening of mental status and myxedema coma can be precipitated by the use of sedatives, especially in older adults. The nurse should discuss the diazepam with the health care provider before administration. The other medications may safely be given to the pt.
21. When teaching a patient with newly diagnosed hypothyroidism about management of the condition, the nurse should a. delay teaching about the condition until the patient has responded to replacement therapy. b. provide written handouts of all instructions for continued reference as the patient improves. c. have a family member teach the patient about the condition when the patient is more alert. d. arrange for daily home visits by home health nurses to repeat the necessary instructions.
B R: Written instructions will be helpful to the patient because initially the hypothyroid patient may be unable to remember to take medications and other aspects of self-care. Teaching should not be delayed, but family members or friends should be included in teaching to assist the patient. The nurse, not a family member, is responsible for patient teaching. Because thyroid replacement does not begin to improve alertness immediately, it is not appropriate to schedule daily home health visits for teaching.
4. A pt is suspected of having a pituitary tumor causing panhypopituitarism. During assessment of the pt, the nurse would expect to find a. elevated blood glucose. b. changes in secondary sex characteristics. c. high blood pressure. d. tachycardia and cardiac palpitations.
B Rationale: Changes in secondary sex characteristics are associated with decreases in FSH and LH. Fasting hypoglycemia and hypotension occur in panhypopituitarism as a result of decreases in ACTH and cortisol. Bradycardia is likely due to the decrease in TSH and thyroid hormones associated with panhypopituitarism. (Cognitive Level: Application Text Reference: p. 1294 NProcess: Assessment NCLEX: Physiological Integrity)
A diabetic patient is started on intensive insulin therapy. The nurse will plan to teach the patient about mealtime coverage using _____ insulin. a. NPH b. lispro c. detemir d. glargine
B Rationale: Rapid or short acting insulin is used for mealtime coverage for patients receiving intensive insulin therapy. NPH, glargine, or detemir will be used as the basal insulin.
9. A finding indicating to the nurse that a 22-year-old patient with respiratory distress is in acute respiratory failure includes a a. shallow breathing pattern. b. partial pressure of arterial oxygen (PaO2) of 45 mm Hg. c. partial pressure of carbon dioxide in arterial gas (PaCO2) of 34 mm Hg. d. respiratory rate of 32/min.
B Rationale: The PaO2 indicates severe hypoxemia and that the nurse should take immediate action to correct this problem. Shallow breathing, rapid respiratory rate, and low PaCO2 can be caused by other factors, such as anxiety or pain. Cognitive Level: Application Text Reference: p. 1806 Nursing Process: Assessment NCLEX: Physiological Integrity
A 32-year-old patient has early alcoholic cirrhosis diagnosed by a liver biopsy. When planning patient teaching, the priority information for the nurse to include is the need for a. vitamin B supplements. b. abstinence from alcohol. c. maintenance of a nutritious diet. d. long-term, low-dose corticosteroids.
B Rationale: The disease progression can be stopped or reversed by alcohol abstinence. The other interventions may be used when cirrhosis becomes more severe to decrease symptoms or complications, but the priority for this patient is to stop the progression of the disease.
13. When the nurse is caring for an obese patient with left lower-lobe pneumonia, gas exchange will be best when the patient is positioned a. on the left side. b. on the right side. c. in the high-Fowler's position. d. in the tripod position.
B Rationale: The patient should be positioned with the "good" lung in the dependent position to improve the match between ventilation and perfusion. The obese patient's abdomen will limit respiratory excursion when sitting in the high-Fowler's or tripod positions. Cognitive Level: Comprehension Text Reference: pp. 1809-1810 Nursing Process: Implementation NCLEX: Physiological Integrity
23. The nurse obtains the vital signs for a patient admitted 2 days ago with gram-negative sepsis: temperature 101.2° F, blood pressure 90/56 mm Hg, pulse 92, respirations 34. Which action should the nurse take next? a. Notify the health care provider of the patient's vital signs. b. Obtain oxygen saturation using pulse oximetry. c. Document the vital signs and continue to monitor. d. Administer PRN acetaminophen (Tylenol) 650 mg.
B Rationale: The patient's increased respiratory rate in combination with the admission diagnosis of gram-negative sepsis indicates that acute respiratory distress syndrome (ARDS) may be developing; the nurse should check for hypoxemia, a hallmark of ARDS. The health care provider should be notified after further assessment of the patient. Documentation and continued monitoring of the vital signs are needed but do not constitute an adequate response to the patient situation. Tylenol administration is appropriate but not the highest priority for this patient. Cognitive Level: Application Text Reference: pp. 1813-1814 Nursing Process: Implementation NCLEX: Physiological Integrity
11. A patient with hypercapnic respiratory failure has a respiratory rate of 8 and an SpO2 of 89%. The patient is increasingly lethargic. Which collaborative intervention will the nurse anticipate? a. Administration of 100% oxygen by non-rebreather mask b. Endotracheal intubation and positive pressure ventilation c. Insertion of a mini-tracheostomy with frequent suctioning d. Initiation of bilevel positive pressure ventilation (BiPAP)
B Rationale: The patient's lethargy, low respiratory rate, and SpO2 indicate the need for mechanical ventilation with ventilator-controlled respiratory rate. Administration of high flow oxygen will not be helpful because the patient's respiratory rate is so low. Insertion of a mini-tracheostomy will facilitate removal of secretions, but it will not improve the patient's respiratory rate or oxygenation. BiPAP requires that the patient initiate an adequate respiratory rate to allow adequate gas exchange. Cognitive Level: Application Text Reference: pp. 1807-1808, 1810 Nursing Process: Planning NCLEX: Physiological Integrity
After 2 months of tuberculosis (TB) treatment with a standard four-drug regimen, a patient continues to have positive sputum smears for acid-fast bacilli (AFB). Which action should the nurse take next? A) Discuss the need to use some different medications to treat the TB. B) Ask the patient whether medications have been taken as directed. C) Schedule the patient for directly observed therapy three times weekly. D) Educate about using a 2-drug regimen for the last 4 months of treatment.
B) Ask the patient whether medications have been taken as directed. Feedback: The first action should be to determine whether the patient has been compliant with drug therapy because negative sputum smears would be expected if the TB bacillus is susceptible to the medications and if the medications have been taken correctly. Depending on whether the patient has been compliant or not, different medications or directly observed therapy may be indicated. A two-drug regimen will be used only if the sputum smears are negative for AFB.
When assessing a 24-year-old patient who has just arrived after an automobile accident, the emergency department nurse notes that the breath sounds are absent on the right side. The nurse will anticipate the need for A) Administration of an inhaled bronchodilator. B) Insertion of a chest tube with a chest drainage system. C) Emergency pericardiocentesis. D) Stabilization of the chest wall with tape.
B) Insertion of a chest tube with a chest drainage system. Feedback: The patient's history and absent breath sounds suggest a right-sided pneumothorax or hemothorax, which will require treatment with a chest tube and drainage. The other therapies would be appropriate for an acute asthma attack, flail chest, or cardiac tamponade, but the patient's clinical manifestations are not consistent with these problems.
If a patient states, "It's hard for me to breathe and I feel short-winded all the time," what is the most appropriate terminology to be applied in documenting this assessment by a nurse? A. Apnea B. Dyspnea C. Tachypnea D. Respiratory fatigue
B. Dyspnea Dyspnea is a subjective description reflective of the patient's statement indicating difficulty in breathing. Apnea refers to absence of breath or breathing. Tachypnea refers to an increased rate of breathing, usually greater than 20 breaths per minute. Respiratory fatigue is subjective and usually refers to the patient exhibiting signs and symptoms associated with a comprehensive respiratory assessment including laborious breathing, use of accessory muscles, and slowing of respirations.
Which of the following nursing interventions is of the highest priority in helping a patient expectorate thick secretions related to pneumonia? A. Humidify the oxygen as able B. Increase fluid intake to 3L/day if tolerated. C. Administer cough suppressant q4hr. D. Teach patient to splint the affected area.
B. Increase fluid intake to 3L/day if tolerated. Although several interventions may help the patient expectorate mucus, the highest priority should be on increasing fluid intake, which will liquefy the secretions so that the patient can expectorate them more easily. Humidifying the oxygen is also helpful, but is not the primary intervention. Teaching the patient to splint the affected area may also be helpful, but does not liquefy the secretions so that they can be removed.
Which of the following things must the nurse working with diabetic clients keep in mind about Hyperglycemic Hyperosmolar Nonketotic Syndrome (HHNS)? A. This syndrome occurs mainly in people with Type I Diabetes B. It has a higher mortality rate than Diabetic Ketoacidosis C. The client with HHNS is in a state of overhydration D. This condition develops very rapidly
B. It has a higher mortality rate than Diabetic Ketoacidosis HHNS occurs only in people with Type II Diabetes. It is a medical emergency and has a higher mortality rate than Diabetic Ketoacidosis. This condition develops very slowly over hours or days.
A nurse admits a client newly diagnosed with diabetes mellitus. When reviewing the client's lab work, the nurse notes that the result consistent with diabetic ketoacidosis is which of the following?
Bicarbonate level of 12 mEq/L A bicarbonate level of 12 mEq/L below the expected reference range. A bicarbonate of less than 15 mWq/L indicates metabolic acidosis, such as diabetic ketoacidosis.
A pt with SOB is being tested for lung cancer. Which diagnostic test will be most conclusive?
Biopsy
What is the pathophysiology of a hemothorax/pneumothorax?
Blood or air has accumulated in the pleural space The lung has collapsed
. A patient with cirrhosis who is being treated with spironolactone (Aldactone) and furosemide (Lasix) has a serum sodium level of 135 mEq/L (135 mmol/L) and serum potassium 3.2 mEq/L (3.2 mmol/L). Before notifying the health care provider, the nurse should a. administer the furosemide and withhold the spironolactone. b. give both drugs as scheduled. c. administer the spironolactone. d. withhold both drugs until talking with the health care provider.
C Rationale: Spironolactone is a potassium-sparing diuretic and will help to increase the patient's potassium level. The nurse does not need to talk with the doctor before giving the spironolactone, although the health care provider should be notified about the low potassium value. The furosemide will further decrease the patient's potassium level and should be held until the nurse talks with the health care provider.
A patient with severe cirrhosis has an episode of bleeding esophageal varices. To detect possible complications of the bleeding episode, it is most important for the nurse to monitor a. prothrombin time. b. bilirubin levels. c. ammonia levels. d. potassium levels.
C Rationale: The blood in the GI tract will be absorbed as protein and may result in an increase in ammonia level since the liver cannot metabolize protein well. The prothrombin time, bilirubin, and potassium levels should also be monitored, but these will not be affected by the bleeding episode.
The doctor is interested in how well a client has controlled their blood glucose since their last visit. What lab values could the nurse evaluate to determine how well the client has controlled their blood glucose over the past three months?
C HbgA1c is a blood test used to determine how well blood glucose has been controlled for the last three months.
37. After a patient with a pituitary adenoma has had a hypophysectomy, the nurse will plan to do discharge teaching about the need for a. insulin use to maintain blood glucose at normal levels. b. Na restriction to prevent fluid retention and hypertension. c. oral corticosteroids to replace endogenous cortisol. d. chemotherapy to prevent reoccurrence of tumor
C R: ADH, cortisol, and thyroid hormone replacement will be needed for life after hypophysectomy. Without the effects of ACTH and cortisol, the blood glucose and serum sodium will be low unless cortisol is replaced. An adenoma is a benign tumor, and chemotherapy will not be needed. (Cognitive Level: Application Text Reference: p. 1293 NProcess: Planning NCLEX: Physiological Integrity)
29. A patient with Cushing syndrome returns to the surgical unit following an adrenalectomy. During the initial postoperative period, the nurse gives the highest priority to a. monitoring for infection. b. protecting the patient's skin. c. maintaining fluid and electrolyte status. d. preventing severe emotional disturbances.
C R: After adrenalectomy, the pt is at risk for circulatory instability caused by fluctuating hormone levels, and the focus of care is to assess and maintain fluid and electrolyte status through the use of IV fluids and corticosteroids. other goals are also important for pt but arent as immediately life-threatening as circulatory collapse
13. During the nursing assessment of a patient with Graves' disease, the nurse notes a bounding, rapid pulse and systolic hypertension. Based on these assessment data, which question is important for the nurse to ask the patient? a. "Do you have any problem with frequent constipation?" b. "Have you noticed any recent decrease in your appetite?" c. "Do you ever have any chest pain?" d. "Have you had recent muscle aches?"
C R: Angina is a possible complication of Graves' disease, especially for a patient with tachycardia and hypertension. The other CMs are associated with hypothyroidism.
12. A patient with Graves' disease is prepared for surgery with drug therapy consisting of 4 weeks of propylthiouracil (PTU) and 10 days of iodine before surgery. When teaching the patient about the drugs, the nurse explains that the drugs are given preoperatively to a. eliminate the risk for tetany during the postoperative period. b. decrease the risk of hypometabolism during and after the surgery. c. normalize metabolism and decrease the size and vascularity of the gland. d. assist in differentiating the thyroid and parathyroid glands during surgery.
C R: Antithyroid drugs and iodine decrease the levels of thyroid hormone and the vascularity of the thyroid gland prior to surgery and lower the risk for postoperative thyrotoxicosis and hemorrhage. Postoperative tetany might be caused by removal of the parathyroid gland during thyroidectomy. The medications will tend to decrease metabolic rate. The medications will not help in differentiating the tissues of the thyroid and parathyroid glands.
6. A patient with an antidiuretic hormone (ADH)-secreting small-cell cancer of the lung is treated with demeclocycline (Declomycin) to control the symptoms of syndrome of inappropriate secretion of antidiuretic hormone (SIADH). The nurse determines that the demeclocycline is effective upon finding that the a. patient's daily weight is stable. b. urine specific gravity is increased. c. patient's urinary output is increased. d. peripheral edema is decreased.
C R: Demeclocycline blocks the action of ADH on the renal tubules and increases urine output. A stable body weight and an increase in urine specific gravity indicate that the SIADH is not corrected. Peripheral edema does not occur with SIADH; a sudden weight gain without edema is a common clinical manifestation of this disorder.
34. A patient has an adrenocortical adenoma causing hyperaldosteronism and is scheduled for laparoscopic surgery to remove the tumor. During care before surgery, the nurse should a. monitor blood glucose level every 4 hours. b. provide a potassium-restricted diet. c. monitor the blood pressure every 4 hours. d. relieve edema by elevating the extremities.
C R: HTN caused by Na retention is a common complication of hyperaldosteronism. Hyperaldosteronism does not cause elevation in blood glucose. pt will be hypokalemic and require potassium supplementation prior to surgery. Edema does not usually occur with hyperaldosteronism. (Cognitive Level: Application Text : pp. 1319-1320 NProcess: Implementation NCLEX: Physiological Integrity)
19. A 72-year-old patient is diagnosed with hypothyroidism, and levothyroxine (Synthroid) is prescribed. During initiation of thyroid replacement for the patient, it is most important for the nurse to assess a. mental status. b. nutritional level. c. cardiac function. d. fluid balance.
C R: In older patients, initiation of levothyroxine therapy can increase myocardial oxygen demand and cause angina or dysrhythmias. The medication is also expected to improve mental status and fluid balance and will increase metabolic rate and nutritional needs, but these changes do not indicate a need to change the therapy.
5. A pt seen at clinic for an upper respiratory infection reports receiving subcutaneous somatotropin (Genotropin) when asked by the nurse about current medications. The nurse questions the pt further about a hx of a. adrenal disease. b. untreated acromegaly. c. a pituitary tumor. d. diabetes insipidus (DI).
C R: Somatotropin is a recombinant growth hormone product used for adults with growth hormone deficiency, such as that caused by a pituitary tumor. The med is not used in adrenal disease or DI. The patient with untreated acromegaly will have an excess of growth hormone.
35. A pt with a possible pheochromocytoma is admitted to the hospital for evaluation and diagnostic testing. During an attack, the nurse will monitor for hypertension and a. hypoglycemia. b. bradycardia. c. headache. d. flushing.
C R: The classic CMs of pheochromocytoma are hypertension, tachycardia, severe headache, diaphoresis, and abdominal or chest pain. Elevated blood glucose may also occur due to sympathetic nervous system stimulation. Bradycardia and flushing would not be expected. (Cognitive Level: Application Text Reference: p. 1320 NProcess: Assessment NCLEX: Physiological Integrity)
14. While assessing a patient who has just arrived in the postanesthesia recovery unit (PACU) after a thyroidectomy, the nurse obtains these data. Which information is most important to communicate to the surgeon? a. The pt is complaining of 7/10 incisional pain. b. The pt's cardiac monitor shows a HR of 112. c. The patient has increasing swelling of the neck. d. The pat's voice is weak and hoarse sounding.
C R: The neck swelling may lead to respiratory difficulty, and rapid intervention is needed to prevent airway obstruction. The incisional pain should be treated but is not unusual after surgery. A heart rate of 112 is not unusual in a pt who has been hyperthyroid and has just arrived in the PACU from surgery. Vocal hoarseness is expected after surgery due to edema.
24. Which of these nursing actions included in the care of a mechanically ventilated patient with acute respiratory distress syndrome (ARDS) is most appropriate for the RN to delegate to an experienced LPN/LVN working in the intensive care unit? a. Placing the patient in the prone position b. Assessment of patient breath sounds c. Administration of enteral tube feedings d. Obtaining the pulmonary artery pressures
C Rationale: Administration of tube feedings is included in LPN/LVN education and scope of practice and can be safely delegated to an LPN/LVN who is experienced in caring for critically ill patients. Placing a patient who is on a ventilator in the prone position requires multiple staff and should be supervised by an RN. Assessment of breath sounds and obtaining pulmonary artery pressures require advanced assessment skills and should be done by the RN caring for a critically ill patient. Cognitive Level: Application Text Reference: pp. 1816-1818 Nursing Process: Implementation NCLEX: Safe and Effective Care Environment
A patient hospitalized with possible acute pancreatitis has severe abdominal pain and nausea and vomiting. The nurse would expect the diagnosis to be confirmed with laboratory testing that reveals elevated serum a. calcium. b. bilirubin. c. amylase. d. potassium.
C Rationale: Amylase is elevated early in acute pancreatitis. Changes in bilirubin, calcium, and potassium levels are not diagnostic for pancreatitis.
When assessing the neurologic status of a patient with a diagnosis of hepatic encephalopathy, the nurse asks the patient to a. stand on one foot. b. ambulate with the eyes closed. c. extend both arms. d. perform the Valsalva maneuver.
C Rationale: Extending the arms allows the nurse to check for asterixis, a classic sign of hepatic encephalopathy. The other tests might also be done as part of the neurologic assessment but would not be diagnostic for hepatic encephalopathy.
17. All the following medications are ordered for a mechanically ventilated patient with acute respiratory distress syndrome (ARDS) and acute renal failure. Which medication should the nurse discuss with the health care provider before administration? a. IV ranitidine (Zantac) 50 mg IV b. sucralfate (Carafate) 1 g per nasogastric tube c. IV gentamicin (Garamycin) 60 mg d. IV methylprednisolone (Solu-Medrol) 40 mg
C Rationale: Gentamicin, which is one of the aminoglycoside antibiotics, is potentially nephrotoxic, and the nurse should clarify the drug and dosage with the health care provider before administration. The other medications are appropriate for the patient with ARDS. Cognitive Level: Application Text Reference: p. 1816 Nursing Process: Implementation NCLEX: Physiological Integrity
A type 1 diabetic patient who was admitted with severe hypoglycemia and treated tells the nurse, "I did not have any of the usual symptoms of hypoglycemia." Which question by the nurse will help identify a possible reason for the patient's hypoglycemic unawareness? a. "Do you use any calcium-channel blocking drugs for blood pressure?" b. "Have you observed any recent skin changes?" c. "Do you notice any bloating feeling after eating?" d. "Have you noticed any painful new ulcerations or sores on your feet?"
C Rationale: Hypoglycemic unawareness is caused by autonomic neuropathy, which would also cause delayed gastric emptying. Calcium-channel blockers are not associated with hypoglycemic unawareness, although -adrenergic blockers can prevent patients from having symptoms of hypoglycemia. Skin changes can occur with diabetes, but these are not associated with autonomic neuropathy. If the patient can feel painful areas on the feet, neuropathy has not occurred.
1. It will be most important for the nurse to check pulse oximetry for which of these patients? a. A patient with emphysema and a respiratory rate of 16 b. A patient with massive obesity who is refusing to get out of bed c. A patient with pneumonia who has just been admitted to the unit d. A patient who has just received morphine sulfate for postoperative pain
C Rationale: Hypoxemia and hypoxemic respiratory failure are caused by disorders that interfere with the transfer of oxygen into the blood, such as pneumonia. The other listed disorders are more likely to cause problems with hypercapnia because of ventilatory failure. Cognitive Level: Application Text Reference: pp. 1799-1800 Nursing Process: Assessment NCLEX: Physiological Integrity
3. When a patient is diagnosed with pulmonary fibrosis, the nurse will teach the patient about the risk for poor oxygenation because of a. too-rapid movement of blood flow through the pulmonary blood vessels. b. incomplete filling of the alveoli with air because of reduced respiratory ability. c. decreased transfer of oxygen into the blood because of thickening of the alveoli. d. mismatch between lung ventilation and blood flow through the blood vessels of the lung.
C Rationale: Pulmonary fibrosis causes the alveolar-capillary interface to become thicker, which increases the amount of time it takes for gas to diffuse across the membrane. Too-rapid pulmonary blood flow is another cause of shunt but does not describe the pathology of pulmonary fibrosis. Decrease in alveolar ventilation will cause hypercapnia. Ventilation and perfusion are matched in pulmonary fibrosis; the problem is with diffusion. Cognitive Level: Application Text Reference: p. 1802 Nursing Process: Implementation NCLEX: Physiological Integrity
26. A nursing assessment of a patient with Cushing syndrome reveals that the patient has truncal obesity and thin arms and legs. An additional manifestation of Cushing syndrome that the nurse would expect to find is a. chronically low blood pressure. b. decreased axillary and pubic hair. c. purplish red streaks on the abdomen. d. bronzed appearance of the skin.
C Rationale: Purplish-red striae on the abdomen are a common clinical manifestation of Cushing syndrome. Hypotension and bronzed-appearing skin are manifestations of Addison's disease. Decreased axillary and pubic hair occur with androgen deficiency.
A patient who has just been diagnosed with type 2 diabetes is 5 ft 4 in (160 cm) tall and weighs 182 pounds (82 kg). A nursing diagnosis of imbalanced nutrition: more than body requirements is developed. Which patient outcome is most important for this patient? a. The patient will have a diet and exercise plan that results in weight loss. b. The patient will state the reasons for eliminating simple sugars in the diet. c. The patient will have a glycosylated hemoglobin level of less than 7%. d. The patient will choose a diet that distributes calories throughout the day.
C Rationale: The complications of diabetes are related to elevated blood glucose, and the most important patient outcome is the reduction of glucose to near-normal levels. The other outcomes are also appropriate but are not as high in priority.
27. A pt with Cushing syndrome is admitted to the hospital to have laparoscopic adrenalectomy. During the admission assessment, the patient tells the nurse, "The worst thing about this disease is how terrible I look. I feel awful about it." best response by the nurse is a. "Let me show you how to dress so that the changes are not so noticeable." b. "I do not think you look bad. Your appearance is just altered by your disease." c. "Most of the physical and mental changes caused by the disease will gradually improve after surgery." d. "You really should not worry about how you look in the hospital. We see many worse things."
C Rationale: The most reassuring communication to the patient is that the physical and emotional changes caused by the Cushing syndrome will resolve after hormone levels return to normal postoperatively. The response beginning "Let me show you how to dress" indicates that the changes are permanent and that the patient's appearance needs disguising. The response beginning, "I do not think you look bad" does not acknowledge the patient's feelings and also fails to communicate that the changes will be resolved after surgery. And the response beginning "You really should not worry about how you look in the hospital" implies that the pt's appearance is not good.
A diabetic patient is admitted with ketoacidosis and the health care provider writes all of the following orders. Which order should the nurse implement first? a. Start an infusion of regular insulin at 50 U/hr. b. Give sodium bicarbonate 50 mEq IV push. c. Infuse 1 liter of normal saline per hour. d. Administer regular IV insulin 30 U.
C Rationale: The most urgent patient problem is the hypovolemia associated with DKA, and the priority is to infuse IV fluids. The other actions can be accomplished after the infusion of normal saline is initiated.
16. When caring for a patient who developed acute respiratory distress syndrome (ARDS) as a result of a urinary tract infection (UTI), the nurse is asked by the patient's family how a urinary tract infection could cause lung damage. Which response by the nurse is appropriate? a. "The infection spread through the circulation from the urinary tract to the lungs." b. "The urinary tract infection produced toxins that damaged the lungs." c. "The infection caused generalized inflammation that damaged the lungs." d. "The fever associated with the infection led to scar tissue formation in the lungs."
C Rationale: The pathophysiologic changes that occur in ARDS are thought to be caused by inflammatory and immune reactions that lead to changes at the alveolar-capillary membrane. ARDS is not directly caused by infection, toxins, or fever. Cognitive Level: Application Text Reference: p. 1813 Nursing Process: Implementation NCLEX: Physiological Integrity
21. When prone positioning is used in the care of a patient with acute respiratory distress syndrome (ARDS), which information obtained by the nurse indicates that the positioning is effective? a. The skin on the patient's back is intact and without redness. b. Sputum and blood cultures show no growth after 24 hours. c. The patient's PaO2 is 90 mm Hg, and the SaO2 is 92%. d. Endotracheal suctioning results in minimal mucous return.
C Rationale: The purpose of prone positioning is to improve the patient's oxygenation as indicated by the PaO2 and SaO2. The other information will be collected but does not indicate whether prone positioning has been effective. Cognitive Level: Application Text Reference: pp. 1817-1818 Nursing Process: Evaluation NCLEX: Physiological Integrity
The nurse is obtaining a focused respiratory assessment from a patient who is in severe respiratory distress. What is most important for the nurse to assess? A) Palpate the chest area for tactile fremitus B) Evaluate for anterior and posterior chest expansion C) Auscultation of lung (breath) sounds D) Percussion of anterior and posterior chest wall
C) Auscultation of lung (breath) sounds Feedback: Rationale: Important assessments obtained during a focused respiratory assessment include auscultation of lung (breath) sounds. Assessment of tactile fremitus has limited value in acute respiratory distress. It is not necessary to assess for both anterior and posterior chest expansion. Percussion of the chest wall is not essential to a focused respiratory assessment.
The nurse assesses a patient with shortness of breath for evidence of long-standing hypoxemia by inspecting A) Chest excursion. B) Spinal curvatures. C) The fingernails and their base D) The respiratory pattern.
C) The fingernails and their base Feedback: Clubbing, a sign of long-standing hypoxemia, is evidenced by an increase in the angle between the base of the nail and the fingernail to 180 degrees or more, usually accompanied by an increase in the depth, bulk, and sponginess of the end of the finger.
Which of the following positions is most appropriate for the nurse to place a patient experiencing an asthma exacerbation? A. Supine B. Lithotomy C. High-Fowler's D. Reverse Trendelenburg
C. High-Fowler'sThe patient experiencing an asthma attack should be placed in high-Fowler's position to allow for optimal chest expansion and enlist the aid of gravity during inspiration.
Which of the following clinical manifestations would the nurse expect to find during assessment of a patient admitted with pneumococcal pneumonia? A. Hyperresonance on percussion B. Fine crackles in all lobes on auscultation C. Increased vocal fremitus on palpation D. Vesicular breath sounds in all lobes
C. Increased vocal fremitus on palpation. A typical physical examination finding for a patient with pneumonia is increased vocal fremitus on palpation. Other signs of pulmonary consolidation include dullness to percussion, bronchial breath sounds, and crackles in the affected area.
The nurse is working with an overweight client who has a high-stress job and smokes. This client has just received a diagnosis of Type II Diabetes and has just been started on an oral hypoglycemic agent. Which of the following goals for the client which if met, would be most likely to lead to an improvement in insulin efficiency to the point the client would no longer require oral hypoglycemic agents? a. Comply with medication regimen 100% for 6 months b. Quit the use of any tobacco products by the end of three months c. Lose a pound a week until weight is in normal range for height and exercise 30 minutes daily d. Practice relaxation techniques for at least five minutes five times a day for at least five months
C. Lose a pound a week until weight is in normal range for height and exercise 30 minutes daily When type II diabetics lose weight through diet and exercise they sometimes have an improvement in insulin efficiency sufficient to the degree they no longer require oral hypoglycemic agents.
When assessing a patient's respiratory status, which of the following nonrespiratory data are most important for the nurse to obtain? A. Height and weight B. Neck circumference C. Occupation and hobbies D. Usual daily fluid intake
C. Occupation and hobbiesMany respiratory problems occur as a result of chronic exposure to inhalation irritants. Common occupational sources of inhalation irritants include mines, granaries, farms, lawn care companies, paint, plastics and rubber manufacture, and building remodeling. Hobbies associated with inhalation irritants include woodworking, metal finishing, furniture refinishing, painting, and ceramics. Daily fluids, height, and weight are more related to respiratory problems secondary to cardiac issues.
A 71-year-old patient is admitted with acute respiratory distress related to cor pulmonale. Which of the following nursing interventions is most appropriate during admission of this patient? A. Delay any physical assessment of the patient and review with the family the patient's history of respiratory problems. B. Perform a comprehensive health history with the patient to review prior respiratory problems. C. Perform a physical assessment of the respiratory system and ask specific questions related to this episode of respiratory distress. D. Complete a full physical examination to determine the effect of the respiratory distress on other body functions.
C. Perform a physical assessment of the respiratory system and ask specific questions related to this episode of respiratory distress.Because the patient is having respiratory difficulty, the nurse should ask specific questions about this episode and perform a physical assessment of this system. Further history taking and physical examination of other body systems can proceed once the patient's acute respiratory distress is being managed.
A patient with an acute pharyngitis is seen at the clinic with fever and severe throat pain that affects swallowing. On inspection the throat is reddened and edematous with patchy yellow exudates. The nurse anticipates that collaborative management will include A. treatment with antibiotics. B. treatment with antifungal agents. C. a throat culture or rapid strep antigen test. D. treatment with medication only if the pharyngitis does not resolve in 3 to 4 days.
C. a throat culture or rapid strep antigen test. Although inadequately treated β-hemolytic streptococcal infections may lead to rheumatic heart disease or glomerulonephritis, antibiotic treatment is not recommended until strep infections are definitely diagnosed with culture or antigen tests. The manifestations of viral and bacterial infections are similar, and appearance is not diagnostic except when candidiasis is present.
A patient with cirrhosis is being treated with spironolactone (Aldactone) tid and furosemide (Lasix) bid. The patient's most recent laboratory results indicate a serum sodium of 134 mEq/L (134 mmol/L) and a serum potassium of 3.2 mEq/L (3.2 mmol/L). Before notifying the physician, the nurse should A. administer only the furosemide B. administer both drugs as ordered C. administer only the spironolactone D. Withhold the furosemide and spironolactone
C. administer only the spironolactone The potassium level is dangerously low. Lasix is potassium depleting, while spironolactone is potassium sparing. You would hold the Lasix and call the physician. This is a good NCLEX question that integrates this course with pharmacology.
A thirty five year old male has been an insulin-dependent diabetic for five years and now is unable to urinate. Which of the following would you most likely suspect? A: Atherosclerosis B: Diabetic nephropathy C: Autonomic neuropathy D: Somatic neuropathy
C: Autonomic neuropathy
Risk factors for type 2 diabetes include all of the following except: a. Advanced age b. Obesity c. Smoking d. Physical inactivity
C: smoking Additional risk factors for type 2 diabetes are a family history of diabetes, impaired glucose metabolism, history of gestational diabetes, and race/ethnicity. African-Americans, Hispanics/Latinos, Asian Americans, Native Hawaiians, Pacific Islanders, and Native Americans are at greater risk of developing diabetes than whites.
A client has been started on long-term therapy with rifampin. Which information about this medication should the nurse provide to the client?
Causes red-orange discoloration of sweat, tears, urine, and feces.
Steps for obtaining sputum culture (in order)
Check order. Obtain appropriate container. Teach pt to inhale deeply several times. Have pt cough deeply from lungs. Send specimen immediately t olab.
When is bubbling normal in the CDU?
Chest tube connected to suction- gentle continuous bubbling is expected in the suction chamber. If a client with a pneumothorax is coughing, sneezing, or just taking a deep breath and exhaling, you may see intermittent bubbling in the water seal chamber As long as there is intermittent bubbling, the client needs the chest tube because air is still leaking out of the pleural space.
What actions should the nurse perform pre thoracentesis?
Chest x-ray and baseline vital signs Positioning: - Sitting up leaning over the bedside table - Sit in a chair backwards, popped up over the back of the chair - Lie on unaffected side with HOB at 45 degrees
What actions should the nurse perform during a thoracentesis?
Client must be very still, no coughing or deep breaths The fluid/blood/exudate is being removed from the pleural space As the fluid is removed, the lung should re-expand Since you are removing fluid, the client could go into a fluid volume deficit Therefore, you should be checking the vital sings
What do anticoagulants prevent?
Clots formation and clots getting bigger
When initially teaching a patient the supraglottic swallow following a radical neck dissection, with which food or fluid should the nurse begin? Cola Applesauce French fries White grape juice
Cola When learning the supraglottic swallow, it may be helpful to start with carbonated beverages because the effervescence provides clues about the liquid's position. Thin, watery fluids should be avoided because they are difficult to swallow and increase the risk of aspiration. Nonpourable pureed foods, such as applesauce, would decrease the risk of aspiration, but carbonated beverages are the better choice with which to start.
fever and chills
Common signs and symptoms that should alert the nurse to the possibility of pneumonia include: A. Fever and chills B. Nonproductive cough C. Night sweats D. Clubbed fingers
A patient with a history of tonsillitis complains of difficulty breathing. Which patient assessment data warrants emergency interventions by the nurse? Bilateral erythema of especially large tonsils Temperature 102.2° F, diaphoresis, and chills Contraction of neck muscles during inspiration β-hemolytic streptococcus in the throat culture
Contraction of neck muscles during inspiration Contraction of neck muscles during inspiration indicates that the patient is using accessory muscles for breathing and is in serious respiratory distress. The reddened and enlarged tonsils indicate pharyngitis. The increased temperature, diaphoresis, and chills indicate an infection, which could be β-hemolytic streptococcus or fungal infection, but not an emergency situation for the patient.
A client is admitted to the medical-surgical floor with a diagnosis of acute pancreatitis. His blood pressure is 136/76 mm Hg, pulse 96 beats/minute, respirations 22 breaths/minute, temperature 99°F (38.3°C), and he has been experiencing severe vomiting for 24 hours. His past medical history reveals hyperlipidemia and alcohol abuse. The physician prescribes a nasogastric (NG) tube for the client. Which of the following is the primary purpose for insertion of the NG tube? A. Empty the stomach of fluids and gas to relieve vomiting. B. Prevent spasms at the sphincter of Oddi. C. Prevent air from forming in the small and large intestines. D. Remove bile from the gallbladder.
Correct answer: A An NG tube is no longer routinely inserted to treat pancreatitis, but if the client has protracted vomiting, the NG tube is inserted to drain fluids and gas and relieve vomiting. An NG tube doesn't prevent spasms at the sphincter of Oddi (a valve in the duodenum that controls the flow of digestive enzymes) or prevent air from forming in the small and large intestine. The common bile duct connects to the pancreas and the gall bladder, and a T tube rather than an NG tube would be used to collect bile drainage from the common bile duct.
A client with cirrhosis of the liver develops ascites. Which of the following orders would the nurse expect? A. Restrict fluid to 1000 mL per day. B. Ambulate 100 ft. three times per day. C. High-sodium diet. D. Maalox 30 ml P.O. BID.
Correct answer: A Fluid restriction is a primary treatment for ascites. Restricting fluids decreases the amount of fluid present in the body, thereby decreasing the fluid that accumulates in the peritoneal space. A high sodium diet would increase fluid retention. Physical activities are usually restricted until ascites is relieved. Loop diuretics (such as furosemide) are usually ordered, and Maalox® (a bismuth subsalicylate) may interfere with the action of the diuretics.
The nurse is doing teaching with the family of a client with liver failure. Which of the following foods should the nurse advise them to limit in the client's diet? A. Meats and beans. B. Butter and gravies. C. Potatoes and pasta. D. Cakes and pastries.
Correct answer: A Meats and beans are high-protein foods and are restricted with liver failure. In liver failure, the liver is unable to metabolize protein adequately, causing protein by-products to build up in the body rather than be excreted. This causes problems such as hepatic encephalopathy (neurologic syndrome that develops as a result of rising blood ammonia levels). Although other nutrients, such as fat and carbohydrates, may be regulated, it's most important to limit protein in the diet of the client with liver failure.
A client who recently underwent cranial surgery develops syndrome of inappropriate antidiuretic hormone (SIADH). Which of the following symptoms should the nurse anticipate? A. Edema and weight gain. B. Excessive urinary output. C. Fluid loss and dehydration. D. Low urine specific gravity.
Correct answer: A Syndrome of inappropriate antidiuretic hormone (SIADH) results in an abnormally high release of antidiuretic hormone, which causes water retention as serum sodium levels fall, leading to edema and weight gain. Because of fluid retention, urine output is low. Fluid is restricted to prevent fluid overload rather than replaced. As the urine becomes more concentrated, the specific gravity increases. Other symptoms include nausea, vomiting, seizures, altered mentation, and coma. SIADH is most common with diseases of the hypothalamus but can also occur with heart failure, Guillain-Barré syndrome, meningitis, encephalitis, head trauma, or brain tumors. It may also be triggered by medications.
The nurse is caring for a client with cirrhosis of the liver. The client has developed ascites and requires a paracentesis. Which of the following symptoms is associated with ascites and should be relieved by the paracentesis? A. Pruritus. B. Dyspnea. C. Jaundice. D. Peripheral neuropathy.
Correct answer: B Ascites (fluid buildup in the abdomen) puts pressure on the diaphragm, resulting in difficulty breathing and dyspnea. Paracentesis (surgical puncture of the abdominal cavity to aspirate fluid) is done to remove fluid from the abdominal cavity and thus reduce pressure on the diaphragm in order to relieve the dyspnea. Pruritus, jaundice, and peripheral neuropathy are signs of cirrhosis that aren't relieved or treated by paracentesis.
A 37-year-old forklift operator presents with shakiness, sweating, anxiety, and palpitations and tells the nurse he has type 1 diabetes mellitus. Which of the follow actions should the nurse do first? A. Inject 1 mg of glucagon subcutaneously. B. Administer 50 mL of 50% glucose I.V. C. Give 4 to 6 oz (118 to 177 mL) of orange juice. D. Give the client four to six glucose tablets.
Correct answer: C Because the client is awake and complaining of symptoms, the nurse should first give him 15 grams of carbohydrate to treat hypoglycemia. This could be 4 to 6 oz of fruit juice, five to six hard candies such as Lifesavers, or 1 tablespoon of sugar. When a client has worsening symptoms of hypoglycemia or is unconscious, treatment includes 1 mg of glucagon subcutaneously or intramuscularly, or 50 mL of 50% glucose I.V. The nurse may also give two to three glucose tablets for a hypoglycemic reaction.
The school nurse is providing information to high school students about influenza prevention. What should the nurse emphasize in teaching to prevent the transmission of the virus (select all that apply)? Cover the nose when coughing. Obtain an influenza vaccination. Stay at home when symptomatic. Drink non-caffeinated fluids daily. Obtain antibiotic therapy promptly.
Cover the nose when coughing. Obtain an influenza vaccination. Stay at home when symptomatic. Covering the nose and mouth when coughing is an effective way to prevent the spread of the virus. Obtaining an influenza vaccination helps prevent the flu. Staying at home helps prevent direct exposure of others to the virus. Drinking fluids helps liquefy secretions but does not prevent influenza. Antibiotic therapy is not used unless the patient develops a secondary bacterial infection.
During a teaching session, the nurse tells the client that 50% to 60% of daily calories should come from carbohydrates. What should the nurse say about the types of carbohydrates that can be eaten? a. Simple carbohydrates are absorbed more rapidly than complex carbohydrates. b. Simple sugars cause a rapid spike in glucose levels and should be avoided c. Simple sugars should never be consumed by someone with diabetes. d. Try to limit simple sugars to between 10% and 20% of daily calories.
D It is recommended that carbohydrates provide 50% to 60% of the daily calories. Approximately 40% to 50% should be from complex carbohydrates. The remaining 10% to 20% of carbohydrates could be from simple sugars. Research provides no evidence that carbohydrates from simple sugars are digested and absorbed more rapidly than are complex carbohydrates, and they do not appear to affect blood sugar control.
A hospitalized diabetic patient receives 12 U of regular insulin mixed with 34 U of NPH insulin at 7:00 AM. The patient is away from the nursing unit for diagnostic testing at noon, when lunch trays are distributed. The most appropriate action by the nurse is to a. save the lunch tray to be provided upon the patient's return to the unit. b. call the diagnostic testing area and ask that a 5% dextrose IV be started. c. ensure that the patient drinks a glass of milk or orange juice at noon in the diagnostic testing area. d. request that the patient be returned to the unit to eat lunch if testing will not be completed promptly.
D Rationale: Consistency for mealtimes assists with regulation of blood glucose, so the best option is for the patient to have lunch at the usual time. Waiting to eat until after the procedure is likely to cause hypoglycemia. Administration of an IV solution is unnecessarily invasive for the patient. A glass of milk or juice will keep the patient from becoming hypoglycemic but will cause a rapid rise in blood glucose because of the rapid absorption of the simple carbohydrate in these items.
A patient recovering from DKA asks the nurse how acidosis occurs. The best response by the nurse is that a. insufficient insulin leads to cellular starvation, and as cells rupture they release organic acids into the blood. b. when an insulin deficit causes hyperglycemia, then proteins are deaminated by the liver, causing acidic by-products. c. excess glucose in the blood is metabolized by the liver into acetone, which is acidic. d. an insulin deficit promotes metabolism of fat stores, which produces large amounts of acidic ketones.
D Rationale: Ketoacidosis is caused by the breakdown of fat stores when glucose is not available for intracellular metabolism. The other responses are inaccurate.
A patient with type 1 diabetes has received diet instruction as part of the treatment plan. The nurse determines a need for additional instruction when the patient says, a. "I may have an occasional alcoholic drink if I include it in my meal plan." b. "I will need a bedtime snack because I take an evening dose of NPH insulin." c. "I will eat meals as scheduled, even if I am not hungry, to prevent hypoglycemia." d. "I may eat whatever I want, as long as I use enough insulin to cover the calories."
D Rationale: Most patients with type 1 diabetes need to plan diet choices very carefully. Patients who are using intensified insulin therapy have considerable flexibility in diet choices but still should restrict dietary intake of items such as fat, protein, and alcohol. The other patient statements are correct and indicate good understanding of the diet instruction.
The nurse teaches the diabetic patient who rides a bicycle to work every day to administer morning insulin into the a. thigh. b. buttock. c. arm. d. abdomen.
D Rationale: Patients should be taught not to administer insulin into a site that will be exercised because exercise will increase the rate of absorption. The thigh, buttock, and arm are all exercised by riding a bicycle.
A program of weight loss and exercise is recommended for a patient with impaired fasting glucose (IFG). When teaching the patient about the reason for these lifestyle changes, the nurse will tell the patient that a. the high insulin levels associated with this syndrome damage the lining of blood vessels, leading to vascular disease. b. although the fasting plasma glucose levels do not currently indicate diabetes, the glycosylated hemoglobin will be elevated. c. the liver is producing excessive glucose, which will eventually exhaust the ability of the pancreas to produce insulin, and exercise will normalize glucose production. d. the onset of diabetes and the associated cardiovascular risks can be delayed or prevented by weight loss and exercise.
D Rationale: The patient with IFG is at risk for developing type 2 diabetes, but this risk can be decreased with lifestyle changes. Glycosylated hemoglobin levels will not be elevated in IFG and the Hb A1C test is not included in prediabetes testing. Elevated insulin levels do not cause the damage to blood vessels that can occur with IFG. The liver does not produce increased levels of glucose in IFG
A patient screened for diabetes at a clinic has a fasting plasma glucose level of 120 mg/dl (6.7 mmol/L). The nurse will plan to teach the patient about a. use of low doses of regular insulin. b. self-monitoring of blood glucose. c. oral hypoglycemic medications. d. maintenance of a healthy weight.
D Rationale: The patient's impaired fasting glucose indicates pre-diabetes and the patient should be counseled about LIFESTYLE CHANGES to prevent the development of type 2 diabetes. The patient with prediabetes does not require insulin or the oral hypoglycemics for glucose control and does not need to self-monitor blood glucose.
A side effect of hydrocortisone is...
Hyperglycemia. A bilateral adrenalectomy is the surgical removal of both adrenal glands. One of the hormones produced by the adrenal glands is cortisol, which is a glucocorticoid. Following an adrenalectomy, this hormone must be replaced endogenously. Side effects of the IV hydrocortisone administered to replace the cortisol are increased gluconeogenesis and hyperglycemia.
A health care provider who has not been immunized for hepatitis B is exposed to the hepatitis B virus (HBV) through a needle stick from an infected patient. The infection control nurse informs the individual that treatment for the exposure should include a. baseline hepatitis B antibody testing now and in 2 months. b. active immunization with hepatitis B vaccine. c. hepatitis B immune globulin (HBIG) injection. d. both the hepatitis B vaccine and HBIG injection.
D Rationale: The recommended treatment for exposure to hepatitis B in unvaccinated individuals is to receive both HBIG and the hepatitis B vaccine, which would provide temporary passive immunity and promote active immunity. Antibody testing may also be done, but this would not provide protection from the exposure.
38. A pt is admitted with possible SIADH. Which information obtained by nurse is most important to communicate rapidly to health care provider? a. The patient complains of a severe headache. b. The patient complains of severe thirst. c. The patient has a urine specific gravity of 1.025. d. The pt has a serum sodium level of 119 mEq/L.
D R: A serum sodium of less than 120 mEq/L increases risk for complications such as seizures and needs rapid correction. The other data are not unusual for a pt with SIADH and do not indicate the need for rapid action. (Cognitive Level: Application Text Reference: p. 1295 NProcess: Assessment NCLEX: Physiological Integrity)
32. A pt who uses every-other-day prednisone therapy for rheumatoid arthritis complains of not feeling as well on the non-prednisone days and asks nurse about taking prednisone daily instead. The best response to the pt is that a. an every-other-day schedule mimics the normal pattern of cortisol secretion from the adrenal gland. b. glucocorticoids are taken on a daily basis only when theyre being used for replacement therapy. c. if it improves the symptoms, it would be acceptable to take half the usual dose every day. d. there is less effect on normal adrenal function when prednisone is taken every other day.
D R: An alternate-day regimen is given to minimize the impact of exogenous glucocorticoids on adrenal gland function. The normal pattern of cortisol secretion is diurnal. Glucocorticoids are taken daily when being used for replacement therapy, but this is not the only indication for daily use. Taking half the usual dose would not achieve the goal of minimizing adrenal gland suppression.
11. When teaching a patient newly diagnosed with Graves' disease about the disorder, the nurse explains that a. restriction of iodine intake is needed to reduce thyroid activity. b. exercise is contraindicated to avoid increasing metabolic rate. c. surgery will eventually be required to remove the thyroid gland. d. antithyroid medications may take several weeks to have an effect.
D R: Improvement usually begins in 1-2 wks w good results at 4-6 weeks. Large doses of iodine are used to inhibit the synthesis of thyroid hormones. Exercise using large muscle groups is encouraged to decrease irritability and hyperactivity associated with high levels of thyroid hormones. Radioactive iodine is the most common trtmt for Graves' disease, although surgery may be used.
40. After receiving change-of-shift report about these pts, which patient should nurse assess first? a. A 22-year-old admitted with SIADH who has a serum sodium level of 130 mEq/L. b. A 31-year-old who has iatrogenic Cushing's syndrome with a capillary blood glucose level of 244 mg/dl. c. A 53-year-old who has Addison's disease and is due for a scheduled dose of hydrocortisone (Solu-Cortef). d. A 70-year-old who recently started levothyroxine (Synthroid) to treat hypothyroidism and has an irregular pulse of 134.
D R: Initiation of thyroid replacement in older adults may cause angina and cardiac dysrhythmias. The pt's high pulse rate needs rapid investigation by the nurse to assess for and intervene with any cardiac problems. The other pts also require nursing assessment and/or actions but are not at risk for life-threatening complications. (Cognitive Level: Application Text Reference: p. 1306 Nursing Process: Planning NCLEX: Physiological Integrity)
10. Which information obtained when caring for a pt who has just been admitted for evaluation of DI will be of greatest concern to the nurse? a. The patient has a urine output of 800 ml/hr. b. The patient's urine specific gravity is 1.003. c. The patient had a recent head injury. d. The patient is confused and lethargic.
D R: Pts with diabetes insipidus compensate for fluid losses by drinking copious amounts of fluids, but a patient who is lethargic will be unable to drink enough fluids and will become hypovolemic. A high urine output, low urine specific gravity, and history of a recent head injury are consistent with DI, but they do not require immediate nursing action to avoid life-threatening complications.
25. A patient with hypoparathyroidism receives instructions from the nurse regarding symptoms of hypocalcemia and hypercalcemia. The nurse teaches the patient that if mild symptoms of hypocalcemia occur, the patient should a. increase daily fluid intake to twice usual amount b. self-administer IM calcium before calling doctor. c. call an ambulance because the symptoms will progress to seizures. d. rebreathe with a paper bag and then seek medical assistance.
D R: Rebreathing may help alleviate mild sx, but it will only temporarily increase ionized calcium level, so the pt should call HCP. There is no need to increase fluid intake. Calcium is not given IM but given slowly through IV route. Mild hypocalcemia is unlikely to progress to seizures.
22. A patient with primary hyperparathyroidism has a serum calcium level of 14 mg/dl (3.5 mmol/L), phosphorus of 1.7 mg/dl (0.55 mmol/L), serum creatinine of 2.2 mg/dl (194 mmol/L), and a high urine calcium. While the patient awaits surgery, the nurse should a. institute seizure precautions such as padded siderails. b. assist the patient to perform range-of-motion exercises QID. c. monitor the patient for positive Chvostek's or Trousseau's sign. d. encourage the pt to drink 4000 ml of fluid daily.
D R: The pt with hypercalcemia is at risk for kidney stones, which may be prevented by a high fluid intake. Seizure precautions and monitoring for Chvostek's or Trousseau's sign are appropriate for hypocalcemic patients. The pt should engage in weight-bearing exercise rather than range-of-motion because weight-bearing decreases calcium loss from bone.
The nurse is scheduled to administer seasonal influenza vaccinations to the residents of a long-term care facility. What would be a contraindication to the administration of the vaccine to a resident? Hypersensitivity to eggs Age greater than 80 years History of upper respiratory infections Chronic obstructive pulmonary disease (COPD)
Hypersensitivity to eggs Although current vaccines are highly purified, and reactions are extremely uncommon, a hypersensitivity to eggs precludes vaccination because the vaccine is produced in eggs. Advanced age and a history of respiratory illness are not contraindications for influenza vaccination.
7. A patient with chronic obstructive pulmonary disease (COPD) arrives in the emergency department complaining of acute respiratory distress. When monitoring the patient, which assessment by the nurse will be of most concern? a. The patient is sitting in the tripod position. b. The patient has bibasilar lung crackles. c. The patient's pulse oximetry indicates an O2 saturation of 91%. d. The patient's respiratory rate has decreased from 30 to 10/min.
D Rationale: A decrease in respiratory rate in a patient with respiratory distress suggests the onset of fatigue and a high risk for respiratory arrest; therefore, the nurse will need to take immediate action. Patients who are experiencing respiratory distress frequently sit in the tripod position because it decreases the work of breathing. Crackles in the lung bases may be the baseline for a patient with COPD. An oxygen saturation of 91% is common in patients with COPD and will provide adequate gas exchange and tissue oxygenation. Cognitive Level: Application Text Reference: p. 1804 Nursing Process: Assessment NCLEX: Physiological Integrity
4. A patient is diagnosed with a large pulmonary embolism. When explaining to the patient what has happened to cause respiratory failure, which information will the nurse include? a. "Oxygen transfer into your blood is slow because of thick membranes between the small air sacs and the lung circulation." b. "Thick secretions in your small airways are blocking air from moving into the small air sacs in your lungs." c. "Large areas of your lungs are getting good blood flow but are not receiving enough air to fill the small air sacs." d. "Blood flow though some areas of your lungs is decreased even though you are taking adequate breaths."
D Rationale: A pulmonary embolus limits blood flow but does not affect ventilation, leading to a ventilation-perfusion mismatch. The response beginning, "Oxygen transfer into your blood is slow because of thick membranes" describes a diffusion problem. The remaining two responses describe ventilation-perfusion mismatch with adequate blood flow but poor ventilation. Cognitive Level: Application Text Reference: p. 1802 Nursing Process: Implementation NCLEX: Physiological Integrity
20. Which statement by the nurse when explaining the purpose of positive end-expiratory pressure (PEEP) to the family members of a patient with ARDS is correct? a. "PEEP will prevent fibrosis of the lung from occurring." b. "PEEP will push more air into the lungs during inhalation." c. "PEEP allows the ventilator to deliver 100% oxygen to the lungs." d. "PEEP prevents the lung air sacs from collapsing during exhalation."
D Rationale: By preventing alveolar collapse during expiration, PEEP improves gas exchange and oxygenation. PEEP will not prevent the fibrotic changes that occur with ARDS, push more air into the lungs, or change the fraction of inspired oxygen (FIO2) delivered to the patient. Cognitive Level: Comprehension Text Reference: p. 1817 Nursing Process: Planning NCLEX: Physiological Integrity
10. While caring for a patient who has been admitted with a pulmonary embolism, the nurse notes a change in the patient's arterial oxyhemoglobin saturation (SpO2) from 94% to 88%. The nurse will a. assist the patient to cough and deep-breathe. b. help the patient to sit in a more upright position. c. suction the patient's oropharynx. d. increase the oxygen flow rate.
D Rationale: Increasing oxygen flow rate will usually improve oxygen saturation in patients with ventilation-perfusion mismatch, as occurs with pulmonary embolism. Because the problem is with perfusion, actions that improve ventilation, such as deep-breathing and coughing, sitting upright, and suctioning, are not likely to improve oxygenation. Cognitive Level: Application Text Reference: pp. 1802, 1807 Nursing Process: Implementation NCLEX: Physiological Integrity
18. After prolonged cardiopulmonary bypass, a patient develops increasing shortness of breath and hypoxemia. To determine whether the patient has acute respiratory distress syndrome (ARDS) or pulmonary edema caused by left ventricular failure, the nurse will anticipate assisting with a. positioning the patient for a chest radiograph. b. drawing blood for arterial blood gases. c. obtaining a ventilation-perfusion scan. d. inserting a pulmonary artery catheter.
D Rationale: Pulmonary artery wedge pressure will remain at normal levels in the patient with ARDS because the fluid in the alveoli is caused by increased permeability of the alveolar-capillary membrane rather than by the backup of fluid from the lungs (as occurs in cardiogenic pulmonary edema). The other tests will not help in differentiating cardiogenic from noncardiogenic pulmonary edema. Cognitive Level: Application Text Reference: p. 1815 Nursing Process: Implementation NCLEX: Physiological Integrity
6. When assessing a patient with chronic lung disease, the nurse finds a sudden onset of agitation and confusion. Which action should the nurse take first? a. Monitor the patient every 10 to 15 minutes. b. Notify the patient's health care provider immediately. c. Attempt to calm and reassure the patient. d. Assess vital signs and pulse oximetry.
D Rationale: The nurse needs to collect additional clinical data to share with the health care provider and to start interventions quickly if appropriate (e.g., increased oxygen flow if hypoxic). The change in the patient's neurologic status may indicate deterioration in respiratory function, and the health care provider should be notified immediately but only after some additional information is obtained. Monitoring the patient and attempting to calm the patient are appropriate actions, but they will not prevent further deterioration of the patient's clinical status and may delay care. Cognitive Level: Application Text Reference: pp. 1804-1805 Nursing Process: Assessment NCLEX: Physiological Integrity
12. A patient in acute respiratory failure as a complication of COPD has a PaCO2 of 65 mm Hg, rhonchi audible in the right lung, and marked fatigue with a weak cough. The nurse will plan to a. allow the patient to rest to help conserve energy. b. arrange for a humidifier to be placed in the patient's room. c. position the patient on the right side with the head of the bed elevated. d. assist the patient with augmented coughing to remove respiratory secretions.
D Rationale: The patient's assessment indicates that assisted coughing is needed to help remove secretions, which will improve PaCO2 and will also help to correct fatigue. If the patient is allowed to rest, the PaCO2 will increase. Humidification may help loosen secretions, but the weak cough effort will prevent the secretions from being cleared. The patient should be positioned with the good lung down to improve gas exchange. Cognitive Level: Application Text Reference: p. 1809 Nursing Process: Planning NCLEX: Physiological Integrity
A nurse is monitoring a client with Graves' disease for the development of thyroid storm. The nurse should report which of the following to the provider?
Hypertension. Graves' disease is a common cause of hyperthyroidism, an imbalance of metabolism caused by overproduction of the thyroid hormone. Thyroid storm, an exaggerated condition of hyperthyroidism, is associated with the development of fever, hypertension, abdominal pain, and tachycardia.
15. The nurse is caring for a patient who was hospitalized 2 days earlier with aspiration pneumonia. Which assessment information is most important to communicate to the health care provider? a. The patient has a cough that is productive of blood-tinged sputum. b. The patient has scattered crackles throughout the posterior lung bases. c. The patient's temperature is 101.5° F after 2 days of IV antibiotic therapy. d. The patient's SpO2 has dropped to 90%, although the O2 flow rate has been increased.
D Rationale: The patient's dropping SpO2 despite having an increase in FIO2 indicates the possibility of acute respiratory distress syndrome (ARDS). The patient's blood-tinged sputum and scattered crackles are not unusual in a patient with pneumonia, although they do require continued monitoring. The continued temperature elevation indicates a possible need to change antibiotics, but this is not as urgent a concern as the progression toward hypoxemia despite an increase in O2 flow rate. Cognitive Level: Application Text Reference: p. 1815 Nursing Process: Assessment NCLEX: Physiological Integrity
A patient is admitted with a metabolic acidosis of unknown origin. Based on this diagnosis, the nurse would expect the patient to have A) Intercostal retractions. B) A low oxygen saturation (SpO2). C) A decrease in venous O2 pressure. D) Kussmaul respirations.
D) Kussmaul respirations. Feedback: Kussmaul (deep and rapid) respirations are a compensatory mechanism for metabolic acidosis. Intercostal retractions, a low oxygen saturation rate, and a decrease in PvO2 would not be caused by acidosis.
A patient with acute shortness of breath is admitted to the hospital. Which action should the nurse take during the initial assessment of the patient? A) Complete a full physical examination to determine the systemic effect of the respiratory distress. B) Obtain a comprehensive health history to determine the extent of any prior respiratory problems. C) Delay the physical assessment and ask family members about any history of respiratory problems. D) Perform a respiratory system assessment and ask specific questions about this episode of respiratory distress.
D) Perform a respiratory system assessment and ask specific questions about this episode of respiratory distress. Feedback: When a patient has severe respiratory distress, only information pertinent to the current episode is obtained, and a more thorough assessment is deferred until later. Obtaining a comprehensive health history or full physical examination is unnecessary until the acute distress has resolved. A focused physical assessment should be done rapidly to help determine the cause of the distress and suggest treatment. Although family members may know about the patient's history of medical problems, the patient is the best informant for these data.
After a patient with right lower-lobe pneumonia has been treated with intravenous (IV) antibiotics for 2 days, which assessment data obtained by the nurse indicates that the treatment has been effective? A) The patient coughs up small amounts of green mucus. B) Increased tactile fremitus is palpable over the right chest. C) Bronchial breath sounds are heard at the right base. D) The patient's white blood cell (WBC) count is 9000/µl.
D) The patient's white blood cell (WBC) count is 9000/µl. Feedback: The normal WBC count indicates that the antibiotics have been effective. All the other data suggest that a change in treatment is needed.
The nurse is caring for patients in the student health center. A patient confides to the nurse that the patient's boyfriend informed her that he tested positive for Hepatitis B. Which of the following responses by the nurse is BEST? a. "That must have been a real shock to you" b. "You should be tested for Hepatitis B" c. "You'll receive the Hepatitis B immune globulin HBIG d. "Have you had unprotected sex with your boyfriend"
D. Hepatitis B is transmitted through parenteral drug abuse and sexual contact. Determine exposure before implementing.
When taking a health history, the nurse screens for manifestations suggestive of diabetes type I. Which of the following manifestations are considered the primary manifestations of diabetes type I and would be most suggestive of diabetes type I and require follow-up investigation? a. Excessive intake of calories, rapid weight gain, and difficulty losing weight b. Poor circulation, wound healing, and leg ulcers, c. Lack of energy, weight gain, and depression d. An increase in three areas: thirst, intake of fluids, and hunger
D. An increase in three areas: thirst, intake of fluids, and hunger The primary manifestations of diabetes type I are polyuria (increased urine output), polydipsia (increased thirst), polyphagia (increased hunger).
Which of the following physical assessment findings in a patient with pneumonia best supports the nursing diagnosis of ineffective airway clearance? A. Oxygen saturation of 85% B. Respiratory rate of 28 C. Presence of greenish sputum D. Basilar crackles
D. Basilar crackles The presence of adventitious breath sounds indicates that there is accumulation of secretions in the lower airways. This would be consistent with a nursing diagnosis of ineffective airway clearance because the patient is retaining secretions.
The nurse is assigned to care for a patient who has anxiety and an exacerbation of asthma. Which of the following is the primary reason for the nurse to carefully inspect the chest wall of this patient? A. Observe for signs of diaphoresis B. Allow time to calm the patient C. Monitor the patient for bilateral chest expansion D. Evaluate the use of intercostal muscles
D. Evaluate the use of intercostal muscles The nurse physically inspects the chest wall to evaluate the use of intercostal (accessory) muscles, which gives an indication of the degree of respiratory distress experienced by the patient.
A nurse is performing assessment for a patient diagnosed with chronic obstructive pulmonary disease (COPD). Which of the following findings should the nurse expect to observe? A. Nonproductive cough B. Prolonged inspiration C. Vesicular breath sounds D. Increased anterior-posterior chest diameter
D. Increased anterior-posterior chest diameter An increased anterior-posterior diameter is a compensatory mechanism experienced by patients with COPD and is caused by air-trapping. Patients with COPD have a productive cough, often expectorating copious amounts of sputum. Because of air-trapping, patients with COPD experience a prolonged expiration because the rate of gas on exhalation takes longer to escape. Chest auscultation for patients with COPD often reveals wheezing, crackles, and other adventitious breath sounds.
After admitting a patient to the medical unit with a diagnosis of pneumonia, the nurse will verify that which of the following physician orders have been completed before administering a dose of cefotetan (Cefotan) to the patient? A. Serum laboratory studies ordered for AM B. Pulmonary function evaluation C. Orthostatic blood pressures D. Sputum culture and sensitivity
D. Sputum culture and sensitivityThe nurse should ensure that the sputum for culture and sensitivity was sent to the laboratory before administering the cefotetan. It is important that the organisms are correctly identified (by the culture) before their numbers are affected by the antibiotic; the test will also determine whether the proper antibiotic has been ordered (sensitivity testing). Although antibiotic administration should not be unduly delayed while waiting for the patient to expectorate sputum, all of the other options will not be affected by the administration of antibiotics.
You are doing some teaching with a client who is starting on a sulfonylurea antidiabetic agent. The client mentions that he usually has a couple of beers each night and takes an aspirin each day to prevent heart attack and/or strokes. Which of the following responses would be best on the part of the nurse? a. As long as you only drink two beers and take one aspirin, this should not be a problem b. The aspirin is alright but you need to give up drinking any alcoholic beverages c. Aspirin and alcohol will cause the stomach to bleed more when on a sulfonylurea drug d. Taking alcohol and/or aspirin with a sulfonylurea drug can cause development of hypoglycemia
D. Taking alcohol and/or aspirin with a sulfonylurea drug can cause development of hypoglycemia Alcohol and/or aspirin taken with a sulfonylurea can cause development of hypoglycemia.
The nurse assesses a patient with shortness of breath for evidence of long-standing hypoxemia by inspecting: A. Chest excursion B. Spinal curvatures C. The respiratory pattern D. The fingernail and its base
D. The fingernail and its base Clubbing, a sign of long-standing hypoxemia, is evidenced by an increase in the angle between the base of the nail and the fingernail to 180 degrees or more, usually accompanied by an increase in the depth, bulk, and sponginess of the end of the finger.
The nurse identifies the nursing diagnosis of activity intolerance for a patient with asthma. The nurse assesses for which of the following etiologic factor for this nursing diagnosis in patients with asthma? A. Anxiety and restlessness B. Effects of medications C. Fear of suffocation D. Work of breathing
D. Work of breathingWhen the patient does not have sufficient gas exchange to engage in activity, the etiologic factor is often the work of breathing. When patients with asthma do not have effective respirations, they use all available energy to breathe and have little left over for purposeful activity.
The most appropriate position to assist a patient with chronic obstructive pulmonary disease (COPD) who is having difficulty breathing would be a A. high Fowler's position without a pillow behind the head. B. semi-Fowler's position with a single pillow behind the head. C. right side-lying position with the head of the bed at 45 degrees' elevation. D. sitting upright and forward position with arms supported on an over-the-bed table.
D. sitting upright and forward position with arms supported on an over-the-bed table.Sitting upright and leaning forward with arms supported on an over-the-bed table would be of most help to this patient, because it allows for expansion of the thoracic cage in all four directions (front, back, and two sides).
When administering oxygen to a patient with COPD with the potential for carbon dioxide narcosis, the nurse should A. never administer oxygen at a rate of more than 2 L/min. B. monitor the patient's use of oxygen to detect oxygen dependency. C. monitor the patient for symptoms of oxygen toxicity, such as paresthesias. D. use ABGs as a guide to determine what FIO2 level meets the patient's needs.
D. use ABGs as a guide to determine what FIO2 level meets the patient's needs. It is critical to start oxygen at low flow rates and then use ABGs as a guide to determine what FIO2 level is sufficient and can be tolerated.
An external insulin pump is prescribed for a client with diabetes mellitus and the client asks the nurse about the functioning of the pump. The nurse bases the response on the information that the pump: a) is timed to release programmed doses of regular or NPH insulin into the bloodstream at specific intervals b) continuously infuses small amounts of NPH insulin into the bloodstream while regularly monitoring blood glucose levels c) is surgically attached to the pancreas and infuses regular insulin into the pancreas, which in turn releases the insulin into the bloodstream d) gives a small continuously dose of regular insulin subcutaneously, and the client can self-administer a bolus with an additional dose form the pump before each meal
D.) gives a small continuously dose of regular insulin subcutaneously, and the client can self-administer a bolus with an additional dose form the pump before each meal An insulin pump provides a small continuous dose of regular insulin subcutaneously throughout the day and night, and the client can self-administer a bolus with an additional dose from the pump before each meal as needed. Regular insulin is used in an insulin pump. An external pump is not attached surgically to the pancreas.
Prediabetes is associated with all of the following except: a. Increased risk of developing type 2 diabetes b. Impaired glucose tolerance c. Increased risk of heart disease and stroke d. Increased risk of developing type 1 diabetes
D: Increased risk of developing type 1 diabetes Persons with elevated glucose levels that do not yet meet the criteria for diabetes are considered to have prediabetes and are at increased risk of developing type 2 diabetes. Weight loss and increasing physical activity can help people with prediabetes prevent or postpone the onset of type 2 diabetes.
What are the causes of pulmonary embolism?
Dehydration Venous stasis from prolonged immobility or surgery, or birth control pills Clotting disorders or heart arrhythmias like A-fib
A nurse is caring for a client with syndrome of inappropriate antidiuretic hormone secretion (SIADH). The nurse should monitor the client closely for what?
Hyponatremia. SIADH is caused by the excessive release of an antidiuretic hormone (ADH). As a result of the excess ADH, the client retains water creating a dilutional hyponatremia.
What are signs and symptoms of pulmonary embolism?
Hypoxemia Decreased PaCO2 Short of breath Cough Increased RR Increased D-dimer (increased with pulmonary embolus) Positive VQ scan Positive spiral CT or CT angiography Hemoptysis (coughing up blood) Increased pulse (because of hypoxia) Chest pain (sharp, stabbing) CXR will show atalectasis Pulmonary hypertension (incrased BP in lungs)
An early manifestation of hypoglycemia.
Drowsiness. Other early manifestations include fatigue, headache, shakiness, and nausea.
1, 2 Coarse crackles are often auscultated in patients diagnosed with pneumonia or heart failure. Rhonchi are auscultated in patients diagnosed with cystic fibrosis. Wheezes are auscultated when the patient is experiencing bronchospasm. Discontinuous, low-pitched lung sounds are auscultated in patients experiencing interstitial edema. Text Reference - p. 489
During a respiratory assessment, the nurse notes coarse crackles upon auscultation of the lung fields. Which diagnoses present with this assessment finding? Select all that apply. 1 Pneumonia 2 Heart failure 3 Cystic fibrosis 4 Bronchospasm 5 Interstitial edema
1, 3, 5 Changes in the respiratory system in the older adult include thickened mucus, decreased chest wall movement, and diminished breath sounds, especially at the lung bases. The PaO2 and SaO2 levels are decreased. Test-Taking Tip: Come to your test prep with a positive attitude about yourself, your nursing knowledge, and your test-taking abilities. A positive attitude is achieved through self-confidence gained by effective study. This means (a) answering questions (assessment), (b) organizing study time (planning), (c) reading and further study (implementation), and (d) answering questions (evaluation). Text Reference - p. 481
During the respiratory assessment of the older adult, the nurse would expect to find which of the following? Select all that apply. 1 Thicker mucus 2 Normal PaO2 and SaO2 3 Decreased chest wall movement 4 Increased breath sounds in the lung apices 5 Diminished breath sounds, particularly at lung bases
Dry, hacking cough
Early Symptoms of acute bronchitis include: A. Large amounts of sputum B.high fever C. Muscle soreness D. Dry , hacking cough
How does bi-level positive airway pressure (BiPAP) work?
Exerts different levels of positive pressure support, along with oxygen
they are more susceptible to upper respiratory infections.
For which of these reasons is it particularly important for older adults to receive influenza immunizations? A. They tend to live alone. B. They cannot tolerate changes in temperature. C. They tend not to seek medical assistance soon enough. D. They are more susceptible to upper respiratory infections.
What are the most common injuries from chest trauma?
Fractures of ribs and sternum
The patient has been diagnosed with head and neck cancer. Along with the treatment for the cancer, what other treatment should the nurse expect? Nasal packing Epistaxis balloon Gastrostomy tube Peripheral skin care
Gastrostomy tube Because 50% of patients with head and neck cancer are malnourished before treatment begins, many patients need enteral feeding via a gastrostomy tube because the effects of treatment make it difficult to take in enough nutrients orally, whether surgery, chemotherapy, or radiation is used. Nasal packing could be used with epistaxis or with nasal or sinus problems. Peripheral skin care would not be expected because it is not related to head and neck cancer.
What action should the nurse take when removing a chest tube?
Have client take a a deep breath and hold (or valsalva) and place an occlusive petroleum dressing over the site.
What treatment interventions are appropriate for a client with open pneumothorax?
Have the client inhale and hold or valsalva or hmmmmmmmmm - These will decrease the intra-thoracic pressure so no more outside air can get into the body Then place a piece of petroleum gauze over the area. Tape down 3 sides. The fourth side acts as an air vent/flutter valve. Have the client sit up (if possible) to expand lungs Trauma clients stay flat, until evaluated for other injuries
What are the two types of chest trauma?
Hemothorax Pneumothorax
What are the common anticoagulant drugs?
Heparin sodium Warfarin Enoxaparin Dabigatrin etexilate
What type of trouble shooting should the nurse implement when things go wrong with the closed drainage system?
If the tubing becomes disconnected, apply another sterile connector at bedside. Reconnect as fast as you can. If the CDU falls over and the water leaks out or shifts to the drainage compartment, do what you can to re-establish the water seal. Set the CDU upright, check all the chambers, and fill the water seal chamber to 2 cm water. Have the patient to deep breathe and cough in case any air went into the pleural space. If there is not water in the water seal chamber then air can collapse the lung. If the chest tube is pulled out - Occlusive dressing taped down on three sides - Otherwise, every time they take a breath, they will pull air into the pleural space
D. It can often be controlled by diet and regular exercise.
In discussing DM with a patient, it is important to base the discussion on which information regarding the disease? A. It can often be cured by insulin therapy. B. It has no cure and is considered "hopeless." C. It has no specific treatment other than use of insulin. D. It can often be controlled by diet and regular exercise.
A nurse is caring for a client with type 2 diabetes mellitus. Which of the following findings should indicate to the nurse that the client has hyperglycemia?
Increased urination and thirst. Hyperglycemia, or elevated blood sugar, is a common complication in clients who have diabetes mellitus. With a deficiency of insulin, the blood glucose rises, leading to an osmotic diuresis and increased urination. Due to osmotic diuresis, thirst is a common finding in a client who has hyperglycemia.
rest assists in keeping the immune system healthy
It is appropriate to teach patients to obtain sufficient rest to help decrease the frequency with which they contract upper respiratory infections. How does rest help prevent respiratory infections? A. Rest enhances the functioning of the cough reflex. B. Rest assists in keeping the immune system healthy. C. Rest allows the body to produce more red blood cells. D. Rest reduces the amount of vitamin C that the body excretes.
C (Feedback: Depending on the severity of the pharyngitis and the degree of pain, warm saline gargles or throat irrigations are used. The benefits of this treatment depend on the degree of heat that is applied. The nurse teaches about these procedures and about the recommended temperature of the solution: high enough to be effective and as warm as the patient can tolerate, usually 105ºF to 110ºF (40.6ºC to 43.3ºC). Irrigating the throat may reduce spasm in the pharyngeal muscles and relieve soreness of the throat. You would not tell the parent teacher organization that there is no real treatment of pharyngitis.)
It is cold season and the school nurse been asked to provide an educational event for the parent teacher organization of the local elementary school. What should the nurse include in teaching about the treatment of pharyngitis? A) Pharyngitis is more common in children whose immunizations are not up to date. B) There are no effective, evidence-based treatments for pharyngitis. C) Use of warm saline gargles or throat irrigations can relieve symptoms. D) Heat may increase the spasms in pharyngeal muscles.
What actions should the nurse take towards maintaining a closed drainage system?
Keep blow level of chest - If you lift it too high--> fluid will go back in - Want gravity drainage Keep tubing straight and free of kinks and dependent loops Tape connections. It must be a closed system. Monitor the water levels in the system Want to see tidaling (fluctuations) with respirations - Fluctuations will stop when the lung has re-expanded, if there is a kink/clot in tubing, or a dependent loop is present in the system
What treatment interventions are appropriate for a patient with a hemothorax/pneumothorax?
Large bore needle is placed into the 2nd intercostal space (by primary healthcare provider) to allow excess air to escape Then the cause is found, and chest tubes will be inserted
What is the chest tube inserted to remove fluid?
Laterally in the lower chest (8th or 9th intercostal space)
A client with TB is being started on antituberculosis therapy with isoniazid. Before giving the client the first dose, the nurse ensures that which baseline study has been completed?
Liver enzyme levels
What is the priority nursing assessment when using PEEP, CPAP, or BiPAP?
Lung sounds
A patient had an open reduction repair of a bilateral nasal fracture. The nurse plans to implement an intervention that focuses on both nursing and medical goals for this patient. Which intervention should the nurse implement? Apply an external splint to the nose. Insert plastic nasal implant surgically. Humidify the air for mouth breathing. Maintain surgical packing in the nose.
Maintain surgical packing in the nose. A goal that is common to nursing and medical management of a patient after rhinoplasty is to prevent the formation of a septal hematoma and potential infections resulting from a septal hematoma. Therefore the nurse helps to keep the nasal packing in the nose. The packing applies direct pressure to oozing blood vessels to stop postoperative bleeding. A medical goal includes realigning the fracture with an external or internal splint. The nurse helps maintain the airway by humidifying inspired air because the nose is unable to do so following surgery because it is swollen and packed with gauze.
A patient whose tracheostomy was inserted 30 minutes ago is recovering in the postanesthesia recovery unit when he coughs and expels the tracheostomy tube. How should the nurse respond? Suction the tracheostomy opening. Maintain the airway with a sterile hemostat. Use an Ambu bag and mask to ventilate the patient. Insert the tracheostomy tube obturator into the stoma.
Maintain the airway with a sterile hemostat. As long as the patient is not in acute respiratory distress after dislodging the tracheostomy tube, the nurse should use a sterile hemostat to maintain an open airway until a sterile tracheostomy tube can be reinserted into the tracheal opening. The tracheostomy is an open surgical wound that has not had time to mature into a stoma. If the patient is in respiratory distress, the nurse will use an Ambu bag and mask to ventilate the patient temporarily.
The patient seeks relief from the symptoms of an upper respiratory infection (URI) that has lasted for 5 days. Which patient assessment should the nurse use to help determine if the URI has developed into acute sinusitis? Coughing Fever, chills Dust allergy Maxillary pain
Maxillary pain The nurse should assess the patient for sinus pain or pressure as a clinical indicator of acute sinusitis. Coughing and fever are nonspecific clinical indicators of a URI. A history of an allergy that is likely to affect the upper respiratory tract is supportive of the sinusitis diagnosis but is not specific for sinusitis.
A nurse is collecting data from a client who was admitted to the unit with diagnosis of ketoacidosis. The nurse should expect the client to exhibit...
Nausea and vomiting. Ketoacidosis causes dehydration, which can result in fluid and electrolyte imbalances that precipitate nausea and vomiting. WRONG ANSWER "Diaphoresis" Diaphoresis is a manifestation of hypoglycemia.
The nurse is reading the results of a tuberculin skin test on a client with no documented health problems. The site has no induration and a 1-mm area of ecchymosis. Which interpretation should the nurse make of these results?
Negative
What treatment interventions are appropriate for a client with fractures of the ribs and sternum?
Non-narcotic analgesic Nerve block to assist with productive coughing Support injured area with hands Not recommended to immobilize with chest binders and straps; this could lead to shallow breathing, atelectasis and pneumonia Observe for complications such as pneumothorax, hemothorax and fail chest
B. health care personnel in contact with the patient using the HEPA respirator masks
Nursing managment of a patient with the diagnosis of pumonary tuberculosis would include: A. placing a client in contact precautions B. health care personnel in contact with the patient using the HEPA respirator masks C. ensuring meal trays have disposable utensils D. increasing the activity level o f a patient as much as possible during the active phase of the illness
A. ensuring sufficient calcium intake to prevent osteoporosis from use of steroid medications
Nutritonal therapy for the patient with chronic obstructive pulmonary disease (COPD) should include increasing fluids to keep mucus thin, resting before eating, eating 4 to 6 small means a day and: A. ensuring sufficient calcium intake to prevent osteoporosis from use of steroid medications B. increasing intake of caffeine in order to boost energy levels C. lying down immediately after meals to allow for adequate rest D. increasing sodium intake
CPAP is indicated for what patients?
Obstructive sleep apnea
1, 2, 3 Bronchial, vesicular, and bronchovesicular sounds are normal breath sounds. Bronchial sounds are loud and high-pitched and resemble air blowing through a hollow pipe. Vesicular sounds are soft, low-pitched, gentle, rustling sounds heard over all lung areas except the major bronchi. Bronchovesicular sounds are medium-pitched sounds heard anteriorly over the mainstem bronchi on either side of the sternum. Bronchophony is an abnormal breath sound and is considered positive (abnormal) if the patient repeats the phrase "ninety nine" and the words are easily understood and are clear and loud. Whispered pectoriloquy is also an abnormal breath sound and is considered positive (abnormal) when the patient whispers "one-two-three," and the almost inaudible voice is transmitted clearly and distinctly. Text Reference - p. 487
On auscultation of a patient's lungs, which breath sounds would the nurse consider normal? Select all that apply. 1 Loud, high-pitched sounds resembling air blowing through a hollow pipe 2 Soft, low-pitched, gentle, rustling sounds heard over all lung areas except the major bronchi 3 Medium-pitched sounds heard anteriorly over the mainstem bronchi on either side of the sternum 4 The patient repeats the phrase "ninety-nine," and the words are easily understood and are clear and loud through the chest. 5 The patient whispers "one-two-three," and the almost inaudible voice is transmitted clearly and distinctly.
What is the pathophysiology of open pneumothorax?
Opening through chest that allows air into the pleural space
A client who has systemic lupus erythematosus and has been on corticosteroids for 10 years. The nurse should observe the client for the development of which complications related to chronic corticosteroid use?
Osteoporosis. This is a common complication of chronic corticosteroid use that leads to an increased risk for fracture.
What treatment interventions are appropriate for a client with pulmonary embolism?
Prevent! - Ambulate and hydrate Oxygen ABGs Decrease pain Heparin sodium, warfarin, enoxaparin Anticoagulant drugs While on warfarin, limit green leafy vegetables and limit foods high in vitamin k Bleeding precautions Surgery Bed rest Elevate extremities to increase venous blood return; decrease pooling TED hose; increase venous return and decrease pooling - Often used with SCDs Warm, moist heat decreases inflammation - Never put cold on a vein = excessive vasoconstriction - Never put hot on a vein = Excessive vasodialation Isometric exercises - decreases stasis
The nurse has given a client taking ethambutol information about the medication. The nurse determines that the client understands the instructions if the client states to report which occurrence immediately?
Problems with visual acuity
The nurse is instructing a client about pursed lip breathing, and the client asks the nurse about its purpose. The nurse should tell the client that the primary purpose of pursed lip breathing is which?
Promote carbon dioxide elimination
All of the following care tasks are needed by a patient admitted for joint replacement surgery who has had a permanent tracheostomy for over 10 years. Which task is appropriate for the nurse to delegate to unlicensed assistive personnel (UAP)? Suction the tracheostomy. Check stoma site for skin breakdown. Complete tracheostomy care using sterile technique. Provide oral care with a toothbrush and tonsil suction tube.
Provide oral care with a toothbrush and tonsil suction tube. Oral care (for a stable patient with a tracheostomy) can be delegated to UAP. A registered nurse would be responsible for assessments (e.g., checking the stoma for skin breakdown) and tracheostomy suctioning and care.
A 62-year-old male has a tracheostomy tube after reconstructive surgery for invasive head and neck cancer. What is most important for the nurse to assess before performing tracheostomy cannula care? Level of consciousness Quality of breath sounds Presence of the gag reflex Tracheostomy cuff pressure
Quality of breath sounds Before performing tracheostomy care, the nurse will auscultate lung sounds to determine the presence of secretions. To prevent aspiration, secretions must be cleared either by coughing or by suctioning before performing tracheostomy cannula care.
The nurse notes that a hospitalized client has experienced a positive reaction to the tuberculin skin test. Which action by the nurse is priority?
Report the findings
A client is to begin a 6-month course of therapy with isoniazid. The nurse should plan to provide which information to the client?
Report yellow eyes or skin immediately
The nurse is caring for a client after pulmonary angiography via catheter insertion into the left groin. The nurse monitors for an allergic reaction to the contrast medium by observing for the presence of what?
Respiratory distress
What is the priority nursing assessment in the care of a patient who has a tracheostomy? Electrolyte levels and daily weights Assessment of speech and swallowing Respiratory rate and oxygen saturation Pain assessment and assessment of mobility
Respiratory rate and oxygen saturation The priority assessment in the care of a patient with a tracheostomy focuses on airway and breathing. These assessments supersede the nurse's assessments that may also be necessary, such as nutritional status, speech, pain, and swallowing ability.
Liver biopsy:
Right side position post procedure to prevent patient from bleeding.
The nurse is reinforcing discharge instructions to the client with pulmonary sarcoidosis. The nurse knows that the client understands the information if the client verbalizes which early sign of exacerbation?
SOB
What are signs and symptoms of a hemothorax/pneumothorax?
SOB Increased HR Diminished breath sounds on the affected side Less movement on the affected side Chest pain Cough Air and blood on the chest x-ray Subcutaneous emphysema is air trapped in the tissue (usually neck, face, and chest)
Rifabutin is prescribed for a client with active Mycobacterium avium complex (MAC) disease with TB. The nurse should monitor for which side/adverse effects of the medication? (SATA)
Signs of hepatitis; Flu-like syndrome; Low neutrophil count; Ocular pain or blurred vision
Which of the following positions is recommended for a pt experiencing a nosebleed?
Sitting up leaning slightly forward
The nurse is reinforcing instructions to a hospitalized client with a diagnosis of emphysema about positions that will enhance the effectiveness of breathing during dyspneic episodes. Which position should the nurse instruct the client to assume?
Sitting up on the side of the bed, leaning on an overbed table.
Which laboratory result should alert the nurse to perform further assessment on pt admitted with respiratory distress?
SpO2 <90%
The nurse is gathering data on a client with a diagnosis of TB. The nurse should review the results of which diagnostic test to confirm this diagnosis?
Sputum culture
What treatment interventions are appropriate for a client with flail chest?
Stabilize the area, intubate, and ventilate Positive pressure ventilation stabilizes the area
When you think pneumothorax, think what?
Sucking chest wound
A client is receiving acetylcysteine 20% solution diluted in 0.9% normal saline by nebulizer. The nurse should have which item available for a possible adverse event after giving this medication?
Suction equipment
Which task can the registered nurse (RN) delegate to unlicensed assistive personnel (UAP) in the care of a stable patient who has a tracheostomy? Assessing the need for suctioning Suctioning the patient's oropharynx Assessing the patient's swallowing ability Maintaining appropriate cuff inflation pressure
Suctioning the patient's oropharynx Providing the individual has been trained in correct technique, UAP may suction the patient's oropharynx. Assessing the need for suctioning should be performed by an RN or licensed practical nurse, whereas swallowing assessment and the maintenance of cuff inflation pressure should be performed solely by the RN.
A postop client has received a dose of naloxone hydrochloride for respiratory depression shortly after transfer to the nursing unit from the PACU. After administration of the medication, the nurse should check the client for which sign/symptoms?
Sudden increase in pain
Which of the following interventions is most appropriate for the pt with an ineffective breathing pattern?
Teach the pt controlled diaphragmatic breathing
The nurse notices clear nasal drainage in a patient newly admitted with facial trauma, including a nasal fracture. What should the nurse do first? Test the drainage for the presence of glucose. Suction the nose to maintain airway clearance. Document the findings and continue monitoring. Apply a drip pad and reassure the patient this is normal.
Test the drainage for the presence of glucose. Clear nasal drainage suggests leakage of cerebrospinal fluid (CSF). The drainage should be tested for the presence of glucose, which would indicate the presence of CSF. Suctioning should not be done. Documenting the findings and monitoring are important after notifying the health care provider. A drip pad may be applied, but the patient should not be reassured that this is normal.
A (Feedback: Clear fluid from either nostril suggests a fracture of the cribriform plate with leakage of cerebrospinal fluid. The symptoms are not indicative of an abrasion of the soft tissue or rupture of a sinus. Clear fluid leakage from the nose would not be indicative of a fracture of the nasal septum.)
The ED nurse is assessing a young gymnast who fell from a balance beam. The gymnast presents with a clear fluid leaking from her nose. What should the ED nurse suspect? A) Fracture of the cribriform plate B) Rupture of an ethmoid sinus C) Abrasion of the soft tissue D) Fracture of the nasal septum
D (Feedback: A cotton tampon may be used to try to stop the bleeding. The use of ice on the bridge of the nose has no scientific rationale for care. Laying the client down on the cot could block the client's airway. Hospital admission is necessary only if the bleeding becomes serious.)
The campus nurse at a university is assessing a 21-year-old student who presents with a severe nosebleed. The site of bleeding appears to be the anterior portion of the nasal septum. The nurse instructs the student to tilt her head forward and the nurse applies pressure to the nose, but the student's nose continues to bleed. Which intervention should the nurse next implement? A) Apply ice to the bridge of her nose B) Lay the patient down on a cot C) Arrange for transfer to the local ED D) Insert a tampon in the affected nare
What is the function of the drainage collection chamber?
The chest tube connects to a 6 food connection tube that leads to the drainage collections chamber. If this chamber fills up get a new CDU
How is a chest tube applied?
The chest tube is sutured to the chest wall and an vaseline or airtight dressing is applied around the chest tube exit site. The chest tube is then connected to a closed chest drainage unit.
D. "Ketone testing should be done each morning when I am in a fasting state."
The diabetic patient has been instructed on ketone testing. Which statement indicates the need for further instruction? A. "Ketones indicate periods of poor glycemic control." B. "I should perform ketone testing whenever I am hyperglycemic." C. "Performing ketone testing during periods of illness is recommended." D. "Ketone testing should be done each morning when I am in a fasting state."
How is the water seal chamber connected?
The drainage chamber and water seal chamber are connected by a straw-like channel that allows the drainage to remain in the first chamber and the air to go down into the water seal chamber. This chamber contains 2 cm of water which acts as a one-way valve. In other words, we are preventing backflow.
A client being discharged from the hospital to home with a diagnosis of TB is worried about the possibility of infecting family members and others. Which information should reassure the client that contaminating family members is not likely?
The family will receive prophylactic therapy, and the client will not be contagious after 2-3 consecutive weeks of medication therapy.
A (Feedback: The nurse stresses the importance of humidification at home and instructs the family to obtain and set up a humidification system before the patient returns home. Air-conditioning may be too cool and too drying for the patient. A water purification system or a radiant heating system is not necessary.)
The home care nurse is assessing the home environment of a patient who will be discharged from the hospital shortly after his laryngectomy. The nurse should inform the patient that he may need to arrange for the installation of which system in his home? A) A humidification system B) An air conditioning system C) A water purification system D) A radiant heating system
ventilate the client manually
The low-pressure alarm sounds on the ventilator. The nurse checks the client and the attempts to determine the cause of the alarm but its unsuccessful. Which *intial* action should the nurse take? A. administer oxygen B.ventilate the client manually C.check the clients vital signs D. start cardiopulmonary resuscitation (CPR)
3 Tachypnea and clubbing of the fingers support the diagnosis of chronic hypoxemia. Pursed-lip breathing, inability to lie in a flat position, and use of accessory muscles to assist with breathing are findings observed in patients with asthma and chronic obstructive pulmonary disease. Voluntary decrease in tidal volume to reduce pain on chest expansion is referred to as splinting, which is a common manifestation of chest trauma or pleurisy. Text Reference - p. 489
The nurse assesses a patient who presents with tachypnea and clubbing of the fingers. Which diagnosis does the nurse anticipate for this patient? 1 Asthma 2 Chest trauma 3 Chronic hypoxemia 4 Chronic pulmonary obstructive disease
Hyperinflation of the lungs
The nurse assesses a patient with emphysema and notes a barrel chest. What is the reason for this patient's chest anomaly? A. Collapse of distal alveoli B. Use of accessory muscles C. Hyperinflation of the lungs D. Long-term, chronic hypoxia
A. Dry Skin E. Constipated F. Cold intolerance
The nurse educator is asking the nursing student to recall the signs and symptoms of hypothyroidism. The nurse educator determines that the student understands this disorder if which are included in the students response? select all that apply A. Dry skin B. Irritability C. Palpation D. Weight loss E. Constipated F. Cold intolerance
C (Feedback: The nurse promptly notifies the surgeon of any active bleeding, which can occur at a variety of sites, including the surgical site, drains, and trachea. The drain should not be removed or connected to suction. Supine positioning would exacerbate the bleeding. Vitamin K would not be administered without an order.)
The nurse has noted the emergence of a significant amount of fresh blood at the drain site of a patient who is postoperative day 1 following total laryngectomy. How should the nurse respond to this development? A) Remove the patient's drain and apply pressure with a sterile gauze. B) Assess the patient, reposition the patient supine, and apply wall suction to the drain. C) Rapidly assess the patient and notify the surgeon about the patient's bleeding. D) Administer a STAT dose of vitamin K to aid coagulation.
3 Vigorous coughing and sneezing causes leakage of the cerebrospinal fluid, which increases the risk of developing meningitis. Seizures are changes in the brain's electrical activity; they are not triggered by vigorous coughing and sneezing. Postoperative headache can be avoided by raising the head of the patient's bed to a 30-degree angle. Pupillary and speech responses are monitored to detect neurologic complications. Text Reference - p. 1191
The nurse informs a patient that has had a hypophysectomy for removal of a pituitary tumor to avoid vigorous coughing and sneezing. What explanation does the nurse give the patient for these instructions? 1 To avoid seizures 2 To avoid headache 3 To prevent cerebrospinal fluid leakage 4 To monitor pupillary and speech responses
1 Indomethacin is a nonsteroidal antiinflammatory drug that helps increase the renal response to antidiuretic hormone. Patients with nephrogenic diabetes insipidus are not responsive to hormonal therapy. Hormonal therapy would not aid in increasing the renal response to antidiuretic hormone. Thiazide diuretics and limiting sodium intake are the primary treatments for nephrogenic diabetes insipidus; the patient has not responded to these treatments. Text Reference - p. 1195
The nurse is caring for a patient diagnosed with nephrogenic diabetes insipidus not responding to primary treatment. Which intervention does the nurse expect to be useful in increasing the renal response to antidiuretic hormone? 1 Administering indomethacin 2 Providing hormonal therapy 3 Administering thiazide diuretics 4 Limiting sodium intake to 3 g/day
2 If intravenous glucose solutions are administered, the serum glucose levels of the patient should be monitored, because hyperglycemia and glycosuria can occur, which can lead to osmotic dieresis and an increase in fluid volume deficit. Monitoring blood pressure is not specifically required for managing osmotic dieresis, but is essential and may be required hourly in patients who are acutely ill. Monitoring fluid and electrolyte levels is required to check the intake and output of fluids and is useful for adjusting fluid levels during fluid replacement, not for the management of osmotic diureses. Specific gravity of urine is not related to osmotic dieresis. Text Reference - p. 1195
The nurse is administering intravenous glucose solution to a patient who is suffering from diabetes insipidus. What parameters would the nurse monitor for the management of osmotic diuresis? 1 Blood pressure 2 Serum glucose levels 3 Fluid and electrolytes 4 Specific gravity of the urine
2 Tuberculosis is characterized by hemoptysis, which is the act of coughing up blood or blood-tinged sputum from the respiratory tract. Wheezing is the term used to describe the musical sounds auscultated during assessment; it indicates some degree of airway obstruction that occurs with asthma and emphysema. Grey sputum often occurs in patients who smoke cigarettes. Clear sputum, slightly whitish sputum, and viscous sputum are often normal findings. Text Reference - p. 483
The nurse is assessing a patient diagnosed with tuberculosis. Which assessment finding supports this diagnosis? 1 Wheezing 2 Hemoptysis 3 Gray sputum 4 Slightly whitish sputum
4 In a patient with pneumothorax, the nurse would find absent fremitus. Decreased fremitus may be found in pleural effusion when the hand is farther from the lung, and in barrel chest where the lung is hyperinflated. Increased fremitus is found in pneumonia, in lung tumors, with thick bronchial secretions, and above a pleural effusion. As the patient's voice moves through a dense tissue or fluid-filled lungs, the vibration is increased. Text Reference - p. 486
The nurse is assessing a patient with a pneumothorax. The nurse expects to note which type of fremitus? 1 Normal fremitus 2 Decreased fremitus 3 Increased fremitus 4 Absent fremitus
1 Patients with pulmonary fibrosis have normal percussion findings. Inspection would reveal tachypnea and palpation would show movement. Auscultation shows crackles or sounds like Velcro being pulled apart. Prolonged expiration occurs with asthma and fremitus over affected area occurs with pneumonia. Egophony over effusion often occurs with pleural effusion. Test-Taking Tip: Chart/exhibit items present a situation and ask a question. A variety of objective and subjective information is presented about the client in formats such as the hospital record (e.g., laboratory test results, results of diagnostic procedures, progress notes, health care provider orders, medication administration record, health history), physical assessment data, and nurse/client interactions. After analyzing the information presented, the test taker answers the question. These questions usually reflect the analyzing level of cognitive thinking. Text Reference - p. 491
The nurse is assessing a patient with a respiratory problem. Which findings indicate the patient is suffering from pulmonary fibrosis? 1 Normal percussion 2 Prolonged expiration 3 Fremitus over affected area 4 Egophony over effusion
2, 3, 6 A person's normal defense mechanisms include one's cough reflex, alveolar macrophages, and the mucociliary clearance system. All of those personal defense mechanisms help protect one from inhaled particles, microorganisms, and toxic gases. The gender of the patient, one's history of chronic lung disease, and the quality of the ambient air have no effect on a person's personal defense mechanism system. Text Reference - p. 480
The nurse is assessing a patient with complaints of chronic wheezing. The nurse knows that the respiratory defense mechanisms include which of the following? Please select all that apply. 1 Gender of patient 2 The cough reflex 3 Alveolar macrophages 4 Quality of ambient air 5 A history of chronic lung disease 6 The mucociliary clearance system
lateral position
The nurse is assigned to care for a client after a left pneumonectomy. Which position is contraindicated for this client? A. lateral position B. low fowlers position C. semi-fowlers postion D. head of the bed elevation to 40 degrees
4 To appropriately locate the pleural space, the patient needs to be positioned sitting upright with elbows on an over-bed table with feet supported. Lying flat would not adequately expand the thorax and permit the provider to position the thoracentesis needle in the correct place. Sitting in the bed would also not allow the HCP to visualize the pleural space for needle insertion. Test-Taking Tip: Avoid taking a wild guess at an answer. However, should you feel insecure about a question, eliminate the alternatives that you believe are definitely incorrect, and reread the information given to make sure you understand the intent of the question. This approach increases your chances of randomly selecting the correct answer or getting a clearer understanding of what is being asked. Although there is no penalty for guessing on the NCLEX examination, the subsequent question will be based, to an extent, on the response you give to the question at hand; that is, if you answer a question incorrectly, the computer will adapt the next question accordingly based on your knowledge and skill performance on the examination up to that point. Text Reference - p. 493
The nurse is assisting the health-care provider (HCP) in performing a diagnostic thoracentesis on a patient. The nurse positions the patient in what position? 1 Lying flat in the fetal position on the unaffected side 2 Sitting in bed with knees slightly flexed and feet flexed 3 Lying flat on the unaffected side with knees slightly flexed 4 Sitting upright with elbows on an over bed table and feet supported
respiratory disress
The nurse is caring for a client after a pulmonary angiography via catheter insertion into the left groin. The nurse monitors for allergic reaction to the contrast medium by observing for the presence of which? A. hypothermia B. respiratory distress C. hematoma in the left groin D.discomfort in left groin
2
The nurse is caring for a client with emphysema receiving oxygen. The nurse should check the oxygen flow rate to ensure the client does not exceed how many L/minute of oxygen? A. 1 B. 2 C. 6 D. 10
preform tracheostomy care maintain aspiration precautions develop an alternate communication method
The nurse is caring for a patient following a total laryngectomy. Which interventions should the nurse anticipate will be needed? (Select all that apply.) A. Administer oral feedings. B. Perform tracheostomy care. C. Maintain aspiration precautions. D. Maintain neutropenic precautions. E. Develop an alternate communication method.
C (Feedback: Discharge teaching for prevention of epistaxis should include the following: avoid forceful nose bleeding, straining, high altitudes, and nasal trauma (nose picking). Adequate humidification may prevent drying of the nasal passages. Keeping nasal passages clear and using a tissue when blowing the nose are not included in discharge teaching for the prevention of epistaxis. Decongestants are not indicated.)
The nurse is caring for a patient in the ED for epistaxis. What information should the nurse include in patient discharge teaching as a way to prevent epistaxis? A) Keep nasal passages clear. B) Use decongestants regularly. C) Humidify the indoor environment. D) Use a tissue when blowing the nose.
1, 2, 3, 4 If the plasma osmolality and serum sodium levels continue to decline below 120 mmol/L, cerebral edema may occur, leading to the manifestations such as coma, lethargy, confusion, and headache. Tachycardia and hypovolemic shock are the complications of diabetes insipidus. Text Reference - p. 1193
The nurse is caring for a patient who has cerebral edema associated with syndrome of inappropriate antidiuretic hormone (SIADH). What clinical manifestation of severe serum sodium level decline does the nurse assess? Select all that apply. 1 Coma 2 Lethargy 3 Confusion 4 Headache 5 Tachycardia 6 Hypovolemic shock
aspiration
The nurse is caring for a patient who has had a partial laryngectomy and is experiencing difficulty swallowing. For which complication is this patient most at risk? A. Aspiration B. Epiglottitis C. Esophageal varicosities D. Paralysis of the vocal cords
B (Feedback: URIs, specifically chronic rhinosinusitis and recurrent acute rhinosinusitis, may be linked to primary or secondary immune deficiency or treatment with immunosuppressive therapy (i.e., for cancer or organ transplantation). Typical symptoms may be blunted or absent due to immunosuppression. No evidence indicates damage to the transplanted organ due to chronic rhinosinusitis. Immunosuppressive drugs do not cause organ rejection.)
The nurse is caring for a patient who has just been diagnosed with chronic rhinosinusitis. While being admitted to the clinic, the patient asks, "Will this chronic infection hurt my new kidney?" What should the nurse know about chronic rhinosinusitis in patients who have had a transplant? A) The patient will have exaggerated symptoms of rhinosinusitis due to immunosuppression. B) Taking immunosuppressive drugs can contribute to chronic rhinosinusitis. C) Chronic rhinosinusitis can damage the transplanted organ. D) Immunosuppressive drugs can cause organ rejection.
1, 2, 3 COPD may result in whitish to yellowish sputum, however, any change in the baseline characteristics of the sputum should be reported. Frothy sputum may indicate pulmonary edema and needs further evaluation. A foul odor in the sputum indicates presence of infection and needs immediate medical intervention. Pink-tinged sputum may indicate pulmonary edema and the patient may need further evaluation. Sputum with brown specks is a common finding in a person who smokes. Yellowish sputum is a normal finding in COPD. Text Reference - p. 482
The nurse is caring for a patient who is a smoker and is diagnosed with chronic obstructive pulmonary disease (COPD). Which sputum characteristics, if present in the patient, may need further evaluation? Select all that apply. 1 Frothy 2 Foul odor 3 Pink tinged 4 Brown specks
1 Once SIADH is diagnosed, the treatment is directed at the underlying causes. Positioning the head of the patient's bed flat enhances venous return to the heart and increases the left arterial filling pressure, thereby reducing the release of antidiuretic hormone (ADH). Ambulation is necessary to improve circulation the mobility of joints. Ambulation is not involved in promoting a reduction of ADH. Turning and positioning the patient frequently, and encouraging range-of-motion exercises maintain skin integrity and joint mobility, especially in bed-ridden patients; these interventions do not promote a reduction of ADH. Text Reference - p. 1194
The nurse is caring for a patient who is diagnosed with the syndrome of inappropriate antidiuretic hormone (SIADH). Which nursing intervention would promote a reduction in the release of antidiuretic hormone (ADH)? 1 Positioning the head of the bed flat 2 Encouraging the patient to ambulate 3 Changing the patient position frequently 4 Instructing the patient to perform range-of-motion exercises
Sitting, facing the side of the bed
The nurse is caring for a patient who is going to have a thoracentesis performed. How should the nurse position the patient for this procedure? A.Flat, prone B. Flat, supine C. Sitting, facing the side of the bed D. Sitting, facing the front of the bed
D (Feedback: A common postoperative complication from this type of surgery is difficulty in swallowing, which creates a potential for aspiration. Cardiovascular complications are less likely at this stage of recovery. The patient's body image should be assessed, but dysphagia has the potential to affect the patient's airway, and is a consequent priority.)
The nurse is caring for a patient who is postoperative day 2 following a total laryngectomy for supraglottic cancer. The nurse should prioritize what assessment? A) Assessment of body image B) Assessment of jugular venous pressure C) Assessment of carotid pulse D) Assessment of swallowing ability
A (Feedback: Cromolyn (Nasalcrom) inhibits the release of histamine and other chemicals. It is prescribed to treat allergic rhinitis. Beta-adrenergic agents lead to bronchodilation and stimulate beta-2 adrenergic receptors in the smooth muscle of the bronchi and bronchioles. It does not affect proton pump action or the sodium-potassium pump in the nasal cells.)
The nurse is caring for a patient who needs education on his medication therapy for allergic rhinitis. The patient is to take cromolyn (Nasalcrom) daily. In providing education for this patient, how should the nurse describe the action of the medication? A) It inhibits the release of histamine and other chemicals. B) It inhibits the action of proton pumps. C) It inhibits the action of the sodium-potassium pump in the nasal epithelium. D) It causes bronchodilation and relaxes smooth muscle in the bronchi.
monitoring airway patency keeping the patient sitting forward compressing the bleeding nostril against the septum and apply ice
The nurse is caring for a patient with epistaxis who is to be taken for x-rays of the skull and face. What nursing intervention(s) will be necessary? (Select all that apply.) A. Monitoring airway patency B. Administering a narcotic analgesic C. Keeping the patient sitting forward D. Encouraging the patient to take sips of water E. Compressing the bleeding nostril against the septum and applying ice
2 The nurse should monitor the pupillary response, speech patterns, and extremity strength to detect neurologic complications. The nurse should ensure the head of the bed is elevated at all times to a 30-degree angle to avoid pressure on the sellaturcica and to decrease headaches, a frequent postoperative problem. The nurse must perform mouth care for the patient every four hours to keep the surgical area clean and free of debris. The nurse must notify the surgeon and send any clear nasal fluid to the laboratory to test for glucose. Test-Taking Tip: Do not read information into questions, and avoid speculating. Reading into questions creates errors in judgment. Text Reference - p. 1191
The nurse is caring for a patient who underwent a transsphenoidal hypophysectomy. What is the most important nursing intervention for this patient? 1 Place the patient in a supine position at all times. 2 Monitor pupillary response and speech patterns. 3 Perform mouth care every 12 hours. 4 Test any clear nasal drainage for potassium.
4 Vigorous coughing, sneezing, and straining at stool may result in cerebrospinal fluid (CSF) leakage after transsphenoidal hypophysectomy. The patient should lie in an elevated position, with the head of the bed at a 30-degree angle, as opposed to lying in the supine position. Tooth brushing should be avoided for a minimum of 10 days to protect the suture line. Reporting clear nasal discharge to the surgeon is important, because elevated glucose levels in the discharge indicate CSF leakage; however, this intervention does not prevent the leakage of CSF.
The nurse is caring for a patient who underwent transsphenoidal hypophysectomy to treat acromegaly. Which is the best nursing action to prevent leakage of cerebrospinal fluid? 1 Having the patient lie down in the supine position 2 Ensuring oral hygiene in the patient by regular brushing 3 Informing the primary health care provider about nasal drainage 4 Instructing the patient to avoid vigorous coughing, sneezing, and straining at stool
A (Feedback: Hoarseness is an early symptom of laryngeal cancer. Dyspnea, dysphagia, and lumps are later signs of laryngeal cancer. Alopecia is not associated with a diagnosis of laryngeal cancer.)
The nurse is caring for a patient whose recent unexplained weight loss and history of smoking have prompted diagnostic testing for cancer. What symptom is most closely associated with the early stages of laryngeal cancer? A) Hoarseness B) Dyspnea C) Dysphagia D) Frequent nosebleeds
1, 2, 4 For a thoracentesis, the nurse should ensure that the patient's informed consent was signed. The patient should be positioned upright with elbows on an over-bed table and feet supported. This position gives appropriate access for needle insertion. A chest x-ray is obtained after the procedure to rule out a pneumothorax. The patient should be instructed not to talk or cough during the procedure, because it can cause injury by displacement of the needle. NPO status, or withholding food and drink, is not required for thoracentesis. Test-Taking Tip: On a test day, eat a normal meal before going to school. If the test is late in the morning, take a high-powered snack with you to eat 20 minutes before the examination. The brain works best when it has the glucose necessary for cellular function. Text Reference - p. 493
The nurse is caring for a patient with a pleural effusion. What nursing interventions are appropriate when preparing this patient for a thoracentesis? Select all that apply. 1 Ensure that the informed consent was signed. 2 Position patient upright with elbows on an over bed table with feet supported. 3 Instruct the patient to cough vigorously during the procedure. 4 Instruct the patient not to eat anything for four hours before the procedure. 5 Obtain chest x-ray after the procedure.
B (Feedback: A compressed nasal sponge may be used. Once the sponge becomes saturated with blood or is moistened with a small amount of saline, it will expand and produce tamponade to halt the bleeding. The packing may remain in place for 48 hours or up to 5 or 6 days if necessary to control bleeding. Antibiotics may be prescribed because of the risk of iatrogenic sinusitis and toxic shock syndrome.)
The nurse is caring for a patient with a severe nosebleed. The physician inserts a nasal sponge and tells the patient it may have to remain in place up to 6 days before it is removed. The nurse should identify that this patient is at increased risk for what? A) Viral sinusitis B) Toxic shock syndrome C) Pharyngitis D) Adenoiditis
3 The hormones secreted by the posterior pituitary gland are antidiuretic hormone and oxytocin. They are produced in the hypothalamus and transported and stored in the posterior pituitary gland. Antidiuretic hormone (ADH) is also referred to as arginine vasopressin, which plays a major role in the regulation of water balance and osmolarity. Oxytocin, growth hormone, and adrenocorticotropic hormone are secreted by the anterior pituitary gland, and are not involved in the regulation of water balance and osmolarity. Text Reference - p. 1193
The nurse is caring for a patient with an alteration in the regulation of water balance and osmolarity. Which hormone will require alteration with medication? 1 Oxytocin 2 Growth hormone 3 Arginine vasopressin 4 Adrenocorticotropic hormone
3 The patient with diabetes insipidus may experience massive diuresis of up to 20 L per day. Severe dehydration and hypovolemic shock may occur if the patient does not consume or receive sufficient fluids to address the urinary losses. The patient may experience nocturia-related weakness and fatigue, but this is of lower priority than preventing dehydration and fluid volume deficit. Diabetes insipidus is a condition of too little ADH. Glucose-lowering agents are not used to treat diabetes insipidus. Whereas diabetes insipidus and diabetes mellitus both result in polydipsia and polyphagia, the mechanism driving these symptoms is entirely different between the two disorders, and treatment is not the same. Diabetes insipidus is a disorder of too little antidiuretic hormone. Decreasing renal responsiveness to a hormone that is already insufficiently present would be deleterious. Text Reference - p. 1195
The nurse is caring for a patient with central diabetes insipidus (DI). What does the nurse recognize is a priority focus of care? 1 Pacing activities and minimizing fatigue 2 Preventing treatment-related hypoglycemia 3 Avoiding dehydration and fluid volume deficit 4 Decreasing renal responsiveness to antidiuretic hormone (ADH)
2 After obtaining blood for an arterial blood gas measurement, the nurse should hold pressure on the puncture site for five minutes to be sure that bleeding has stopped. An artery is an elastic vessel under much higher pressure than veins, and significant blood loss or hematoma formation could occur if the time is insufficient. Text Reference - p. 491
The nurse is caring for a patient with chronic obstructive pulmonary disease (COPD) and pneumonia who has a prescription for arterial blood gases to be drawn. What is the minimum length of time the nurse should plan to hold pressure on the puncture site? 1 2 Minutes 2 5 Minutes 3 10 Minutes 4 15 Minutes
2 The patient is in respiratory distress. Starting at the lung bases is the most preferred method in a patient in respiratory distress, because the increased respiratory rate and shortness of breath may tire the patient easily. Generally, auscultation should proceed from the lung apices to the bases, comparing opposite areas of the chest, unless the patient is in respiratory distress or will tire easily. During any auscultation, the nurse listens to at least one cycle of inspiration and expiration with each placement of the stethoscope. Text Reference - p. 490
The nurse is caring for a patient with shortness of breath and a respiratory rate of 28 breaths/minute. Which is the most preferred method to auscultate the chest of a patient in this condition? 1 Starting at the apices 2 Starting at the lung bases 3 Listening to and comparing opposite areas of the chest 4 Listening to a complete inspiratory and expiratory cycle
4 A patient with a serum sodium level of 116 mEq/L is severely hyponatremic and needs replacement with a hypertonic saline solution (at least 3 to 5 percent) slowly to correct the neurologic effects of the severe hyponatremia. Lorazepam and carbamazepine are used for seizures but would not be indicated in this case since the only way the seizures will cease are with correction of the hyponatremia. Lasix should not be given to a patient with a serum sodium level under 125 mEq/L, because it will cause further loss of sodium. Text Reference - p. 1194
The nurse is caring for a patient with syndrome of inappropriate antidiuretic hormone (SIADH) who has muscle twitching, vomiting, severe abdominal cramps, and begins to have seizures. The nurse assesses a serum sodium level of 116 mEq/L. What is the priority nursing action for this patient? 1 Administer lorazepam IV slowly. 2 Administer carbamazepine by mouth. 3 Administer furosemide 40 mg IV push. 4 Administer hypertonic saline solution slowly.
1, 4, 5 In the acute care setting the patient's total fluid intake is restricted to no more than 1000 mL/day, including that taken with medications. The nurse should implement seizure precautions and set the bed alarm to protect the patient from injury, because of the potential for an alteration in mental status. The nurse should provide the patient with frequent oral care and ice chips to decrease discomfort related to thirst from the fluid restrictions. The head of the bed should be flat or elevated no more than 10 degrees to enhance venous return to the heart and increase left atrial filling pressure, thereby reducing the release of antidiuretic hormone. Frequent turning, positioning, and range-of-motion exercises are important to maintain skin integrity and joint mobility. Text Reference - p. 1193
The nurse is caring for a patient with syndrome of inappropriate antidiuretic hormone secretion in the acute care setting. What nursing interventions are important for this patient? Select all that apply. 1 Restrict fluid intake to no more than 1000 mL/day. 2 Elevate the head of the bed to an angle of 30 degrees. 3 Avoid frequent repositioning of the patient. 4 Implement seizure precautions and set the bed alarm. 5 Provide the patient with ice chips to decrease thirst.
A. Providing adequate rest periods B. Maintaining adequate fluid intake D. Monitoring vital signs and respiratory status E. Providing oral hygiene before and after meals
The nurse is caring for a patient with viral pneumonia. Which intervention(s) should the nurse expect to be included in the care plan? (Select all that apply.) A. Providing adequate rest periods B. Maintaining adequate fluid intake C. administering organism-specific antibiotics D. Monitoring vital signs and respiratory status E. Providing oral hygiene before and after meals
B. Laryngeal stridor
The nurse is caring for a post-op parathyroidectomy client. Which would require the nurses immediate attention? A. Incision pain B. Laryngeal stridor C. Difficulty voiding D. Abdominal cramps
4 There are certain age-related changes to the respiratory system that increase a patient's risk for developing respiratory disorders. The patient's symptoms suggest pneumonia. Based on these data, the nurse suspects that the current symptoms are caused by decreased alveolar macrophage functions, which are responsible for protecting the lungs against bacterial infection. Hematemesis, increased compliance, and an alpha1-antitrypsin deficiency are not the likely causes for this patient's symptoms. Text Reference - p. 481
The nurse is caring for an older adult patient who presents with a cough, chest pain, shortness of breath, a heart rate of 110 beats/minute, a respiratory rate of 30 breaths/minute, and a temperature of 101° F. Based on these data, what does the nurse suspect is causing the patient's symptoms? 1 Hematemesis 2 Increased compliance 3 Alpha1-antitrypsin deficiency 4 Decreased alveolar macrophage function
a man who is an inspector for the postal service
The nurse is caring for several clients with respiratory disorders. Which client is atleast risk for developing a tuberculosis infection? A. an uninsured man who is homeless B. a women newly immigrated from korea C.a man who is an inspector for the U.S postal service D. an older women admitted from a long-term care facility
1 When assessing the effect of COPD on the patient's nutritional-metabolic pattern, the nurse should ask if the patient has experienced any weight loss. Asking about trouble getting to the toilet assesses the effect that COPD has on the patient's elimination patterns. Asking the patient about waking in the middle of the night with breathing issues assesses the patient's sleep-rest pattern. Asking the patient about pain associated with breathing assesses the patient's cognitive-perceptual pattern. Text Reference - p. 483
The nurse is conducting a health history interview with a patient diagnosed with chronic obstructive pulmonary disease (COPD). Which question is appropriate when assessing the patient's nutritional-metabolic pattern? 1 "Have you lost any weight recently?" 2 "Do you have trouble getting to the toilet?" 3 "Does your breathing wake you up in the night?" 4 "Do you have any pain associated with breathing?"
B (Feedback: Considering the known risk factors for cancer of the larynx, it is essential to assess the patient's history of alcohol intake. Infection is a risk in the postoperative period, but not an appropriate answer based on the patient's history. Depression and nonadherence are risks in the postoperative phase, but would not be critical short-term assessments.)
The nurse is conducting a presurgical interview for a patient with laryngeal cancer. The patient states that he drinks approximately six to eight shots of vodka per day. It is imperative that the nurse inform the surgical team so the patient can be assessed for what? A) Increased risk for infection B) Delirium tremens C) Depression D) Nonadherence to postoperative care
C (Feedback: Tracheoesophageal puncture is simple and has few complications. It is associated with high phonation success, good phonation quality, and steady long-term results. As a result, it is preferred over esophageal speech, and electric larynx or ASL.)
The nurse is creating a care plan for a patient who is status post-total laryngectomy. Much of the plan consists of a long-term postoperative communication plan for alaryngeal communication. What form of alaryngeal communication will likely be chosen? A) Esophageal speech B) Electric larynx C) Tracheoesophageal puncture D) American sign language (ASL)
C (Feedback: Management of acute laryngitis includes resting the voice, avoiding irritants (including smoking), resting, and inhaling cool steam or an aerosol. Fluid intake should be increased. Warm clothes on the throat will not help relieve the symptoms of acute laryngitis.)
The nurse is creating a plan of care for a patient diagnosed with acute laryngitis. What intervention should be included in the patient's plan of care? A) Place warm cloths on the patient's throat, as needed. B) Have the patient inhale warm steam three times daily. C) Encourage the patient to limit speech whenever possible. D) Limit the patient's fluid intake to 1.5 L/day.
B (Feedback: The nurse explains how to apply direct pressure to the nose with the thumb and the index finger for 15 minutes in case of a recurrent nosebleed. If recurrent bleeding cannot be stopped, the patient is instructed to seek additional medical attention. ASA is not contraindicated in most cases and the patient should avoiding blowing the nose for an extended period of time, not just 45 minutes.)
The nurse is doing discharge teaching in the ED with a patient who had a nosebleed. What should the nurse include in the discharge teaching of this patient? A) Avoid blowing the nose for the next 45 minutes. B) In case of recurrence, apply direct pressure for 15 minutes. C) Do not take aspirin for the next 2 weeks. D) Seek immediate medical attention if the nosebleed recurs.
D (Feedback: The use of topical decongestants is controversial because of the potential for a rebound effect. The patient should hold his or her head back for maximal distribution of the spray. Only the patient should use the bottle.)
The nurse is explaining the safe and effective administration of nasal spray to a patient with seasonal allergies. What information is most important to include in this teaching? A) Finish the bottle of nasal spray to clear the infection effectively. B) Nasal spray can only be shared between immediate family members. C) Nasal spray should be administered in a prone position. D) Overuse of nasal spray may cause rebound congestion.
sputum culture
The nurse is gathering data on a client with a diagnosis of tuberculosis(TB). The nurse should review the results of which diagnostic test to confirm diagnosis? A. Chest x-ray B. Bronchoscopy C.Sputum Culture D. Tuberculin skin test
promote carbon dioxide elimination
The nurse is instructing a client about pursed lip breathing, and the client asks the nurse about its purpose. The nurse should tell the client that the *primary* purpose of pursed lip breathing is which? A. promote oxygen intake B.strengthen the diaphragm C. strengthen the intercostal muscles D. promote carbon dioxide elimination
3 Electrolyte solutions should be taken during meals, because it dilutes the solution by allowing it to mix with the food, which prevents irritation and damage to the gastrointestinal tract. Administration of demeclocycline blocks the effect of antidiuretic hormone on the renal tubules, resulting in more dilute urine. Chewing sugarless gum and ice chips decreases thirst. Diluting the electrolyte solution before administration prevents gastrointestinal damage. Test-Taking Tip: Identify option components as correct or incorrect. This may help you identify a wrong answer. Text Reference - p. 1194
The nurse is instructing a patient regarding self-management of syndrome of inappropriate antidiuretic hormone (SIADH). The patient has gastric ulceration as a comorbid condition. Which statement by the patient needs correction? 1 "I should take demeclocycline as prescribed." 2 "I should chew sugarless gum and ice chips." 3 "I should drink the electrolyte solution after meals." 4 "I should dilute the electrolyte solution before administration."
4 The abbreviation SpO2 is used to indicate the oxygen saturation of hemoglobin as measured by pulse oximetry. If there is doubt about the accuracy of the SpO2 reading, obtain an ABG analysis to verify the results. The findings should not be documented as normal. The nurse should not increase oxygen flow without further assessment. Test-Taking Tip: Start by reading each of the answer options carefully. Usually at least one of them will be clearly wrong. Eliminate this one from consideration. Now you have reduced the number of response choices by one and improved the odds. Continue to analyze the options. If you can eliminate one more choice in a four-option question, you have reduced the odds to 50/50. While you are eliminating the wrong choices, recall often occurs. One of the options may serve as a trigger that causes you to remember what a few seconds ago had seemed completely forgotten. Text Reference - p. 478
The nurse is measuring a patient's pulse oximetry and obtains an SpO2 reading of 88%, and then another reading of 87%. The patient is receiving oxygen per nasal cannula at 2 L/minute and states feeling short of breath. Previous readings were above 94%. What should the nurse do next? 1 Document the findings. 2 Increase the flow of oxygen delivery. 3 Check the pulse oximetry again in an hour. 4 Notify the health care provider for a possible arterial blood gas (ABG) analysis.
A. " I am urinating a lot"
The nurse is monitoring a client who has been newly diagnosed with diabetes mellitus for signs of complications. Which statement made by the client would indicate hyperglycemia and thus warrant health care provider notification? A. " I am urinating a lot" B. " My pulse is really slow" C. " I am sweating for no reason" D. " My blood pressure is really high"
persistent hoarseness
The nurse is performing an admission assessment on a patient who is scheduled for several diagnostic respiratory procedures. Which symptom(s) reported by the patient would make the nurse suspect the patient may have laryngeal cancer? A. Anemia B. Difficulty swallowing C. Persistent hoarseness D. Sleep apnea and snoring
" You should see your primary health care provider"
The patient tells the LPN/LVN that she has been hoarse for the past 2½ weeks. Which response by the nurse is most appropriate? A."Try to talk as little as possible." B. "Gargle with warm, slightly salted water." C. "You should see your primary health care provider." D. "Use a high-humidity vaporizer two or three times a day."
B, C, E (Feedback: The nurse also assesses the patient's general state of nutrition, including height and weight and body mass index, and reviews laboratory values that assist in determining the patient's nutritional status (albumin, protein, glucose, and electrolyte levels). The white blood cell count and the platelet count would not normally assist in determining the patient's nutritional status.)
The nurse is performing an assessment on a patient who has been diagnosed with cancer of the larynx. Part of the nurse's assessment addresses the patient's general state of nutrition. Which laboratory values would be assessed when determining the nutritional status of the patient? Select all that apply. A) White blood cell count B) Protein level C) Albumin level D) Platelet count E) Glucose level
C (Feedback: Informational materials (written and audiovisual) about the surgery are given to the patient and family for review and reinforcement. The nurse never gives personal contact information to the patient. Nothing in the scenario indicates that a referral to a social worker or psychologist is necessary. False reassurance must always be avoided.)
The nurse is performing preoperative teaching with a patient who has cancer of the larynx. After completing patient teaching, what would be most important for the nurse to do? A) Give the patient his or her cell phone number. B) Refer the patient to a social worker or psychologist. C) Provide the patient with audiovisual materials about the surgery. D) Reassure the patient and family that everything will be alright.
B (Feedback: Beconase should be avoided in patients with recurrent epistaxis, glaucoma, and cataracts. Sinustop Pro and Afrin are pseudoephedrine and do not have a side effect of epistaxis. Singulair is a bronchodilator and does not have epistaxis as a side effect.)
The nurse is performing the health interview of a patient with chronic rhinosinusitis who experiences frequent nose bleeds. The nurse asks the patient about her current medication regimen. Which medication would put the patient at a higher risk for recurrent epistaxis? A) Afrin B) Beconase C) Sinustop Pro D) Singulair
C (Feedback: Each of the listed diagnoses is valid, but ineffective airway clearance is the priority nursing diagnosis for all conditions.)
The nurse is planning the care of a patient who is scheduled for a laryngectomy. The nurse should assign the highest priority to which postoperative nursing diagnosis? A) Anxiety related to diagnosis of cancer B) Altered nutrition related to swallowing difficulties C) Ineffective airway clearance related to airway alterations D) Impaired verbal communication related to removal of the larynx
A. Activities should be resumed gradually C. a sputum culture is needed every 2 to 4 weeks once medication therapy has started E. Cover the mouth and nose when coughing of sneezing and confine used tissues to plastic bags
The nurse is preparing a list of home care instructions for the client who has been hospitalized and treated for tuberculosis. Which instructions should the nurse reinforce? select all that apply A. Activities should be resumed gradually B. avoid contact with other individuals except family members, for atleast 6 months C.a sputum culture is needed every 2 to 4 weeks once medications therapy is initiated D. Respiratory isolation is not necessary because family members have already been exposed E. Cover the mouth and nose when coughing or sneezing and confine used tissues to plastic bags F. When one sputum culture is negative, the client is no longer considered infectious and can usually return to work
B (Feedback: The patient with a laryngectomy is a risk for airway occlusion and respiratory distress. As in all nursing situations, assessment of the airway is a priority over other potential complications and assessment parameters.)
The nurse is providing care for a patient who has just been admitted to the postsurgical unit following a laryngectomy. What assessment should the nurse prioritize? A) The patient's swallowing ability B) The patient's airway patency C) The patient's carotid pulses D) Signs and symptoms of infection
3 Injury to the phrenic nerve above C3 results in a complete spinal cord injury, causing complete diaphragm paralysis and dependence on a mechanical ventilator. Dysrhythmia, alveolar collapse, and impaired cardiac output are not causes of diaphragm paralysis that necessitate mechanical ventilation. Text Reference - p. 478
The nurse is providing care to a patient who is being mechanically ventilated due to diaphragm paralysis. Which is the most likely reason for the current clinical manifestations and treatment plan? 1 Dysrhythmia 2 Alveolar collapse 3 Spinal cord injury 4 Impaired cardiac output
3 Alveolar macrophages rapidly phagocytize inhaled foreign particles, such as bacteria, and often fail as a result of cigarette smoking. The cough reflex, filtration of air, and mucociliary clearance are not the respiratory defense mechanisms that may have failed and caused pneumonia. Text Reference - p. 480
The nurse is providing care to a patient who is diagnosed with pneumonia. The patient smokes one pack of cigarettes per day. Which respiratory defense mechanism may have failed, causing the patient's diagnosis? 1 Cough reflex 2 Filtration of air 3 Alveolar macrophages 4 Mucociliary clearance system
C. Recurrent infections D. Delayed wound healing E. Peripheral vascular disease
The nurse is providing discharge teaching to a patient recently diagnosed with type 2 DM. The nurse should include information on which long-term consequences of poor glycemic control? (Select all that apply.) A. Depression B. Hypertension C. Recurrent infections D. Delayed wound healing E. Peripheral vascular disease
A (Feedback: Patient teaching is an important aspect of nursing care for the patient with acute rhinosinusitis. The nurse instructs the patient about symptoms of complications that require immediate follow-up. Referral to a physician is indicated if periorbital edema and severe pain on palpation occur. Clear drainage and blood-tinged mucus do not require follow-up if the patient has acute rhinosinusitis. A persistent headache does not necessarily warrant immediate follow-up.)
The nurse is providing patient teaching to a patient diagnosed with acute rhinosinusitis. For what possible complication should the nurse teach the patient to seek immediate follow-up? A) Periorbital edema B) Headache unrelieved by OTC medications C) Clear drainage from nose D) Blood-tinged mucus when blowing the nose
One of the benefits of Glargine (Lantus) insulin is its ability to: a. Release insulin rapidly throughout the day to help control basal glucose. b. Release insulin evenly throughout the day and control basal glucose levels. c. Simplify the dosing and better control blood glucose levels during the day. d. Cause hypoglycemia with other manifestation of other adverse reactions.
b. Release insulin evenly throughout the day and control basal glucose levels. Glargine (Lantus) insulin is designed to release insulin evenly throughout the day and control basal glucose levels.
D (Feedback: Handwashing remains the most effective preventive measure to reduce the transmission of organisms. Taking prescribed antibiotics, using warm salt-water gargles, and dressing warmly do not suppress transmission. Antibiotics are not prescribed for a cold.)
The nurse is providing patient teaching to a young mother who has brought her 3-month-old infant to the clinic for a well-baby checkup. What action should the nurse recommend to the woman to prevent the transmission of organisms to her infant during the cold season? A) Take preventative antibiotics, as ordered. B) Gargle with warm salt water regularly. C) Dress herself and her infant warmly. D) Wash her hands frequently.
shortness of breath
The nurse is reinforcing discharge instructions to the client with pulmonary sarcoidosis. The nurse knows that the client understands the information if the client verbalizes which *early* sign of exacerbation? A. fever B. fatigue C. weight loss D. shortness of breath
" I should not be contagious after 2 to 3 weeks of medication therapy."
The nurse is reinforcing discharge teaching with a client diagnosed with TB and has been on medication for 1 and a half weeks. The nurse knows that the client has understood the information if which statement is made? A. " I can't shop at the mall for the next 6 months" B. " I need to continue medication therapy for 2 months." C. " I can return to work if a sputum culture comes back negative." D. " I should not be contagious after 2 to 3 weeks of medication therapy."
A. " I can eat foods that contain potassium."
The nurse is reinforcing discharge teaching with a client who has Cushings Syndrome. Which statement by the client indicated that the instructions related to dietary management were understood? A. " I can eat foods that contain potassium." B. " I will need to limit the amount of protein in my diet." C. " I am fortunate that i can eat all the salty foots I enjoy." D. " I am fortunate that I do not need to follow my special diet."
C. Monitor blood glucose levels frequently
The nurse is reinforcing instructions with a client with diabetes mellitus who is recovering from DKA regarding measures to prevent a recurrence. Which instruction is important for the nurse to emphasize? A.Eat six small meals daily B. Test the urine ketones level C. Monitor blood glucose levels frequently D. Receive appropriate follow- up health care
2 A pleural effusion involves the build-up of fluid in the pleural cavity around the lungs. Presence of fluid in the pleural cavity may result in decreased tactile fremitus, dullness on percussion, and absence of breath sounds. The patient may also have unequal chest movement due to fluid collection. Therefore, Patient B is likely to have a pleural effusion. Patient A is likely to have asthma, as evidenced by pursed-lip breathing, use of accessory muscles for breathing, increased AP diameter of chest, and presence of wheezing. Patient C is likely to have chronic obstructive pulmonary disease (COPD), as evidenced by the tripod position, use of accessory muscles for breathing, hyperresonance on percussion, and presence of coarse crackles. Patient D is likely to have cystic fibrosis, as evidenced by an increased AP diameter of the chest, finger clubbing, presence of rhonchi, and a history of repeated pulmonary infections. Text Reference - p. 489
The nurse is reviewing the assessment findings of four patients. Which patient should be evaluated for pleural effusion? 1 Patient A 2 Patient B 3 Patient C 4 Patient D
A. Increased heart rate B. Increased appetite C. Emotional instability E. Hyperactivity with increasing sense of fatigue
The nurse is reviewing the history of a patient suspected of having hyperthyroidism. Which manifestation(s) would be supportive of the diagnosis? (Select all that apply.) A. Increased heart rate B. Increased appetite C. Emotional instability D. Mental sluggishness E. Hyperactivity with increasing sense of fatigue
2 Patients with nephrogenic diabetes insipidus will not be able to increase urine osmolality to greater than 300 mOsm/kg. Patients with central diabetes insipidus show a significant decrease in urine volume below 200 mL/hr and a dramatic increase in the urine osmolality from 100 to 600 mOsm/kg. When an antidiuretic hormone analog such as desmopressin is administered, if the patient has central diabetes, the kidneys respond by forming concentrated urine. Text Reference - p. 1195
The nurse is reviewing the results of four diagnostic tests for diabetes insipidus (DI). Which patient's results indicate nephrogenic DI? 1. Patient A 2. Patient B 3. Patient C 4. Patient D
B. Providing the patient with tissues to blow her nose
The nurse is supervising the care of a patient by unlicensed assistive personnel (UAP) after hypophysectomy. Which action by the UAP would prompt the nurse to intervene immediately? A. Encouraging the patient to drink water regularly B. Providing the patient with tissues to blow her nose C. Assisting the patient to the toilet while remaining upright D. Reminding the patient to wear sequential compression devices while in bed
2 Respiratory problems that have a strong genetic link include cystic fibrosis, chronic obstructive pulmonary disease (COPD) resulting from α1-antitrypsin deficiency, and asthma. If people have a family history of these respiratory problems, they have a much greater risk of developing them. A family history of tuberculosis, pulmonary fibrosis, and multiple sclerosis indicates no increased risk for a person to develop one of those diseases. Text Reference - p. 483
The nurse is taking a health history of a patient with respiratory distress. The nurse knows that respiratory problems that have a strong genetic link include: 1 Tuberculosis 2 Cystic fibrosis 3 Multiple sclerosis 4 Pulmonary fibrosis
C. Excessive thirst D. Frequent urination E. Increased appetite
The nurse is teaching a class on diabetes to a group of adults in the community. The nurse should be sure to include information on which classic symptoms of diabetes? (Select all that apply.) A. Hypertension B. Vision changes C. Excessive thirst D. Frequent urination E. Increased appetite F. Recurrent urinary tract infections (UTIs)
1 Motion is the most likely cause of the low SpO2 for this patient with Parkinson's disease. Anemia, dark skin color, and thick acrylic nails, as well as low perfusion, bright fluorescent lights, and intravascular dyes also may cause an inaccurate pulse oximetry result. There is no mention of these or any reason to suspect these in this question. Text Reference - p. 478
The patient with Parkinson's disease has a pulse oximetry reading of 72% but the patient is not displaying any other signs of decreased oxygenation. What most likely is contributing to the patient's low SpO2 level? 1 Motion 2 Anemia 3 Dark skin color 4 Thick acrylic nails
C (Feedback: Patient education is essential when assisting the patient in the use of all medications. To prevent possible drug interactions, the patient is cautioned to read drug labels before taking any OTC medications. Some Web sites are reliable and valid information sources, but this is not always the case. Patients do not necessarily need to limit themselves to one pharmacy, though checking for potential interactions is important. Not all OTC medications are safe additions to prescription medication regimens.)
The nurse is teaching a patient with allergic rhinitis about the safe and effective use of his medications. What would be the most essential information to give this patient about preventing possible drug interactions? A) Prescription medications can be safely supplemented with OTC medications. B) Use only one pharmacy so the pharmacist can check drug interactions. C) Read drug labels carefully before taking OTC medications. D) Consult the Internet before selecting an OTC medication.
report the findings
The nurse notes that a hospitalized client has experience a positive reaction to the tuberculin skin test. Which action by the nurse is priority? A.report the findings B. document the findings in the clients record C. call the employee health service department D. call the radiology department for a chest x-ray
Extra protein is required to repair damage tissues
The nurse notes that the respiratory symptoms of the patient with chronic obstructive pulmonary disease (COPD) have affected his nutrition. Which would most help improve the patient's nutrition? A. Eat three large meals to increase stomach fullness. B. Extra protein is required to repair damaged tissues. C. Exercise before eating each meal three times a day. D. Drink six to eight glasses of caffeinated fluids each day.
C. " I need to buy special diabetic foods"
The nurse provides dietary instructions to a client with diabetes mellitus regarding the prescribed diabetic diet. Which statement made by the client indicated the need for further teaching? A. " Ill eat a balanced meal plan" B. " I need to drink diet soft drinks" C. " I need to buy special diabetic foods" D. " I will snack on fruit instead of cake"
B (Feedback: Due to the risk for aspiration, the nurse keeps a suction setup available in the hospital and instructs the family to do so at home for use if needed. TPN is not indicated and small meals do not necessarily reduce the risk of aspiration. Physical therapists do not address swallowing ability.)
The nurse recognizes that aspiration is a potential complication of a laryngectomy. How should the nurse best manage this risk? A) Facilitate total parenteral nutrition (TPN). B) Keep a complete suction setup at the bedside. C) Feed the patient several small meals daily. D) Refer the patient for occupational therapy.
B. Shakiness
The nurse reinforces teaching with a client with diabetes mellitus regarding differentiating between hypoglycemia and ketoacidosis. The client demonstrates an understanding of the teaching by stating that glucose will be taken if which symptom develops? A. Polyuria B. Shakiness C. Blurred Vision D. Fruity breath odor
3 The blood sample for ABG studies should be immediately analyzed, because the blood gases may get released or absorbed, making the results inaccurate. However, if the test cannot be performed immediately, the sample should be stored in ice. Storing the sample in a utility tray in room temperature may interfere with results. Expelling the air bubbles from the syringe prevents external air from getting absorbed. Collecting the sample in a heparinized syringe prevents the blood from clotting. Applying pressure on the puncture site for five minutes helps prevent hematoma formation or arterial bleeding. It does not interfere with the results. Text Reference - p. 491
The nurse suspects that the arterial blood gas (ABG) results of a patient may be incorrect. Which nursing error while collecting the sample may interfere with the results? 1 Expelling all the air bubbles from the syringe 2 Collecting the sample in a heparinized syringe 3 Storing the syringe in the utility tray until dispatched 4 Applying pressure on the puncture site for five minutes
C (Feedback: Hereditary angioedema is an inherited condition that is characterized by episodes of life-threatening laryngeal edema. No information supports lost days of work or reduced cardiac function.)
The occupational health nurse is obtaining a patient history during a pre-employment physical. During the history, the patient states that he has hereditary angioedema. The nurse should identify what implication of this health condition? A) It will result in increased loss of work days. B) It may cause episodes of weakness due to reduced cardiac output. C) It can cause life-threatening airway obstruction. D) It is unlikely to interfere with the individual's health.
1 The sound of the cough, sputum production, and description, as well as pattern, of the cough's occurrence (including acute or chronic) and what its occurrence is related to are the first assessments to be made to determine the severity. Frequency of the cough will not provide a lot of information. Family history can help to determine a genetic cause of the cough. Hematemesis is vomiting blood and is not as important as hemoptysis. Smoking is an important risk factor for chronic obstructive lung disease (COPD) and lung cancer and may cause a cough. Medications may or may not contribute to a cough as well as residence location. Weight loss, activity intolerance, and orthopnea may be related to respiratory or cardiac problems, but are not as important when dealing with a cough. Text Reference - p. 482
The patient is calling the clinic complaining of a cough. What assessment should be made first before the nurse advises the patient? 1 Cough sound, sputum production, and pattern 2 Frequency, a family history, and hematemesis 3 Smoking, medications, and residence location 4 Weight loss, activity tolerance, and orthopnea
3 Arterial blood gases are used to assess the efficiency of gas transfer in the lung and tissue oxygenation, as is pulse oximetry. Thoracentesis is used to obtain specimens for diagnostic evaluation, remove pleural fluid, or instill medication into the pleural space. Bronchoscopy is used for diagnostic purposes, to obtain biopsy specimens, and to assess changes resulting from treatment. Pulmonary function tests measure lung volumes and airflow to diagnose pulmonary disease, monitor disease progression, evaluate disability, and evaluate response to bronchodilators. Text Reference - p. 491
The patient is hospitalized with pneumonia. Which diagnostic test should be used to measure the efficiency of gas transfer in the lung and tissue oxygenation? 1 Thoracentesis 2 Bronchoscopy 3 Arterial blood gases 4 Pulmonary function tests
A. Iodine
The patient presents to the clinic with a gross enlargement of the anterior neck. The nurse knows that this is most likely caused by a deficiency in which substance? A. Iodine B. Sodium C. Calcium D. Vitamin D
2, 5 The care for the patient with SIADH will include limiting fluids to 1000 mL per day or less to decrease weight, increase osmolality, and improve symptoms. The head of the bed should be kept elevated at 10 degrees or less to enhance venous return to the heart and increase left atrial filling pressure, thereby reducing the release of ADH. The weights should be done daily along with intake and output. Signs of hyponatremia should be monitored and frequent turning, positioning, and range of motion exercises are important to maintain skin integrity and joint mobility. Text Reference - p. 1193
The patient with systemic lupus erythematosus has been diagnosed with syndrome of inappropriate antidiuretic hormone (SIADH). What should the nurse expect to include in this patient's plan of care? Select all that apply. 1 Obtain weekly weights 2 Limit fluids to 1000 mL per day 3 Monitor for signs of hypernatremia 4 Minimize turning and range of motion 5 Keep the head of the bed at 10 degrees or less elevation
B (Feedback: Hemorrhage is a potential complication of a tonsillectomy. Increased pulse, fever, and restlessness may indicate a postoperative hemorrhage. Difficulty ambulating and bradycardia are not common complications in a patient after a tonsillectomy. Infrequent swallowing does not indicate hemorrhage; frequent swallowing does.)
The perioperative nurse has admitted a patient who has just underwent a tonsillectomy. The nurse's postoperative assessment should prioritize which of the following potential complications of this surgery? A) Difficulty ambulating B) Hemorrhage C) Infrequent swallowing D) Bradycardia
Which of the following assessment findings in the pt with pneumonia most indicates a need to remind the pt to cough and deep breathe?
The pt develops coarse wheezes and crackles
A pt with end-stage COPD has a nursing diagnosis of Impaired Gas Exchange. Which assessment finding shows that interventions have been effective?
The pt's SpO2 is 92% on 2L of oxygen
3, 5 Dilutional hyponatremia occurs in cases of SIADH due to increased water reabsorption in the renal tubules and impairment in water excretion caused by the inability to suppress the secretions of antidiuretic hormone (ADH). A low serum osmolality will suppress the release of ADH, resulting in decreased water reabsorption and more concentrated plasma. Text Reference - p. 1193
The registered nurse is preparing to teach a group of nursing students about the pathophysiologic events associated with syndrome of inappropriate antidiuretic hormone (SIADH). Which events should the nurse include in the teaching plan? Select all that apply. 1 Decreased serum osmolality 2 Occurrence of hypernatremia 3 Increased water reabsorption 4 Decreased water reabsorption 5 Occurrence of dilutional hyponatremia
"The disease is characterized by decreased lung expansion."
The student nurse is caring for a patient with a restrictive respiratory disease. Which description demonstrates the student's knowledge of the disease? A. "The disease is characterized by increased lung volumes." B. "The disease is characterized by decreased lung expansion." C. "The disease is characterized by an obstruction in the lungs." D. "The disease is characterized by narrowed tracheobronchial tree openings."
emphysema chronic bronchitis
The student nurse is preparing a report about COPD. The student would be correct in including which disease(s) in the report? (Select all that apply.) A. Emphysema B. Bronchial asthma C. Chronic bronchitis D. Pleurisy with effusion E. Pulmonary tuberculosis
4 Cushing syndrome is caused by excessive secretion of adrenocorticotropic hormone.Acromegaly occurs due to excess secretion of growth hormone. Conn's disease occurs due to increased secretion of aldosterone hormone. Graves' disease occurs due to the excess secretion of thyroid hormone. Text Reference - p. 1190
The syndrome of inappropriate antidiuretic hormone (SIADH) is characterized by inappropriate secretion of ADH, which disrupts the fluid and electrolyte balance. Increased intravascular volume is one of the characteristic features of SIADH. Decreased ADH, excessive urine output, and increased serum osmolality are the features of diabetes insipidus. Text Reference - p. 1193
What are the different types of thoracic (chest) procedures?
Thoracentisis Chest tubes
Cilia
To defend against exposure to foreign particles, the mucous membrane of the respiratory tract contains tiny, hairlike projections. What are these called? A. Cilia B. Alveoli C. Surfactants D. Chemoreceptors
Cycloserine is added to the medication regimen for a client with TB. Which instruction should the nurse reinforce in the client-teaching plan regarding this medication?
To return to the clinic weekly for serum drug-level testing
The patient has decided to use the voice rehabilitation that offers the best speech quality even though it must be cleaned regularly. The nurse knows that this is what kind of voice rehabilitation? Electromyograph Intraoral electrolarynx Neck type electrolarynx Transesophageal puncture
Transesophageal puncture The transesophageal puncture provides a fistula between the esophagus and trachea with a one-way valved prosthesis to prevent aspiration from the esophagus to the trachea. Air moves from the lungs, vibrates against the esophagus, and words are formed with the tongue and lips as the air moves out the mouth. The electromyography and both electrolarynx methods produce low-pitched mechanical sounds.
C. rising levels of air pollution
Two of the most prevalent causative factors for the development of COPD are cigarette smoking and: A. high serum alpha-antitrypsin B. exposure to mycobacterium tuberculosis C. rising levels of air pollution D. frequent upper respiratory infections
Where is the chest tube inserted to remove air?
Upper anterior chest (2nd intercostal space)
The low-pressure alarm sounds on the ventilator. The nurse checks the client then attempts to determine the cause of the alarm but is unsuccessful. Which initial action should be done?
Ventilate the client manually
1, 4 There are three categories of age-related changes that impact the respiratory system, including changes in structure, defense mechanisms, and respiratory control. A decrease in cilia function and cell-mediated immunity are both age-related defense mechanism changes. Decreased chest wall compliance is a structural change. Decreased response to hypoxemia is a respiratory control change. Decreased respiratory muscle strength is an age-related structural change. Text Reference - p. 481
What are age-related changes to the respiratory system's defense mechanisms? Select all that apply. 1 Decreased cilia function 2 Decreased chest wall compliance 3 Decreased response to hypoxemia 4 Decreased cell-mediated immunity 5 Decreased respiratory muscle strength
Antihistamines, corticosteroids, and decongestants
What are some commonly prescribed drugs used for allergic rhinitis and sinusitis? A. Antihistamines, beta blockers, and aspirin B. corticosteriods, anginals, and anticoagulants C. Corticosteroids, alpha antagonists, and aspirin D. Antihistamines, corticosteroids, and decongestants
2 Surfactant is a lipoprotein that lowers the surface tension in the alveoli and reduces the amount of pressure needed to inflate the alveoli, making them less likely to collapse. Therefore, the primary reason that atelectasis occurs is due to a surfactant deficiency. Deficiencies in platelets, red blood cells, and white blood cells do not cause atelectasis. Text Reference - p. 477
What is the cause of atelectasis? 1 Platelet deficiency 2 Surfactant deficiency 3 Red blood cell deficiency 4 White blood cell deficiency
3 The diaphragm is the major muscle of respiration. It is a sheet of internal skeletal muscle. An accessory muscle is a relatively rare anatomic duplication of muscle that may appear anywhere in the muscular system. The intercostal muscles are several groups of muscles that run between the ribs and help form and move the chest wall. The abdominal muscles support the trunk, allow movement, and hold organs in place by regulating internal abdominal pressure. They also assist in expelling air during labored breathing. Text Reference - p. 478
What is the major muscle of respiration? 1 Accessory muscle 2 Intercostal muscle 3 Diaphragm muscle 4 Abdominal muscle
Alveoli
What structure allows for gas exchange with the pulmonary capillaries during respiration? A. Lungs B. Alveoli C. Bronchi D. Trachea
Which term should be used to document the musical sounds generated by airflow through narrowed airways?
Wheezes
D. Parathyroid
When a patient has alterations in serum calcium and phosphorus levels, which gland is dysfunctional? A. Thyroid B. Adrenal C. Pancreas D. Parathyroid
4 When a skin test is administered for TB bacilli, the nurse should chart the site of administration by drawing a diagram of the forearm and hand and labeling the injection sites. The nurse should ensure that the injection is given intradermally. The nurse should circle the area with a pen and instruct the patient not to remove the mark. The diameter of the induration should be measured for reading the test. The reddened flat area is not included in the measurement. Text Reference - p. 490
When administering and reading the tuberculosis (TB) skin test, what measures should the nurse take? 1 Ensure that the injection is given subcutaneously. 2 Do not use a pen around the test area to mark the site. 3 Include the reddened flat areas on the skin when measuring the induration. 4 Draw a diagram of the forearm and hand and label the injection sites in the patient's chart.
D. Epinephrine
When an individual is under a great deal of stress for a prolonged period of time, which hormone would be expected to be secreted in increased amounts? A. Insulin B. Estrogen C. Thyroxine D. Epinephrine
1, 3, 5 The patient with sleep apnea may have insomnia or abrupt awakenings. Patients with cardiovascular disease (e.g., heart failure that may affect respiratory health) may need to sleep with the head elevated on several pillows (orthopnea). Sleeping more than eight hours per night or needing to urinate during the night is not indicative of impaired respiratory health. Text Reference - p. 483
When assessing a patient's sleep-rest pattern related to respiratory health, about what should the nurse ask the patient? Select all that apply. 1 Do you have trouble falling asleep? 2 Do you need to urinate during the night? 3 Do you awaken abruptly during the night? 4 Do you sleep more than eight hours per night? 5 Do you need to sleep with the head elevated?
3 Coarse crackles are a series of long-duration, discontinuous, low-pitched sounds caused by air passing through an airway intermittently occluded by mucus, an unstable bronchial wall, or a fold of mucosa. Coarse crackles are evident on inspiration and at times expiration. Stridor is a continuous crowing sound of constant pitch from partial obstruction of larynx or trachea. Rhonchi are a continuous rumbling, snoring, or rattling sound from obstruction of large airways with secretions. Bronchovesicular sounds are normal sounds heard anteriorly over the mainstem bronchi on either side of the sternum and posteriorly between the scapulae with a medium pitch and intensity. Text Reference - p. 489
When auscultating the lower lungs of a patient, the nurse hears breath sounds that are of long-duration, discontinuous, low-pitched caused by air passing through an airway intermittently occluded by mucus on inspiration and sometimes on expiration. How should the nurse document these sounds? 1 Stridor 2 Rhonchi 3 Coarse crackles 4 Bronchovesicular
D. I will notify my health care provider if my blood glucose level is consistently greater than 250mg/dL
When the nurse is reinforcing instructions to a client who has been newly diagnosed with type 1 diabetes mellitus, which statement by the client would indicate that the teaching has been effective? A. " I will stop taking my insulin if im too sick to eat." B. " I will decrease my insulin during times of illness." C. " I will adjust my insulin dose according to the level of glucose in my urine." D. I will notify my health care provider if my blood glucose level is consistently greater than 250mg/dL
4 When a patient with type 1 diabetes has hyperglycemia and ketonemia causing metabolic acidosis, the physiologic response is to increase the respiratory rate and tidal volume to blow off the excess CO2. Vomiting and increased urination may occur with hyperglycemia, but not as physiologic responses to metabolic acidosis. The heart rate will increase. Text Reference - p. 481
When the patient is experiencing metabolic acidosis secondary to type 1 diabetes mellitus, what physiologic response should the nurse expect to assess in the patient? 1 Vomiting 2 Increased urination 3 Decreased heart rate 4 Rapid respiratory rate
3, 4 Age-related changes to respiratory control include decreased responses to hypoxemia and hypercapnia. A less forceful cough is an age-related change to respiratory defense mechanisms. Calcification of the costal cartilage and a decrease in functional alveoli are age-related structural changes to the respiratory system. Text Reference - p. 481
Which age-related changes to respiratory control may be observed while assessing an older adult? Select all that apply. 1 Less forceful cough 2 Calcification of costal cartilage 3 Decreased response to hypoxemia 4 Decreased response to hypercapnia 5 Decreased number of functional alveoli
3 Symmetric chest expansion and contraction is a normal assessment finding. Positive egophony and fine crackles are not normal findings. Bronchial or bronchovesicular sounds heard in the peripheral lung fields are abnormal breath sounds. Test-Taking Tip: Practicing a few relaxation techniques may prove helpful on the day of an examination. Relaxation techniques such as deep breathing, imagery, head rolling, shoulder shrugging, rotating and stretching of the neck, leg lifts, and heel lifts with feet flat on the floor can effectively reduce tension while causing little or no distraction to those around you. It is recommended that you practice one or two of these techniques intermittently to avoid becoming tense. The more anxious and tense you become, the longer it will take you to relax. Text Reference - p. 486
Which assessment finding of the respiratory system does the nurse interpret as normal? 1 Positive egophony 2 Fine crackles over the distal lung fields 3 Symmetric chest expansion and contraction 4 Bronchial breath sounds in the lower lung fields
2 Excess GH after closure of the epiphyseal plates results in acromegaly. When there is excess GH before the epiphyseal plates close, then gigantism can result. Dwarfism is associated with a deficiency, not an excess of GH and cretinism can result as an effect of congenital hypothyroidism. Text Reference - p. 1190
Which condition can result if hypersecretion of growth hormone (GH) occurs after epiphyseal plate closure? 1 Dwarfism 2 Acromegaly 3 Gigantism 4 Cretinism
4, 5 Pleural effusion is a condition in which there is an accumulation of fluids in the pleural space. Pleural effusion can be caused by a blockage of lymphatic drainage and an imbalance between intravascular and oncotic fluid pressure. Rib fracture, phrenic nerve paralysis, and irritation of the parietal pleura do not cause pleural effusion. Text Reference - p. 486
Which conditions cause pleural effusion? Select all that apply. 1 Rib fracture 2 Phrenic nerve paralysis 3 Irritation of parietal pleura 4 Blockage of lymphatic drainage 5 Imbalance between intravascular and oncotic fluid pressure
1 The cough reflex is a protective reflex action that clears the airway by a high pressure, high velocity flow of air. Coughing is effective in removing secretions in the main airways. Below the larynx, movement of mucus is accomplished by the mucociliary clearance system or the mucociliary escalator. Secretions below the subsegmental level must be moved upward by the mucociliary escalator before they can be removed by coughing. Alveolar macrophages rapidly phagocytize foreign particles, such as bacteria. The debris is moved by the cilia in the bronchi for removal. Reflex bronchoconstriction is a protective mechanism, triggered by inhaling irritating substances. Text Reference - p. 481 v
Which defense mechanism is effective for removing secretions in the main airways? 1 Cough reflex 2 Mucociliary escalator 3 Alveolar macrophages 4 Reflex bronchoconstriction
4 Ectopic adrenocorticotropic hormone production is more common in men. Graves' disease affects four to eight times more women than men. Thyroid nodules affect up to four times more women than men. Hyperaldosteronism affects two times as many as women as men. Text Reference - p. 1190
Which endocrine problem is more common in men than in women? 1 Graves' disease 2 Thyroid nodules 3 Hyperaldosteronism 4 Ectopic adrenocorticotropic hormone production
1, 3, 4 Desmopressin acetate is a synthetic analogue of natural pituitary hormone, 8-arginine vasopressin (ADH), an antidiuretic hormone that affects renal water conservation. Assessing the weight of the patient when the patient is using desmopressin acetate is important, because weight gain is a side effect of desmopressin acetate. The use of desmopressin acetate may also alter the mental status of the patient, causing hallucinations and depression. Desmporessin acetate may cause fluctuations in sodium levels. Therefore, regular assessment of sodium levels of the patient is necessary. Reduced skin turgor and dried mucous membranes are the signs of severe dehydration, which are not side effects of desmopressin acetate. Text Reference - p. 1195
Which factors should the nurse assess in a patient who is on desmopressin acetate (DDAVP)? Select all that apply. 1 Weight 2 Skin turgor 3 Mental status 4 Sodium levels 5 Mucus membranes
clubbing of the fingers
Which finding in a female patient should indicate to the LPN/LVN that the patient is likely to have a respiratory problem? A. Clubbing of the fingers B. Inverted breast nipples C. Inability to rotate the shoulder joint A. A fine maculopapular rash over the anterior of her chest
2 The alveoli are interconnected by the pores of Kohn, which allow the passage of air from alveolus to alveolus. The main stem bronchi subdivide to form the lobar, segmental, and subsegmental bronchi. Further divisions form bronchioles, which cause bronchoconstriction and bronchodilation. Lungs are lined by a membrane called the visceral pleura. The chest cavity is lined with a membrane called the parietal pleura. Test-Taking Tip: Reread the question if the answers do not seem to make sense, because you may have missed words such as not or except in the statement. Text Reference - p. 477
Which interconnected structure allows the movement of air between the alveoli? 1 Bronchioles 2 Pores of Kohn 3 Visceral pleura 4 Parietal pleura
3 Adventitious is the term used to describe abnormal breath sounds such as crackles, rhonchi, wheezes, and a pleural friction rub. The three normal breath sounds are vesicular, bronchovesicular, and bronchial sounds. Vesicular sounds are relatively soft, low-pitched, gentle, rustling sounds. Bronchial sounds are louder and higher pitched; they resemble air blowing through a hollow pipe. Bronchovesicular sounds have a medium pitch and intensity and are heard anteriorly over the main stem bronchi on either side of the sternum and posteriorly between the scapulae. Text Reference - p. 488
Which is the term used to describe abnormal breath sounds? 1 Vesicular 2 Bronchial 3 Adventitious 4 Bronchovesicular
2 Polyuria and polydipsia are the major clinical manifestations of diabetes insipidus. Therefore strict monitoring of fluid intake and output is a priority nursing intervention. Diet education and finger-stick blood glucose measurements are not high-priority interventions for diabetes insipidus. Constipation can be a secondary problem, as a result of dehydration. Text Reference - p. 1194
Which nursing intervention is most important for a patient with diabetes insipidus? 1 Providing dietary education 2 Monitoring fluid intake and output 3 Assessing for constipation every day 4 Obtaining a finger-stick blood glucose level
2, 4 When assessing the effects that a respiratory diagnosis has on activity-exercise patterns, the nurse will ask the patient if walking is impacted by dyspnea and how many flights of steps the patient can walk up before dyspnea occurs. Asking the patient about urinary incontinence with coughing is appropriate when assessing elimination patterns. Asking the patient if the spouse wakes him or her up in the middle of the night due to snoring will assess sleep-rest patterns. Asking the patient if he or she feels full quickly when eating assesses the patient's nutritional-metabolic pattern. Test-Taking Tip: Identify option components as correct or incorrect. This may help you identify a wrong answer. Text Reference - p. 484
Which questions are appropriate when assessing the effects of the patient's respiratory diagnosis on activity-exercise patterns? Select all that apply. 1 "Are you ever incontinent of urine when you cough?" 2 "Do you have trouble walking due to shortness of breath?" 3 "Does your spouse wake you in the middle of the night due to snoring?" 4 "How many flights of stairs can you walk up before you are short of breath?" 5 "Do you ever feel full very quickly when eating due to your breathing issues?"
2, 3, 4 Acromegaly is a disorder in which there is increased secretion of growth hormone (GH). Enlargement of the feet and hands occurs as a result of overgrowth of bones and tissue. GH antagonizes the action of insulin, and therefore blood glucose is elevated. Headaches also are common if the increased secretion of GH is caused by a pituitary adenoma, which increases pressure on the optic nerve. The skin becomes thick and leathery. The patient's weight may increase, but there is no change in height because acromegaly occurs after epiphyseal closure. Text Reference - p. 1190
Which signs and symptoms would the nurse expect to assess in a patient who is diagnosed with acromegaly? Select all that apply. 1 Fragile skin 2 Increased shoe size 3 Elevated blood glucose 4 Complaint of headaches 5 Increased height and weight
3, 5 The ABG is used to monitor oxygenation, acid-base balance, and carbon dioxide levels in the blood. The ABG is not used to monitor white blood cells or SpO2 in the blood. Text Reference - p. 478
Which statements by the student nurse indicate a need for further teaching about arterial blood gas (ABG) analysis? Select all that apply. 1 "The ABG is used to monitor oxygenation." 2 "The ABG is used to monitor acid-base balance." 3 "The ABG is used to monitor white blood cells in the blood." 4 "The ABG is used to monitor levels of carbon dioxide in the blood." 5 "The ABG is used to monitor the arterial oxygen saturation (SpO2) in the blood."
2, 5 The patient's PaO2 and SpO2 indicate inadequate oxygenation. Complications associated with these findings include tissue hypoxia and cardiac arrhythmia. Atelectasis, pleural effusion, and pulmonary edema are not complications associated with these data. Text Reference - p. 480
While assessing a patient, the nurse finds that the partial pressure of oxygen (PaO2) is 39 percent and the peripheral capillary oxygen saturation (SpO2) is 72 percent. Which complications are associated with these assessment findings? Select all that apply. 1 Atelectasis 2 Tissue hypoxia 3 Pleural effusion 4 Pulmonary edema 5 Cardiac arrhythmia
4 The nurse assesses fremitus in a patient by palpating the chest with his or her hands in the manner described. If the patient has pleural effusion, there will be distance between the lung tissue and the nurse's hand, which will decrease the vibrations felt. Fremitus would be increased in cases of pneumonia and lung tumors, because the vibration of sound would be transmitted more through solid tissue. The nurse would not detect any fremitus in a patient with pneumothorax because the condition prevents any vibration from transmitting. Text Reference - p. 486
While palpating a patient's chest, the nurse places the palmar surface of the hands with hyperextended fingers against the patient's chest. The nurse asks the patient to say "ninety-nine" in a deeper, louder than normal voice. The nurse finds that the vibration of the sound is decreased in the patient. What does this finding indicate? 1 The patient has pneumonia 2 The patient has a lung tumor 3 The patient has pneumothorax 4 The patient has pleural effusion
How does positive end expiatory pressure (PEEP)?
With PEEP, the client is on the ventilator On end expiration, the vent exerts pressure down into the lungs to keep the alveoli open Improves gas exchanges and decreases the work of breathing It exapnds and realigns the ribs so they can start growing back together PEEP may also be used to treat pulmonary edema or severe hypoxemia. The classic reason to use PEEP is Acute Respiratory Distress Syndrome (ARDS)
Can a client have a chest tubes inserted to remove fluid and air?
Yes They are y-connected together and attached to a closed chest drainage unit (CDU)
How does the water seal chamber work?
You may see bubbling in this chamber when the client coughs, sneezes, or exhales You will see a slight rise and fall of water in the water seal tube as the client tubes This fluctuation is called tidaling and is normal. If tidaling stops, it usually means that the lung has re-expanded. The air exits the water seal chamber and enters the third chamber called the suction control chamber. This allows any air to be vented out through the air vent found at the top of the suction control chamber.
The nurse assisting in the admission of a client with diabetic ketoacidosis will anticipate the physician ordering which of the following types of intravenous solution if the client cannot take fluids orally? a. 0.45% normal saline solution b. Lactated Ringer's solution c. 0.9 normal saline solution d. 5% dextrose in water (D5W)
a. 0.45% normal saline solution
Which of the following factors are risks for the development of diabetes mellitus? (Select all that apply.) a. Age over 45 years b. Overweight with a waist/hip ratio >1 c. Having a consistent HDL level above 40 mg/dl d. Maintaining a sedentary lifestyle
a. Age over 45 years b. Overweight with a waist/hip ratio >1 d. Maintaining a sedentary lifestyle Aging results in reduced ability of beta cells to respond with insulin effectively. Overweight with waist/hip ratio increase is part of the metabolic syndrome of DM II. There is an increase in atherosclerosis with DM due to the metabolic syndrome and sedentary lifestyle.
Which laboratory test should a nurse anticipate a physician would order when an older person is identified as high-risk for diabetes mellitus? (Select all that apply.) a. Fasting Plasma Glucose (FPG) b. Two-hour Oral Glucose Tolerance Test (OGTT) c. Glycosylated hemoglobin (HbA1C) d. Finger stick glucose three times daily
a. Fasting Plasma Glucose (FPG) b. Two-hour Oral Glucose Tolerance Test (OGTT) When an older person is identified as high-risk for diabetes, appropriate testing would include FPG and OGTT. A FPG greater than 140 mg/dL usually indicates diabetes. The OGTT is to determine how the body responds to the ingestion of carbohydrates in a meal. HbA1C evaluates long-term glucose control. A finger stick glucose three times daily spot-checks blood glucose levels.
A pt is admitted to the hospital with a diagnosis of Cushing syndrome. On physical assessment of the patient, the nurse would expect to find a. HTN, peripheral edema, and petechiae b. weight loss, buffalo hump, moon face with acne c. abdominal and buttock striae, truncal obesity, and hypotension d. anorexia, signs of dehydration, and hyper pigmentation of the skin
a. HTN, peripheral edema, and petechiae (R- effects of glucocorticoid excess include weight gain from accumulation and redistribution of adipose tissue, Na and water retention, glucose intolerance, protein wasting, loss of bone structure, loss of collagen, capillary fragility. CM of corticosteroid deficiency include hypotension, dehydration, weight loss, hyperpigmentation of skin.)
After having a transverse colostomy constructed for colon cancer, discharge planning for home care would include teaching about the ostomy appliance. Information appropriate for this intervention would include: a. Instructing the client to report redness, swelling, fever, or pain at the site to the physician for evaluation of infection b. Nothing can be done about the concerns of odor with the appliance. c. Ordering appliances through the client's health care provider d. The appliance will not be needed when traveling.
a. Instructing the client to report redness, swelling, fever, or pain at the site to the physician for evaluation of infection Signs and symptoms for monitoring infection at the ostomy site are a priority evaluation for clients with new ostomies. The remaining actions are not appropriate. There are supplies avaliable for clients to help control odor that may be incurred because of the ostomy. Although a prescription for ostomy supplies is needed, you can order the supplies from any medical supplier. Dependent on the location and trainability of the ostomy, appliances are almost always worn throughout the day and when traveling
When caring for a patient with primary hyperaldosteronism, the nurse would question a physician's order for the use of a. Lasix b. amiloride (midamor) c. spironolactone (aldactone) d. aminoglutethimide (cytadren)
a. Lasix37 (R- hyperaldosteronism is an excess of aldosterone, which is manifested by sodium and water retention and potassium excretion. Lasix is a potassium-wasting diuretic that would increase the potassium deficiency. Aminoglutethimide blocks aldosterone synthesis; amiloride is apotassium-sparing diuretic; and spironolactone blocks mineralocorticoid receptors in the kidney, increasing secretion of sodium and water and retention of potassium.)
A client has just had surgery for colon cancer. Which of the following disorders might the client develop? a. Peritonitis b. Diverticulosis c. Partial bowel obstruction d. Complete bowel obstruction
a. Peritonitis Bowel spillage could occur during surgery, resulting in peritonitis. Complete or partial bowel obstruction may occur before bowel resection. Diverticulosis doesn't result from surgery or colon cancer.
Which of the following is accurate pertaining to physical exercise and type 1 diabetes mellitus? a. Physical exercise can slow the progression of diabetes mellitus. b. Strenuous exercise is beneficial when the blood glucose is high. c. Patients who take insulin and engage in strenuous physical exercise might experience hyperglycemia. d. Adjusting insulin regimen allows for safe participation in all forms of exercise.
a. Physical exercise can slow the progression of diabetes mellitus. Physical exercise slows the progression of diabetes mellitus, because exercise has beneficial effects on carbohydrate metabolism and insulin sensitivity. Strenuous exercise can cause retinal damage, and can cause hypoglycemia. Insulin and foods both must be adjusted to allow safe participation in exercise.
Which one of the following methods/techniques will the nurse use when giving insulin to a thin person? [Hint] A. Pinch the skin up and use a 90 degree angle B. Use a 45 degree angle with the skin pinched up C. Massage the area of injection after injecting the insulin D. Warm the skin with a warmed towel or washcloth prior to the injection
a. Pinch the skin up and use a 90 degree angle The best angle for a thin person is 90 degrees with the skin pinched up. The area is not massaged and it is not necessary to warm it.
Physician's orders for a client with acute pancreatitis include the following: strict NPO, NG tube to low intermittent suction. The nurse recognizes that these interventions will: a. Reduce the secretion of pancreatic enzymes b. Decrease the client's need for insulin c. Prevent secretion of gastric acid d. Eliminate the need for analgesia
a. Reduce the secretion of pancreatic enzymes
Radiation therapy is used to treat colon cancer before surgery for which of the following reasons? a. Reducing the size of the tumor b. Eliminating the malignant cells c. Curing the cancer d. Helping the bowel heal after surgery
a. Reducing the size of the tumor Radiation therapy is used to treat colon cancer before surgery to reduce the size of the tumor, making it easier to be resected. Radiation therapy isn't curative, can't eliminate the malignant cells (though it helps define tumor margins), can could slow postoperative healing.
Which of the following diets is most commonly associated with colon cancer? a. low fiber, high fat b. low fat high fiber c. low protein, high carb d. low carb, high protein
a. a. low fiber, high fat low fiber, high fat diet reduced motility and increases the chance of constipation. The metabolic end products of this type of diet are carcinogenic. A LOW FAT HIGH FIBER diet is recommended to help avoid colon cancer. Carbohydrates and protein aren't necessarily associated with colon cancer.
The guidelines for Carbohydrate Counting as medical nutrition therapy for diabetes mellitus includes all of the following EXCEPT: a. Flexibility in types and amounts of foods consumed b. Unlimited intake of total fat, saturated fat and cholesterol c. Including adequate servings of fruits, vegetables and the dairy group d. Applicable to with either Type 1 or Type 2 diabetes mellitus
b. Unlimited intake of total fat, saturated fat and cholesterol
A patient newly diagnosed with Type I DM is being seen by the home health nurse. The doctors orders include: 1200 calorie ADA diet, 15 units NPH insulin before breakfast, and check blood sugar qid. When the nurse visits the patient at 5 pm, the nurse observes the man performing blood sugar analysis. The result is 50 mg/dL. The nurse would expect the patient to be a. confused with cold, clammy skin an pulse of 110 b. lethargic with hot dry dkin and rapid deep respirations c. alert and cooperative with BP of 130/80 and respirations of 12 d. short of breath, with distended neck veins and bounding pulse of 96.
a. confused with cold, clammy skin an pulse of 110 hypoglycemia
pt with acromegaly is treated w a transphenoidal hypophysectomy. Postoperatively, the nurse a. ensures that any clear nasal drainage is tested for glucose b. maintains the patient flat in bed to prevent cerebrospinal fluid leak c. assists the patient with toothbrushing Q4H to keep the surgical area clean d. encourages deep breathing and coughing to prevent respiratory complications
a. ensures any clear nasal drainage is tested for glucose (R- a transphenoidal hypophysectomy involves entry into the sella turcica through an incision in the upper lip and gingiva into the floor of the nose and the sphenoid sinuses. Postoperative clear nasal drainage with glucose content indicates CSF leakage from an open connection to the brain, putting the pt at risk for meningitis. After surgery, the pt is positioned with head elevated to avoid pressure on the sella turcica, coughing and straining are avoided to prevent increased ICP and CSF leakage, and although mouth care is required Q4H toothbrushing should not be performed for 7-10post sx.)
Preoperative instructions for the patient scheduled for a subtotal thyroidectomy includes teaching the patient a. how to support the head w hands when moving b. that coughing should due avoided to prevent pressure on the incision c. that the head and neck will need to remain immobile until the incision heals d. that any tingling around the lips or in the fingers after surgery is expected and temporary
a. how to support the head with the hands when moving (R- to prevent strain on suture line postoperatively, head must be manually supported while turning and moving in bed, but range-of-motion exercise for the head and neck are also taught preoperatively to be gradually implemented after surgery. There is no contraindication for coughing and deep breathing, and they should be carrier out postoperatively. Tingling around the lips or fingers is a sign of hypocalcemia, which may occur if the parathyroid glands are inadvertently removed during surgery, and should be reported immediately.)
During care of a patient with syndrome of inappropriate ADH (SIADH), the nurse should a. monitor neurologic status Q2H or more often if needed b. keep the HOB elevated to prevent ADH release c. teach the patient receiving treatment with diuretics to restrict sodium intake d. notify the physician if the p's blood pressure decreases more than 20mmHg from baseline
a. monitor neurologic status Q2H or more often if needed R- the pt with SIADH has marked dilution hyponatremia and should be monitored for decreased neurologic function and convulsions every 2 hours. ADH release is reduced by keeping the HOB flat to increase left atrial filling pressure, and sodium intake is supplemented because of hyponatremia and sodium loss caused by diuretics. A reduction in BP indicates a reduction in total fluid vo and is an expected outcome of treatment.)
The goal for pre-prandial blood glucose for those with Type 1 diabetes mellitus is: a. <80 mg/dl b. < 130 mg/dl c. <180 mg/dl d. <6%
b. < 130 mg/dl
he goal for pre-prandial blood glucose for those with Type 1 diabetes mellitus is: a. <80 mg/dl b. < 130 mg/dl c. <180 mg/dl d. <6%
b. < 130 mg/dl
Which of the following symptoms is a client with colon cancer most likely to exhibit? a. A change in appetite b. A change in bowel habits c. An increase in body weight d. An increase in body temperature
b. A change in bowel habits The most common complaint of the client with colon cancer is a change in bowel habits. The client may have anorexia, secondary abdominal distention, or weight loss. Fever isn't associated with colon cancer.
The nurse is planning dietary changes for a client following an episode of pancreatitis. Which diet is suitable for the client? a. Low calorie, low carbohydrate b. High calorie, low fat c. High protein, high fat d. Low protein, high carbohydrate
b. High calorie, low fat
While preparing the client for a colonoscopy, the nurse's responsibilities include: a. Explaining the risks and benefits of the exam b. Instructing the client about the bowel preparation prior to the test c, Instructing the client about medication that will be used to sedate the client d. Explaining the results of the exam
b. Instructing the client about the bowel preparation prior to the test The nurse is responsible for instructing the client about the bowel preparation prior to the test. Answers 1, 3, 4 are the physician's responsibility.
Which of these laboratory values noted by the nurse when reviewing the chart of a diabetic patient indicates the need for further assessment of the patient? a. Fasting blood glucose of 130 mg/dl b. Noon blood glucose of 52 mg/dl c. Glycosylated hemoglobin of 6.9% d. Hemoglobin A1C of 5.8%
b. Noon blood glucose of 52 mg/dl The nurse should assess the patient with a blood glucose level of 52 mg/dl for symptoms of hypoglycemia, and give the patient some carbohydrate-containing beverage such as orange juice. The other values are within an acceptable range for a diabetic patient.
A 1200-calorie diet and exercise are prescribed for a patient with newly diagnosed type 2 diabetes. The patient tells the nurse, "I hate to exercise! Can't I just follow the diet to keep my glucose under control?" The nurse teaches the patient that the major purpose of exercise for diabetics is to a. increase energy and sense of well-being, which will help with body image. b. facilitate weight loss, which will decrease peripheral insulin resistance. c. improve cardiovascular endurance, which is important for diabetics. d. set a successful pattern, which will help in making other needed changes.
b. facilitate weight loss, which will decrease peripheral insulin resistance. Rationale: Exercise is essential to decrease insulin resistance and improve blood glucose control. Increased energy, improved cardiovascular endurance, and setting a pattern of success are secondary benefits of exercise, but they are not the major reason.
When the patient with parathyroid disease experiences symptoms of hypocalcemia, a measure that can be used to temporarily raise serum calcium levels is to a. administer IV normal saline b. have the patient rebreathe in a paper bag c. administer Lasix as ordered d. administer oral phosphorous supplements
b. have the patient rebreathe in a paper bag (R- rebreathing in a paper bag promotes CO2 retention in blood, which lowers pH and creates an acidosis. An academia enhances solubility and ionization of calcium, increasing the proportion of total body Ca available in physiologically active form and relieving the sx of hypocalcemia. Saline promotes calcium excretion, as does Lasix. Phosphate levels in blood are reciprocal to calcium and an increase in phosphate promotes calcium excretion.)
An appropriate nursing intervention for the patient with hyperparathyroidism is to a. pad side rails as a seizure precaution b. increase fluid intake to 3000 to 4000ml/day c. maintain bed rest to prevent pathologic frxturs d. monitor the patient for Trousseau's phenomenon or Chvostek's sign
b. increase fluid intake to 3000 to 4000ml/day (R-high fluid intake is indicated in hyperparathyroidism to dilute hypercalcemia and flush the kidneys so that calcium stone formation is reduced.)
A patient with SIADH is treated with water restriction and administration of IV fluids. The nurses evaluates that treatment has been effective when the patient experiences a. increased urine output, decreased serum sodium, and increased urine specific gravity b. increased urine output, increased serum sodium, and decreased urine specific gravity c. decreased urine output, increased serum sodium, and decreased urine specific gravity d. decreased urine output, decreased serum sodium, and increased urine specific gravity
b. increased urine output, increased serum sodium, and decreased urine specific gravity (rationale- the patient with SIADH has water retention with hyponatremia, decreased urine output and concentrated urine with high specific gravity. improvement in the patient's condition reflected by increased urine output, normalization of serum sodium, and more water in the urine, decreasing the specific gravity.)
The most important nursing intervention during the medical and surgical treatment of the patient with a pheochromocytoma is a. administering IV fluids b. monitoring blood pressure c. monitoring I&O and daily weights d. administering B-adrenergic blocking agents
b. monitoring blood pressure38 (R- a pheochromocytoma is a catecholamine-producing tumor of the adrenal medulla, which may cause severe, episodic HTN; severe, pounding headache; and profuse sweating. Monitoring for dangerously high BP before surgery is critical, as is monitoring for BP fluctuation during medical and surgical tx.)
A patient who is admitted with acute hepatic encephalopathy and ascites receives instructions about appropriate diet. The nurse determines that the teaching has been effective when the patient's choice of foods from the menu includes a. an omelet with cheese and mushrooms and milk. b. pancakes with butter and honey and orange juice. c. baked beans with ham, cornbread, potatoes, and coffee. d. baked chicken with french-fries, low-fiber bread, and tea.
b. pancakes with butter and honey and orange juice. B Rationale: The patient with acute hepatic encephalopathy is placed on a LOW-protein diet to decrease ammonia levels. The other choices are all higher in protein and would not be as appropriate for this patient. In addition, the patient's ascites indicate that a low-sodium diet is needed and the other choices are all high in sodium.
A pt with hypothyroidism is treated h Synthroid. When teaching the pt about the therapy, the nurse a. explains that caloric intake must be reduced when drug therapy is started b. provides written instruction for all information related to the medication therapy c. assures the patient that a return to normal function will occur with replacement therapy d. informs the patient that medications must be taken until hormone balance is reestablished
b. provides written instruction for all information related to the medication therapy (R- bc of mental sluggishness, inattentiveness, and memory loss that occur with hypothyroidism, it is important to provide written instructions and repeat information when teaching pt. Caloric intake can be increased when drug therapy is started, because of an increased metabolic rate, and replacement therapy must be taken for life. Although most pts return to a normal state w treatment, cardiovascular conditions and psychoses may persist.)
When caring for a patient with nephrogenic DI, the nurse would expect treatment to include a. fluid restriction b. thiazide diuretics c. a high-sodium diet d. chlorpropamide (DIabinese)
b. thiazide diuretics (Rationale- in nephrogenic Di the kidney is unable to respond to ADH, so vasopressin or hormone analogs are not effective. Thiazide diuretics slow the glomerular filtration rate in the kidney and produce a decrease in urine output. Low-sodium diets are also thought to decrease urine output. Fluids are not restricted, because the patient could become easily dehydrated.)
Which of the following persons would most likely be diagnosed with diabetes mellitus? A 44-year-old: a. Caucasian woman. b. Asian woman. c. African-American woman. d. Hispanic male.
c. African-American woman. Age-specific prevalence of diagnosed diabetes mellitus (DM) is higher for African-Americans and Hispanics than for Caucasians. Among those younger than 75, black women had the highest incidence.
Which of the following diabetes drugs acts by decreasing the amount of glucose produced by the liver? a. Sulfonylureas b. Meglitinides c. Biguanides d. Alpha-glucosidase inhibitors
c. Biguanides Biguanides, such as metformin, lower blood glucose by reducing the amount of glucose produced by the liver. Sulfonylureas and Meglitinides stimulate the beta cells of the pancreas to produce more insulin. Alpha-glucosidase inhibitors block the breakdown of starches and some sugars, which helps to reduce blood glucose levels
The nurse is caring for a patient with a diagnosis of hypothyroidism. Which nursing diagnosis should the nurse most seriously consider when analyzing the needs of the patient? a. High risk for aspiration related to severe vomiting b. Diarrhea related to increased peristalsis c. Hypothermia related to slowed metabolic rate d. Oral mucous membrane, altered related to disease process
c. Hypothermia related to slowed metabolic rate Thyroid hormone deficiency results in reduction in the metabolic rate, resulting in hypothermia, and does predispose the older adult to a host of other health-related issues. One quarter of affected elderly experience constipation.
A patient with Addison's disease comes to the emergency department with complaints of N/V/D, and fever. The nurse would expect collaborative care to include a. parenteral injections of ACTH b. IV administration of vasopressors c. IV administration of hydrocortisone d. IV administration of D5W with 20mEq of KCl
c. IV administration of hydrocortisone (R- vom and dia are early indicators of addisonian crisis and fever indicates an infection, which s causing additional stress for pt. trtmt of crisis requires immediate glucocorticoid replacement, and IV hydrocortisone, fluids, Na and glucose are necessary for 24hours. Addison's disease is a primary insufficiency of adrenal gland, and ACTH is not effective, nor would vasopressors be effective w fluid deficiency of Addison's. Potassium levels are incd in Addison's dz, and KCl would be contraindicated.)
During treatment of a patient with a Minnesota balloon tamponade for bleeding esophageal varices, which nursing action will be included in the plan of care? a. Encourage the patient to cough and deep breathe. b. Insert the tube and verify its position q4hr. c. Monitor the patient for shortness of breath. d. Deflate the gastric balloon q8-12hr.
c. Monitor the patient for shortness of breath. Rationale: The most common complication of balloon tamponade is aspiration pneumonia. In addition, if the gastric balloon ruptures, the esophageal balloon may slip upward and occlude the airway. Coughing increases the pressure on the varices and increases the risk for bleeding. The health care provider inserts the tube and verifies the position. The esophageal balloon is deflated every 8 to 12 hours to avoid necrosis, but if the gastric balloon is deflated, the esophageal balloon may occlude the airway.
A nurse cares for a client following a liver biopsy. Which nursing care plan reflects proper care? a. Position in a dorsal recumbent position, with one pillow under the head b. Bed rest for 24 hours, with a pressure dressing over the biopsy site c. Position to a right side-lying position, with a pillow under the biopsy site d. Neurological checks of lower extremities every hour
c. Position to a right side-lying position, with a pillow under the biopsy site Positioning the client in a right side-lying position with a pillow under the biopsy site reflects proper care. Answer 1 does not permit the necessary pressure applied to the biopsy site. B ed rest is only required for several hours. There is no reason to do neurological checks.
A patient with DI is treated with nasal desmopression. The nurse recognize that the drug is not having an adequate therapeutic effect the the patient experiences a. headache and weight gain b. nasal irritation and nausea c. a urine specific gravity of 1.002 d. an oral intake greater than urinary output
c. a urine specific gravity of 1.002 (rationale- normal urine specific gravity is 1.003 to 1.030, and urine with a specific gravity of 1.002 is very dilute, indicating that there continues to be excessive loss of water and that treatment of DI is inadequate. H/A, weight gain, and oral intake greater the urinary output are signs of volume excess that occur with overmedication. Nasal irritation & nausea may also indicate overmedication.)
Mr. L. has a seven-year history of hepatic cirrhosis. He was brought to the emergency room because he began vomiting large amounts of dark-red blood. An Esophageal Balloon Tamponade tube was inserted to tamponade the bleeding esophageal varices. While the balloon tamponade is in place, the nurse caring for Mr. L. gives the highest priority to a. assessing his stools for occult blood. b. evaluating capillary refill in extremities. c. auscultating breath sounds. d. performing frequent mouth care.
c. auscultating breath sounds. Rationale: Airway obstruction and aspiration of gastric contents are potential serious complications of balloon tamponade. Frequent assessment of the client's respiratory status is the priority.
When providing discharge instructions to a pt following a subtotal thyroidectomy, the nurse advises the patient to a. never miss a daily dose of thyroid replacement therapy b. avoid regular exercise until thyroid function is normalized c. avoid eating foods such as soybeans, turnips, and rutabagas d. use warm salt water gargles several times a day to relieve throat pain
c. avoid eating foods such as soybeans, turnips, and rutabagas (when pt has subtotal thyroidectomy, thyroid replacement therapy is not given, because exogenous hormone inhibits pituitary production of TSH and delays or prevents the restoration of thyroid tissue regeneration. However, pt should avoid goitrogens, foods that inhibit thyroid, such as soybeans, turnips, rutabagas, and peanut skins. REgular exercise stimulates thyroid gland and is encourage. Salt water gargles are used for dryness and irritation of mouth and throat following radioactive iodine therapy.)
To prevent complications in the patient with Cushing syndrome, the nurse monitors the pt for a. hypotension b. hypoglycemia c. cardiac arrhythmias d. decreased cardiac output
c. cardiac arrhythmias (R- electrolyte changes that occur in Cushing syndrome include Na retention and K excretion by kidney, resulting in hypokalemia, -may lead to cardiac arrhythmias/ arrest. Hypotension, hypoglycemia, decreased cardiac strength and output are characteristic of adrenal insufficiency.)
A patient received 6 units of REGULAR INSULIN 3 hours ago. The nurse would be MOST concerned if which of the following was observed? a. kussmaul respirations and diaphoresis b. anorexia and lethargy c. diaphoresis and trembling d. headache and polyuria
c. diaphoresis and trembling indicates hypoglycemia
A patient is admitted to the hospital in thyrotoxic crisis. On physical assessment of the patient, the nurse would expect to find a. hoarseness and laryngeal stridor b. bulging eyeballs and arrhythmias c. elevated temperature and signs of heart failure d. lethargy progressing suddenly to impairment of consciousness
c. elevated temperature and signs of heart failure (R- a hyperthyroid crisis results in marked manifs of hyperthyroidism, w fever tachycardia, heart failure, shock, hyperthermia, agitation, N/V/D, delirium, and coma. Although exophthalmos may be present in pt w Gravs', it is not a signif factor in hyperthyroid crisis. Hoarsness and laryngeal stridor are ch of tetany of hypoparathyroidsm, and lethargy progressing to coma is characteristic of myxedema coma, a complication of hypothyroidism.
A patient is scheduled for bilateral adrenalectomy. During the postoperative period, the nurse would expect administration of corticosteroids to be a. reduced to promote wound healing b. withheld until sx of hypocortisolism appear c. increased to promote an adequate response to the stress of surgery d. reduced bc excessive hormones are released during surgical manipulation of the glands
c. increased to promote an adequate response to the stress of surgery (R- although pt with Cushing syndrome has excess corticosteroids, removal of the glands and the stress of surgery require that high doses of cortisone be administered postoperatively for several days. The nurse should monitor the pt postoperatively to detect whether large amounts of hormones were released during surgical manipulation and to ensure the healing is satisfactory.)
A patient with acute pancreatitis has a nasogastric (NG) tube to suction and is NPO. The nurse explains to the patient that the major purpose of this treatment is a. control of fluid and electrolyte imbalance. b. relief from nausea and vomiting. c. reduction of pancreatic enzymes. d. removal of the precipitating irritants.
c. reduction of pancreatic enzymes. Rationale: Pancreatic enzymes are released when the patient eats. NG suction and NPO status decrease the release of these enzymes. Fluid and electrolyte imbalances will be caused by NG suction and require that the patient receive IV fluids to prevent this. The patient's nausea and vomiting may decrease, but this is not the major reason for these treatments. The pancreatic enzymes that precipitate the pancreatitis are not removed by NG suction.
During assessment of the pt with acromegaly, the nurse would expect the patient to report a. infertility b. dry, irritated skin c. undesirable changes in appearance d. an increase in height of 2 to 3 inches per year
c. undesirable changes in appearance (R- the increased production of GH in acromegaly causes an increase in thickness and width of bones and enlargement of soft tissues, resulting in marked changes in facial features, oily and coarse skin, and speech difficulties. Height is not increased in adults w GH excess bc the epiphyses of the bones are closed, and infertility is not a common finding because growth hormone is usually the only pituitary hormone involved in acromegaly.)
Parathyroid hormone regulates...
calcium, phosphorus, and magnesium balance within the blood and bone by maintaining a balance between the mineral levels in the blood and bone. Hyperparathyroidism is associated with hypercalcemia; therefore, a decrease in the calcium level indicates an improvement in the client's condition.
client newly diagnosed with diabetes mellitus has been stabilized with daily insulin injections. A nurse prepares a discharge teaching plan regarding the insulin and plans to reinforce which of the following concepts? a) always keep insulin vials refrigerated b) ketones in the urine signify a need for less insulin c) increase the amount of insulin before unusual exercise d) systematically rotate insulin injections within one anatomic site
d) systematically rotate insulin injections within one anatomic site Insulin doses should not be adjusted nor increased before unusual exercise. If ketones are found in the urine, it possibly may indicate the need for additional insulin. To minimize the discomfort associated with insulin injections, insulin should be administered at room temperature. Injection sites should be rotated systematically within one anatomic site.
A pt with Grave's dz asks the nurse what caused the disorder. The best response by the nurse is a. "The cause of Grave's disease is not known, although it is thought to be genetic." b. "It is usually associated with goiter formation from an iodine deficiency over a long period of time." c. "Antibodies develop against thyroid tissue and destroy it, causing a deficiency of thyroid hormones" d. "In genetically susceptible persons antibodies form that attack thyroid tissue and stimulate overproduction of thyroid hormones."
d. "In genetically susceptible persons antibodies form that attack thyroid tissue and stimulate overproduction of thyroid hormones." (R- The antibodies present in Graves' disease that attack thyroid tissue cause hyperplasia of the gland and stimulate TSH receptors on the thyroid and activate the production of thyroid hormones, creating hyperthyroidism. disease is not directly genetic, but individuals appear to have a genetic susceptibility to become sensitized to develop autoimmune antibodies. Goiter formation from insufficient iodine intake is usually associated with hypothyroidism.)
When working in the community, the nurse will recommend routine screening for diabetes when the person has one or more of seven risk criteria. Which of the following persons that the nurse comes in contact with most needs to be screened for diabetes based on the seven risk criteria? a. A woman who is at 90% of standard body weight after delivering an eight-pound baby b. A middle-aged Caucasian male c. An older client who is hypotensive d. A client with an HDL cholesterol level of 40 mg/dl and a triglyceride level of 300 mg/dl
d. A client with an HDL cholesterol level of 40 mg/dl and a triglyceride level of 300 mg/dl The seven risk criteria include: greater than 120% of standard body weight, Certain races but not including Caucasian, delivery of a baby weighing more than 9 pounds or a diagnosis of gestational diabetes, hypertensive, HDL greater than 35 mg/dl or triglyceride level greater than 250 or a triglyceride level of greater than 250 mg/dl, and, lastly, impaired glucose tolerance or impaired fasting glucose on prior testing.
The nurse working in the physician's office is reviewing lab results on the clients seen that day. One of the clients who has classic diabetic symptoms had an eight-hour fasting plasma glucose test done. The nurse realizes that diagnostic criteria developed by the American Diabetes Association for diabetes include classic diabetic symptoms plus which of the following fasting plasma glucose levels? a. Greater than 106 mg/dl b. Greater than 126 mg/dl c. Higher than 140 mg/dl d. Higher than 160 mg/dl
d. Higher than 160 mg/dl
A frail elderly patient with a diagnosis of type 2 diabetes mellitus has been ill with pneumonia. The client's intake has been very poor, and she is admitted to the hospital for observation and management as needed. What is the most likely problem with this patient? a. Insulin resistance has developed. b. Diabetic ketoacidosis is occurring. c. Hypoglycemia unawareness is developing. d. Hyperglycemic hyperosmolar non-ketotic coma
d. Hyperglycemic hyperosmolar non-ketotic coma Illness, especially with the frail elderly patient whose appetite is poor, can result in dehydration and HHNC. Insulin resistance usually is indicated by a daily insulin requirement of 200 units or more. Diabetic ketoacidosis, an acute metabolic condition, usually is caused by absent or markedly decreased amounts of insulin.
Proliferative retinopathy is often treated using: a. Tonometry b. Fluorescein angiogram c. Antibiotics d. Laser surgery
d. Laser surgery Scatter laser treatment is used to shrink abnormal blood vessels in an effort to preserve vision. When there is significant bleeding in the eye, it is removed in a procedure known as vitrectomy. Tonometry is a diagnostic test that measures pressure inside the eye. A fluorescein angiogram is a diagnostic test that traces the flow of dye through the blood vessels in the retina; it is used to detect macular edema.
A pt suspected of having acromegaly has an elevated plasma growth hormone level. In acromegaly, the nurse would also expect the pt's diagnostic results to include a. hyperinsulinemia b. a plasma glucose of less than 70 c. decreased growth hormone levels with an oral glucose challenge test d. a serum sometomedin C (insulin-like growth-factor) of more than 300
d. a serum somatomedin C (Insulin-like-growth-factor) of more than 300 (R- a normal response to growth hormone secretion is stimulation of the liver to produce somatomedin C which stimulates growth of bones and soft tissue. The increased levels of somatomedin C normally inhibit growth hormone, but in acromegaly the pituitary gland secretes GH despite elevated somatomedin C levels.)
Causes of primary hypothyroidism in adults include a. malignant or benign thyroid nodules b. surgical removal or failure of the pituitary gland c. surgical removal or radiation of thyroid gland d. autoimmune-induced atrophy of the gland
d. autoimmune-induced atrophy of the gland (R- both Graves disease and Hasimotos thyroiditis are autoimmune disorders that eventually destroy thyroid gland, leading to primary hypothyroidism. Thyroid tumors most often result in hyperthyroidism. Secondary hypothyroidism occurs as a result of pituitary failure, and iatrogenic hypothyroidism results from thyroidectomy or radiation of the thyroid gland.)
A college student who has type 1 diabetes normally walks each evening as part of an exercise regimen. The student now plans to take a swimming class every day at 1:00 PM. The clinic nurse teaches the patient to a. delay eating the noon meal until after the swimming class. b. increase the morning dose of neutral protamine Hagedorn (NPH) insulin on days of the swimming class. c. time the morning insulin injection so that the peak occurs while swimming. d. check glucose level before, during, and after swimming.
d. check glucose level before, during, and after swimming. Rationale: The change in exercise will affect blood glucose, and the patient will need to monitor glucose carefully to determine the need for changes in diet and insulin administration. Because exercise tends to decrease blood glucose, patients are advised to eat before exercising. Increasing the morning NPH or timing the insulin to peak during exercise may lead to hypoglycemia, especially with the increased exercise.
Physical changes of hypothyroidism that must be monitored when replacement therapy is started include a. achlorhydria and constipation b. slowed mental processes and lethargy c. anemia and increased capillary fragility d. decreased cardiac contractility and coronary atherosclerosis
d. decreased cardiac contractility and coronary atherosclerosis (R- hypothyroidism affects the heart in many ways, causing cardiomyopathy, coronary atherosclerosis, bradycardia, pericardial effusions, and weakened cardiac contractility. when thyroid replacement therapy is started, myocardial oxygen consumption is increased and resultant oxygen demand may cause angina, cardiac arrhythmias, and HFs. It is important to monitor pts with compromised cardiac status when starting replacement therapy.)
When lactulose (Cephulac) 30 ml QID is ordered for a patient with advanced cirrhosis, the patient complains that it causes diarrhea. The nurse explains to the patient that it is still important to take the drug because the lactulose will a. promote fluid loss. b. prevent constipation. c. prevent gastrointestinal (GI) bleeding. d. improve nervous system function.
d. improve nervous system function. Rationale: The purpose for lactulose in the patient with cirrhosis is to lower ammonia levels and prevent encephalopathy. Although the medication may promote fluid loss through the stool, prevent constipation, and prevent bearing down during bowel movements (which could lead to esophageal bleeding), the medication is not ordered for these purposes for this patient.
In a patient with central diabetes insipidus, administration of aqueous vasopressin during a water deprivation test will result in a a. decrease in body weight b. increase in urinary output c. decrease in blood pressure d. increase in urine osmolality
d. increase in urine osmolality (R- pt with DI has a deficiency of ADH with excessive loss of water from the kidney, hypovolemia, hypernatreamia, and dilute urine with a low specific gravity. When vasopressin is administered, the symptoms are reversed, with water retention, decreased urinary output that increases urine osmolality, and an increase in BP.)
The nurse is performing discharge teaching for a patient with Addison's disease. It is MOST important for the nurse to instruct the patient about: a. signs and symptoms of infection b. fluid and electrolyte balance c. seizure precautions d. steroid replacement
d. steroid replacement steroid replacement is the most important information the client needs to know.
Colon cancer is most closely associated with which of the following conditions? a. appendicitis b. hemorroids c. hiatal hernia d. ulcerative colitis
d. ulcerative colitis Chronic ulcerative colitis, granulomas, and familial polyps seem to increase a person's chance of developing colon cancer. The other conditions listed have no known effect on the colon cancer risk.
Acromegaly
is a disorder caused by the excessive production of somatotropin (growth hormone) after closing the epiphyses (the "growth plate" at the ends of the long bones) by the pituitary gland. It results in gradual enlargement of body tissues including the bones of the face, jaw, hands, feet, and skull.
Which clinical manifestation should the nurse expect to note when assessing a client with Addison's disease? 1. Edema 2. Obesity 3. Hirsutism 4. Hypotension
rationale Common manifestations of Addison's disease include postural hypotension from fluid loss, syncope, muscle weakness, anorexia, nausea, vomiting, abdominal cramps, weight loss, depression, and irritability. The manifestations in options 1, 2, and 3 are not associated with Addison's disease.
A nurse is reviewing the postoperative prescriptions for a client who had a transsphenoidal hypophysectomy. Which health care provider's prescription, if noted on the record, indicates the need for clarification? 1. Instruct the client about the need for a Medic-Alert bracelet. 2. Apply a loose dressing if any clear drainage is noted. 3. Monitor vital signs and neurological status. 4. Instruct the client to avoid blowing the nose. Ans. 2
rationale The nurse should observe for clear nasal drainage, constant swallowing, and a severe, persistent, generalized, or frontal headache. These signs and symptoms indicate cerebrospinal fluid leak into the sinuses. If clear drainage is noted following this procedure, the health care provider needs to be notified immediately. Options 1, 3, and 4 indicate appropriate postoperative interventions.